Beginning Algebra
Beginning Algebra
Beginning Algebra
v. 1.0
This is the book Beginning Algebra (v. 1.0).
This book was accessible as of December 29, 2012, and it was downloaded then by Andy Schmitz
(http://lardbucket.org) in an effort to preserve the availability of this book.
Normally, the author and publisher would be credited here. However, the publisher has asked for the customary
Creative Commons attribution to the original publisher, authors, title, and book URI to be removed. Additionally,
per the publisher's request, their name has been removed in some passages. More information is available on this
project's attribution page (http://2012books.lardbucket.org/attribution.html?utm_source=header).
For more information on the source of this book, or why it is available for free, please see the project's home page
(http://2012books.lardbucket.org/). You can browse or download additional books there.
ii
Table of Contents
About the Author .................................................................................................................. 1
Acknowledgments................................................................................................................. 2
Preface..................................................................................................................................... 3
Chapter 1: Real Numbers and Their Operations ............................................................ 5
Real Numbers and the Number Line ............................................................................................................ 6
Adding and Subtracting Integers ............................................................................................................... 31
Multiplying and Dividing Integers ............................................................................................................. 47
Fractions ....................................................................................................................................................... 67
Review of Decimals and Percents ............................................................................................................... 99
Exponents and Square Roots .................................................................................................................... 127
Order of Operations ................................................................................................................................... 149
Review Exercises and Sample Exam......................................................................................................... 167
Chapter 2: Linear Equations and Inequalities ............................................................ 184
Introduction to Algebra............................................................................................................................. 185
Simplifying Algebraic Expressions........................................................................................................... 206
Solving Linear Equations: Part I ............................................................................................................... 227
Solving Linear Equations: Part II .............................................................................................................. 249
Applications of Linear Equations ............................................................................................................. 270
Ratio and Proportion Applications .......................................................................................................... 305
Introduction to Inequalities and Interval Notation ............................................................................... 327
Linear Inequalities (One Variable) ........................................................................................................... 346
Review Exercises and Sample Exam......................................................................................................... 372
Chapter 3: Graphing Lines .............................................................................................. 390
Rectangular Coordinate System ............................................................................................................... 391
Graph by Plotting Points ........................................................................................................................... 420
Graph Using Intercepts ............................................................................................................................. 447
Graph Using the y-Intercept and Slope ................................................................................................... 470
Finding Linear Equations .......................................................................................................................... 502
Parallel and Perpendicular Lines ............................................................................................................. 529
Introduction to Functions ......................................................................................................................... 548
Linear Inequalities (Two Variables) ......................................................................................................... 578
Review Exercises and Sample Exam......................................................................................................... 597
iii
Chapter 4: Solving Linear Systems................................................................................ 619
Solving Linear Systems by Graphing ....................................................................................................... 620
Solving Linear Systems by Substitution .................................................................................................. 644
Solving Linear Systems by Elimination ................................................................................................... 664
Applications of Linear Systems ................................................................................................................ 689
Solving Systems of Linear Inequalities (Two Variables)........................................................................ 715
Review Exercises and Sample Exam......................................................................................................... 732
Chapter 5: Polynomials and Their Operations ........................................................... 746
Rules of Exponents..................................................................................................................................... 747
Introduction to Polynomials..................................................................................................................... 775
Adding and Subtracting Polynomials ...................................................................................................... 797
Multiplying Polynomials ........................................................................................................................... 816
Dividing Polynomials................................................................................................................................. 841
Negative Exponents ................................................................................................................................... 864
Review Exercises and Sample Exam......................................................................................................... 887
Chapter 6: Factoring and Solving by Factoring .......................................................... 903
Introduction to Factoring ......................................................................................................................... 904
Factoring Trinomials of the Form x^2 + bx + c........................................................................................ 935
Factoring Trinomials of the Form ax^2 + bx + c...................................................................................... 955
Factoring Special Binomials...................................................................................................................... 978
General Guidelines for Factoring Polynomials ....................................................................................... 999
Solving Equations by Factoring .............................................................................................................. 1013
Applications Involving Quadratic Equations ........................................................................................ 1038
Review Exercises and Sample Exam....................................................................................................... 1063
Chapter 7: Rational Expressions and Equations....................................................... 1077
Simplifying Rational Expressions........................................................................................................... 1078
Multiplying and Dividing Rational Expressions ................................................................................... 1107
Adding and Subtracting Rational Expressions...................................................................................... 1127
Complex Rational Expressions................................................................................................................ 1152
Solving Rational Equations ..................................................................................................................... 1172
Applications of Rational Equations ........................................................................................................ 1192
Variation ................................................................................................................................................... 1219
Review Exercises and Sample Exam....................................................................................................... 1241
iv
Chapter 8: Radical Expressions and Equations......................................................... 1256
Radicals ..................................................................................................................................................... 1257
Simplifying Radical Expressions............................................................................................................. 1285
Adding and Subtracting Radical Expressions ....................................................................................... 1313
Multiplying and Dividing Radical Expressions ..................................................................................... 1330
Rational Exponents .................................................................................................................................. 1360
Solving Radical Equations ....................................................................................................................... 1385
Review Exercises and Sample Exam....................................................................................................... 1409
Chapter 9: Solving Quadratic Equations and Graphing Parabolas ....................... 1425
Extracting Square Roots .......................................................................................................................... 1426
Completing the Square ............................................................................................................................ 1449
Quadratic Formula ................................................................................................................................... 1469
Guidelines for Solving Quadratic Equations and Applications............................................................ 1488
Graphing Parabolas.................................................................................................................................. 1514
Introduction to Complex Numbers and Complex Solutions................................................................ 1553
Review Exercises and Sample Exam....................................................................................................... 1575
Appendix: Geometric Figures ....................................................................................... 1592
Plane .......................................................................................................................................................... 1593
Solid ........................................................................................................................................................... 1595
v
About the Author
John Redden earned his degrees at California State
University–Northridge and Glendale Community
College. He is now a professor of mathematics at the
College of the Sequoias, located in Visalia, California.
With over a decade of experience working with students
to develop their algebra skills, he knows just where they
struggle and how to present complex techniques in
more understandable ways. His student-friendly and
commonsense approach carries over to his writing of Elementary Algebra and various
other open-source learning resources.
1
Acknowledgments
I would like to thank the following reviewers whose feedback helped improve the
final product:
I would also like to acknowledge Michael Boezi and Vanessa Gennarelli of Unnamed
Publisher. The success of this project is in large part due to their vision and
expertise. Finally, a special heartfelt thank-you is due to my wife, Tracy, who spent
countless hours proofreading and editing these pages—all this while maintaining a
tight schedule for our family. Without her, this textbook would not have been
possible.
2
Preface
It is essential to lay a solid foundation in mathematics if a student is to be
competitive in today’s global market. The importance of algebra, in particular,
cannot be overstated, as it is the basis of all mathematical modeling used in
applications found in all disciplines. Traditionally, the study of algebra is separated
into a two parts, elementary algebra and intermediate algebra. This textbook,
Elementary Algebra, is the first part, written in a clear and concise manner, making
no assumption of prior algebra experience. It carefully guides students from the
basics to the more advanced techniques required to be successful in the next
course.
This text is, by far, the best elementary algebra textbook offered under a Creative
Commons license. It is written in such a way as to maintain maximum flexibility
and usability. A modular format was carefully integrated into the design. For
example, certain topics, like functions, can be covered or omitted without
compromising the overall flow of the text. An introduction of square roots in
Chapter 1 is another example that allows for instructors wishing to include the
quadratic formula early to do so. Topics such as these are carefully included to
enhance the flexibility throughout. This textbook will effectively enable traditional
or nontraditional approaches to elementary algebra. This, in addition to robust and
diverse exercise sets, provides the base for an excellent individualized textbook
instructors can use free of needless edition changes and excessive costs! A few other
differences are highlighted below:
3
Preface
mathematical equations. It also clearly lays out the steps required to build the skills
needed to solve these equations and interpret the results. With robust and diverse
exercise sets, students have the opportunity to solve plenty of practice problems. In
addition to embedded video examples and other online learning resources, the
importance of practice with pencil and paper is stressed. This text respects the
traditional approaches to algebra pedagogy while enhancing it with the technology
available today. In addition, textual notation is introduced as a means to
communicate solutions electronically throughout the text. While it is important to
obtain the skills to solve problems correctly, it is just as important to communicate
those solutions with others effectively in the modern era of instant
communications.
Unnamed Publisher is the only publisher today willing to put in the resources that
it takes to produce a quality, peer-reviewed textbook and allow it to be published
under a Creative Commons license. They have the system that implements the
customizable, affordable, and open textbook of the twenty-first century. In fact, this
textbook was specifically designed and written to fully maximize the potential of
the Unnamed Publisher system. I feel that my partnership with Unnamed Publisher
has produced a truly fine example in Elementary Algebra, which demonstrates what
is possible in the future of publishing.
4
Chapter 1
Real Numbers and Their Operations
5
Chapter 1 Real Numbers and Their Operations
LEARNING OBJECTIVES
Definitions
6. The set of natural numbers The set of integers7, denoted Z, consists of both positive and negative whole
combined with zero {0, 1, 2, 3, numbers, as well as zero.
4, 5, …}.
6
Chapter 1 Real Numbers and Their Operations
Notice that the sets of natural and whole numbers are both subsets of the set of
integers.
Rational numbers8, denoted Q, are defined as any number of the form ab, where a
and b are integers and b is nonzero. Decimals that repeat or terminate are rational.
For example,
The set of integers is a subset of the set of rational numbers because every integer
can be expressed as a ratio of the integer and 1. In other words, any integer can be
written over 1 and can be considered a rational number. For example,
Irrational numbers9 are defined as any number that cannot be written as a ratio of
two integers. Nonterminating decimals that do not repeat are irrational. For
example,
The set of real numbers10, denoted R, is defined as the set of all rational numbers
combined with the set of all irrational numbers. Therefore, all the numbers defined
so far are subsets of the set of real numbers. In summary,
Number Line
A real number line11, or simply number line, allows us to visually display real
numbers by associating them with unique points on a line. The real number
associated with a point is called a coordinate12. A point on the real number line
that is associated with a coordinate is called its graph13.
To construct a number line, draw a horizontal line with arrows on both ends to
indicate that it continues without bound. Next, choose any point to represent the
number zero; this point is called the origin14.
Mark off consistent lengths on both sides of the origin and label each tick mark to
define the scale. Positive real numbers lie to the right of the origin and negative
real numbers lie to the left. The number zero (0) is neither positive nor negative.
Typically, each tick represents one unit.
The graph of each real number is shown as a dot at the appropriate point on the
number line. A partial graph of the set of integers Z follows:
Solution: Graph the numbers on a number line with a scale where each tick mark
represents 13 unit.
When comparing real numbers on a number line, the larger number will always lie
to the right of the smaller one. It is clear that 15 is greater than 5, but it may not be
so clear to see that −1 is greater than −5 until we graph each number on a number
line.
We next define symbols that denote an order relationship between real numbers.
Since the graph of −120 is to the left of the graph of –10 on the number line, that
number is less than −10. We could write an equivalent statement as follows:
Similarly, since the graph of zero is to the right of the graph of any negative
number on the number line, zero is greater than any negative number.
The symbols < and > are used to denote strict inequalities16, and the symbols ≤ and
≥ are used to denote inclusive inequalities17. In some situations, more than one
symbol can be correctly applied. For example, the following two statements are
both true:
The logical use of the word “or” requires that only one of the conditions need be
true: the “less than” or the “equal to.”
Solution: Use > because the graph of −2 is to the right of the graph of −12 on a
number line. Therefore, −2 > −12, which reads “negative two is greater than negative
twelve.”
In this text, we will often point out the equivalent notation used to express
mathematical quantities electronically using the standard symbols available on a
keyboard. We begin with the equivalent textual notation for inequalities:
Many calculators, computer algebra systems, and programming languages use this
notation.
Opposites
The opposite18 of any real number a is −a. Opposite real numbers are the same
distance from the origin on a number line, but their graphs lie on opposite sides of
the origin and the numbers have opposite signs.
Next, consider the opposite of a negative number. Given the integer −7, the integer
the same distance from the origin and with the opposite sign is +7, or just 7.
Therefore, we say that the opposite of −7 is −(−7) = 7. This idea leads to what is often
referred to as the double-negative property19. For any real number a,
3
Answer: 4
Solution: Start with the innermost parentheses by finding the opposite of +4.
Answer: 4
Answer: −2
Tip
Answer: −5
Video Solution
The absolute value20 of a real number a, denoted |a|, is defined as the distance
between zero (the origin) and the graph of that real number on the number line.
Since it is a distance, it is always positive. For example,
Both 4 and −4 are four units from the origin, as illustrated below:
Example 6: Simplify:
a. |−12|
b. |12|
Solution: Both −12 and 12 are twelve units from the origin on a number line.
Therefore,
Answers: a. 12; b. 12
The absolute value can be expressed textually using the notation abs(a). We often
encounter negative absolute values, such as − |3| or −abs(3). Notice that the
negative sign is in front of the absolute value symbol. In this case, work the absolute
value first and then find the opposite of the result.
Try not to confuse this with the double-negative property, which states that
−(−7) = +7.
Solution: First, find the opposite of −7 inside the absolute value. Then find the
opposite of the result.
Answer: −7
At this point, we can determine what real numbers have a particular absolute value.
For example,
Think of a real number whose distance to the origin is 5 units. There are two
origin, namely, {±5}. The symbol (±) is read “plus or minus” and indicates that
solutions: the distance to the right of the origin and the distance to the left of the
Here we wish to find a value for which the distance to the origin is negative. Since
negative distance is not defined, this equation has no solution. If an equation has no
solution, we say the solution is the empty set: Ø.
KEY TAKEAWAYS
TOPIC EXERCISES
Determine whether the following real numbers are integers, rational, or irrational.
1
7. 2
8. −3
9. 4.5
10. −5
⎯⎯⎯⎯
11. 0.36
⎯⎯
12. 0.3
13. 1.001000100001 …
⎯⎯⎯⎯⎯⎯
14. 1.001
15. e = 2.71828 …
⎯⎯
16. √ 7 = 2.645751 …
17. −7
18. 3.14
22
19. 7
20. 1.33
21. 0
22. 8,675,309
True or false.
Choose an appropriate scale and graph the following sets of real numbers on a
number line.
29. {−3, 0 3}
31. {−2,
3}
1 2 5
− 3
, 3
,
32. {− , 2}
5 1 1
2
, − 2
, 0, 2
33. {− 7 , 1}
5 2
, 0, 7
34. { – 5, – 2, – 1, 0}
35. { − 3, − 2, 0, 2, 5}
41. −7 ___ 0
42. 30 ___ 2
46. 0 ___ 0
True or false.
51. 5 ≠7
52. 4 =5
53. 1 ≠1
55. 4 ≤4
56. −12 ≥0
58. 3 >3
60. 0 =0
67. 10 < 20
69. −4 ≠0
70. 30 ≥ −1
71. 0 =0
72. e ≈ 2.718
Part D: Opposites
Simplify.
79. −(−9)
80. − (−
5)
3
81. −(10)
82. −(3)
83. −(5)
84. − (
4)
3
85. − (−1)
86. − (− (−1))
87. − (− (1))
88. − (− (−3))
89. − (− (− (−11)))
1
90. What is the opposite of −
2
Simplify.
101. |20|
102. |−20|
103. |−33|
104. |−0.75||
105. |−
2|
| 5|
106. |
3|
|8|
107. |0|
108. |1|
109. − |12|
110. − |−20|
111. − |20|
112. − |−8|
113. − |7|
114. − |− |
3
| 16 |
115. − (− |
| 9 |)
8|
116. |−(−2)||
117. − |−(−3)||
119. − (− |−45||)
120. − |− (−21)||
121. abs(6)
122. abs(−7)
123. −abs(5)
124. −abs(−19)
125. − (−abs(9))
126. −abs(−(−12))
127. || ? || =9
128. || ? || = 15
129. || ? || =0
130. || ? || =1
131. || ? || = −8
132. || ? || = −20
133. |?| − 10 = −2
134. |?|| + 5 = 14
ANSWERS
3: {0, 1, 2, 3, 4, 5, 6, 7, 8, 9}
5: {2, 4, 6, 8, 10}
7: Rational
9: Rational
11: Rational
13: Irrational
15: Irrational
19: Rational
23: True
25: False
27: True
29:
31:
33:
35:
37:
39:
41: <
43: >
45: >
47: <
49: =
51: True
53: False
55: True
57: True
59: True
73: −7 <0
75: 0 ≥ −1
77: −2 = −2
79: 9
81: −10
83: −5
85: 1
87: 1
89: 11
91: −π
93: Larger
95: <
97: >
99: <
101: 20
103: 33
105: 2/5
107: 0
109: −12
111: −20
113: −7
115: 8/9
117: −3
119: 45
121: 6
123: −5
125: 9
127: ±9
129: 0
131: Ø, No solution
133: ±8
135: >
137: <
139: <
LEARNING OBJECTIVES
Visualize adding 3 + 2 on the number line by moving from zero three units to the
right then another two units to the right, as illustrated below:
In this example, the illustration shows (−3) + (−2) = −5, which leads to the following
two properties of real numbers.
Next, we will explore addition of numbers with unlike signs. To add 3 + (−7), first
move from the origin three units to the right, then move seven units to the left as
shown:
31
Chapter 1 Real Numbers and Their Operations
In this case, we can see that adding a negative number is equivalent to subtraction:
It is tempting to say that a positive number plus a negative number is negative, but
that is not always true: 7 + (−3) = 7 − 3 = 4. The result of adding numbers with
unlike signs may be positive or negative. The sign of the result is the same as the
sign of the number with the greatest distance from the origin. For example, the
following results depend on the sign of the number 12 because it is farther from
zero than 5:
Solution: Here −25 is the greater distance from the origin. Therefore, the result is
negative.
Answer: −11
Given any real numbers a, b, and c, we have the following properties of addition:
Associative property23: (a + b) + c = a + (b + c)
Example 2: Simplify:
a. 5 + 0
b. 10 + (−10)
Solution:
a. Adding zero to any real number results in the same real number.
(a + b) + c = a + (b + c).
23. Given real numbers a, b and c,
Answers: a. 5; b. 0
Example 3: Simplify:
a. (3 + 7) + 4
b. 3 + (7 + 4)
a.
b.
These two examples both result in 14: changing the grouping of the numbers does
not change the result.
Answers: a. 14; b. 14
At this point, we highlight the fact that addition is commutative: the order in which
we add does not matter and yields the same result.
We will use these properties, along with the double-negative property for real
numbers, to perform more involved sequential operations. To simplify things, we
will make it a general rule to first replace all sequential operations with either
addition or subtraction and then perform each operation in order from left to right.
Solution: Replace the sequential operations and then perform them from left to
right.
Answer: 9
Solution:
Answer: −4
Answer: 15
Video Solution
Often we find the need to translate English sentences involving addition and
subtraction to mathematical statements. Listed below are some key words that
translate to the given operation.
Solution: The key word “difference” implies that we should subtract the numbers.
Solution: The initial key word to focus on is “sum”; this means that we will be
adding the five numbers. The first five positive integers are {1, 2, 3, 4, 5}. Recall that
0 is neither positive nor negative.
One application of the absolute value is to find the distance between any two points
on a number line. For real numbers a and b, the distance formula for a number
line25 is given as,
Solution: On the graph we see that the distance between the two given integers is 5
units.
Answer: 5 units
Solution: Use the distance formula for a number line d = |b − a||, where a = −4
and b = 7.
Answer: 11 units
It turns out that it does not matter which points are used for a and b; the absolute
value always ensures a positive result.
d = |7 − (−4)||
d = |−4 − 7|
= |7 + 4||
= |−11|
= |11|
= 11
= 11
Try this! Determine the distance between −12 and −9 on the number line.
Answer: 3
Video Solution
KEY TAKEAWAYS
TOPIC EXERCISES
1. 24 + (−18)
2. 9 + (−11)
3. −31 +5
4. −12 + 15
5. −30 + (−8)
6. −50 + (−25)
7. −7 + (−7)
8. −13 − (−13)
9. 8 − 12 + 5
10. −3 −7+4
11. −1 −2−3−4
14. 2 − 7 + (−9)
26. Determine the sum of the integers in the set {−2, −1, 0, 1, 2}.
41. Mandy made a $200 deposit into her checking account on Tuesday. She
then wrote 4 checks for $50.00, $125.00, $60.00, and $45.00. How much more
than her deposit did she spend?
42. The quarterback ran the ball three times in last Sunday’s football game.
He gained 7 yards on one run but lost 3 yards and 8 yards on the other two.
What was his total yardage running for the game?
43. The revenue for a local photographer for the month is $1,200. His costs
include a studio rental of $600, props costing $105, materials fees of $135,
and a make-up artist who charges $120. What is his total profit for the
month?
44. An airplane flying at 30,000 feet lost 2,500 feet in altitude and then rose
1,200 feet. What is the new altitude of the plane?
45. The temperature was 22° at 6:00 p.m. and dropped 26° by midnight. What
was the temperature at midnight?
46. A nurse has 30 milliliters of saline solution but needs 75 milliliters of the
solution. How much more does she need?
47. The width of a rectangle is 2 inches less than its length. If the length
measures 16 inches, determine the width.
48. The base of a triangle is 3 feet shorter than its height. If the height
measures 5 feet, find the length of the base.
49. −3 and 12
58. The daily high temperature was recorded as 91°F and the low was
recorded as 63°F. What was the temperature range for the day?
59. A student earned 67 points on his lowest test and 87 points on his best.
Calculate his test score range.
60. On a busy day, a certain website may have 12,500 hits. On a slow day, it
may have as few as 750 hits. Calculate the range of the number of hits.
62. Demonstrate the associative property of addition with any three real
numbers.
ANSWERS
1: 6
3: −26
5: −38
7: −14
9: 1
11: −10
13: −1
15: 0
17: 1
19: −2
21: 29
23: 2
25: 55
27: 4
29: −11
31: −2
33: −36
35: 6
37: 3
39: 1
41: $80
43: $240
45: −4°
47: 14 inches
49: 15 units
51: 15 units
53: 13 units
55: 20 units
57: 265°F
59: 20 points
LEARNING OBJECTIVES
We begin with a review of what it means to multiply and divide signed numbers.
The result of multiplying real numbers is called the product26 and the result of
dividing is called the quotient27. Recall that multiplication is equivalent to adding:
Clearly, the product of two positive numbers is positive. Similarly, the product of a
positive number and negative number can be written as shown:
We see that the product of a positive number and a negative number is negative.
Next, explore the results of multiplying two negative numbers. Consider the
products in the following illustration and try to identify the pattern:
47
Chapter 1 Real Numbers and Their Operations
This shows that the product of two negative numbers is positive. To summarize,
The rules for division are the same because division can always be rewritten as
multiplication:
The rules for multiplication and division should not be confused with the fact that
the sum of two negative numbers is negative.
Example 1: Simplify:
a. (−3) + (−5)
b. (−3) (−5)
Solution: Here we add and multiply the same two negative numbers.
Answers: a. −8; b. 15
(a ⋅ b) ⋅ c = a ⋅ (b ⋅ c) .
and c,
Example 2: Simplify:
a. 5 ⋅ 0
b. 10 ⋅ 1
Solution:
b. Multiplying any real number by one results in the same real number.
Answers: a. 0; b. 10
Example 3: Simplify:
a. (3 ⋅ 7) ⋅ 2
b. 3 ⋅ (7 ⋅ 2)
Solution:
a.
b.
The value of each expression is 42. Changing the grouping of the numbers does not
change the result.
Answers: a. 42; b. 42
Answer: −30
Solution: Perform the division first; otherwise, the result will be incorrect.
Answer: 25
Notice that the order in which we multiply and divide does affect the final result.
Therefore, it is important to perform the operations of multiplication and division
as they appear from left to right.
Answer: 6
Video Solution
Within text-based applications, the symbol used for multiplication is the asterisk32
(*) and the symbol used for division is the forward slash (/).
The set of odd integers34 is the set of all integers that are not evenly divisible by 2.
A prime number35 is an integer greater than 1 that is divisible only by 1 and itself.
The smallest prime number is 2 and the rest are necessarily odd.
Any integer greater than 1 that is not prime is called a composite number36 and
can be written as a product of primes. When a composite number, such as 30, is
written as a product, 30 = 2 ⋅ 15, we say that 2 ⋅ 15 is a factorization37 of 30 and
that 2 and 15 are factors38. Note that factors divide the number evenly. We can
continue to write composite factors as products until only a product of primes
remains.
Since the prime factorization is unique, it does not matter how we choose to
initially factor the number because the end result is the same.
Some tests (called divisibility tests) useful for finding prime factors of composite
numbers follow:
Often we find the need to translate English sentences that include multiplication
and division terms to mathematical statements. Listed below are some key words
that translate to the given operation.
Example 9: What is the product of the first three positive even integers?
Solution: The first three positive even integers are {2, 4, 6} and the key word
“product” implies that we should multiply.
Answer: The product of the first three positive even integers is 48.
Example 10: Joe is able to drive 342 miles on 18 gallons of gasoline. How many miles
per gallon of gas is this?
Solution: The key word “per” indicates that we must divide the number of miles
driven by the number of gallons used:
In everyday life, we often wish to use a single value that typifies a set of values. One
way to do this is to use what is called the arithmetic mean40 or average41. To
calculate an average, divide the sum of the values in the set by the number of values
in that set.
Example 11: A student earns 75, 86, and 94 on his first three exams. What is the
student’s test average?
In this case, the dividend42 12 is evenly divided by the divisor43 6 to obtain the
40. A numerical value that typifies quotient, 2. It is true in general that if we multiply the divisor by the quotient we
a set of numbers. It is obtain the dividend. Now consider the case where the dividend is zero and the
calculated by adding up the divisor is nonzero:
numbers in the set and
dividing by the number of
elements in the set.
This demonstrates that zero divided by any nonzero real number must be zero. Now
consider a nonzero number divided by zero:
The zero-factor property of multiplication states that any real number times 0 is 0.
We conclude that there is no real number such that 0⋅? = 12 and thus, the
quotient is left undefined44. Try 12 ÷ 0 on a calculator. What does it say? For our
purposes, we will simply write “undefined.”
We are left to consider the case where the dividend and divisor are both zero.
KEY TAKEAWAYS
TOPIC EXERCISES
1. 5(−7)
2. −3(−8)
3. 2 (−4) (−9)
4. −3 ⋅2⋅5
6. 0 (−12) (−5)
9. −100 ÷ 25
10. 25 ÷ 5(−5)
12. −5 ⋅ 10 ÷ 2(−5)
14. 6*(−3)/(−9)
15. 20/(−5)*2
16. −50/2*5
25. Determine the product of the first three positive even integers.
26. Determine the product of the first three positive odd integers.
27. 105
28. 78
29. 138
30. 154
31. 165
32. 330
The distance traveled D is equal to the average rate r times the time traveled t at
that rate: D = rt. Determine the distance traveled given the rate and the time.
45. A student club ran a fund-raiser in the quad selling hot dogs. The
students sold 122 hot dog meals for $3.00 each. Their costs included $50.00
for the hot dogs and buns, $25.00 for individually wrapped packages of
chips, and $35.00 for the sodas. What was their profit?
46. A 230-pound man loses 4 pounds each week for 8 weeks. How much does
he weigh at the end of 8 weeks?
47. Mary found that she was able to drive 264 miles on 12 gallons of gas. How
many miles per gallon does her car get?
48. After filling his car with gasoline, Bill noted that his odometer reading
was 45,346 miles. After using his car for a week, he filled up his tank with 14
gallons of gas and noted that his odometer read 45,724 miles. In that week,
how many miles per gallon did Bill’s car get?
49. 0 ÷9
50. 15 ÷0
51. 4(−7) ÷0
55. −8 − 5 + (−13)
58. 49 ÷ 7 ÷ (−1)
59. 3 ⋅ 4 ÷ 12
60. 0 − (−8) − 12
61. −8 ⋅ 4(−3) ÷ 2
62. 0/(−3*8*5)
63. (−4*3)/(2*(−3))
64. −16/(−2*2)*3
65. −44/11*2
66. −5*3/(−15)
67. 4*3*2/6
69. During 5 consecutive winter days, the daily lows were −7°, −3°, 0°, −5°, and
−10°. Calculate the average low temperature.
70. On a very cold day the temperature was recorded every 4 hours with the
following results: −16°, −10°, 2°, 6°, −5°, and −13°. Determine the average
temperature.
71. A student earns 9, 8, 10, 7, and 6 points on the first 5 chemistry quizzes.
What is her quiz average?
72. A website tracked hits on its homepage over the Thanksgiving holiday.
The number of hits for each day from Thursday to Sunday was 12,250; 4,400;
7,750; and 10,200, respectively. What was the average number of hits per day
over the holiday period?
76. Discuss division involving 0. With examples, explain why the result is
sometimes 0 and why it is sometimes undefined.
78. Research and discuss other divisibility tests. Provide an example for each
test.
79. The arithmetic mean is one way to typify a set of values. Research other
methods used to typify a set of values.
ANSWERS
1: −35
3: 72
5: 0
7: 1
9: −4
11: −9
13: 15
15: −8
17: −33
19: −60
21: −12
23: 12
25: 48
27: 3 ⋅5⋅7
29: 2 ⋅ 3 ⋅ 23
31: 3 ⋅ 5 ⋅ 11
33: 60
35: 18
37: 5
41: 75 miles
45: $256.00
49: 0
51: Undefined
53: 0
55: −26
57: 1
59: 1
61: 48
63: 2
65: −8
67: 4
69: −5°
71: 8 points
1.4 Fractions
LEARNING OBJECTIVES
Reducing
The integer above the fraction bar is called the numerator48 and the integer below
is called the denominator49. The numerator is often called the “part” and the
denominator is often called the “whole.” Equivalent fractions50 are two equal
ratios expressed using different numerators and denominators. For example,
67
Chapter 1 Real Numbers and Their Operations
Making use of the multiplicative identity property and the fact that 25
25
= 1, we have
Dividing 25
25
and replacing this factor with a 1 is called canceling52. Together, these
basic steps for finding equivalent fractions define the process of reducing53. Since
factors divide their product evenly, we achieve the same result by dividing both the
numerator and denominator by 25 as follows:
54. Finding equivalent fractions We achieve the same result by dividing the numerator and denominator by the
where the numerator and the greatest common factor (GCF)55. The GCF is the largest number that divides both
denominator share no common the numerator and denominator evenly. One way to find the GCF of 50 and 100 is to
integer factor other than 1.
list all the factors of each and identify the largest number that appears in both lists.
55. The largest shared factor of Remember, each number is also a factor of itself.
any number of integers.
1.4 Fractions 68
Chapter 1 Real Numbers and Their Operations
Common factors are listed in bold, and we see that the greatest common factor is
50. We use the following notation to indicate the GCF of two numbers: GCF(50, 100) =
50. After determining the GCF, reduce by dividing both the numerator and the
denominator as follows:
Solution: Rewrite the numerator and denominator as a product of primes and then
cancel.
Alternatively, we achieve the same result if we divide both the numerator and
denominator by the GCF(105, 300). A quick way to find the GCF of the two numbers
requires us to first write each as a product of primes. The GCF is the product of all
the common prime factors.
1.4 Fractions 69
Chapter 1 Real Numbers and Their Operations
In this case, the common prime factors are 3 and 5 and the greatest common factor
of 105 and 300 is 15.
7
Answer: 20
1
Answer: 3
Video Solution
1.4 Fractions 70
Chapter 1 Real Numbers and Their Operations
Note that the denominator of the fractional part of the mixed number remains the
same as the denominator of the original fraction.
Answer: 4 3
5
1.4 Fractions 71
Chapter 1 Real Numbers and Their Operations
26
Answer: 7
21
Answer: 2
Video Solution
In this section, assume that a, b, c, and d are all nonzero integers. The product of
two fractions is the fraction formed by the product of the numerators and the
product of the denominators. In other words, to multiply fractions, multiply the
numerators and multiply the denominators:
1.4 Fractions 72
Chapter 1 Real Numbers and Their Operations
Answer: 10
21
Example 5: Multiply: 59 (− 4 ).
1
Solution: Recall that the product of a positive number and a negative number is
negative.
5
Answer: − 36
1.4 Fractions 73
Chapter 1 Real Numbers and Their Operations
Answer: 3 56
Two real numbers whose product is 1 are called reciprocals59. Therefore, ab and b
a
are reciprocals because ab ⋅ ba = ab
ab
= .1For example,
Because their product is 1, 23 and 32 are reciprocals. Some other reciprocals are listed
below:
58. Cancelling common factors in This definition is important because dividing fractions requires that you multiply
the numerator and the the dividend by the reciprocal of the divisor.
denominator of fractions
before multiplying.
1.4 Fractions 74
Chapter 1 Real Numbers and Their Operations
14
Answer: 15
You also need to be aware of other forms of notation that indicate division: / and —.
For example,
Or
1.4 Fractions 75
Chapter 1 Real Numbers and Their Operations
Note
Before multiplying, look for common factors to cancel; this eliminates the need to
reduce the end result.
5
2
Example 8: Divide: 7
.
4
Solution:
1.4 Fractions 76
Chapter 1 Real Numbers and Their Operations
10
Answer: 7
Example 9: Divide: 23 ÷ 6.
1.4 Fractions 77
Chapter 1 Real Numbers and Their Operations
1
Answer: 9
Also, note that we only cancel when working with multiplication. Rewrite any
division problem as a product before canceling.
Answer: 1 12
13
Video Solution
Negative fractions are indicated with the negative sign in front of the fraction bar,
in the numerator, or in the denominator. All such forms are equivalent and
interchangeable.
1.4 Fractions 78
Chapter 1 Real Numbers and Their Operations
3
Example 10: Subtract: 12
15
− 15 .
Solution: The two fractions have a common denominator 15. Therefore, subtract
the numerators and write the result over the common denominator:
3
Answer: 5
Most problems that you are likely to encounter will have unlike denominators62.
In this case, first find equivalent fractions with a common denominator before
adding or subtracting the numerators. One way to obtain equivalent fractions is to
61. A denominator that is shared divide the numerator and the denominator by the same number. We now review a
by more than one fraction. technique for finding equivalent fractions by multiplying the numerator and the
denominator by the same number. It should be clear that 5/5 is equal to 1 and that 1
62. Denominators of fractions that
are not the same. multiplied times any number is that number:
1.4 Fractions 79
Chapter 1 Real Numbers and Their Operations
We have equivalent fractions 12 = 105. Use this idea to find equivalent fractions with
a common denominator to add or subtract fractions. The steps are outlined in the
following example.
7 3
Example 11: Subtract: 15 − 10 .
Solution:
Common multiples are listed in bold, and the least common multiple is 30.
Step 2: Multiply the numerator and the denominator of each fraction by values that
result in equivalent fractions with the determined common denominator.
1.4 Fractions 80
Chapter 1 Real Numbers and Their Operations
Step 3: Add or subtract the numerators, write the result over the common
denominator and then reduce if possible.
1
Answer: 6
The least common multiple of the denominators is called the least common
denominator (LCD)64. Finding the LCD is often the difficult step. It is worth finding
because if any common multiple other than the least is used, then there will be
more steps involved when reducing.
5 1
Example 12: Add: 10 + 18 .
Solution: First, determine that the LCM(10, 18) is 90 and then find equivalent
fractions with 90 as the denominator.
1.4 Fractions 81
Chapter 1 Real Numbers and Their Operations
5
Answer: 9
2 5
Try this! Add: 30 + 21 .
32
Answer: 105
Video Solution
1.4 Fractions 82
Chapter 1 Real Numbers and Their Operations
Answer: 2 13
30
Answer: −1 37
Video Solution
1.4 Fractions 83
Chapter 1 Real Numbers and Their Operations
Example 14: How many 12 inch thick paperback books can be stacked to fit on a
shelf that is 1 12 feet in height?
Solution: First, determine the height of the shelf in inches. To do this, use the fact
that there are 12 inches in 1 foot and multiply as follows:
Next, determine how many notebooks will fit by dividing the height of the shelf by
the thickness of each book.
1.4 Fractions 84
Chapter 1 Real Numbers and Their Operations
KEY TAKEAWAYS
• Fractions are not unique; there are many ways to express the same ratio.
Find equivalent fractions by multiplying or dividing the numerator and
the denominator by the same real number.
• Equivalent fractions in lowest terms are generally preferred. It is a good
practice to always reduce.
• In algebra, improper fractions are generally preferred. However, in real-
life applications, mixed number equivalents are often preferred. We may
present answers as improper fractions unless the original question
contains mixed numbers, or it is an answer to a real-world or geometric
application.
• Multiplying fractions does not require a common denominator; multiply
the numerators and multiply the denominators to obtain the product. It
is a best practice to cancel any common factors in the numerator and
the denominator before multiplying.
• Reciprocals are rational numbers whose product is equal to 1. Given a
fraction ba , its reciprocal is ba .
• Divide fractions by multiplying the dividend by the reciprocal of the
divisor. In other words, multiply the numerator by the reciprocal of the
denominator.
• Rewrite any division problem as a product before canceling.
• Adding or subtracting fractions requires a common denominator. When
the denominators of any number of fractions are the same, simply add
or subtract the numerators and write the result over the common
denominator.
• Before adding or subtracting fractions, ensure that the denominators
are the same by finding equivalent fractions with a common
denominator. Multiply the numerator and the denominator of each
fraction by the appropriate value to find the equivalent fractions.
• Typically, it is best to convert all mixed numbers to improper fractions
before beginning the process of adding, subtracting, multiplying, or
dividing.
1.4 Fractions 85
Chapter 1 Real Numbers and Their Operations
TOPIC EXERCISES
5
1.
30
6
2.
24
30
3.
70
18
4.
27
44
5.
84
54
6.
90
135
7.
30
105
8.
300
18
9.
6
256
10.
16
126
11.
45
52
12.
234
54
13.
162
2000
14.
3000
1.4 Fractions 86
Chapter 1 Real Numbers and Their Operations
270
15.
360
3
16. 4
4
1
17. 2
2
7
18. 5
15
1
19. 1
2
5
20. 3
8
3
21. 1
4
1
22. −2
2
3
23. −1
4
15
24.
2
9
25.
2
40
26.
13
103
27.
25
73
28.
10
52
29. − 7
1.4 Fractions 87
Chapter 1 Real Numbers and Their Operations
59
30. −
6
31. 2
3
⋅ 5
7
32. 1
5
⋅ 4
8
33. 1
2
⋅ 1
3
34. 3
4
⋅ 20
9
35. 5
7
⋅ 49
10
36. 2
3
⋅ 9
12
37. 6
14
⋅ 21
12
38. 44
15
⋅ 15
11
39. 3 3
4
⋅2 1
3
40. 2 7
10
⋅5 5
6
(− 2 )
3 5
41.
11
(5)
4 9
42. −
5
43. (−
5) (− )
9 3
10
(− )
6 14
44. 7
3
45. (− ) (− 8 )
9 4
12
1.4 Fractions 88
Chapter 1 Real Numbers and Their Operations
(− )
3 4
46. −
8 15
1 1 1
47. 7 ⋅ 2
⋅ 3
3 15 7
48.
5
⋅ 21
⋅ 27
2 1 4
49.
5
⋅3 8
⋅ 5
4 2 5
50. 2 ⋅ ⋅2
9 5 11
1
51.
2
8
52.
5
2
53. −
3
4
54. −
3
55. 10
56. −4
1
57. 2
3
5
58. 1
8
1 2
59.
2
÷ 3
5 1
60.
9
÷ 3
61.
5
8
÷ (− 45 )
1.4 Fractions 89
Chapter 1 Real Numbers and Their Operations
62. (−
5)
2 15
÷ 3
− 67
63.
− 67
− 12
64. 1
4
− 10
3
65. 5
− 20
2
3
66. 9
2
30
50
67. 5
3
1
2
68.
2
5
69. 2
5
−6
70. 5
4
71. 2 1
2
÷ 5
3
72. 4 2
3
÷3 1
2
73. 5 ÷2 3
5
74. 4 3
5
÷ 23
1.4 Fractions 90
Chapter 1 Real Numbers and Their Operations
75. 17
20
− 5
20
76. 4
9
− 13
9
77. 3
5
+ 1
5
78. 11
15
+ 9
15
79. 5
7
−2 1
7
80. 5 1
8
−1 1
8
81. 1
2
+ 1
3
82. 1
5
− 1
4
83. 3
4
− 5
2
84. 3
8
+ 7
16
85. 7
15
− 3
10
86. 3
10
+ 2
14
87. 2
30
+ 5
21
88. 3
18
− 1
24
89. 5 1
2
+2 1
3
90. 1 3
4
+2 1
10
1 1 1
91.
2
+ 3
+ 6
2 3 2
92.
3
+ 5
− 9
7 3 2
93.
3
− 2
+ 15
1.4 Fractions 91
Chapter 1 Real Numbers and Their Operations
9 3 3
94.
4
− 2
+ 8
1 2 1
95. 1 +2 −1
3 5 15
2 1 1
96.
3
−4 2
+3 6
6 3
97. 1 − +
16 18
1 1
98. 3 − −
21 15
3 7 1
99.
14
⋅ 3
÷ 8
100.
1
2
⋅ (− 45 ) ÷ 14
15
1 3 1
101.
2
÷ 4
⋅ 5
5 5 5
102. − ÷ ⋅
9 3 2
5 9 3
103.
12
− 21
+ 9
3 5 1
104. − − +
10 12 20
4 1
105.
5
÷4⋅ 2
5 2
106.
3
÷ 15 ⋅ 3
3 4
107. What is the product of 16
and 9
?
5 25
109. What is the quotient of 9
and 3
?
1.4 Fractions 92
Chapter 1 Real Numbers and Their Operations
1 9 2
111. Subtract 6
from the sum of 2
and 3
.
1 3 6
112. Subtract 4
from the sum of 4
and 5
.
113. What is the total width when 3 boards, each with a width of 2 5
8
inches,
are glued together?
115. A board that is 5 14 feet long is to be cut into 7 pieces of equal length.
What is length of each piece?
3
116. How many 4
inch thick notebooks can be stacked into a box that is 2
feet high?
1
119. Each lap around the track measures 4
mile. How many laps are
required to complete a 2 12 mile run?
122. Explain the difference between the LCM and the GCF. Give an example.
1.4 Fractions 93
Chapter 1 Real Numbers and Their Operations
7 1
125. Explain how to determine which fraction is larger, 16
or 2
.
1.4 Fractions 94
Chapter 1 Real Numbers and Their Operations
ANSWERS
1: 1/6
3: 3/7
5: 11/21
7: 9/2
9: 3
11: 14/5
13: 1/3
15: 3/4
17: 5/2
19: 3/2
21: 7/4
23: −7/4
1
25: 4
2
3
27: 4
25
3
29: −7 7
31: 10/21
33: 1/6
35: 7/2
37: 3/4
1.4 Fractions 95
Chapter 1 Real Numbers and Their Operations
39: 8 3
4
41: −15/22
43: 27/50
45: 3/8
47: 1/42
49: 1
51: 2
53: −3/2
55: 1/10
57: 3/7
59: 3/4
61: −25/32
63: 1
65: 40/3
67: 9/25
69: 25/2
71: 1 1
2
73: 1 12
13
75: 3/5
77: 4/5
1.4 Fractions 96
Chapter 1 Real Numbers and Their Operations
79: −1 3
7
81: 5/6
83: −7/4
85: 1/6
87: 32/105
5
89: 7
6
91: 1
93: 29/30
95: 2 2
3
97: 19/24
99: 4
101: 2/15
103: 9/28
105: 1/10
107: 1/12
109: 1/15
111: 5
113: 7 7
8
inches
3
115: 4
feet
117: 11 students
1.4 Fractions 97
Chapter 1 Real Numbers and Their Operations
119: 10 laps
1.4 Fractions 98
Chapter 1 Real Numbers and Their Operations
LEARNING OBJECTIVES
Decimals
In this section, we provide a brief review of the decimal system. A real number in
decimal form, a decimal65 consists of a decimal point, digits (0 through 9) to the left
of the decimal point representing the whole number part, and digits to the right of
the decimal point representing the fractional part. The digits represent powers of
10 as shown in the set {…, 1,000, 100, 10, 1, 1/10, 1/100, 1/1,000, …} according to the
following diagram:
For example, the decimal 538.3 can be written in the following expanded form:
65. A real number expressed using After simplifying, we obtain the mixed number 538 103 . Use this process to convert
the decimal system. decimals to mixed numbers.
99
Chapter 1 Real Numbers and Their Operations
Solution: In this example, 32 is the whole part and the decimal ends in the
hundredths place. Therefore, the fractional part will be 15/100, and we can write
Answer: 32.15 = 32 3
20
Answer: 3
4
= 0.75
If the division never ends, then use a bar over the repeating digit (or block of digits)
to indicate a repeating decimal.
Solution: Use long division to convert the fractional part to a decimal and then add
the whole part.
At this point, we can see that the long division will continue to repeat. When this is
the case, use a bar over the repeating digit to indicate that it continues forever:
Then write
⎯⎯
Answer: 2 5
6
= 2.83
To add or subtract decimals, align them vertically with the decimal point and add
corresponding place values. Recall that sometimes you need to borrow from or
carry over to the adjoining column (regrouping).
Solution: Note that trailing zeros to the right of the decimal point do not change
the value of the decimal, 23.25 = 23.250. In this case, you need to borrow from the
tenths place (regroup) to subtract the digits in the hundredths place.
Answer: 31.078
Multiply decimals the same way you multiply whole numbers. The number of
decimal places in the product will be the sum of the decimal places found in each of
the factors.
Answer: 39.664
When dividing decimals, move the decimal points of both the dividend and the
divisor so that the divisor is a whole number. Remember to move the decimal the
same number of places for both the dividend and divisor.
Solution: Move the decimal point to turn the divisor into a whole number: 624.
Move the decimal points of both the divisor and dividend two places to the right.
Next, divide.
Answer: 5.34
1. If the digit to the right of the specified place is 4 or less, then leave the
specified digit unchanged and drop all subsequent digits.
2. If the digit to the right of the specified place is 5 or greater, then
increase the value of the digit in the specified place by 1 and drop all
subsequent digits.
Recall that decimals with trailing zeros to the right of the decimal point can be
dropped. For example, round 5.635457 to the nearest thousandth:
After rounding off, be sure to use the appropriate notation ( ≈ ) to indicate that the
number is now an approximation. When working with US currency, we typically
round off to two decimal places, or the nearest hundredth.
a. 1/3 of $10.25.
b. 1/4 of $10.25.
Solution: In this context, the key word “of” indicates that we should multiply.
Definition of Percent
For example,
Each sector is proportional to the size of the part out of the whole. The sum of the
percentages presented in a pie chart must be 100%. To work with percentages
effectively, you have to know how to convert percents to decimals or fractions and
back again.
Percents to Decimals
Solution: Treat 152% as 152.0% and move the decimal two places to the left.
Answer: 1.52
Answer: 0.0275
Answer: 2.15
Video Solution
Solution: First, convert the decimal to a fraction of 100 and apply the definition.
You can achieve the same result by moving the decimal two places to the right and
adding a percent sign.
Answer: 23%.
You can achieve the same result by moving the decimal two places to the right and
adding a percent sign.
Answer: 235%
Solution:
Answer: 520%
Sometimes we can use the definition of percent and find an equivalent fraction with
a denominator of 100.
13
Example 13: Convert 25
to a percent.
Solution: Notice that the denominator 25 is a factor of 100. Use the definition of
percent by finding an equivalent fraction with 100 in the denominator.
Answer: 52%
This is a very specialized technique because 100 may not be a multiple of the
denominator.
Solution: Notice that the denominator 3 is not a factor of 100. In this case, it is best
to multiply by 1 in the form of 100%.
Answer: 33 1
3
%
Answer: 66 2
3
%
Video Solution
When converting percents to fractions, apply the definition of percent and then
reduce.
Solution:
7
Answer: 25
Applying the definition of percent is equivalent to removing the percent sign and
1
multiplying by 100 .
Solution: First, convert to an improper fraction and then apply the definition of
percent.
2
Answer: 3
1
Answer: 31
Video Solution
Example 17: Using the given pie chart, calculate the total number of students that
were 21 years old or younger if the total US community college enrollment in 2009
was 11.7 million.
Solution: From the pie chart we can determine that 47% of the total 11.7 million
students were 21 years old or younger.
Convert 47% to a decimal and multiply as indicated by the key word “of.”
KEY TAKEAWAYS
TOPIC EXERCISES
Part A: Decimals
1. 45.8
2. 15.4
3. 1.82
4. 2.55
5. 4.72
6. 3.14
Write as a decimal.
7. 2 4
5
8. 5 1
5
9. 3 1
8
10. 1 3
20
3
11. 8
5
12. 8
13. 1 1
3
14. 2 1
6
1
28. Find 5
of $33.26.
2
29. Find 3
of $15.25.
3
30. Find 4
of $15.50.
31. A gymnast scores 8.8 on the vault, 9.3 on the uneven bars, 9.1 on the
balance beam, and 9.8 on the floor exercise. What is her overall average?
32. To calculate a batting average, divide the player’s number of hits by the
total number of at-bats and round off the result to three decimal places. If a
player has 62 hits in 195 at-bats, then what is his batting average?
33. 43%
34. 25%
35. 33%
36. 100%
37. 150%
38. 215%
1
39.
2
%
3
40. 2 %
4
1
41. 1 %
2
2
42. 3 %
3
43. 0.025%
44. 0.0001%
45. 1.75%
46. 20.34%
47. 0%
48. 1%
49. 3.05%
50. 5.003%
58. What is 9 1
2
% of $1,200?
59. If the bill at a restaurant comes to $32.50, what is the amount of a 15%
tip?
60. Calculate the total cost, including a 20% tip, of a meal totaling $37.50.
61. If an item costs $45.25, then what is the total after adding 8.25% for tax?
62. If an item costs $36.95, then what is the total after adding 9¼% tax?
63. A retail outlet is offering 15% off the original $29.99 price of branded
sweaters. What is the price after the discount?
65. If a local mayor of a town with a population of 40,000 people enjoys a 72%
favorable rating in the polls, then how many people view the mayor
unfavorably?
66. If a person earning $3,200 per month spends 32% of his monthly income
on housing, then how much does he spend on housing each month?
67. 0.67
68. 0.98
69. 1.30
70. 2.25
57
71.
100
99
72.
100
1
73.
5
2
74.
3
25
75.
8
1
76. 3
4
17
77.
50
1
78. 7
79. 0.0023
80. 0.000005
81. 20
82. 100
83. 20%
84. 80%
85. 57%
86. 97%
1
87. 5 %
2
2
88. 1 %
3
89. 75%
90. 32%
91. 400%
92. 230%
93. 100%
1
94.
8
%
5
95.
12
%
5
96. 5 7 %
1
97. 33 %
3
7
98. 3 %
31
99. 0.7%
100. 0.05%
101. 1.2%
102. 12.5%
The course grade weighting for a traditional mathematics course with 1,200 total
points is shown in the pie chart below. Use the chart to answer the following
questions.
105. How many points will each of the four regular exams be worth?
A website had 12,000 unique users in the fall of 2009. Answer the questions based on
the pie chart below depicting total Web browser usage.
108. How many users used a browser other than Internet Explorer?
The 2009 employment status of 11.7 million full-time community college students is
given in the following pie chart. Use the chart to answer the following questions.
Round off each answer to the nearest hundredth.
Source: American
Association of
Community Colleges.
113. How many full-time students were unemployed or employed part time?
114. How many full-time students also worked part time or full time?
The pie chart below depicts all US households categorized by income. The total
number of households in 2007 was about 111,600,000. Use the chart to answer the
following questions.
Source: US Census
Bureau.
121. Research and discuss simple interest and how it is calculated. Make up
an example and share the solution.
ANSWERS
1: 45 4
5
3: 1 41
50
5: 4 18
25
7: 2.8
9: 3.125
11: 0.375
⎯⎯
13: 1.3
15: 8.94
17: 53.451
19: 75.638
21: 0.6765
23: 7.23
25: 40.6
27: $3.33
29: $10.17
31: 9.25
33: 0.43
35: 0.33
37: 1.5
39: 0.005
41: 0.015
43: 0.00025
45: 0.0175
47: 0
49: 0.0305
51: 0.005
53: 0
55: 150
57: $235.20
59: $4.88
61: $48.98
63: $25.49
67: 67%
69: 130%
71: 57%
73: 20%
75: 312.5%
77: 34%
79: 0.23%
81: 2,000%
1
83:
5
57
85:
100
11
87:
200
3
89:
4
91: 4
93: 1
1
95:
240
1
97:
3
7
99:
1000
3
101:
250
LEARNING OBJECTIVES
If a number is repeated as a factor numerous times, then we can write the product
in a more compact form using exponential notation69. For example,
The base70 is the factor, and the positive integer exponent71 indicates the number
of times the base is repeated as a factor. In the above example, the base is 5 and the
exponent is 4. In general, if a is the base that is repeated as a factor n times, then
69. The compact notation When the exponent is 2, we call the result a square72. For example,
ax 2 + bx + c = 0. used
when a factor is repeated
multiple times.
127
Chapter 1 Real Numbers and Their Operations
It is important to study the difference between the ways the last two examples are
calculated. In the example (−7)2 , the base is −7 as indicated by the parentheses. In
the example −52 , the base is 5, not −5, so only the 5 is squared and the result
remains negative. To illustrate this, write
This subtle distinction is very important because it determines the sign of the
result.
The textual notation for exponents is usually denoted using the caret73 (^) symbol
as follows:
Answer: 144
Video Solution
The notation 33 can be read two ways: “three cubed” or “3 raised to the third
power.” As before, the base can be any real number.
Note that the result of cubing a negative number is negative. The cube of an integer
is called a perfect cube76. The ability to recognize perfect cubes is useful in our
study of algebra. The cubes of the integers from 1 to 10 should be memorized. A
partial list of perfect cubes follows:
Answer: −8
If the exponent is greater than 3, then the notation an is read “a raised to the nth
power.”
Notice that the result of a negative base with an even exponent is positive. The
result of a negative base with an odd exponent is negative. These facts are often
confused when negative numbers are involved. Study the following four examples
carefully:
The parentheses indicate that the negative number is to be used as the base.
Example 1: Calculate:
a. (− 13 )
3
b. (− 13 )
4
Answers: a. − 271 ; b. 1
81
Video Solution
Think of finding the square root77 of a number as the inverse of squaring a number.
In other words, to determine the square root of 25 the question is, “What number
squared equals 25?” Actually, there are two answers to this question, 5 and −5.
When asked for the square root of a number, we implicitly mean the principal
(nonnegative) square root78. Therefore we have,
⎯⎯⎯⎯
As an example, √25 = 5, which is read “square root of 25 equals 5.” The symbol √
is called the radical sign79 and 25 is called the radicand80. The alternative textual
notation for square roots follows:
⎯⎯⎯⎯⎯⎯⎯⎯⎯⎯⎯
Example 2: Simplify: √10,000.
Example 3: Simplify: √ 19 .
⎯⎯⎯
1
Answer: 3
Given a and b as positive real numbers, use the following property to simplify
square roots whose radicands are not squares:
The idea is to identify the largest square factor of the radicand and then apply the
⎯⎯
property shown above. As an example, to simplify √8 notice that 8 is not a perfect
square. However, 8 = 4 ⋅ 2 and thus has a perfect square factor other than 1. Apply
the property as follows:
⎯⎯
Here 2√2 is a simplified irrational number. You are often asked to find an
approximate answer rounded off to a certain decimal place. In that case, use a
calculator to find the decimal approximation using either the original problem or
the simplified equivalent.
On a calculator, try 2.83^2. What do you expect? Why is the answer not what you
would expect?
⎯⎯⎯⎯⎯
It is important to mention that the radicand must be positive. For example, √−9 is
undefined since there is no real number that when squared is negative. Try taking
the square root of a negative number on your calculator. What does it say? Note:
taking the square root of a negative number is defined later in the course.
⎯⎯⎯⎯
Answer: √75 ≈ 8.66
⎯⎯⎯⎯ ⎯⎯
As a check, calculate √75 and 5√3 on a calculator and verify that the both results
are approximately 8.66.
⎯⎯⎯⎯⎯⎯
Example 5: Simplify: √180.
Solution:
Since the question did not ask for an approximate answer, we present the exact
answer.
⎯⎯
Answer: 6√5
⎯⎯⎯⎯⎯⎯
Example 5: Simplify: −5√162.
Solution:
⎯⎯
Answer: −45√2
Try this! Simplify and give an approximate answer rounded to the nearest
⎯⎯⎯⎯⎯⎯
hundredth: √128.
⎯⎯
Answer: 8√2 ≈ 11.31
Video Solution
84. Given any right triangle with Example 6: If the two legs of a right triangle measure 3 units and 4 units, then find
legs measuring a and b units the length of the hypotenuse.
and hypotenuse measuring c
units, then a2 + b2 = c2.
Solution: Given the lengths of the legs of a right triangle, use the formula
⎯⎯⎯⎯⎯⎯⎯⎯⎯⎯⎯
c = √a2 + b2 to find the length of the hypotenuse.
Answer: c = 5 units
When finding the hypotenuse of a right triangle using the Pythagorean theorem,
the radicand is not always a perfect square.
Example 7: If the two legs of a right triangle measure 2 units and 6 units, find the
length of the hypotenuse.
Solution:
⎯⎯⎯⎯
Answer: c = 2√10 units
KEY TAKEAWAYS
TOPIC EXERCISES
Simplify.
2
1. 10
2
2. 12
2
3. (−9)
2
4. −12
5. 11 ^2
6. (−20) ^2
2
7. 0
2
8. 1
2
9. −(−8)
2
10. −(13)
11. (
2)
1 2
12. (−
3)
2 2
13. 0.5^2
14. 1.25^2
15. (−2.6)^2
16. −(−5.1)^2
17. (2
3)
1 2
18. (5
4)
3 2
19. Determine the area of a square given that a side measures 5 inches.
20. Determine the area of a square given that a side measures 2.3 feet.
{0, , 2}.
23. List the squares of all the rational numbers in the set
1 2 4 5
3
, 3
, 1, 3
, 3
{0, 2}
24. List the squares of all the rational numbers in the set
1 3 5
2
, 1, 2
, 2, .
Simplify.
3
25. 5
6
26. 2
4
27. (−1)
3
28. (−3)
4
29. −1
4
30. (−2)
3
31. −7
3
32. (−7)
3
33. −(−3)
4
34. −(−10)
20
35. (−1)
21
36. (−1)
37. (−6) ^3
38. −3 ^4
39. 1 ^ 100
40. 0 ^ 100
41. −(
2)
1 3
42. (
2)
1 6
43. (
2)
5 3
44. (−
4)
3 4
{− 3}
47. List all the cubes of the rational numbers in the set
2 1 1 2
3
, − 3
, 0, 3
, .
{− 7}
48. List all the cubes of the rational numbers in the set
3 1 1 3
7
, − 7
, 0, 7
, .
⎯⎯⎯⎯⎯⎯
49. √ 121
⎯⎯⎯⎯
50. √ 81
⎯⎯⎯⎯⎯⎯
51. √ 100
⎯⎯⎯⎯⎯⎯
52. √ 169
⎯⎯⎯⎯
53. −√ 25
⎯⎯⎯⎯⎯⎯
54. −√ 144
⎯⎯⎯⎯
55. √ 12
⎯⎯⎯⎯
56. √ 27
⎯⎯⎯⎯
57. √ 45
⎯⎯⎯⎯
58. √ 50
⎯⎯⎯⎯
59. √ 98
⎯⎯⎯⎯⎯⎯⎯⎯
60. √ 2000
61. √
⎯1⎯⎯
4
62. √
⎯⎯⎯⎯
9
⎯
16
63. √
⎯5⎯⎯
9
64. √
⎯⎯⎯⎯
8
⎯
36
⎯⎯⎯⎯⎯⎯⎯
65. √ 0.64
⎯⎯⎯⎯⎯⎯⎯
66. √ 0.81
⎯⎯⎯⎯⎯2⎯
67. √ 30
⎯⎯⎯⎯⎯2⎯
68. √ 15
⎯⎯⎯⎯⎯⎯⎯⎯2⎯
69. √ (−2)
70. √ (−5)
⎯⎯⎯⎯⎯⎯⎯⎯⎯2⎯
⎯⎯⎯⎯⎯
71. √ −9
⎯⎯⎯⎯⎯⎯⎯
72. √ −16
⎯⎯⎯⎯
73. 3√ 16
⎯⎯⎯⎯
74. 5√ 18
⎯⎯⎯⎯
75. −2√ 36
⎯⎯⎯⎯
76. −3√ 32
⎯⎯⎯⎯⎯⎯
77. 6√ 200
⎯⎯⎯⎯
78. 10√ 27
⎯⎯
79. √ 2
⎯⎯
80. √ 3
⎯⎯⎯⎯
81. √ 10
⎯⎯⎯⎯
82. √ 15
⎯⎯
83. 2√ 3
⎯⎯
84. 5√ 2
⎯⎯
85. −6√ 5
⎯⎯
86. −4√ 6
87. sqrt(79)
88. sqrt(54)
89. −sqrt(162)
90. −sqrt(86)
91. If the two legs of a right triangle measure 6 units and 8 units, then find
the length of the hypotenuse.
92. If the two legs of a right triangle measure 5 units and 12 units, then find
the length of the hypotenuse.
93. If the two legs of a right triangle measure 9 units and 12 units, then find
the length of the hypotenuse.
3
94. If the two legs of a right triangle measure 2
units and 2 units, then find
the length of the hypotenuse.
95. If the two legs of a right triangle both measure 1 unit, then find the
length of the hypotenuse.
96. If the two legs of a right triangle measure 1 unit and 5 units, then find
the length of the hypotenuse.
97. If the two legs of a right triangle measure 2 units and 4 units, then find
the length of the hypotenuse.
98. If the two legs of a right triangle measure 3 units and 9 units, then find
the length of the hypotenuse.
99. Why is the result of an exponent of 2 called a square? Why is the result of
an exponent of 3 called a cube?
ANSWERS
1: 100
3: 81
5: 121
7: 0
9: −64
11: 1/4
13: .25
15: 6.76
4
17: 5
9
21: {0, 1, 4, 9, 16, 25, 36, 49, 64, 81, 100, 121, 144, 169, 196, 225}
25: 125
27: 1
29: −1
31: −343
33: 27
35: 1
37: −216
39: 1
1
41: −
8
125
43:
8
47: {− }
8 1 1 8
27
, − 27
, 0, 27
, 27
49: 11
51: 10
53: −5
⎯⎯
55: 2√ 3
⎯⎯
57: 3√ 5
⎯⎯
59: 7√ 2
1
61:
2
√5
63:
3
65: 0.8
67: 30
69: 2
73: 12
75: −12
⎯⎯
77: 60√ 2
79: 1.41
81: 3.16
83: 3.46
85: −13.42
87: 8.89
89: −12.73
91: 10 units
93: 15 units
⎯⎯
95: √ 2 units
⎯⎯
97: 2√ 5 units
LEARNING OBJECTIVES
Grouping Symbols
All of the above grouping symbols, as well as absolute value, have the same order of
precedence. Perform operations inside the innermost grouping symbol or absolute
value first.
Solution: Perform the operations within the parentheses first. In this case, first
subtract 12 from 4.
149
Chapter 1 Real Numbers and Their Operations
Answer: 13
Solution:
Answer: −24
5−||4−(−3)||
|−3|−(5−7)
Example 3: Simplify: .
Solution: The fraction bar groups the numerator and denominator. They should be
simplified separately.
Answer: − 25
Answer: 15
Video Solution
Caution: Note that multiplication and division operations must be worked from left to
right.
Solution: First, evaluate 52 and then perform multiplication and division as they
appear from left to right.
Answer: 24
Because multiplication and division operations should be worked from left to right,
it is sometimes correct to perform division before multiplication.
Answer: 19
Answer: 21
Answer: 59
Answer: −8
Example 9: Simplify: (− 23 ) ÷ [ 53 − (− 12 ) ].
2 3
Solution:
32
Answer: 129
We are less likely to make a mistake if we work one operation at a time. Some
problems may involve an absolute value, in which case we assign it the same order
of precedence as parentheses.
Solution: We begin by evaluating the absolute value and then the exponent
(−2)4 = (−2) (−2) (−2) (−2) = +16.
Answer: −10
Answer: 13
Video Solution
KEY TAKEAWAYS
TOPIC EXERCISES
Simplify.
1. −7 −3⋅5
2. 3 +2⋅3
3. −3(2) − 62
2
4. 2(−3) + 5(−4)
5. 6/3 *2
6. 6/(3 * 2)
1 3 2
7. − − ⋅
2 5 3
5 1 5
8.
8
÷ 2
− 6
2
9. 3.2 − 6.9 ÷ 2.3
11. 2 + 3(−2) − 7
12. 8 ÷2−3⋅2
13. 3 + 6 2 ÷ 12
14. 5 − 4 2 ÷ (−8)
15. −9 − 3 ⋅ 2 ÷ 3(−2)
16. −2 − 3 2 + (−2) 2
17. 12 ÷ 6 ⋅ 2 − 22
18. 4 ⋅ 3 ÷ 12 ⋅ 2 − (−2) 2
2
19. (−5) − 2(5)2 ÷ 10
20. −3(4 − 7) + 2
21. (−2 + 7) 2 − 10 2
22. 10 − 7(3 + 2) + 7 2
23. −7 − 3 (4 − 2 ⋅ 8)
24. 5 − 3 [6 − (2 + 7)]
[2 (7 − 5) ÷ 4 ⋅ (−2) + (−3) ]
3
26. −3
27. −7
2
− [−20 − (−3) 2 ] − (−10)
2
30. −7.3 + (−9.3) 2 − 37.8 ÷ 1.8
31. 2 − 7 ( 3 2 − 3 + 4 ⋅ 3)
32. (
2)
1 2
− (− 23 )
2
33. (
2)
1 3
+ (−2) 3
34. (−
3)
1 2
− (− 23 )
3
1 1 1
35.
3
− 2
⋅ 5
5 3 14
36.
8
÷ 2
⋅ 15
− ( 12 )
2 3
37. 5 ⋅ 15
(5 − )
5 3 4
38.
17 35
39.
3
16
÷ ( 12
5
− 1
2
+ 23 ) ⋅ 4
40. (
3)
2 2
− ( 12 )
2
[ 4 ⋅ (−4) − 2]
1 3 2 2
41. 2
⋅ [( 23 ) − ( 12 ) ] ÷ (−2) 2
2 2
42. 6
(−5) +3
22
43.
−4 2 +2⋅7
(−3.2−3.3)(8.7−4.7)
44.
(−4.7+3.9+2.1)
2−[3−(5−7) ]
2
3(6−3 2 )
45.
2+3⋅6−4⋅3
46.
2 2 −3 2
(2+7)⋅2−2 3
47.
10+9 2 +3 3
(−1−3)2 −15
−3⋅(−7+2 2 )−5
48.
50. 4 + 3 * ((−3) ^ 3 + 5 ^ 2) / 6 − 2 ^ 2
52. Joe bought four 8-foot 2-by-4 boards for $24.00. How much did he spend
per linear foot?
53. Margaret bought two cases of soda at the local discount store for $23.52.
If each case contained 24 bottles, how much did she spend per bottle?
54. Billy earns $12.00 per hour and “time and a half” for every hour he works
over 40 hours a week. What is his pay for 47 hours of work this week?
56. Mark and Janet carpooled home from college for the Thanksgiving
holiday. They shared the driving, but Mark drove twice as far as Janet. If
Janet drove 135 miles, then how many miles was the entire trip?
Simplify.
57. 3 + 2 |−5||
58. 9 − 4 |−3|
63. −(|−8| − 5)
2
65. −4 + 2 ||2 2 − 3 2 ||
67. − |(−5)
| 2
+ 4 2 ÷ 8||
| |
69. −2 [7 − (4 + |−7|)]
70. 3 − 7 |−2 − 3| + 4 3
72. (−4)
2
− ||−5 + (−2) 3 || − 3 2
− || 12 − (− 43 ) ||
2 2
73.
3 | |
74. −30 | ÷ 15 ||
10 1
| 3
− 2
75. (−4)
3
− (2 − |−4|) ÷ ||−3 2 + 7||
1 1
77. and −
2 4
3 2
78. − and −
4 3
5 3
79. − and −
8 4
7 3
80. − and 7
5
1 1
83. 3 and −2
5 3
3
84. 5 and 0
4
87. Discuss the importance of proper grouping and give some examples.
88. Experiment with the order of operations on a calculator and share your
results.
ANSWERS
1: −22
3: −42
5: 4
9
7: −
10
9: 7.24
11: −11
13: 6
15: −5
17: 0
19: 20
21: −75
23: 29
25: −33
27: −10
29: 67.22
31: −124
63
33: −
8
7
35:
30
13
37:
24
9
39: 7
41: 50
43: −17
1
45: −
3
5
47:
59
49: −1
51: $32.50
53: $0.49
55: 20 marbles
57: 13
59: −8
61: 18
63: −9
65: 6
67: −27
69: 8
71: 1
11
73: −
18
75: −63
3
77: unit
4
1
79: unit
8
8
83: 5 units
15
167
Chapter 1 Real Numbers and Their Operations
REVIEW EXERCISES
Choose an appropriate scale and graph the following sets of real numbers on a
number line.
1. {−4, 0, 4}
3. {−12, −3, 9}
4. {−10, 8, 10}
5. 0 ___ −9
6. −75 ___ −5
8. − (−23) ___ 23
11. || ? || =2
12. || ? || =1
13. || ? || = −7
14. || ? || =0
Simplify.
19. 12 + (−7)
20. 20 + (−32)
22. −8 − (−8)
25. (7 − 10) − 3
26. (−19 + 6) − 2
27. −8 and 14
33. An airplane flying at 22,000 feet descended 8,500 feet and then ascended
5,000 feet. What is the new altitude of the airplane?
34. The width of a rectangle is 5 inches less than its length. If the length
measures 22 inches, then determine the width.
Simplify.
35. 10 ÷5⋅2
36. 36 ÷6⋅2
39. −8 (−5) ÷0
43. James was able to drive 234 miles on 9 gallons of gasoline. How many
miles per gallon did he get?
44. If a bus travels at an average speed of 54 miles per hour for 3 hours, then
how far does the bus travel?
Fractions
180
45.
300
252
46.
324
23
47. Convert to a mixed number: .
8
Simplify.
(− 7 )
3 2
49.
5
(− 3 )
5 1
50. −
8
3 6
51. − ÷ 7
4
4 28
52.
15
÷ 3
53. 4 4
5
÷6
54. 5 ÷8 1
3
5 15
55.
4
÷ 2
⋅6
5 3 5
56.
24
÷ 2
÷ 12
1 1
57.
12
− 4
5 3
58.
6
− 14
3 2 1
59.
4
+ 3
− 12
3 5 1
60.
10
+ 12
− 6
61. Subtract 2
3
from the sum of − 1
2
and 2
9
.
5 1 7
62. Subtract 6
from the difference of 3
and 2
.
Convert to a decimal.
71. 7.2%
72. 5 3
8
%
73. 147%
74. 27 1
2
%
Convert to a percent.
75. 0.055
76. 1.75
9
77. 10
5
78. 6
79. Mary purchased 3 boxes of t-shirts for a total of $126. If each box
contains 24 t-shirts, then what is the cost of each t-shirt?
80. A retail outlet is offering 12% off the original $39.99 price of tennis shoes.
What is the price after the discount?
81. If an item costs $129.99, then what is the total after adding 7 1
4
% sales
tax?
Simplify.
2
83. 8
84. (−5)
2
2
85. −4
2
86. −(−3)
87. (
9)
2 2
88. (1 )
2
2
3
3
89. 3
3
90. (−4)
91. (
5)
2 3
92. (−
6)
1 3
4
93. −(−2)
5
94. −(−1)
⎯⎯⎯⎯
95. √ 49
⎯⎯⎯⎯⎯⎯
96. √ 225
⎯⎯⎯⎯
97. 2√ 25
⎯⎯⎯⎯⎯⎯
98. −√ 121
⎯⎯⎯⎯
99. 3√ 50
⎯⎯⎯⎯
100. −4√ 12
101. √
⎯4⎯⎯
9
102. √
⎯⎯⎯⎯
8
⎯
25
105. Determine the length of the diagonal of a square with sides measuring 3
centimeters.
Order of Operations
Simplify.
107. −5 (2) − 72
108. 1 − 4 2 + 2(−3) 2
+ 3(6 − 2 ⋅ 4)
3
109. 2
111. −2
3
+ 6 (3 2 − 4) + (−3) 2
2
112. 5 − 40 ÷ 5(−2) 2 − (−4)
[ 9 (−3) − 4]
3 2 2 2
113.
4
114. (
2)
1 2 3 9 1
− 4
÷ 16
− 3
2−3(6−3 2 )
2
115.
4⋅5−5 2
(2⋅8−6 ) −10
2 2 2
7 3 −(2(−5) −7)
116. 3
121. 7
8
and − 1
5
122. −5 1
2
and −1 1
4
SAMPLE EXAM
4. Convert to a fraction: 33 1
3
%.
5. Convert to a percent: 2 3
4
.
75
6. Reduce: .
225
2 2 2
7. a. (−7) ; b. −(−7) ; c. −7
3 3 3
8. a. (−3) ; b. −(−3) ; c. −3
Simplify.
10. − (− (−1))
11. 2
3
+ 1
5
− 3
10
14.
1
2
⋅ (− 45 ) ÷ 14
15
3 1 2
15.
5
⋅ 2
− 3
16. 4 ⋅ 5 − 20 ÷ 5 ⋅ 2
17. 10 − 7 (3 − 8) − 5 2
[5 − (7 − |−2|) + 15 ⋅ 2 ÷ 3]
1 2
19. 3
20. √
⎯⎯⎯⎯
1
⎯
16
⎯⎯⎯⎯
21. 3√ 72
24. A student earns 9, 8, 10, 7, and 8 points on the first 5 chemistry quizzes.
What is her quiz average?
25. An 8 34 foot plank is cut into 5 pieces of equal length. What is the length
of each piece?
1:
3:
5: >
7: <
9: >
11: ±2
13: Ø, No Solution
15: −8 ≤0
19: 5
21: −16
23: 9
25: −6
27: 22 units
29: 15
31: −10
35: 4
37: 36
39: Undefined
41: −54
3
45:
5
47: 2 7
8
6
49: −
35
7
51: −
8
4
53:
5
55: 1
1
57: −
6
4
59:
3
17
61: −
18
65: 5 8
25
67: 8.222
69: 21.6999
71: 0.072
73: 1.47
75: 5.5%
77: 90%
79: $1.75
81: $139.41
83: 64
85: −16
87: 4/81
89: 27
8
91:
125
93: −16
95: 7
97: 10
⎯⎯
99: 15√ 2
2
101:
3
⎯⎯
105: 3√ 2 centimeters
107: −59
109: −22
111: 31
113: 3
115: 5
117: −12
119: 36 units
43
121: units
40
3: <
5: 275%
17
11:
30
13: 48
11
15: −
30
17: 20
19: 10
⎯⎯
21: 18√ 2
23: −90
25: 1 3
4
feet
184
Chapter 2 Linear Equations and Inequalities
LEARNING OBJECTIVES
Preliminary Definitions
In algebra, letters are used to represent numbers. The letters used to represent
these numbers are called variables1. Combinations of variables and numbers along
with mathematical operations form algebraic expressions2, or just expressions.
The following are some examples of expressions with one variable, x:
185
Chapter 2 Linear Equations and Inequalities
The third term in this expression, −1, is a constant term because it is written
without a variable factor. While a variable represents an unknown quantity and
may change, the constant term does not change.
Example 1: List all coefficients and variable parts of each term: 5x 2 − 4xy − y 2 .
5x 2 5 x2
−4xy −4 xy
−y 2 −1 y2
−3(x + y) + (x + y) .
Example 2: List all coefficients and variable parts of each term:
3 2
−3(x + y) (x + y)
3 3
−3
(x + y) (x + y)
2 2
1
Try this! List all coefficients and variable parts of the expression:
−5a2 + ab − 2b2 − 3.
Video Solution
Example 3: Evaluate:
a. 2x + 3 , where x = −4
b. 23 y, where y = 9
Solution: To avoid common errors, it is a best practice to first replace all variables
with parentheses and then replace, or substitute9, the given value.
a.
b.
Answers: a. −5; b. 6
If parentheses are not used in part (a) of the previous example, the result is quite
different: 2x + 3 = 2 − 4 + 4. Without parentheses, the first operation is
subtraction, which leads to an incorrect result.
Solution: After substituting the given values for the variables, simplify using the
order of operations.
Answer: 13
Solution:
Answer: 11
Solution:
Answer: 11/4
The answer to the previous example is 114, which can be written as a mixed number
2 34. In algebra, improper fractions are generally preferred. Unless the original
problem has mixed numbers in it, or it is an answer to a real-world application,
solutions will be expressed as reduced improper fractions.
Solution: The order of operations requires us to perform the operations within the
parentheses first.
Answer: 0
Answer: 17
Answer: 35
10. The algebraic expression
b2 − 4ac.
Video Solution
The main difference between algebra and arithmetic is the organized use of
variables. This idea leads to reusable formulas11, which are mathematical models
using algebraic expressions to describe common applications. For example, the area
of a rectangle is modeled by the formula:
In this equation, variables are used to describe the relationship between the area of
a rectangle and the length of its sides. The area is the product of the length and
width of the rectangle. If the length of a rectangle measures 3 meters and the width
measures 2 meters, then the area can be calculated using the formula as follows:
Example 9: The cost of a daily truck rental is $48.00 plus an additional $0.45 for
every mile driven. This cost in dollars can be modeled by the formula
cost = 0.45x + 48, where x represents the number of miles driven in one day. Use
this formula to calculate the cost to rent the truck for a day and drive it 120 miles.
Solution: Use the formula to find the cost when the number of miles x = 120.
Example 10: Jim’s road trip takes 2 12 hours at an average speed of 66 miles per
hour. How far does he travel?
Solution: Substitute the appropriate values into the formula and then simplify.
Example 11: A wooden box is 1 foot in length, 5 inches wide, and 6 inches high.
Find the volume of the box in cubic inches.
Solution: Take care to ensure that all the units are consistent and use 12 inches for
the length instead of 1 foot.
Example 12: Calculate the simple interest earned on a 2-year investment of $1,250
at an annual interest rate of 3 34 %.
13. Modeled by the formula
I = prt, where p represents
the principal amount invested Solution: Convert 3 3
4
%to a decimal number before using it in the formula.
at an annual interest rate r for
t years.
Use this value for r and the fact that p = $1,250 and t = 2 years to calculate the simple
interest.
Try this! The perimeter of a rectangle is given by the formula P = 2l + 2w, where
l represents the length and w represents the width. Use the formula to calculate the
perimeter of a rectangle with a length of 5 feet and a width of 2 12 feet.
Answer: 15 feet
Video Solution
KEY TAKEAWAYS
TOPIC EXERCISES
Part A: Definitions
List all of the coefficients and variable parts of the following expressions.
1. 4x −1
2. – 7x 2 − 2x + 1
3. −x 2 + 5x − 3
2
4. 3x 2 y 2 − xy + 7
3
1 1 5
5.
3
y2 − 2
y+ 7
6. −4a2 b + 5ab 2 − ab + 1
7. 2(a + b) − 3(a + b)
3 5
8. 5(x + 2) 2 − 2 (x + 2) − 7
9. m 2 n − mn 2 + 10mn − 27
10. x 4 − 2x 3 − 3x 2 − 4x − 1
Evaluate.
11. x + 3 , where x = −4
12. 2x − 3 , where x = −3
3
15.
4
a, where a = 32
1
19. −2a + 1, where a = − 3
3
20. 4x + 3 , where x = 16
1
21. −x + , where x = −2
2
2 1 1
22.
3
x− 2
, where x =− 4
2
For each problem below, evaluate b − 4ac, given the following values for a, b,
and c.
23. a = 1, b = 2, c = 3
24. a = 3, b =– 4 , c =– 1
25. a =– 6, b = 0, c =– 2
1 2
26. a = ,b = 1, c =
2 3
1 1
27. a = −3, b = − ,c =
2 9
2
28. a = −13, b = − ,c =0
3
Evaluate.
34. 3x 2 − 5x + 1 , where x = −3
35. y 2 − y − 6, where y = 0
1
36. 1 − y 2 , where y = − 2
37. (x + 3) (x − 2) , where x = −4
38. (y − 5) (y + 6) , where y = 5
h = 0.01 .
45. 86°F
46. 95°F
47. −13°F
48. 14°F
49. 32°F
50. 0°F
3 5
54. b = inches and h = inches
4 8
55. A certain cellular phone plan charges $23.00 per month plus $0.09 for
each minute of usage. The monthly charge is given by the formula
monthly charge = 0.09x + 23 , where x represents the number of
minutes of usage per month. What is the charge for a month with 5 hours of
usage?
56. A taxi service charges $3.75 plus $1.15 per mile given by the formula
charge = 1.15x + 3.75 , where x represents the number of miles
driven. What is the charge for a 17-mile ride?
57. If a calculator is sold for $14.95, then the revenue in dollars, R, generated
by this item is given by the formula R = 14.95q , where q represents the
number of calculators sold. Use the formula to determine the revenue
generated by this item if 35 calculators are sold.
58. Yearly subscriptions to a tutoring website can be sold for $49.95. The
revenue in dollars, R, generated by subscription sales is given by the formula
R = 49.95q , where q represents the number of yearly subscriptions sold.
Use the formula to calculate the revenue generated by selling 250
subscriptions.
59. The cost of producing pens with the company logo printed on them
consists of a onetime setup fee of $175 plus $0.85 for each pen produced.
This cost can be calculated using the formula C = 175 + 0.85q , where q
represents the number of pens produced. Use the formula to calculate the
cost of producing 2,000 pens.
65. The formula for the area of a rectangle in square units is given by
A = l ⋅ w , where l represents the length and w represents the width. Use
this formula to calculate the area of a rectangle with length 12 centimeters
and width 3 centimeters.
70. Each side of a square deck measures 8 feet. Determine the area and
perimeter of the deck.
72. If a trunk measures 3 feet by 2 feet and is 2½ feet tall, then what is the
volume of the trunk?
73. The interior of an industrial freezer measures 3 feet wide by 3 feet deep
and 4 feet high. What is the volume of the freezer?
75. If the trip from Fresno to Sacramento can be made by car in 2½ hours at
an average speed of 67 miles per hour, then how far is Sacramento from
Fresno?
76. A high-speed train averages 170 miles per hour. How far can it travel in
1½ hours?
77. A jumbo jet can cruises at an average speed of 550 miles per hour. How
far can it travel in 4 hours?
78. A fighter jet reaches a top speed of 1,316 miles per hour. How far will the
jet travel if it can sustain this speed for 15 minutes?
79. The Hubble Space Telescope is in low earth orbit traveling at an average
speed of 16,950 miles per hour. What distance does it travel in 1½ hours?
80. Earth orbits the sun a speed of about 66,600 miles per hour. How far does
earth travel around the sun in 1 day?
87. Research and discuss the history of the symbols for addition (+) and
subtraction (−).
88. What are mathematical models and why are they useful in everyday life?
89. Find and post a useful formula. Demonstrate its use with some values.
90. Discuss the history and importance of the variable. How can you denote a
variable when you run out of letters?
91. Find and post a useful resource describing the Greek alphabet.
ANSWERS
5: Coefficients: {− 7}
; variable parts: {y 2 , y}
1 1 5
2
, 3
,
11: −1
13: 0
15: 24
17: 90
19: 5/3
21: 5/2
23: −8
25: −48
27: 19/12
29: 48
31: −5
33: 0
35: −6
37: 6
39: −17
41: 20.04
43: 1.32
45: 30°C
47: −25°C
49: 0°C
55: $50
57: $523.25
59: $1,875.00
63: 60 feet
81: $300
83: $76.80
85: $1,785
LEARNING OBJECTIVES
Distributive Property
The properties of real numbers are important in our study of algebra because a
variable is simply a letter that represents a real number. In particular, the
distributive property14 states that given any real numbers a, b, and c,
Certainly, if the contents of the parentheses can be simplified, do that first. On the
other hand, when the contents of parentheses cannot be simplified, multiply every
term within the parentheses by the factor outside of the parentheses using the
distributive property. Applying the distributive property allows you to multiply and
remove the parentheses.
and c, a (b + c) = ab + ac
14. Given any real numbers a, b,
206
Chapter 2 Linear Equations and Inequalities
Answer: 35y + 10
Solution: Multiply −3 times each of the coefficients of the terms inside the
parentheses.
Solution: Apply the distributive property by multiplying only the terms grouped
within the parentheses by 5.
Answer: 9x − 12y + 3
Division in algebra is often indicated using the fraction bar rather than with the
symbol (÷). And sometimes it is useful to rewrite expressions involving division as
products:
25x 2 −5x+10
Example 5: Divide: 5
.
Solution: First, treat this as 15 times the expression in the numerator and then
distribute.
Answer: 5x 2 − x + 2
Answer: −3x + 9y − 1
Video Solution
Terms with the same variable parts are called like terms15, or similar terms16.
Furthermore, constant terms are considered to be like terms. If an algebraic
expression contains like terms, apply the distributive property as follows:
In other words, if the variable parts of terms are exactly the same, then we may add
or subtract the coefficients to obtain the coefficient of a single term with the same
variable part. This process is called combining like terms17. For example,
Notice that the variable factors and their exponents do not change. Combining like
terms in this manner, so that the expression contains no other similar terms, is
called simplifying the expression18. Use this idea to simplify algebraic expressions
with multiple like terms.
Answer: −a + 11b
In the previous example, rearranging the terms is typically performed mentally and
is not shown in the presentation of the solution.
Example 7: Simplify: x 2 + 3x + 2 + 4x 2 − 5x − 7.
Solution: Identify the like terms and add the corresponding coefficients.
Answer: 5x 2 − 2x − 5
Solution: Remember to leave the variable factors and their exponents unchanged
in the resulting combined term.
Answer: 9x 2 y − 5xy 2
Example 9: Simplify: 12 a − 13 b + 34 a + .b
Answer: 54 a + 23 b
Solution: Consider the variable part to be x(x + y) . Then this expression has two
3
Answer: 14x(x + y)
3
Answer: −9x + 5y
Video Solution
When simplifying, we will often have to combine like terms after we apply the
distributive property. This step is consistent with the order of operations:
multiplication before addition.
In the example above, it is important to point out that you can remove the
parentheses and collect like terms because you multiply the second quantity by −7,
Answer: −3x + 15
Video Solution
Solution: Multiply each term within the parentheses by −1 and then combine like
terms.
Answer: 2x 2 + 2x + 1
When distributing a negative number, all of the signs within the parentheses will
change. Note that 5x in the example above is a separate term; hence the
distributive property does not apply to it.
Answer: −2x2 + 8x + 11
Caution
It is worth repeating that you must follow the order of operations: multiply and
divide before adding and subtracting!
Answer: 3x 2 − 6x + 29
Video Solution
Next, identify the key words and translate them into a mathematical expression.
Answer: −8x 2 + x − 14
KEY TAKEAWAYS
expressions, a (b + c) = ab + ac.
• Use the distributive property when multiplying grouped algebraic
TOPIC EXERCISES
Multiply.
1. 3 (3x − 2)
2. 12(−5y + 1)
3. −2 (x + 1)
4. 5(a − b)
(8x − 16)
5
5.
8
(10x − 5)
3
6. −
5
7. (2x + 3) ⋅ 2
8. (5x − 1) ⋅ 5
9. (−x + 7) (−3)
12. −(x − 1)
1
13.
3
(2x + 5)
3
14. − (y − 2)
4
15. −3 (2a + 5b − c)
16. − (2y 2 − 5y + 7)
17. 5 (y 2 − 6y − 9)
18. −6 (5x 2 + 2x − 1)
(4a − 6a + 4)
1 2
22.
2
(9y − 3y + 27)
1 2
23. −
3
25. 6 ( x + 12 )
1 1
3
x2 − 6
26. −2 (3x 3 − 2x 2 + x − 3)
20x+30y−10z
27.
10
−4a+20b−8c
28.
4
3x 2 −9x+81
29.
−3
−15y 2 +20y−5
30.
5
Translate the following sentences into algebraic expressions and then simplify.
Simplify.
35. 2x − 3x
1 4 3
39. − x+ + x
4 5 8
40. 2x − 4x + 7x − x
42. 5x − 7x + 8y + 2y
43. −8α + 2β − 5α − 6β
44. −6α + 7β − 2α + β
45. 3x + 5 − 2y + 7 − 5x + 3y
46. – y + 8x − 3 + 14x + 1 − y
3 2 1 3
50.
8
a− 7
b− 4
a+ 14
b
53. x 2 − y 2 + 2x 2 − 3y
1 2 1 1
54.
2
x2 − 3
y2 − 8
x2 + 5
y2
3 4 1 1
55.
16
a2 − 5
+ 4
a2 − 4
1 3 7 1
56.
5
y2 − 4
+ 10
y2 − 2
60. m 2 n 2 − mn + mn − 3m 2 n + 4m 2 n 2
1 2
62.
5
(x + 2) 3 − 3
(x + 2) 3
63. −3x (x 2 − 1) + 5x (x 2 − 1)
64. 5 (x − 3) − 8 (x − 3)
Simplify.
67. 5 (2x − 3) + 7
68. −2 (4y + 2) − 3y
69. 5x − 2(4x − 5)
70. 3 − (2x + 7)
71. 2x − (3x − 4y − 1)
73.
1
2
y− 3
4
x − ( 23 y − 1
5
x)
1 3 3 1
74.
5
a− 4
b+ 15
a− 2
b
(x − y) + x − 2y
2
75.
3
77. (2x 2 − 7x + 1) + (x 2 + 7x − 5)
79. − (x 2 − 3x + 8) + x 2 − 12
83. 10 − 5 (x 2 − 3x − 1)
84. 4 + 6 (y 2 − 9)
85.
3
4
x − ( 12 x 2 + 2
3
x − 75 )
86. −
7
3
x 2 + (− 1
6
x 2 + 7x − 1)
89. −4 (2x 2 + 3x − 2) + 5 (x 2 − 4x − 1)
90. 2 (3x 2 − 7x + 1) − 3 (x 2 + 5x − 1)
95. −5 (4α − 2β + 1) + 10 (α − 3β + 2)
Translate the following sentences into algebraic expressions and then simplify.
103. Explain how to subtract one expression from another. Give some
examples and demonstrate the importance of the order in which subtraction
is performed.
105. Can you apply the distributive property to the expression 5(abc)?
Explain why or why not and give some examples.
106. How can you check to see if you have simplified an expression
correctly? Give some examples.
ANSWERS
1: 9x −6
3: −2x −2
5: 5x − 10
7: 4x +6
9: 3x − 21
11: −2a + 3b
2 5
13:
3
x+ 3
17: 5y 2 − 30y − 45
19: 7x 2 − 3x + 11
21: 21x 2 − 6x − 3
25: 2x 2 −x+3
27: 2x + 3y − z
29: −x 2 + 3x − 27
31: 50x 2 − 18
33: 5x 2 − 40
35: −x
37: −5y − 30
1 4
39:
8
x+ 5
41: y
43: −13α − 4β
45: −2x + y + 12
47: 6xy +2
49: x −y
51: −8xy +4
53: 3x 2 − y 2 − 3y
7 21
55:
16
a2 − 20
57: 8x 2 y − 8xy 2
61: 5(x + y)
2
63: 2x (x 2 − 1)
65: −8 (2x + 7)
67: 10x −8
69: −3x + 10
71: −x + 4y + 1
11 1
73: − x− y
20 6
5 8
75:
3
x− 3
y
77: 3x 2 −4
79: 3x − 20
91: −x 2 y + 4xy 2
93: −3ab + 10
97: 5x −9
99: −2x −7
LEARNING OBJECTIVES
1. Identify linear equations with one variable and verify their solutions.
2. Use the properties of equality to solve basic linear equations.
3. Use multiple steps to solve linear equations by isolating the variable.
4. Solve linear equations where the coefficients are fractions or decimals.
Learning how to solve various algebraic equations is one of the main goals in
algebra. This section introduces the basic techniques used for solving linear
equations with one variable.
A solution21 to a linear equation is any value that can replace the variable to
produce a true statement. The variable in the linear equation 2x + 3 = 13 is x, and
the solution is x = 5 . To verify this, substitute the value 5 for x and check that you
obtain a true statement.
227
Chapter 2 Linear Equations and Inequalities
Answer: No, it is not a solution, because a = − 12 does not satisfy the equation.
Recall that when evaluating expressions, it is a good practice to first replace all
variables with parentheses, then substitute the appropriate values. By making use
of parentheses we avoid some common errors when working the order of
operations.
Example 3: Is y = −3 a solution to 2y − 5 = −y − 14 ?
Answer: No
Video Solution
Here we can see that the two linear equations 3x − 5 = 16 and 3x = 21 are
equivalent because they share the same solution set, namely, {7}. The goal is to
develop a systematic process to find equivalent equations until the variable is
23. Equations with the same isolated:
solution set.
Note
Solution: To isolate the variable x on the left side, subtract 3 from both sides.
Answer: The solution is x = −8 . To check that this is true, substitute −8 into the
original equation and simplify to see that it is satisfied: x + 3 = −8 + 3 = −5 ✓.
In the previous example, after subtracting 3 from both sides, you get x + 0 = −8.
By the additive identity property of real numbers, this is equivalent to x = −8 .
This step is often left out in the presentation of the solution.
If the variable term of the equation (including the coefficient) is isolated, then apply
the multiplication or division property of equality to obtain an equivalent equation
with the variable isolated. In other words, our goal is to obtain an equivalent
equation with x or 1x isolated on one side of the equal sign.
In the previous example, after dividing both sides by −5, x is left with a coefficient of
1, because −5−5
= 1. In fact, when we say “isolate the variable,” we mean to change
the coefficient of the variable to 1, because 1x = 7 is equivalent to x = 7 . This step
is often left out of the instructional examples even though its omission is sometimes
a source of confusion.
Example 6: Solve: 2 = 5 + x.
Solution: Isolate the variable x by subtracting 5 from both sides of the equation.
Answer: The solution is −3, and checking the solution shows that 2 = 5 − 3.
Video Solution
A linear equation of the form ax + b = c takes two steps to solve. First, use the
appropriate equality property of addition or subtraction to isolate the variable
term. Next, isolate the variable using the equality property of multiplication or
division. Checking solutions in the following examples is left to the reader.
Solution:
Solution:
Solution:
1
Answer: The solution is 18
.
Solution:
Recall that −y is equivalent to −1y ; divide both sides of the equation by −1.
Answer: x = −3
Video Solution
To solve an equation like 34 x = 1, we can isolate the variable by dividing both sides
by the coefficient. For example,
On the left side of the equal sign, the fraction cancels. On the right side, we have a
complex fraction and multiply by the reciprocal of the coefficient. You can save a
step by recognizing this and start by multiplying both sides of the equation by the
reciprocal of the coefficient.
Solution: Isolate the variable term using the addition property of equality and then
multiply both sides of the equation by the reciprocal of the coefficient 53 .
Solution:
Answer: x = 15
2
Video Solution
KEY TAKEAWAYS
TOPIC EXERCISES
1. x − 6 = 20; x = 26
2. y + 7 = −6; y = −13
3. −x + 5 = 17; x = 12
4. −2y = 44; y = 11
5. 4x = −24; x = −6
6. 5x − 1 = 34; x = −7
7. −2a − 7 = −7; a = 0
1
8. − x − 4 = −5; x = −3
3
1 2 1 11
9. − x+ =− ; x=
2 3 4 6
1 1 1 1
13.
2
y− 3
= 3
y+ 6
;y =3
4 1 2 1 1
14. − y+ =− y− ;y =
3 9 3 9 3
Solve.
15. x + 3 = 13
16. y − 4 = 22
17. −6 + x = 12
18. 9 + y = −4
1 1
19. x − =
2 3
2 1
20. x + =−
3 5
1 1
21. x +2 =3
2 3
23. 4x = −44
24. −9x = 63
25. −y = 13
26. −x = −10
27. −9x =0
29. 27 = 18y
30. 14 = −7x
t 1
34. − =
12 4
7 1
35. − x=
3 2
x
36.
5
= −3
4 2
37.
9
y=− 3
5 5
38. − y=−
8 2
Solve.
39. 5x + 7 = 32
40. 4x − 3 = 21
41. 3a − 7 = 23
51. 3 − 2y = −11
52. −4 − 7a = 24
53. −10 = 2x − 5
54. 24 = 6 − 12y
5 1 2
55.
6
x− 2
= 3
1 1 2
56.
2
x+ 3
= 5
2 1
57. 4a − =−
3 6
3 1 1
58.
5
x− 2
= 10
4 1 1
59. − y+ =
5 3 15
9 4 4
60. − x+ =
16 3 3
61. −x + 5 = 14
62. −y − 7 = −12
63. 75 − a = 200
64. 15 =5−x
65. −8 = 4 − 2x
66. 33 − x = 33
67. 18 =6−y
1
72. 70 = 50 − y
2
Translate the following sentences into linear equations and then solve.
Find a linear equation of the form ax + b = 0 with the given solution, where a
and b are integers. (Answers may vary.)
83. x =2
84. x = −3
1
85. x =− 2
2
86. x = 3
87. How many steps are needed to solve any equation of the form
ax + b = c? Explain.
ANSWERS
1: Yes
3: No
5: Yes
7: Yes
9: Yes
11: Yes
13: Yes
15: 10
17: 18
19: 5/6
21: 5/6
23: −11
25: −13
27: 0
29: 3/2
31: −7
33: −3/2
35: −3/14
37: −3/2
39: 5
41: 10
43: 1/3
45: 1/5
47: 2
49: 14.2
51: 7
53: −5/2
55: 7/5
57: 1/8
59: 1/3
61: −9
63: −125
65: 6
67: −12
69: 5.3
71: 1/16
73: 2x + 5 = 15 ; x = 5
75: 5x − 6 = 4; x = 2
n
77:
8
= 5; n = 40
79: 3n + 4 = 25 ; n = 7
2
81: − x = 20 ; x = −30
3
83: x −2=0
85: 2x +1=0
LEARNING OBJECTIVES
Linear equations typically are not given in standard form, so solving them requires
additional steps. These additional steps include simplifying expressions on each
side of the equal sign using the order of operations.
We will often encounter linear equations where the expressions on each side of the
equal sign can be simplified. Typically, this involves combining same-side like
terms26. If this is the case, then it is best to simplify each side first before solving.
Solution: First, combine the like terms on each side of the equal sign.
249
Chapter 2 Linear Equations and Inequalities
Solution: To “move” the term 5y to the left side, subtract it on both sides.
Always check to see that the solution is correct by substituting the solution back
into the original equation and simplifying to see if you obtain a true statement.
When solving linear equations, the goal is to determine what value, if any, will
produce a true statement when substituted in the original equation. Do this by
isolating the variable using the following steps:
Step 1: Simplify both sides of the equation using the order of operations and
combine all same-side like terms.
Solution: Simplify the linear expression on the left side before solving.
To check,
Solution: First, simplify the expressions on both sides of the equal sign.
Answer: x = 1
Video Solution
There are three different types of equations. Up to this point, we have been solving
conditional equations28. These are equations that are true for particular values. An
identity29 is an equation that is true for all possible values of the variable. For
example,
has no solution. We use the empty set, ∅, to indicate that there are no solutions.
If the end result of solving an equation is a true statement, like 0 = 0, then the
equation is an identity and any real number is a solution. If solving results in a false
statement, like 0 = 1, then the equation is a contradiction and there is no solution.
Solution:
Solution:
If it is hard to believe that any real number is a solution to the equation in the
previous example, then choose your favorite real number, and substitute it in the
equation to see that it leads to a true statement. Choose x = 7 and check:
Answer: R
Video Solution
The coefficients of linear equations may be any real number, even decimals and
fractions. When decimals and fractions are used, it is possible to use the
multiplication property of equality to clear the coefficients in a single step. If given
decimal coefficients, then multiply by an appropriate power of 10 to clear the
decimals. If given fractional coefficients, then multiply both sides of the equation by
the least common multiple of the denominators (LCD).
Solution: Notice that all decimal coefficients are expressed with digits in the tenths
place; this suggests that we can clear the decimals by multiplying both sides by 10.
Take care to distribute 10 to each term on both sides of the equation.
Example 8: Solve: 13 x + 15 = 15 x − 1
.
multiple of the given denominators. In this case, the LCM (3, 5) = 15.
Solution: Clear the fractions by multiplying both sides by the least common
It is important to know that these techniques only work for equations. Do not try to
clear fractions when simplifying expressions. As a reminder,
If we divide both sides by r, we obtain the formula t = Dr. Use this formula to find
the time, given the distance and the rate.
If we divide both sides by t, we obtain the formula r = Dt. Use this formula to find
the rate, given the distance traveled and the time it takes to travel that distance.
Using the techniques learned up to this point, we now have three equivalent
31. A formula that summarizes formulas relating distance, average rate, and time:
whole classes of problems.
When given a literal equation, it is often necessary to solve for one of the variables
in terms of the others. Use the properties of equality to isolate the indicated
variable.
Answer: a = P−b
2
x+y
Example 10: Solve for y: z = 2
.
Answer: y = 2z − x
Answer: b = 2a−c
3
Video Solution
KEY TAKEAWAYS
TOPIC EXERCISES
1. 2 (3x + 5) − 6 = 3x − 8; x = −4
2. −x + 17 − 8x = 9 − x; x = −1
1
3. 4 (3x − 7) − 3 (x + 2) = −1; x = 3
4. −5 − 2 (x − 5) = − (x + 3) ; x = −8
5. 7 − 2 ( 12 x − 6) = x − 1; x = 10
2 4
6. 3x − (9x − 2) = 0; x =
3 9
Solve.
7. 4x − 7 = 7x + 5
8. −5x + 3 = −8x − 9
9. 3x − 5 = 2x − 17
10. −2y − 52 = 3y + 13
11. −4x + 2 = 7x − 20
12. 4x − 3 = 6x − 15
13. 9x − 25 = 12x − 25
1 1 1 1
20.
3
x− 2
=− 4
x− 3
1 2 1 3
21. − y+ = y+
10 5 5 10
20 5 5
22. x − = x+
3 2 6
2 1 5 37
23.
3
y+ 2
= 8
y+ 24
1 4 10 1 2
24.
3
+ 3
x= 7
x+ 3
− 21
x
8 11 7 1
25.
9
− 18
x= 6
− 2
x
1 4 1
26.
3
− 9x = 9
+ 2
x
27. 12x − 5 + 9x = 44
28. 10 − 6x − 13 = 12
29. −2 + 4x + 9 = 7x + 8 − 2x
31. 3a + 5 − a = 2a + 7
32. −7b + 3 = 2 − 5b + 1 − 2b
33. 7x − 2 + 3x = 4 + 2x − 2
34. −3x + 8 − 4x + 2 = 10
35. 6x + 2 − 3x = −2x − 13
37. −x − 2 + 4x = 5 + 3x − 7
5 4 1 3 1 1
40.
8
− 3
x+ 3
=− 9
x− 4
+ 3
x
Solve.
41. −5 (2y − 3) + 2 = 12
42. 3 (5x + 4) + 5x = −8
43. 4 − 2 (x − 5) = −2
44. 10 − 5 (3x + 1) = 5 (x − 4)
45. 9 − (x + 7) = 2 (x − 1)
47. 3x − 2 (x + 1) = x + 5
48. 5x − 3 (2x − 1) = 2 (x − 3)
49. −6 (x − 1) − 3x = 3 (x + 8)
(5x + 10) =
3 1
50. − (4x − 12)
5 2
54. −9 (x − 3) − 3x = −3 (4x + 9)
55. 3 − 2 (x + 4) = −3 (4x − 5)
56. 12 − 2 (2x + 1) = 4 (x − 1)
57. 3 (x + 5) − 2 (2x + 3) = 7x + 9
59. −3 (2a − 3) + 2 = 3 (a + 7)
1 1
61.
2
(2x + 1) − 4
(8x + 2) = 3 (x − 4)
(6x − 3) −
2 1 3
62. − = (4x + 1)
3 2 2
(2x − 5) = 0
1 1
63.
2
(3x − 1) + 3
1 1 1
64.
3
(x − 2) + 5
= 9
(3x + 3)
65. −2 (2x − 7) − (x + 3) = 6 (x − 1)
68. 5 (x − 2) − (4x − 1) = −2 (3 − x)
1
77. Solve for h: A= 2
bh .
3b−2c
82. Solve for c: a= 3
.
x y
87. Solve for y:
3
− 5
= 1.
3 1 1
88. Solve for y:
4
x− 5
y= 2
.
Translate the following sentences into linear equations and then solve.
1 3 2
92. The sum of
2
x and 4
is equal to
3
x.
93. A number n divided by 5 is equal to the sum of twice the number and 3.
94. Negative ten times a number n is equal to the sum of three times the
number and 13.
98. Post some real-world linear formulas not presented in this section.
ANSWERS
1: Yes
3: No
5: Yes
7: −4
9: −12
11: 2
13: 0
15: −3
17: 3.1
19: −26/15
21: 1/3
23: 25
25: −5/2
27: 7/3
29: −1
31: ∅
33: 1/2
35: −3
37: R
39: −3/5
41: 1/2
43: 8
45: 4/3
47: ∅
49: −3/2
51: 5
53: −14
55: 2
57: 0
59: −10/9
61: 3
63: 1
65: 17/11
67: ∅
69: 7/6
A
71: w = l
P−2l
73: w = 2
75: b =P−a−c
2A
77: h = b
−ax+c
79: y = b
5z−y
81: x = 2
83: b = y − mx
3x−6
85: y = 2
5x−15
87: y = 3
89: 3x + 5 = 2x + 7 ; x = 2
91: 5x − 25 = 3x − 51 ; x = −13
n 5
93:
5
= 2n + 3; n = − 3
LEARNING OBJECTIVES
Algebra simplifies the process of solving real-world problems. This is done by using
letters to represent unknowns, restating problems in the form of equations, and
offering systematic techniques for solving those equations. To solve problems using
algebra, first translate the wording of the problem into mathematical statements
that describe the relationships between the given information and the unknowns.
Usually, this translation to mathematical statements is the difficult step in the
process. The key to the translation is to carefully read the problem and identify
certain key words and phrases.
270
Chapter 2 Linear Equations and Inequalities
One-half of a number.
1
2
x
Solution: First, choose a variable for the unknown number and identify the key
words and phrases.
Remember that subtraction is not commutative. For this reason, take care when
setting up differences. In this example, 4 − 2x = 16 is an incorrect translation.
Answer: 2x − 4 = 16
Example 2: Translate: When 7 is subtracted from 3 times the sum of a number and
12, the result is 20.
Answer: 3 (n + 12) − 7 = 20
To understand why parentheses are needed, study the structures of the following
two sentences and their translations:
The key is to focus on the phrase “3 times the sum.” This prompts us to group the
sum within parentheses and then multiply by 3. Once an application is translated
into an algebraic equation, solve it using the techniques you have learned.
Step 1: Read the problem several times, identify the key words and phrases, and
organize the given information.
Step 3: Translate and set up an algebraic equation that models the problem.
Step 5: Finally, answer the question in sentence form and make sure it makes sense
(check it).
For now, set up all of your equations using only one variable. Avoid two variables by
looking for a relationship between the unknowns.
Example 3: A larger integer is 2 less than 3 times a smaller integer. The sum of the
two integers is 18. Find the integers.
Solution:
Use the first sentence to identify the larger integer in terms of the variable x: “A
larger integer is 2 less than 3 times a smaller.”
Set up an equation: Add the expressions that represent the two integers, and set the
resulting expression equal to 18 as indicated in the second sentence: “The sum of
the two integers is 18.”
Back substitute: Use the expression 3x − 2 to find the larger integer—this is called
back substituting32.
Example 4: The difference between two integers is 2. The larger integer is 6 less
than twice the smaller. Find the integers.
Solution: Use the relationship between the two integers in the second sentence,
“The larger integer is 6 less than twice the smaller,” to identify the unknowns in
terms of one variable.
Since the difference is positive, subtract the smaller integer from the larger.
Solve.
Answer: The two integers are 8 and 10. These integers clearly solve the problem.
It is worth mentioning again that you can often find solutions to simple problems
by guessing and checking. This is so because the numbers are chosen to simplify the
process of solving, so that the algebraic steps are not too tedious. You learn how to
set up algebraic equations with easier problems, so that you can use these ideas to
solve more difficult problems later.
Example 5: The sum of two consecutive even integers is 46. Find the integers.
Solve.
It should be clear that consecutive even integers are separated by two units.
However, it may not be so clear that odd integers are as well.
Example 6: The sum of two consecutive odd integers is 36. Find the integers.
Add the two odd integers and set the expression equal to 36.
Solve.
The algebraic setup for even and odd integer problems is the same. A common
mistake is to use x and x + 3 when identifying the variables for consecutive odd
integers. This is incorrect because adding 3 to an odd number yields an even
number: for example, 5 + 3 = 8. An incorrect setup is very likely to lead to a decimal
answer, which may be an indication that the problem was set up incorrectly.
Example 7: The sum of three consecutive integers is 24. Find the integers.
Solve.
Try this! The sum of three consecutive odd integers is 87. Find the integers.
Video Solution
Recall that the perimeter33 of a polygon is the sum of the lengths of all the outside
edges. In addition, it is helpful to review the following perimeter formulas
(π ≈ 3.14159).
Keep in mind that you are looking for a relationship between the unknowns so that
you can set up algebraic equations using only one variable. When working with
geometry problems, it is often helpful to draw a picture.
Example 8: A rectangle has a perimeter measuring 64 feet. The length is 4 feet more
than 3 times the width. Find the dimensions of the rectangle.
Solution: The sentence “The length is 4 feet more than 3 times the width” gives
the relationship between the two variables.
Once you have set up an algebraic equation with one variable, solve for the width,
w.
Answer: The rectangle measures 7 feet by 25 feet. To check, add all of the sides:
Example 9: Two sides of a triangle are 5 and 7 inches longer than the third side. If
the perimeter measures 21 inches, find the length of each side.
Solution: The first sentence describes the relationships between the unknowns.
Substitute these expressions into the appropriate formula and use 21 for the
perimeter P.
Back substitute.
Answer: The three sides of the triangle measure 3 inches, 8 inches, and 10 inches.
The check is left to the reader.
Try this! The length of a rectangle is 1 foot less than twice its width. If the
perimeter is 46 feet, find the dimensions.
Video Solution
The amount of tax is this rate times the original cost of the item. The original cost
of the item is what you are asked to find.
Use this equation to solve for c, the original cost of the item.
Answer: The cost of the item before taxes is $48.95. Check this by multiplying $48.95
by 0.0725 to obtain the tax and add it to this cost.
Example 11: Given a 5 18 %annual interest rate, how long will it take $1,200 to yield
$307.50 in simple interest?
Solution:
Organize the data needed to use the simple interest formula I = prt.
Next, substitute all of the known quantities into the formula and then solve for the
only unknown, t.
Example 12: Mary invested her total savings of $3,400 in two accounts. Her mutual
fund account earned 8% last year and her CD earned 5%. If her total interest for the
year was $245, how much was in each account?
Solution: The relationship between the two unknowns is that they total $3,400.
When a total is involved, a common technique used to avoid two variables is to
represent the second unknown as the difference of the total and the first unknown.
The total interest is the sum of the interest earned from each account.
Interest
earned in
I = Prt = x ⋅ 0.08 ⋅ 1 = 0.08x
the mutual
fund:
Interest
earned in I = Prt = (3,400 − x) ⋅ 0.05 ⋅ 1 = 0.05(3,400 − x)
the CD:
This equation models the problem with one variable. Solve for x.
Back substitute.
Example 13: Joe has a handful of dimes and quarters that values $5.30. He has one
fewer than twice as many dimes than quarters. How many of each coin does he
have?
To determine the total value of a number of coins, multiply the number of coins by
the value of each coin. For example, 5 quarters have a value $0.25 ⋅ 5 = $1.25.
Answer: Joe has 12 quarters and 23 dimes. Check by multiplying $0.25 ⋅ 12 = $3.00
and $0.10 ⋅ 23 = $2.30. Then add to obtain the correct amount: $3.00 + $2.30 = $5.30.
Try this! A total amount of $5,900 is invested in two accounts. One account earns
3.5% interest and another earns 4.5%. If the interest for 1 year is $229.50, then how
much is invested in each account?
Video Solution
Uniform motion refers to movement at a speed, or rate, that does not change. We
can determine the distance traveled by multiplying the average rate by the time
traveled at that rate with the formula D = r ⋅ t. Applications involving uniform
motion usually have a lot of data, so it helps to first organize the data in a chart and
then set up an algebraic equation that models the problem.
Example 14: Two trains leave the station at the same time traveling in opposite
directions. One travels at 70 miles per hour and the other at 60 miles per hour. How
long does it take for the distance between them to reach 390 miles?
Solution: First, identify the unknown quantity and organize the data.
The given information is filled in on the following chart. The time for each train is
equal.
To avoid introducing two more variables, use the formula D = r ⋅ t to fill in the
unknown distances traveled by each train.
The algebraic setup is defined by the distance column. The problem asks for the
time it takes for the total distance to reach 390 miles.
Solve for t.
Answer: It takes 3 hours for the distance between the trains to reach 390 miles.
Example 15: A train traveling nonstop to its destination is able to make the trip at
an average speed of 72 miles per hour. On the return trip, the train makes several
stops and is only able to average 48 miles per hour. If the return trip takes 2 hours
longer than the initial trip to the destination, then what is the travel time each
way?
Solution: First, identify the unknown quantity and organize the data.
The algebraic setup is again defined by the distance column. In this case, the
distance to the destination and back is the same, and the equation is
Solve for t.
Try this! Mary departs for school on a bicycle at an average rate of 6 miles per
hour. Her sister Kate, running late, leaves 15 minutes later and cycles at twice that
speed. How long will it take Kate to catch up to Mary? Be careful! Pay attention to
the units given in the problem.
Video Solution
KEY TAKEAWAYS
TOPIC EXERCISES
Part A: Translate
8. The sum of three times a number and five times that same number is 24.
9. Ten is subtracted from twice some number and the result is the sum of the
number and 2.
10. Six less than some number is ten times the sum of that number and 5.
11. A larger integer is 1 more than twice another integer. If the sum of the
integers is 25, find the integers.
12. If a larger integer is 2 more than 4 times another integer and their
difference is 32, find the integers.
13. One integer is 30 more than another integer. If the difference between
the larger and twice the smaller is 8, find the integers.
14. The quotient of some number and 4 is 22. Find the number.
15. Eight times a number is decreased by three times the same number,
giving a difference of 20. What is the number?
16. One integer is two units less than another. If their sum is −22, find the
two integers.
17. The sum of two consecutive integers is 139. Find the integers.
18. The sum of three consecutive integers is 63. Find the integers.
19. The sum of three consecutive integers is 279. Find the integers.
20. The difference of twice the smaller of two consecutive integers and the
larger is 39. Find the integers.
21. If the smaller of two consecutive integers is subtracted from two times
the larger, then the result is 17. Find the integers.
22. The sum of two consecutive even integers is 46. Find the integers.
23. The sum of two consecutive even integers is 238. Find the integers.
24. The sum of three consecutive even integers is 96. Find the integers.
25. If the smaller of two consecutive even integers is subtracted from 3 times
the larger the result is 42. Find the integers.
26. The sum of three consecutive even integers is 90. Find the integers.
27. The sum of two consecutive odd integers is 68. Find the integers.
28. The sum of two consecutive odd integers is 180. Find the integers.
29. The sum of three consecutive odd integers is 57. Find the integers.
30. If the smaller of two consecutive odd integers is subtracted from twice
the larger the result is 23. Find the integers.
31. Twice the sum of two consecutive odd integers is 32. Find the integers.
32. The difference between twice the larger of two consecutive odd integers
and the smaller is 59. Find the integers.
33. If the perimeter of a square is 48 inches, then find the length of each
side.
34. The length of a rectangle is 2 inches longer than its width. If the
perimeter is 36 inches, find the length and width.
35. The length of a rectangle is 2 feet less than twice its width. If the
perimeter is 26 feet, find the length and width.
36. The width of a rectangle is 2 centimeters less than one-half its length. If
the perimeter is 56 centimeters, find the length and width.
37. The length of a rectangle is 3 feet less than twice its width. If the
perimeter is 54 feet, find the dimensions of the rectangle.
38. If the length of a rectangle is twice as long as the width and its perimeter
measures 72 inches, find the dimensions of the rectangle.
40. An isosceles triangle whose base is one-half as long as the other two
equal sides has a perimeter of 25 centimeters. Find the length of each side.
41. Each of the two equal legs of an isosceles triangle are twice the length of
the base. If the perimeter is 105 centimeters, then how long is each leg?
42. A triangle has sides whose measures are consecutive even integers. If the
perimeter is 42 inches, find the measure of each side.
43. A triangle has sides whose measures are consecutive odd integers. If the
perimeter is 21 inches, find the measure of each side.
44. A triangle has sides whose measures are consecutive integers. If the
perimeter is 102 inches, then find the measure of each side.
45. The circumference of a circle measures 50π units. Find the radius.
46. The circumference of a circle measures 10π units. Find the radius.
50. The diameter of a circle is 13 feet. Calculate the exact value of the
circumference.
53. For how many years must $10,000 be invested at an 8½% annual interest
rate to yield $4,250 in simple interest?
54. For how many years must $1,000 be invested at a 7.75% annual interest
rate to yield $503.75 in simple interest?
55. At what annual interest rate must $2,500 be invested for 3 years in order
to yield $412.50 in simple interest?
56. At what annual interest rate must $500 be invested for 2 years in order to
yield $93.50 in simple interest?
57. If the simple interest earned for 1 year was $47.25 and the annual rate
was 6.3%, what was the principal?
58. If the simple interest earned for 2 years was $369.60 and the annual rate
was 5¼%, what was the principal?
59. Joe invested last year’s $2,500 tax return in two different accounts. He
put most of the money in a money market account earning 5% simple
interest. He invested the rest in a CD earning 8% simple interest. How much
did he put in each account if the total interest for the year was $138.50?
60. James invested $1,600 in two accounts. One account earns 4.25% simple
interest and the other earns 8.5%. If the interest after 1 year was $85, how
much did he invest in each account?
61. Jane has her $5,400 savings invested in two accounts. She has part of it in
a CD at 3% annual interest and the rest in a savings account that earns 2%
annual interest. If the simple interest earned from both accounts is $140 for
the year, then how much does she have in each account?
62. Marty put last year’s bonus of $2,400 into two accounts. He invested part
in a CD with 2.5% annual interest and the rest in a money market fund with
1.3% annual interest. His total interest for the year was $42.00. How much
did he invest in each account?
63. Alice puts money into two accounts, one with 2% annual interest and
another with 3% annual interest. She invests 3 times as much in the higher
yielding account as she does in the lower yielding account. If her total
interest for the year is $27.50, how much did she invest in each account?
64. Jim invested an inheritance in two separate banks. One bank offered 5.5%
annual interest rate and the other 6¼%. He invested twice as much in the
higher yielding bank account than he did in the other. If his total simple
interest for 1 year was $4,860, then what was the amount of his inheritance?
65. If an item is advertised to cost $29.99 plus 9.25% tax, what is the total
cost?
66. If an item is advertised to cost $32.98 plus 8¾% tax, what is the total cost?
67. An item, including an 8.75% tax, cost $46.49. What is the original pretax
cost of the item?
68. An item, including a 5.48% tax, cost $17.82. What is the original pretax
cost of the item?
69. If a meal costs $32.75, what is the total after adding a 15% tip?
70. How much is a 15% tip on a restaurant bill that totals $33.33?
71. Ray has a handful of dimes and nickels valuing $3.05. He has 5 more
dimes than he does nickels. How many of each coin does he have?
72. Jill has 3 fewer half-dollars than she has quarters. The value of all 27 of
her coins adds to $9.75. How many of each coin does Jill have?
73. Cathy has to deposit $410 worth of five- and ten-dollar bills. She has 1
fewer than three times as many tens as she does five-dollar bills. How many
of each bill does she have to deposit?
74. Billy has a pile of quarters, dimes, and nickels that values $3.75. He has 3
more dimes than quarters and 5 more nickels than quarters. How many of
each coin does Billy have?
75. Mary has a jar with one-dollar bills, half-dollar coins, and quarters
valuing $14.00. She has twice as many quarters than she does half-dollar
coins and the same amount of half-dollar coins as one-dollar bills. How
many of each does she have?
76. Chad has a bill-fold of one-, five-, and ten-dollar bills totaling $118. He
has 2 more than 3 times as many ones as he does five-dollar bills and 1 fewer
ten- than five-dollar bills. How many of each bill does Chad have?
77. Two cars leave a location traveling in opposite directions. If one car
averages 55 miles per hour and the other averages 65 miles per hour, then
how long will it take for them to separate a distance of 300 miles?
78. Two planes leave the airport at the same time traveling in opposite
directions. The average speeds for the planes are 450 miles per hour and 395
miles per hour. How long will it take the planes to be a distance of 1,478.75
miles apart?
79. Bill and Ted are racing across the country. Bill leaves 1 hour earlier than
Ted and travels at an average rate of 60 miles per hour. If Ted intends to
catch up at a rate of 70 miles per hour, then how long will it take?
80. Two brothers leave from the same location, one in a car and the other on
a bicycle, to meet up at their grandmother’s house for dinner. If one brother
averages 30 miles per hour in the car and the other averages 12 miles per
hour on the bicycle, then it takes the brother on the bicycle 1 hour less than
3 times as long as the other in the car. How long does it take each of them to
make the trip?
81. A commercial airline pilot flew at an average speed of 350 miles per hour
before being informed that his destination airfield may be closed due to
poor weather conditions. In an attempt to arrive before the storm, he
increased his speed 400 miles per hour and flew for another 3 hours. If the
total distance flown was 2,950 miles, then how long did the trip take?
82. Two brothers drove the 2,793 miles from Los Angeles to New York. One of
the brothers, driving during the day, was able to average 70 miles per hour,
and the other, driving at night, was able to average 53 miles per hour. If the
brother driving at night drove 3 hours less than the brother driving in the
day, then how many hours did they each drive?
83. Joe and Ellen live 21 miles apart. Departing at the same time, they cycle
toward each other. If Joe averages 8 miles per hour and Ellen averages 6
miles per hour, how long will it take them to meet?
85. Jaime and Alex leave the same location and travel in opposite directions.
Traffic conditions enabled Alex to average 14 miles per hour faster than
Jaime. After 1½ hours they are 159 miles apart. Find the speed at which each
was able to travel.
86. Jane and Holly live 51 miles apart and leave at the same time traveling
toward each other to meet for lunch. Jane traveled on the freeway at twice
the average speed as Holly. They were able to meet in a half hour. At what
rate did each travel?
89. Research and compare simple interest and compound interest. What is
the difference?
91. Research ways to show that a repeating decimal is rational. Share your
findings on the discussion board.
ANSWERS
1: x + 6 = 37
3: 5x − 14 = 1
5: 5 (x + 6) = 20
7: 10 − (x + 3) = 5
9: 2x − 10 = x + 2
11: 8, 17
13: 22, 52
15: 4
17: 69, 70
21: 15, 16
25: 18, 20
27: 33, 35
31: 7, 9
33: 12 inches
39: 21 centimeters
45: 25 units
51: $271.25
53: 5 years
55: 5.5%
57: $750.00
59: Joe invested $2,050 in the money market account and $450 in the CD.
65: $32.76
67: $42.75
69: $37.66
79: 6 hours
81: 8 hours
83: 1½ hours
LEARNING OBJECTIVES
Definitions
All of the above are equivalent forms used to express a ratio. However, the most
familiar way to express a ratio is in the form of a fraction. When writing ratios, it is
important to pay attention to the units. If the units are the same, then the ratio can
be written without them.
Solution:
305
Chapter 2 Linear Equations and Inequalities
Answer: 1 to 4
If the units are different, then we must be sure to include them because the ratio
represents a rate39.
Solution:
Rates are useful when determining unit cost40, or the price of each unit. We use the
unit cost to compare values when the quantities are not the same. To determine the
unit cost, divide the cost by the number of units.
Example 3: A local supermarket offers a pack of 12 sodas for $3.48 on sale, and the
local discount warehouse offers the soda in a 36-can case for $11.52. Which is the
better value?
39. A ratio where the units for the
numerator and the
denominator are different. Solution: Divide the cost by the number of cans to obtain the unit price.
40. The price of each unit.
$3.48 $11.52
= $0.29/can = $0.32/can
12 cans 36 cans
Answer: The supermarket sale price of $3.48 for a 12-pack is a better value at $0.29
per can.
If we clear the fractions by multiplying both sides of the proportion by the product
of the denominators, 8, then we obtain the following true statement:
Given any nonzero real numbers a, b, c, and d that satisfy a proportion, multiply
both sides by the product of the denominators to obtain the following:
This shows that cross products are equal, and is commonly referred to as cross
multiplication42.
Solving Proportions
a c
42. If b = d
, then ad = bc.
Answer: n = 5
2
Example 5: Solve: 15 5
x = 6.
Answer: x = 18
n+3
Example 6: Solve: 5 = 72.
Answer: n = 29
2
Answer: n = 13
9
Video Solution
When setting up proportions, consistency with the units of each ratio is critical.
Units for the numerators should be the same and units for the denominators should
also be the same.
Since you are looking for the number of dentists who prefer the brand name out of
a total of 600 surveyed, construct the ratios with the number of dentists who prefer
the brand in the numerator and the total number surveyed in the denominator.
Answer: The claim suggests that 400 out of 600 dentists surveyed prefer the brand
name.
Example 8: In Tulare County, 3 out of every 7 voters said yes to Proposition 40. If
42,000 people voted, how many said no to Proposition 40?
Solution: The problem gives the ratio of voters who said yes, but it asks for the
number who said no.
If 3 out of 7 said yes, then we can assume 4 out of 7 said no. Set up the ratios with
the number of voters who said no in the numerator and the total number of voters
in the denominator.
Example 9: The sum of two integers in the ratio of 4 to 5 is 27. Find the integers.
Solution: The sum of two integers is 27; use this relationship to avoid two variables.
The integers are given to be in the ratio of 4 to 5. Set up the following proportion:
Try this! A recipe calls for 5 tablespoons of sugar for every 8 cups of flour. How
many tablespoons of sugar are required for 32 cups of flour?
Video Solution
We may write ABC ~ RST and conclude that all of the corresponding angles are
equal. The notation indicates that angle A corresponds to angle R and that the
measures of these angles are equal: A = R.
In addition, the measures of other pairs of corresponding angles are equal: B = S and
C = T.
Use uppercase letters for angles and a lowercase letter to denote the side opposite
of the given angle. Denote the proportionality of the sides as follows:
Solution: Draw a picture and identify the variables pictorially. Represent the
remaining unknown sides by s and t. Set up proportions and solve for the missing
sides.
The reduced ratio of any two corresponding sides of similar triangles is called the
scale factor44. In the previous example, the ratio of the two given sides a and r is
Therefore, triangle ABC is similar to triangle RST with a scale factor of 1/3. This
means that each leg of triangle ABC is 1/3 of the measure of the corresponding legs
of triangle RST. Also, another interesting fact is that the perimeters of similar
triangles are in the same proportion as their sides and share the same scale factor.
Example 11: If a triangle ABC has a perimeter of 12 units and is similar to RST with a
scale factor of 1/3, then find the perimeter of triangle RST.
Solution:
Scale factor 1/3 implies that the perimeters are in proportion to this ratio. Set up a
proportion as follows:
KEY TAKEAWAYS
TOPIC EXERCISES
3. 96 feet : 72 feet
240 miles
4.
4 hours
96 feet
5.
3 seconds
6,000 revolutions
6.
4 minutes
7. Google’s average 2008 stock price and earnings per share were $465.66 and
$14.89, respectively. What was Google’s average price-to-earnings ratio in
2008? (Source: Wolfram Alpha)
8. The F-22 Raptor has two engines that each produce 35,000 pounds of
thrust. If the takeoff weight of this fighter jet is 50,000 pounds, calculate the
plane’s thrust-to-weight ratio. (Source: USAF)
10. Joe and Mary wish to take a road trip together and need to decide whose
car they will take. Joe calculated that his car is able to travel 210 miles on 12
gallons of gasoline. Mary calculates that her car travels 300 miles on 19
gallons. Which of their cars gets more miles to the gallon?
Solve.
2 n
11.
3
= 150
7 21
12. n = 5
1 5
13.
3
= n
12 6
14.
5
= n
n 3
15.
8
=− 2
n 5
16.
3
=− 7
2n
17. 8 = 3
5
18. n = −30
1
19. 1 = n−1
1
20. −1 =− n+1
40 5
21. − n =− 3
2n+1 3
22.
3
=− 5
5 2
23.
3n+3
= 3
n+1 1
24.
2n−1
= 3
5n+7 n−1
25.
5
= 2
n+7
26. −2n +3= 6
27. Find two numbers in the ratio of 3 to 5 whose sum is 160. (Hint: Use n and
160 − n to represent the two numbers.)
31. A larger integer is 5 more than a smaller integer. If the two integers have
a ratio of 6 to 5 find the integers.
32. A larger integer is 7 less than twice a smaller integer. If the two integers
have a ratio of 2 to 3 find the integers.
x−2y 3y
34.
3
=− 5
2x+4y 3
35.
2x−4y
= 2
x+y 3
36. x−y = 5
Part C: Applications
37. If 4 out of every 5 voters support the governor, then how many of the
1,200 people surveyed support the governor?
38. If 1 out of every 3 voters surveyed said they voted yes on Proposition 23,
then how many of the 600 people surveyed voted yes?
39. Out of 460 students surveyed, the ratio to support the student union
remodel project was 3 to 5. How many students were in favor of the
remodel?
40. An estimated 5 out of 7 students carry credit card debt. Estimate the
number of students that carry credit card debt out of a total of 14,000
students.
42. In the year 2009 it was estimated that there would be 838 deaths in the
United States for every 100,000 people. If the total US population was
estimated to be 307,212,123 people, then how many deaths in the United
States were expected in 2009? (Source: CIA World Factbook)
43. In the year 2009 it was estimated that there would be 1,382 births in the
United States for every 100,000 people. If the total US population was
estimated to be 307,212,123 people, then how many births in the United
States were expected in 2009? (Source: CIA World Factbook)
44. If 2 out of every 7 voters approve of a sales tax increase then determine
the number of voters out of the 588 surveyed who do not support the
increase.
45. A recipe calls for 1 cup of lemon juice to make 4 cups of lemonade. How
much lemon juice is needed to make 2 gallons of lemonade?
46. The classic “Shirley Temple” cocktail requires 1 part cherry syrup to 4
parts lemon-lime soda. How much cherry syrup is needed to mix the cocktail
given a 12-ounce can of lemon-lime soda?
47. A printer prints 30 pages in 1 minute. How long will it take to print a
720-page booklet?
48. A typist can type 75 words per minute. How long will it take to type 72
pages if there are approximately 300 words per page?
1
49. On a particular map, every 16
inch represents 1 mile. How many miles
does 3 12 inches represent?
51. A candy store offers mixed candy at $3.75 for every half-pound. How
much will 2.6 pounds of candy cost?
52. Mixed nuts are priced at $6.45 per pound. How many pounds of mixed
nuts can be purchased with $20.00?
53. Corn at the farmers market is bundled and priced at $1.33 for 6 ears. How
many ears can be purchased with $15.00?
54. If 4 pizzas cost $21.00, then how much will 16 pizzas cost?
3
55. A sweetened breakfast cereal contains 110 calories in one 4
-cup serving.
How many calories are in a 1 78 -cup serving?
57. A 200-pound man would weigh about 33.2 pounds on the moon. How
much will a 150-pound man weigh on the moon?
58. A 200-pound man would weigh about 75.4 pounds on Mars. How much
will a 150-pound man weigh on Mars?
60. There is a 1 out of 6 chance of rolling a 7 with two six-sided dice. How
many times can we expect a 7 to come up in 300 rolls?
61. The ratio of peanuts to all nuts in a certain brand of packaged mixed nuts
is 3 to 5. If the package contains 475 nuts, then how many peanuts can we
expect?
62. A mixed bag of marbles is packaged with a ratio of 6 orange marbles for
every 5 red marbles. If the package contains 216 orange marbles, then how
many red marbles can we expect?
64. If a video monitor is produced in the width to height ratio of 16:9 and the
width of the monitor is 40 inches, then what is the height?
If triangle ABC is similar to triangle RST, find the remaining two sides given the
information.
67. b = 2, c = 4, r = 6, and s = 4
70. c = 2, r = 7, s = 9, and t = 4
71. At the same time of day, a tree casts a 12-foot shadow while a 6-foot man
casts a 3-foot shadow. Estimate the height of the tree.
72. At the same time of day, a father and son, standing side by side, cast a
4-foot and 2-foot shadow, respectively. If the father is 6 feet tall, then how
tall is his son?
73. If the 6-8-10 right triangle ABC is similar to RST with a scale factor of 2/3,
then find the perimeter of triangle RST.
74. If the 3-4-5 right triangle ABC is similar to RST with a scale factor of 5,
then find the perimeter of triangle RST.
77. The perimeter of an isosceles triangle ABC, where the two equal sides
each measure twice that of the base, is 60 units. If the base of a similar
triangle measures 6 units, then find its perimeter.
78. The perimeter of an isosceles triangle ABC measures 11 units and its two
equal sides measure 4 units. If triangle ABC is similar to triangle RST and
triangle RST has a perimeter of 22 units, then find all the sides of triangle
RST.
79. A 6-8-10 right triangle ABC is similar to a triangle RST with perimeter 72
units. Find the length of each leg of triangle RST.
84. Research and discuss the various aspect ratios that are available in
modern media devices.
ANSWERS
1: 2:5
3: 4:3
7: 31.27
11: n = 100
13: n = 15
15: n = −12
17: n = 12
19: n =2
21: n = 24
3
23: n = 2
19
25: n =− 5
31: 25, 30
33: 3/4
35: 10
47: 24 minutes
49: 56 miles
51: $19.50
53: 66 ears
59: 60 times
65: t = 20, r = 12
67: a = 3, t = 8
69: r = 8, b = 15
71: 24 feet
73: 36 units
75: 2 units
77: 30 units
LEARNING OBJECTIVES
Unbounded Intervals
To express the solution graphically, draw a number line and shade in all the values
that are solutions to the inequality. Interval notation is textual and uses specific
notation as follows:
327
Chapter 2 Linear Equations and Inequalities
included. Infinity is an upper bound to the real numbers, but is not itself a real
number: it cannot be included in the solution set.
Now compare the interval notation in the previous example to that of the strict, or
noninclusive, inequality that follows:
Strict inequalities50 imply that solutions may get very close to the boundary point,
in this case 2, but not actually include it. Denote this idea with an open dot on the
number line and a round parenthesis in interval notation.
Solution: Use an open dot at 3 and shade all real numbers strictly less than 3. Use
negative infinity51 (−∞) to indicate that the solution set is unbounded to the left on
a number line.
(−inf, 5].
Any real number less than 3 in the shaded region on the number line will satisfy at
least one of the two given inequalities.
Example 4: Graph and give the interval notation equivalent: x < 3 or x ≥ −1.
Solution: Both solution sets are graphed above the union, which is graphed below.
When you combine both solution sets and form the union, you can see that all real
numbers satisfy the original compound inequality.
In summary,
and
Bounded Intervals
An inequality such as
reads “−1 one is less than or equal to x and x is less than three.” This is a compound
inequality because it can be decomposed as follows:
The logical “and” requires that both conditions must be true. Both inequalities are
satisfied by all the elements in the intersection54, denoted ∩, of the solution sets of
each.
Example 5: Graph and give the interval notation equivalent: x < 3 and x ≥ −1.
Solution: Determine the intersection, or overlap, of the two solution sets. The
solutions to each inequality are sketched above the number line as a means to
determine the intersection, which is graphed on the number line below.
solution graphically and/or with interval notation, in this case [−1, 3).
there are infinitely many real numbers between −1 and 3, we must express the
Example 6: Graph and give the interval notation equivalent: − 32 < x < 2.
Example 7: Graph and give the interval notation equivalent: −5 < x ≤ 15.
Solution: Shade all real numbers between −5 and 15, and indicate that the upper
bound, 15, is included in the solution set by using a closed dot.
In the previous two examples, we did not decompose the inequalities; instead we
chose to think of all real numbers between the two given bounds.
In summary,
Set-Builder Notation
In this text, we use interval notation. However, other resources that you are likely
to encounter use an alternate method for describing sets called set-builder
notation55. We have used set notation to list the elements such as the integers
The braces group the elements of the set and the ellipsis marks indicate that the
integers continue forever. In this section, we wish to describe intervals of real
numbers—for example, the real numbers greater than or equal to 2.
Since the set is too large to list, set-builder notation allows us to describe it using
familiar mathematical notation. An example of set-builder notation follows:
Here x∈R describes the type of number, where the symbol (∈) is read “element of.”
This implies that the variable x represents a real number. The vertical bar (|) is read
“such that.” Finally, the statement x ≥ 2 is the condition that describes the set
using mathematical notation. At this point in our study of algebra, it is assumed
that all variables represent real numbers. For this reason, you can omit the “∈R”
and write {x||x ≥ 2} , which is read “the set of all real numbers x such that x is
greater than or equal to 2.”
KEY TAKEAWAYS
TOPIC EXERCISES
Graph all solutions on a number line and provide the corresponding interval
notation.
1. x ≤ 10
2. x > −5
3. x >0
4. x ≤0
5. x ≤ −3
6. x ≥ −1
7. −4 <x
8. 1 ≥x
1
9. x <− 2
3
10. x ≥− 2
3
11. x ≥ −1 4
3
12. x < 4
Graph all solutions on a number line and give the corresponding interval notation.
13. −2 <x<5
14. −5 ≤ x ≤ −1
15. −5 < x ≤ 20
16. 0 ≤ x < 15
17. 10 < x ≤ 40
19. 0 < x ≤ 50
3 1
22. − ≤x≤
4 2
1
23. −1 ≤x<1 2
1 1
24. −1 <x<−
2 2
26. x < −2 or x ≥ 4
27. x ≤ 0 or x > 10
4 1
30. x ≤− or x > −
3 3
33. x < 3 or x ≥ 3
34. x ≤ 0 or x > 0
36. x ≥ −3 or x > 0
37. x ≥ 5 or x > 0
38. x < 15 or x ≤ 10
41. x ≥ −5 and x ≤ −1
44. x ≤ 5 and x ≥ 5
45. x ≤ 0 and x ≥ 0
49. (−∞, 7]
50. (−4, ∞)
51. [− 1
2
, ∞)
52. (−∞, − 3)
54. (−20, 0]
55. (−14, − 2)
56. [
3]
2 4
3
,
57. (−
2)
3 1
4
,
58. (−∞, − 8)
59. (8, ∞)
72. Explain why we do not use a bracket in interval notation when infinity is
an endpoint.
ANSWERS
1: (−∞, 10]
3: (0, ∞)
5: (−∞, − 3]
7: (−4, ∞)
9: (−∞, − 12 )
11: [−1 3
4
, ∞)
13: (−2, 5)
21: (− 5
8
, 1
8 )
23: [−1, 1 12 )
29: (−∞, − 23 ) ∪ ( 13 , ∞)
31: R
33: R
35: (−∞, 2)
37: (0, ∞)
39: (−2, 3)
41: [−5, − 1]
43: ∅
45: {0}
47: (0, ∞)
49: x ≤7
1
51: x ≥− 2
53: −8 < x ≤ 10
59: x >8
61: x ≤ −2 or x ≥ 0
65: x < 27
67: x >5
69: −6 <x<6
LEARNING OBJECTIVES
Solution: Substitute the values for x, simplify, and check to see if we obtain a true
56. A mathematical statement statement.
relating a linear expression as
either less than or greater than
another.
346
Chapter 2 Linear Equations and Inequalities
All but one of the techniques learned for solving linear equations apply to solving
linear inequalities. You may add or subtract any real number to both sides of an
inequality, and you may multiply or divide both sides by any positive real number to
create equivalent inequalities. For example,
Both subtracting 7 from each side and dividing each side by +5 results in an
equivalent inequality that is true.
Solution:
It is helpful to take a minute and choose a few values in and out of the solution set,
substitute them into the original inequality, and then verify the results. As
indicated, you should expect x = 0 to solve the original inequality, but x = 5
should not.
Checking in this manner gives a good indication that the inequality is solved
correctly. This can be done mentally.
When working with linear inequalities, a different rule applies when multiplying or
dividing by a negative number. To illustrate the problem, consider the true
statement 10 > −5 and divide both sides by −5.
The same problem occurs when multiplying by a negative number. This leads to the
following new rule: when multiplying or dividing by a negative number, reverse the
inequality. It is easy to forget to do this so take special care to watch for negative
coefficients.
We use these properties to obtain an equivalent inequality59, one with the same
solution set, where the variable is isolated. The process is similar to solving linear
equations.
Solution:
Solution:
Solution:
Answer: [−4, ∞)
Video Solution
Solve each inequality individually, and the intersection of the two solution sets
solves the original compound inequality. While this method works, there is another
method that usually requires fewer steps. Apply the properties of this section to all
three parts of the compound inequality with the goal of isolating the variable in the
middle of the statement to determine the bounds of the solution set.
Solution:
Solution:
The answer above can be written in an equivalent form, where smaller numbers lie
to the left and the larger numbers lie to the right, as they appear on a number line.
Answer: (−4, 3]
Video Solution
For compound inequalities with the word “or” you must work both inequalities
separately and then consider the union of the solution sets. Values in this union
solve either inequality.
Solution: Solve each inequality and form the union by combining the solution sets.
Video Solution
Some of the key words and phrases that indicate inequalities are summarized
below:
A number is at least 5.
x≥5
A number is 5 or more inclusive.
A number is at most 3.
x≤3
A number is 3 or less inclusive.
As with all applications, carefully read the problem several times and look for key
words and phrases. Identify the unknowns and assign variables. Next, translate the
wording into a mathematical inequality. Finally, use the properties you have
learned to solve the inequality and express the solution graphically or in interval
notation.
Solution: First, choose a variable for the unknown number and identify the key
words and phrases.
Answer: 2n − 5 ≤ 25. The key phrase “is at most” indicates that the quantity has a
maximum value of 25 or smaller.
Example 10: The temperature in the desert can range from 10°C to 45°C in one
24-hour period. Find the equivalent range in degrees Fahrenheit, F, given that
C = 59 (F − 32).
Example 11: In the first four events of a meet, a gymnast scores 7.5, 8.2, 8.5, and 9.0.
What must she score on the fifth event to average at least 8.5?
Solution: The average must be at least 8.5; this means that the average must be
greater than or equal to 8.5.
KEY TAKEAWAYS
TOPIC EXERCISES
1. 2x − 3 < 6; x = −1
2. −3x + 1 ≤ 0; x = −2
3. 5x − 20 > 0; x = 3
1 3 1
4.
2
x+1>− 4
; x=− 4
5. −5 < 7x + 1 < 9; x = 0
9. 2x + 1 < −3 or 2x + 1 ≥ 5 ; x = 2
Solve and graph the solution set. In addition, present the solution set in interval
notation.
11. x +5>1
12. x − 3 < −4
13. 6x ≤ 24
14. 4x > −8
15. −7x ≤ 14
17. 7x − 3 ≤ 25
22. −8x + 1 ≤ 29
1
23. 7 x−3<1
1 1 2
24.
2
x− 3
> 3
5 1 1
25.
3
x+ 2
≤ 3
3 1 5
26. − x− ≥
4 2 2
1 3 1
27. − x+ <−
5 4 5
2
28. − x + 1 < −3
3
30. −7 (x − 2) + 1 < 15
33. 5 − 3 (2x − 6) ≥ −1
35. 2 (x − 7) − 3 (x + 3) ≤ −3
36. 5x − 3 > 3x + 7
37. 4 (3x − 2) ≤ −2 (x + 3) + 12
43. The sum of three times a number and 4 is greater than negative 8.
44. The sum of 7 and three times a number is less than or equal to 1.
45. When a number is subtracted from 10, the result is at most 12.
46. When 5 times a number is subtracted from 6, the result is at least 26.
47. If five is added to three times a number, then the result is less than
twenty.
48. If three is subtracted from two times a number, then the result is greater
than or equal to nine.
49. Bill earns $12.00 for the day plus $0.25 for every person he gets to
register to vote. How many people must he register to earn at least $50.00
for the day?
50. With a golf club membership costing $100 per month, each round of golf
costs only $25.00. How many rounds of golf can a member play if he wishes
to keep his costs to $250 per month at most?
51. Joe earned scores of 72, 85, and 75 on his first three algebra exams. What
must he score on the fourth exam to average at least 80?
52. Maurice earned 4, 7, and 9 points out of 10 on the first three quizzes.
What must he score on the fourth quiz to average at least 7?
53. A computer is set to shut down if the temperature exceeds 40°C. Give an
equivalent statement using degrees Fahrenheit. (Hint: C = 59 (F − 32).)
Solve and graph the solution set. In addition, present the solution set in interval
notation.
55. −1 <x+3<5
57. −2 ≤ 4x + 6 < 10
58. −10 ≤ 3x − 1 ≤ −4
1 3 2 1
64. − < x− ≤
2 4 3 2
65. −3 ≤ 3(x − 1) ≤ 3
( 2 x − 1) +
3 1 1 3 3
69. − ≤ <
2 4 4 2
1
70. −4 ≤− (3x + 12) < 4
3
71. −2 ≤ 12 − 2(x − 3) ≤ 20
76. 7x + 4 ≤ 4 or 6x − 5 ≥ 1
1 1
80.
3
x + 3 ≥ −2 or 3
x+3≤2
81. 3x + 7 ≤ 7 or − 5x + 6 > 6
89. Five more than two times some number is between 15 and 25.
90. Four subtracted from three times some number is between −4 and 14.
91. Clint wishes to earn a B, which is at least 80 but less than 90. What range
must he score on the fourth exam if the first three were 65, 75, and 90?
93. The average temperature in London ranges from 23°C in the summer to
14°C in the winter. Find the equivalent range in degrees Fahrenheit.
94. If the base of a triangle measures 5 inches, then in what range must the
height be for the area to be between 10 square inches and 20 square inches?
95. A rectangle has a length of 7 inches. Find all possible widths if the area is
to be at least 14 square inches and at most 28 square inches.
96. A rectangle has a width of 3 centimeters. Find all possible lengths, if the
perimeter must be at least 12 centimeters and at most 26 centimeters.
97. The perimeter of a square must be between 40 feet and 200 feet. Find the
length of all possible sides that satisfy this condition.
98. If two times an angle is between 180 degrees and 270 degrees, then what
are the bounds of the original angle?
99. If three times an angle is between 270 degrees and 360 degrees then what
are the bounds of the original angle?
100. Research and discuss the use of set-builder notation with intersections
and unions.
101. Can we combine logical “or” into one statement like we do for logical
“and”?
ANSWERS
1: Yes
3: No
5: Yes
7: Yes
9: Yes
13: x ≤ 4 ; (−∞, 4]
15: x ≥ −2 ; [−2, ∞)
17: x ≤ 4 ; (−∞, 4]
3 (
21: x <− 2
; −∞, − 23 )
10 ( ]
1
25: x ≤− ; −∞, − 1
10
4 ( 4
27: x > 19
; 19 , ∞)
31: ∅
33: x ≤ 4 ; (−∞, 4]
37: x ≤ 1 ; (−∞, 1]
39: R
2 [2
41: x ≥ 1
; 1 , ∞)
43: n > −4
45: n ≥ −2
47: n <5
53: The computer will shut down when the temperature exceeds 104°F.
2 [ )
3
63: 0 ≤x< ; 0, 3
2
65: 0 ≤ x ≤ 2 ; [0, 2]
77: R
81: x ≤ 0 ; (−∞, 0]
372
Chapter 2 Linear Equations and Inequalities
REVIEW EXERCISES
Introduction to Algebra
Evaluate.
1. 2x + 7 , where x = −4
2. −4x + 1 , where x = −2
2 1 3
3.
3
y− 2
, where y = 5
3 5 2
4. − y+ , where y =
4 3 3
2 1
5. b − 4ac, where a = 5, b = −2 , and c = 2
2 1
6. b − 4ac, where a = − ,b = −1 , and c = −3
4
7. 2x 2 − x + 3 , where x = −3
8. 5x 2 − 2x + 4 , where x = −1
10. A bus traveled for 1 23 hours at an average speed of 48 miles per hour.
What distance did the bus travel?
Multiply.
13. −5 (3x − 2)
14. (6x − 9) ⋅ 3
(4x − 8x + 32)
3 2
15.
4
16. −20 ( x − 54 )
1 2
10
x2 − 5
17. − (3a − 2b + 5c − 1)
18. −6 (y 3 + 3y 2 − 7y + 5)
Simplify.
19. 5a − 7b − 3a + 5b
20. 6x 2 − 4x + 7x 2 − 3x
3 1 1 1
21.
5
xy + 2
− 10
xy − 4
3 4 1 1
22. − a− b+ a− 7
b
4 21 3
24. y 2 − 3y + 5 − y 2 + 9
25. −8 (8x − 3) − 7
26. 7 − (6x − 9)
30. 10 − 5 (x 2 − x + 1) − (3x 2 + 5x − 1)
34. 4x − 3 = −3x ; x = −2
1
35. 8x + 2 = 5x + 1 ; x = − 3
36. 2x + 4 = 3x − 2 ; x = −1
Solve.
37. y + 23 = 25
38. −3x = 54
x
39.
4
=8
5 2
40.
2
x= 3
41. 7x − 5 = −54
42. −2x + 7 = 43
43. 7x +3=0
44. 4x +5=5
45. 1 = 10 − 3x
46. 10 − 5y = 15
47. 7 − y = 28
48. 33 − x = 16
5 1 3
49.
6
x+ 3
= 2
2 1 1
50. − y+ =−
3 5 3
Solve.
53. 5x − 2 = 3x + 6
54. 7x + 1 = 2x − 29
56. 6y − 13 = 3 + 7y
57. 8y + 6 − 3y = 22 − 3y
58. 12 − 5y + 6 = y − 6
59. 5 − 2 (7x − 1) = 2x + 1
60. 10 − 5 (x − 1) = 5 − x
61. 2x − (3x − 4) = 7 − x
62. 9x − 3 (2x + 1) = 3x − 3
3 1
65.
2
(4x − 3) + 4
=1
3 1
66.
4
− 6
(4x − 9) = 2
67.
2
3
(9x − 3) + 1
2
= 3 (2x − 12 )
68. 1 − 5
4
(4x − 1) = 5 ( 12 − x)
75. A larger integer is 3 more than twice a smaller integer. If their sum is 39,
then find the integers.
76. A larger integer is 5 more than 3 times a smaller integer. If their sum is
49, then find the integers.
77. The sum of three consecutive odd integers is 45. Find the integers.
78. The sum of three consecutive even integers is 72. Find the integers.
79. The sum of three consecutive integers is 60. Find the integers.
80. The length of a rectangle is 7 centimeters less than twice its width. If the
perimeter measures 46 centimeters, then find the dimensions of the
rectangle.
81. A triangle has sides whose measures are consecutive even integers. If the
perimeter is 24 meters, then find the measure of each side.
82. The circumference of a circle measures 24π inches. Find the radius of the
circle.
83. Mary invested $1,800 in two different accounts. One account earned 3.5%
simple interest and the other earned 4.8%. If the total interest after 1 year
was $79.25, then how much did she invest in each account?
84. James has $6 in dimes and quarters. If he has 4 fewer quarters than he
does dimes, then how many of each coin does he have?
85. Two brothers leave the house at the same time traveling in opposite
directions. One averages 40 miles per hour and the other 36 miles per hour.
How long does it take for the distance between them to reach 114 miles?
86. Driving to her grandmother’s house, Jill made several stops and was only
able to average 40 miles per hour. The return trip took 2 hours less time
because she drove nonstop and was able to average 60 miles per hour. How
long did it take Jill to drive home from her grandmother’s house?
Solve.
3 n
87.
4
= 8
7 28
88.
3
= n
6 30
89. n = 11
n 2
90.
5
= 3
3n−1 1
91.
3
= 2
4 1
92.
2n+5
=− 3
1
93. −3 = n−1
2 1
94.
n−6
= 2n+1
96. A larger number is 2 less than twice a smaller number. If the two
numbers are in the proportion 5 to 9, then find the numbers.
97. A recipe calls for 1½ teaspoons of vanilla extract for every 3 cups of
batter. How many teaspoons of vanilla extract should be used with 7 cups of
batter?
If triangle ABC is similar to triangle RST, then find the remaining two sides given the
following.
101. At the same time of day, a pole casts a 27-foot shadow and 4-foot boy
casts a 6-foot shadow. Calculate the height of the pole.
Graph all solutions on a number line and provide the corresponding interval
notation.
103. x < −1
104. x ≤ 10
105. x ≥0
106. x > −2
1 3
107. − ≤x<
2 2
109. x < 5 or x ≥ 15
111. (−∞, 3)
112. [−4, ∞)
113. (−2, 2)
114. (−3, 8]
Solve and graph. In addition, present the solution set in interval notation.
117. x + 2 > −1
118. −4x ≥ 16
119. 9x + 4 ≤ −5
120. 5x − 7 < 13
121. 7x + 5 − 8x ≥ 15
122. 5x − 6 + 3x < 2 + 9x − 5
123. 3x − (x − 4) > x + 4
124. 3 (2x − 1) − 3 (x − 2) ≤ 2 (x + 4)
125. 2 − 5 (x − 4) > 12
126. 3x − 5 (x − 2) ≥ 11 − 5x
127. −1 < 2x + 5 ≤ 11
1 7
128. −2 ≤ x− ≤2
4 2
129. 5x + 3 < −2 or 6x − 5 ≥ 7
130. 20 − 3x ≤ 5 or 5 − 2x ≥ 25
SAMPLE EXAM
2
1. Evaluate b − 4ac, where a = −1, b = −2 , and c = 1
2
.
Simplify.
3. 5 − 2 (4x − 1)
1 2 1 3
4.
4
x− 3
y+ 2
x− 5
y
6. 3x − (x 2 + 5x − 1) + (x 2 − x + 4)
Solve.
7. 2 − 5x = 27
1 3 1
8.
2
x− 4
=− 8
9. 5x − 7 = 3x − 5
10. 3 (y − 3) − (4y + 2) = 1
11. 5 (x − 2) − 3 (x + 2) = 2x − 3
5 n
12.
8
= 32
3 6
13.
n+1
=− 4
Solve and graph the solution set. In addition, present the solution set in interval
notation.
15. 2x + 3 > 23
16. 5 (−2x + 1) ≤ 35
18. −9 ≤ 3 (x + 4) ≤ 21
19. 6 (x − 13 ) < −2 or 1
5
(x + 10) ≥ 3
20. An algebra student earns 75, 79, and 89 points on the first three quizzes.
What must she score on the fourth quiz to earn an average of at least 80?
21. The sum of three consecutive odd integers is 117. Find the integers.
22. The length of a rectangle is 6 inches less than twice the width. If the
perimeter measures 39 inches, then find the dimensions of the rectangle.
23. Millie invested her $5,350 savings in two accounts. One account earns 5%
annual interest and the other earns 6.2% in annual interest. If she earned
$317.30 simple interest in 1 year, then how much was in each account?
24. Because of traffic, Joe was only able to drive an average of 42 miles per
hour on the trip to a conference. He was able to average 63 miles per hour
on the return trip and it took 1 hour less time. How long did it take Joe to
drive home from the conference?
1: −1
3: −1/10
5: −6
7: 24
9: $180
13: −15x + 10
15: 3x 2 − 6x + 24
17: −3a + 2b − 5c + 1
19: 2a − 2b
1 1
21:
2
xy + 4
25: −64x + 17
27: 6x 2 − 9x + 9
31: −3x −2
33: No
35: Yes
37: 2
39: 32
41: −7
43: −3/7
45: 3
47: −21
49: 7/5
51: 5
53: 4
55: 12
57: 2
59: 3/8
61: Ø
63: 8
65: 7/8
67: R
69: 11/3
y−1
71: x = 9
P−2w
73: l = 2
75: 12, 27
87: 6
89: 11/5
91: 5/6
93: 2/3
95: 12, 15
97: 3½ teaspoons
99: t = 4, r = 4/3
101: 18 feet
103: (−∞, − 1)
105: [0, ∞)
107: [− 1
2
, 3
2 )
111: x <3
113: −2 <x<2
115: x < 1 or x ≥ 3
119: x ≤ −1 ; (−∞, − 1]
1: 6
3: −8x +7
5: 2ab + 2a + b
7: −5
9: 1
11: Ø
13: −3
21: The three odd integers are 37, 39, and 41.
23: Millie invested $1,200 in the account earning 5% annual interest and
$4,150 in the account earning 6.2%.
390
Chapter 3 Graphing Lines
LEARNING OBJECTIVES
The rectangular coordinate system1 consists of two real number lines that
intersect at a right angle. The horizontal number line is called the x-axis2, and the
vertical number line is called the y-axis3. These two number lines define a flat
surface called a plane4, and each point on this plane is associated with an ordered
pair5 of real numbers (x, y). The first number is called the x-coordinate, and the
second number is called the y-coordinate. The intersection of the two axes is
known as the origin6, which corresponds to the point (0, 0).
391
Chapter 3 Graphing Lines
An ordered pair (x, y) represents the position of a point relative to the origin. The x-
coordinate represents a position to the right of the origin if it is positive and to the
left of the origin if it is negative. The y-coordinate represents a position above the
origin if it is positive and below the origin if it is negative. Using this system, every
position (point) in the plane is uniquely identified. For example, the pair (2, 3)
denotes the position relative to the origin as shown:
Example 1: Plot the ordered pair (−3, 5) and determine the quadrant in which it
lies.
Ordered pairs with 0 as one of the coordinates do not lie in a quadrant; these points
are on one axis or the other (or the point is the origin if both coordinates are 0).
Also, the scale indicated on the x-axis may be different from the scale indicated on
the y-axis. Choose a scale that is convenient for the given situation.
Example 2: Plot this set of ordered pairs: {(4, 0), (−6, 0), (0, 3), (−2, 6), (−4, −6)}.
Solution: Each tick mark on the x-axis represents 2 units and each tick mark on the
y-axis represents 3 units.
Example 3: Plot this set of ordered pairs: {(−6, −5), (−3, −3), (0, −1), (3, 1), (6, 3)}.
Solution:
In this example, the points appear to be collinear9, or to lie on the same line. The
entire chapter focuses on finding and expressing points with this property.
Try this! Plot the set of points {(5, 3), (−3, 2), (−2, −4), (4, −3)} and indicate in which
quadrant they lie. ([Link: Click here for printable graph paper in PDF.])
Answer:
Video Solution
9. Describes points that lie on the Graphs are used in everyday life to display data visually. A line graph10 consists of a
same line. set of related data values graphed on a coordinate plane and connected by line
10. A set of related data values
segments. Typically, the independent quantity, such as time, is displayed on the x-
graphed on a coordinate plane axis and the dependent quantity, such as distance traveled, on the y-axis.
and connected by line
segments.
Example 4: The following line graph shows the number of mathematics and
statistics bachelor’s degrees awarded in the United States each year since 1970.
a. How many mathematics and statistics bachelor’s degrees were awarded in 1975?
b. In which years were the number of mathematics and statistics degrees awarded
at the low of 11,000?
Solution:
a. The scale on the x-axis represents time since 1970, so to determine the number of
degrees awarded in 1975, read the y-value of the graph at x = 5.
b. To find the year a particular number of degrees was awarded, first look at the y-
axis. In this case, 11,000 degrees is represented by 11 on the y-axis; look to the right
to see in which years this occurred.
The y-value of 11 occurs at two data points, one where x = 10 and the other where x
= 30. These values correspond to the years 1980 and 2000, respectively.
Answers:
a. In the year 1975, 18,000 mathematics and statistics degrees were awarded.
b. In the years 1980 and 2000, the lows of 11,000 mathematics and statistics degrees
were awarded.
Distance Formula
Frequently you need to calculate the distance between two points in a plane. To do
this, form a right triangle using the two points as vertices of the triangle and then
11. Given any right triangle with
legs measuring a and b units apply the Pythagorean theorem. Recall that the Pythagorean theorem11 states that
and hypotenuse measuring c if given any right triangle with legs measuring a and b units, then the square of the
units, then a2 + b2 = c2. measure of the hypotenuse c is equal to the sum of the squares of the legs:
Solution: Form a right triangle by drawing horizontal and vertical lines through
the two points. This creates a right triangle as shown below:
The length of leg b is calculated by finding the distance between the x-values of the
given points, and the length of leg a is calculated by finding the distance between
the given y-values.
Next, use the Pythagorean theorem to find the length of the hypotenuse.
d = √(x 2 − x 1 )2 + (y 2 − y 1 )
(x 2 , y 2 ), calculate the ⎯⎯⎯⎯⎯⎯⎯⎯⎯⎯⎯⎯⎯⎯⎯⎯⎯⎯⎯⎯⎯⎯⎯⎯⎯⎯⎯⎯⎯⎯⎯⎯⎯⎯⎯⎯2⎯
distance d between them using
d = √(x 2 − x 1 )2 + (y 2 − y 1 ) .
the formula
⎯⎯⎯⎯⎯⎯⎯⎯⎯⎯⎯⎯⎯⎯⎯⎯⎯⎯⎯⎯⎯⎯⎯⎯⎯⎯⎯⎯⎯⎯⎯⎯⎯⎯⎯⎯2⎯
Example 6: Calculate the distance between (−3, −1) and (−2, 4).
It is a good practice to include the formula in its general form as a part of the
written solution before substituting values for the variables. This improves
readability and reduces the chance for errors.
⎯⎯⎯⎯
Answer: √26 units
Try this! Calculate the distance between (−7, 5) and (−1, 13).
Answer: 10 units
Video Solution
Example 7: Do the three points (1, −1), (3, −3), and (3, 1) form a right triangle?
Solution: The Pythagorean theorem states that having side lengths that satisfy the
property a2 + b2 = c2 is a necessary and sufficient condition of right triangles. In
other words, if you can show that the sum of the squares of the leg lengths of the
triangle is equal to the square of the length of the hypotenuse, then the figure must
be a right triangle. First, calculate the length of each side using the distance
formula.
Answer: Yes, the three points form a right triangle. In fact, since two of the legs are
equal in length, the points form an isosceles right triangle.
Midpoint Formula
The point that bisects the line segment formed by two points, (x 1 , y 1 ) and
(x 2 , y 2 ), is called the midpoint13 and is given by the following formula:
The midpoint is an ordered pair formed by finding the average of the x-values and
the average of the y-values of the given points.
Example 8: Calculate the midpoint between (−1, −2) and (7, 4).
Solution: First, calculate the average of the x- and y-values of the given points.
( 2 )
an ordered pair given by
x 1 +x 2 y 1 +y 2
2
, .
Next, form the midpoint as an ordered pair using the averaged coordinates.
Answer: (3, 1)
To verify that this is indeed the midpoint, calculate the distance between the two
given points and verify that the result is equal to the sum of the two equal distances
from the endpoints to this midpoint. This verification is left to the reader as an
exercise.
Try this! Find the midpoint between (−6, 5) and (6, −11).
Video Solution
KEY TAKEAWAYS
= √(x 2 − x 1 )2 + (y 2 − y 1 ) , is derived
then the triangle must be a right triangle.
⎯⎯⎯⎯⎯⎯⎯⎯⎯⎯⎯⎯⎯⎯⎯⎯⎯⎯⎯⎯⎯⎯⎯⎯⎯⎯⎯⎯⎯⎯⎯⎯⎯⎯⎯⎯2⎯
• The distance formula, d
from the Pythagorean theorem and gives us the distance between any
two points, (x 1 , y 1 ) and (x 2 , y 2 ), in a rectangular coordinate plane.
TOPIC EXERCISES
1.
2.
3.
4.
5.
6.
16. {(−1.2, −1.2), (−0.3, −0.3), (0, 0), (0.6, 0.6), (1.2, 1.2)}
17. (−3, 2)
18. (5, 7)
23. (− 1
8
, − 58 )
24. ( 34 , − 14 )
The average price of a gallon of regular unleaded gasoline in US cities is given in the
following line graph. Use the graph to answer the following questions.
Source: Bureau of
Labor Statistics.
29. What was the average price of a gallon of unleaded gasoline in 2004?
30. What was the average price of a gallon of unleaded gasoline in 1976?
31. In which years were the average price of a gallon of unleaded gasoline
$1.20?
32. What is the price increase of a gallon of gasoline from 1980 to 2008?
33. What was the percentage increase in the price of a gallon of unleaded
gasoline from 1976 to 1980?
34. What was the percentage increase in the price of a gallon of unleaded
gasoline from 2000 to 2008?
The average price of all-purpose white flour in US cities from 1980 to 2008 is given in
the following line graph. Use the graph to answer the questions that follow.
Source: Bureau of
Labor Statistics.
35. What was the average price per pound of all-purpose white flour in 2000?
36. What was the average price per pound of all-purpose white flour in 2008?
37. In which year did the price of flour average $0.25 per pound?
38. In which years did the price of flour average $0.20 per pound?
39. What was the percentage increase in flour from the year 2000 to 2008?
40. What was the percentage increase in flour from the year 1992 to 2000?
41. The percentage of total high school graduates who enrolled in college.
Year Percentage
1969 36%
1979 40%
1989 47%
1999 42%
Exam Temperature
8:00 am 60
12:00 pm 72
4:00 pm 75
8:00 pm 67
12:00 am 60
4:00 am 55
Calculate the area of the shape formed by connecting the following set of vertices.
58. (−
3)
and ( )
1 1 5 11
2
, 2
, − 3
59. (−
3)
and (1, − 13 )
1 2
3
,
60. ( , − 34 )and ( 32 , 4)
1 1
2
For each problem, show that the three points form a right triangle.
Isosceles triangles have two legs of equal length. For each problem, show that the
following points form an isosceles triangle.
Calculate the area and the perimeter of the triangles formed by the following set of
vertices.
93. (− , − 13 )and ( 32 , 3)
1 7
2
94. ( , − 23 )and ( 18 , − 12 )
3
4
95. (
4)
and (− , − 32 )
5 1 1
3
, 6
96. (− , − 52 )and ( 10 , − 14 )
1 7
5
97. Given the right triangle formed by the vertices (0, 0), (6, 0), and (6, 8),
show that the midpoints of the sides form a right triangle.
98. Given the isosceles triangle formed by the vertices (−10, −12), (0, 12), and
(10, −12), show that the midpoints of the sides also form an isosceles
triangle.
99. Calculate the area of the triangle formed by the vertices (−4, −3), (−1, 1),
and (2, −3). (Hint: The vertices form an isosceles triangle.)
100. Calculate the area of the triangle formed by the vertices (−2, 1), (4, 1),
and (1, −5).
101. Research and discuss the life and contributions to mathematics of René
Descartes.
102. Research and discuss the history of the right triangle and the
Pythagorean theorem.
104. Explain why you cannot use a ruler to calculate distance on a graph.
105. How do you bisect a line segment with only a compass and a
straightedge?
ANSWERS
1: A: (3, 5); B: (−2, 3); C: (−5, 0); D: (1, −3); E: (−3, −4)
3: A: (0, 6); B: (−4, 3); C: (−8, 0); D: (−6, −6); E: (8, −9)
5: A: (−10, 25); B: (30, 20); C: (0, 10); D: (15, 0); E: (25, −10)
7:
9:
11:
13:
15:
17: QII
19: QIII
21: QII
23: QIII
25: QIV
27: QII
29: $1.80
33: 100%
35: $0.30
37: 1992
39: 67%
41:
51: 5 units
53: 13 units
55: 15 units
⎯⎯⎯⎯
61: √ 10 units
⎯⎯⎯⎯
63: 2√ 17 units
⎯⎯⎯⎯
65: 3√ 13 units
69: Proof
71: Proof
73: Proof
75: Proof
77: Proof
⎯⎯
81: Perimeter: 8 + 4√ 2 units; area: 8 square units
83: (−4, 2)
85: (0, 0)
87: (2, 0)
93: (1/2, 1)
LEARNING OBJECTIVES
Solution: Substitute the x- and y-values into the equation to determine whether the
ordered pair produces a true statement.
420
Chapter 3 Graphing Lines
Example 2: Are ( 1
2
, − 3)and (−5, 14) solutions to y = 2x − 4 ?
Solution: Substitute the x-values and simplify to see if the corresponding y-values
are obtained.
Answer: ( 1
2
, − 3)is a solution, and (−5, 14) is not.
Answer: Yes
Video Solution
When given linear equations with two variables, we can solve for one of the
variables, usually y, and obtain an equivalent equation as follows:
Written in this form, we can see that y depends on x. Here x is the independent
variable15 and y is the dependent variable16.
Example 3: Find ordered pair solutions to the equation 5x − y = 14 with the given
x-values {−2, −1, 0, 4, 6}.
Answer: {(−2, −24), (−1, −19), (0, −14), (4, 6), (6, 16)}
In the previous example, certain x-values are given, but that is not always going to
be the case. When treating x as the independent variable, we can choose any values
for x and then substitute them into the equation to find the corresponding y-values.
This method produces as many ordered pair solutions as we wish.
Next, choose any set of x-values. Usually we choose some negative values and some
positive values. In this case, we will find the corresponding y-values when x is {−2,
−1, 0, 1, 2}. Make the substitutions required to fill in the following table (often
referred to as a t-chart):
Answer: {(−2, 11), (−1, 8), (0, 5), (1, 2), (2, −1)}. Since there are infinitely many
ordered pair solutions, answers may vary depending on the choice of values for the
independent variable.
Answer: {(−2, −11), (−1, −6), (0, −1), (1, 4), (2, 9)} (answers may vary)
Video Solution
Since the solutions to linear equations are ordered pairs, they can be graphed using
the rectangular coordinate system. The set of all solutions to a linear equation can
be represented on a rectangular coordinate plane using a straight line connecting at
least two points; this line is called its graph17. To illustrate this, plot five ordered
pair solutions, {(−2, 11), (−1, 8), (0, 5), (1, 2), (2, −1)}, to the linear equation
6x + 2y = 10 .
Notice that the points are collinear; this will be the case for any linear equation.
Draw a line through the points with a straightedge, and add arrows on either end to
indicate that the graph extends indefinitely.
Solution:
Step2: Choose at least two x-values and find the corresponding y-values. In this
section, we will choose five real numbers to use as x-values. It is a good practice to
choose 0 and some negative numbers, as well as some positive numbers.
Five ordered pair solutions are {(−2, −6), (−1, −4), (0, −2), (1, 0), (2, 2)}
Step 3: Choose an appropriate scale, plot the points, and draw a line through them
using a straightedge. In this case, choose a scale where each tick mark on the y-axis
represents 2 units because all the y-values are multiples of 2.
Answer:
It will not always be the case that y can be solved in terms of x with integer
coefficients. In fact, the coefficients often turn out to be fractions.
Solution:
Remember that you can choose any real number for the independent variable x, so
choose wisely here. Since the denominator of the coefficient of the variable x is 2,
you can avoid fractions by choosing multiples of 2 for the x-values. In this case,
choose the set of x-values {−6, −4, −2, 0, 2} and find the corresponding y-values.
Five solutions are {(−6, −10), (−4, −5), (−2, 0), (0, 5), (2, 10)}. Here we choose to scale
the x-axis with multiples of 2 and the y-axis with multiples of 5.
Answer:
Answer: {(−2, 4), (0, 3), (2, 2), (4, 1), (6, 0)}
Video Solution
Solution: Since the given equation does not have a variable x, we can rewrite it with
a 0 coefficient for x.
Choose any five values for x and see that the corresponding y-value is always −2.
We now have five ordered pair solutions to plot {(−2, −2), (−1, −2), (0, −2), (1, −2),
(2, −2)}.
Answer:
When the coefficient for the variable x is 0, the graph is a horizontal line. In
general, the equation for a horizontal line18 can be written in the form y = k ,
where k represents any real number.
Solution: Since the given equation does not have a variable y, rewrite it with a 0
coefficient for y.
Choose any five values for y and see that the corresponding x-value is always 3.
We now have five ordered pair solutions to plot: {(3, −2), (3, −1), (3, 0), (3, 1), (3, 2)}.
Answer:
When the coefficient for the variable y is 0, the graph is a vertical line. In general,
the equation for a vertical line19 can be written as x = k , where k represents any
real number.
Try this! Graph y = 5 and x = −2 on the same set of axes and determine where
they intersect.
Answer: (−2, 5)
Video Solution
KEY TAKEAWAYS
TOPIC EXERCISES
1. 5x − 2y = 4 ; (−1, 1)
2. 3x − 4y = 10 ; (2, −1)
3. −3x + y = −6 ; (4, 6)
5. −x + y = −7 ; (5, −2)
6. 9x − 3y = 6 ; (0, −2)
1 1 1
7.
2
x+ 3
y=− 6
; (1, −2)
3 1
8.
4
x− 2
y = −1; (2, 1)
9. 4x − 3y = 1 ; ( 12 , 1
3 )
5 (5 )
9
10. −10x + 2y = − ; 1 , 1
10
1
11. y = x + 3 ; (6, 3)
3
5
14. y =− x + 1 ; (8, −5)
8
4 (
15. y =− 1
2
x+ 3
; − 1
2
, 1)
2 (2
16. y =− 1
3
x− 1
; 1 , − 23 )
17. y = 2 ; (−3, 2)
20. x = 0 ; (1, 0)
3 1
23. y =− x+ ; {−2, 0, 2}
4 2
25. y = −4 ; {−3, 0, 3}
1 3
26. y = x+ ; {−1/4, 0, 1/4}
2 4
27. 2x − 3y = 1 ; {0, 1, 2}
3 1
31.
5
x+ 10
y = 2; {−15, −10, −5}
1
33. y = x − 1 ; {−5, 0, 5}
2
3
34. y =− x + 2 ; {0, 2, 4}
4
3 1
38.
5
x+ 10
y = 2; {−20, −10, −5}
Given the set of x-values {−2, −1, 0, 1, 2}, find the corresponding y-values and graph
them.
39. y =x+1
40. y = −x + 1
41. y = 2x − 1
42. y = −3x + 2
43. y = 5x − 10
44. 5x + y = 15
45. 3x −y=9
46. 6x − 3y = 9
47. y = −5
48. y =3
49. y = 2x − 1
50. y = −5x + 3
51. y = −4x + 2
52. y = 10x − 20
1
53. y =− x+2
2
1
54. y = x−1
3
2
55. y = x−6
3
2
56. y =− x+2
3
57. y =x
58. y = −x
60. x + 5y = 5
61. 6x −y=2
62. 4x + y = 12
63. −x + 5y = 0
64. x + 2y = 0
1
65.
10
x−y=3
3
66.
2
x + 5y = 30
67. y =4
68. y = −10
69. x =4
70. x = −1
71. y =0
72. x =0
3
73. y = 4
5
74. x =− 4
75. Graph the lines y = −4 and x = 2 on the same set of axes. Where do
they intersect?
76. Graph the lines y = 5 and x = −5 on the same set of axes. Where do
they intersect?
3
79. y =− x+6
5
3
80. y = x−3
5
81. y = −3
82. x = −5
83. 3x − 2y = 6
ANSWERS
1: No
3: Yes
5: Yes
7: Yes
9: Yes
11: No
13: Yes
15: Yes
17: Yes
19: Yes
39:
41:
43:
45:
47:
49:
51:
53:
55:
57:
59:
61:
63:
65:
67:
69:
71:
73:
75:
77: y =0
79:
81:
83:
LEARNING OBJECTIVES
The x-intercept20 is the point where the graph of a line intersects the x-axis. The y-
intercept21 is the point where the graph of a line intersects the y-axis. These points
have the form (x, 0) and (0, y), respectively.
To find the x- and y-intercepts algebraically, use the fact that all x-intercepts have a
20. The point (or points) where a y-value of zero and all y-intercepts have an x-value of zero. To find the y-intercept,
graph intersects the x-axis, set x = 0 and determine the corresponding y-value. Similarly, to find the x-
expressed as an ordered pair intercept, set y = 0 and determine the corresponding x-value.
(x, 0).
447
Chapter 3 Graphing Lines
Hence the y-intercept is (0, 6). Note that this linear equation is graphed above.
Keep in mind that the intercepts are ordered pairs and not numbers. In other
words, the x-intercept is not x = 2 but rather (2, 0). In addition, not all graphs
necessarily have both intercepts: for example,
The horizontal line graphed above has a y-intercept of (0, −2) and no x-intercept.
The vertical line graphed above has an x-intercept (3, 0) and no y-intercept.
Video Solution
Since two points determine a line, we can use the x- and y-intercepts to graph linear
equations. We have just outlined an easy method for finding intercepts; now we
outline the steps for graphing lines using the intercepts.
Solution:
Step 2: Plot the intercepts and draw the line through them. Use a straightedge to
create a nice straight line. Add an arrow on either end to indicate that the line
continues indefinitely in either direction.
Answer:
Next, graph the two points and draw a line through them with a straightedge.
Answer:
Solution:
Here the x- and y-intercepts are actually the same point, the origin. We will need at
least one more point so that we can graph the line. Choose any value for x and
determine the corresponding value for y.
Use the ordered pair solutions (0, 0), (−1, 2), and (1, −2) to graph the line.
Answer:
To summarize, any linear equation can be graphed by finding two points and
connecting them with a line drawn with a straightedge. Two important and useful
points are the x- and y-intercepts; find these points by substituting y = 0 and x = 0,
respectively. This method for finding intercepts will be used throughout our study
of algebra.
Video Solution
The x- and y-intercepts are important points on any graph. This chapter will focus
on the graphs of linear equations. However, at this point, we can use these ideas to
determine intercepts of nonlinear graphs. Remember that intercepts are ordered
pairs that indicate where the graph intersects the axes.
Solution: We see that the graph intersects the x-axis in two places. This graph has
two x-intercepts, namely, (−4, 0) and (2, 0). Furthermore, the graph intersects the y-
axis in one place. The only y-intercept is (0, −3).
In our study of algebra, we will see that some graphs have many intercepts. Also, we
will see that some graphs do not have any.
Solution: This is a graph of a circle; we can see that it does not intersect either axis.
Therefore, this graph does not have any intercepts.
Answer: None
KEY TAKEAWAYS
• Since two points determine any line, we can graph lines using the x- and
y-intercepts.
• To find the x-intercept, set y = 0 and solve for x.
• To find the y-intercept, set x = 0 and solve for y.
• This method of finding x- and y-intercepts will be used throughout our
study of algebra because it works for any equation.
• To graph a line, find the intercepts, if they exist, and draw a straight line
through them. Use a straightedge to create the line and include arrows
on either end to indicate that the line extends infinitely in either
direction.
• Horizontal and vertical lines do not always have both x- and y-
intercepts.
TOPIC EXERCISES
Part A: Intercepts
1.
2.
3.
4.
5.
6.
7. 5x − 4y = 20
8. −2x + 7y = −28
9. x −y=3
10. −x +y=0
11. 3x − 4y = 1
12. −2x + 5y = 3
13. 1
4
x− 1
3
y=1
14. − 2
5
x+ 3
4
y=2
15. y =6
16. y = −3
17. x =2
18. x = −1
19. y = mx + b
20. ax + by = c
21. 3x + 4y = 12
22. −2x + 3y = 6
23. 5x − 2y = 10
24. −4x − 8y = 16
1 1
25. − x+ y=1
2 3
3 1
26.
4
x− 2
y = −3
5
27. 2x − y = 10
2
7
28. 2x − y = −14
3
29. 4x − y = −8
30. 6x −y=6
31. – x + 2y = 1
32. 3x + 4y = 6
33. 2x + y = −1
34. −2x + 6y = 3
36. −25x + 3y = 75
37. 4x + 2y = 0
38. 3x −y=0
39. −12x + 6y = −4
40. 3x + 12y = −4
41. y = 2x + 4
42. y = −x + 3
1
43. y = x+1
2
2
44. y = x−3
3
2
45. y =− x+1
5
5 5
46. y =− x−
8 4
7 7
47. y =− x−
8 2
3
48. y = −x + 2
49. y =3
3
50. y = 2
51. x =5
52. x = −2
53. y = 5x
54. y = −x
55.
56.
57.
58.
59.
60.
61.
62.
63.
64.
68. How many intercepts can a circle have? Draw circles showing all possible
numbers of intercepts.
69. Research and post the definitions of line segment, ray, and line. Why are
the arrows important?
ANSWERS
21:
23:
25:
27:
29:
31:
33:
35:
37:
39:
41:
43:
45:
47:
49:
51:
53:
61: x-intercepts: (−3, 0), (0, 0), (2, 0); y-intercept: (0, 0)
63: x-intercepts: (−4, 0), (4, 0); y-intercepts: (0, −4), (0, 4)
LEARNING OBJECTIVES
Slope
The steepness of any incline can be measured as the ratio of the vertical change to
the horizontal change. For example, a 5% incline can be written as 5/100, which
means that for every 100 feet forward, the height increases 5 feet.
In mathematics, we call the incline of a line the slope22 and use the letter m to
denote it. The vertical change is called the rise23 and the horizontal change is called
the run24.
470
Chapter 3 Graphing Lines
Solution: From the given points on the graph, count 3 units down and 4 units right.
Answer: m = − 34
Here we have a negative slope, which means that for every 4 units of movement to
the right, the vertical change is 3 units downward. There are four geometric cases
for the value of the slope.
Reading the graph from left to right, we see that lines with an upward incline have
positive slopes and lines with a downward incline have negative slopes.
The slope of a horizontal line is 0. If the line is vertical, then the run is 0:
Answer: m = 23
Video Solution
Calculating the slope can be difficult if the graph does not have points with integer
coordinates. Therefore, we next develop a formula that allows us to calculate the
slope algebraically. Given any two points (x 1 , y 1 ) and (x 2 , y 2 ), we can obtain the
rise and run by subtracting the corresponding coordinates.
This leads us to the slope formula25. Given any two points (x 1 , y 1 ) and (x 2 , y 2 ),
25. Given two points (x 1 , y 1 ) and the slope is given by
(x 2 , y 2 ), then the slope of the
line is given by the formula
rise y 2 −y 1
m= run = x 2 −x 1.
Example 2: Find the slope of the line passing through (−3, −5) and (2, 1).
Solution: Given (−3, −5) and (2, 1), calculate the difference of the y-values divided by
the difference of the x-values. Since subtraction is not commutative, take care to be
consistent when subtracting the coordinates.
Answer: m = 65
We can graph the line described in the previous example and verify that the slope is
6/5.
Certainly the graph is optional; the beauty of the slope formula is that we can
obtain the slope, given two points, using only algebra.
Example 3: Find the slope of the line passing through (−4, 3) and (−1, −7).
Solution:
Answer: m = − 10
3
When using the slope formula, take care to be consistent since order does matter.
You must subtract the coordinates of the first point from the coordinates of the
second point for both the numerator and the denominator in the same order.
Example 4: Find the slope of the line passing through (7, −2) and (−5, −2).
Solution:
Answer: m = 0. As an exercise, plot the given two points and verify that they lie on
a horizontal line.
Example 5: Find the slope of the line passing through (−4, −3) and (−4, 5).
Solution:
Answer: The slope m is undefined. As an exercise, plot the given two points and
verify that they lie on a vertical line.
Try this! Calculate the slope of the line passing through (−2, 3) and (5, −5).
Answer: m = − 87
Video Solution
When considering the slope as a rate of change it is important to include the correct
units.
Example 6: A Corvette Coupe was purchased new in 1970 for about $5,200 and
depreciated in value over time until it was sold in 1985 for $1,300. At this point, the
car was beginning to be considered a classic and started to increase in value. In the
year 2000, when the car was 30 years old, it sold at auction for $10,450. The
following line graph depicts the value of the car over time.
a. Determine the rate at which the car depreciated in value from 1970 to 1985.
b. Determine the rate at which the car appreciated in value from 1985 to 2000.
Solution: Notice that the value depends on the age of the car and that the slope
measures the rate in dollars per year.
a. The slope of the line segment depicting the value for the first 15 years is
Answer: The value of the car depreciated $260 per year from 1970 to 1985.
b. The slope of the line segment depicting the value for the next 15 years is
Answer: The value of the car appreciated $610 per year from 1985 to 2000.
To this point, we have learned how to graph lines by plotting points and by using
the x- and y-intercepts. In addition, we have seen that we need only two points to
graph a line. In this section, we outline a process to easily determine two points
using the y-intercept and the slope. The equation of any nonvertical line can be
written in slope-intercept form26 y = mx + b . In this form, we can identify the
slope, m, and the y-intercept, (0, b).
It is not always the case that the linear equation is given in slope-intercept form.
When it is given in standard form, you have to first solve for y to obtain slope-
intercept form.
Solution: Begin by solving for y. To do this, apply the properties of equality to first
isolate 5y and then divide both sides by 5.
Starting from the point (0, 6), use the slope to mark another point 3 units down and
5 units to the right.
It is not necessary to check that the second point, (5, 3), solves the original linear
equation. However, we do it here for the sake of completeness.
Marking off the slope in this fashion produces as many ordered pair solutions as we
desire. Notice that if we mark off the slope again, from the point (5, 3), then we
obtain the x-intercept, (10, 0).
Example 9: Graph: −x + 2y = 4 .
Solution: In this example, we outline the general steps for graphing a line using
slope-intercept form.
Step 3: Plot the y-intercept and use the slope to find another ordered pair solution.
Starting from the y-intercept, mark off the slope and identify a second point. In this
case, mark a point after a rise of 1 unit and a run of 2 units.
Step 4: Draw the line through the two points with a straightedge.
Answer:
In this example, we notice that we could get the x-intercept by marking off the
slope in a different but equivalent manner. Consider the slope as the ratio of two
negative numbers as follows:
We could obtain another point on the line by marking off the equivalent slope down
1 unit and left 2 units. We do this twice to obtain the x-intercept, (−4, 0).
Marking off the slope multiple times is not necessarily always going to give us the x-
intercept, but when it does, we obtain a valuable point with little effort. In fact, it is
a good practice to mark off the slope multiple times; doing so allows you to obtain
more points on the line and produce a more accurate graph.
We can see that the x-value of the x-intercept is a mixed number between 2 and 3.
To algebraically find x-intercepts, recall that we must set y = 0 and solve for x.
Answer:
Answer:
Video Solution
KEY TAKEAWAYS
• Slope measures the steepness of a line as rise over run. A positive rise
denotes a vertical change up, and a negative rise denotes a vertical
change down. A positive run denotes a horizontal change right, and a
negative run denotes a horizontal change left.
• Horizontal lines have a slope of zero, and vertical lines have undefined
slopes.
• Given any two points on a line, we can algebraically calculate the slope
rise y 2 −y 1
using the slope formula, m = run = x 2 −x 1 .
• Any nonvertical line can be written in slope-intercept form,
y = mx + b , from which we can determine, by inspection, the slope
m and y-intercept (0, b).
• If we know the y-intercept and slope of a line, then we can easily graph
it. First, plot the y-intercept, and from this point use the slope as rise
over run to mark another point on the line. Finally, draw a line through
these two points with a straightedge and add an arrow on either end to
indicate that it extends indefinitely.
• We can obtain as many points on the line as we wish by marking off the
slope multiple times.
TOPIC EXERCISES
Part A: Slope
1.
2.
3.
4.
5.
6.
15. ( 12 , − 13 ) and (− 1
2
, 2
3 )
16. (− 3
4
, 3
2 ) and ( 4 , − 2 )
1 1
17. (− 1
3
, 5
8 ) and ( 2 , − 4 )
1 3
18. (− 3
5
, − 32 ) and ( 101 , 4
5 )
23. A roof drops 4 feet for every 12 feet forward. Determine the slope of the
roof.
24. A road drops 300 feet for every 5,280 feet forward. Determine the slope of
the road.
25. The following graph gives the US population of persons 65 years old and
over. At what rate did this population increase from 2000 to 2008?
Source: US Census
Bureau.
26. The following graph gives total consumer credit outstanding in the
United States. At what rate did consumer credit increase from 2002 to 2008?
Source: US Census
Bureau.
27. A commercial van was purchased new for $20,000 and is expected to be
worth $4,000 in 8 years. Determine the rate at which the van depreciates in
value.
28. A commercial-grade copy machine was purchased new for $4,800 and
will be considered worthless in 6 years. Determine the rate at which the
copy machine depreciates in value.
29. Find y if the slope of the line passing through (−2, 3) and (4, y) is 12.
30. Find y if the slope of the line passing through (5, y) and (6, −1) is 10.
31. Find y if the slope of the line passing through (5, y) and (−4, 2) is 0.
32. Find x if the slope of the line passing through (−3, 2) and (x, 5) is
undefined.
Express the given linear equation in slope-intercept form and identify the slope and
y-intercept.
33. 6x − 5y = 30
34. −2x + 7y = 28
35. 9x − y = 17
36. x − 3y = 18
37. 2x − 3y = 0
38. −6x + 3y = 0
39. 2
3
x− 5
4
y = 10
40. − 4
3
x+ 1
5
y = −5
1
41. m = and (0, −2)
3
2
42. m =− and (0, 4)
3
2
51. y = x−2
3
1
52. y =− x+1
3
53. y = −3x + 6
54. y = 3x + 1
3
55. y = x
5
3
56. y =− 7
x
57. y = −8
58. y =7
59. y = −x + 2
60. y =x+1
1 3
61. y = x+
2 2
3 5
62. y =− x+
4 2
63. 4x +y=7
64. 3x −y=5
65. 5x − 2y = 10
66. −2x + 3y = 18
67. x −y=0
68. x +y=0
1 1
69.
2
x− 3
y=1
2 1
70. − x+ y=2
3 2
71. 3x + 2y = 1
72. 5x + 3y = 1
73. On the same set of axes, graph the three lines, where y = 3
2
x + b and
b = {−2, 0, 2}.
74. On the same set of axes, graph the three lines, where y = mx + 1 and
m = {−1/2, 0, 1/2}.
75. Name three methods for graphing lines. Discuss the pros and cons of
each method.
76. Choose a linear equation and graph it three different ways. Scan the
work and share it on the discussion board.
78. How are equivalent fractions useful when working with slopes?
Δy
80. Research and discuss the alternative notation for slope: m = Δx
.
ANSWERS
7: −1/2
9: −1
11: 2
13: −8/3
15: −1
17: −33/20
19: 0
21: Undefined
23: −1/3
29: 75
31: 2
6
33: y = x − 6 ; slope: 6/5; y-intercept: (0, −6)
5
8
39: y = x − 8 ; slope: 8/15; y-intercept: (0, −8)
15
41:
43:
45:
47:
49:
51:
53:
55:
57:
59:
61:
63:
65:
67:
69:
71:
73:
LEARNING OBJECTIVES
Given the algebraic equation of a line, we are able to graph it in a number of ways.
In this section, we will be given a geometric description of a line and be asked to
find the algebraic equation. Finding the equation of a line can be accomplished in a
number of ways, the first of which makes use of slope-intercept form, y = mx + b .
If we know the slope, m, and the y-intercept, (0, b), we can construct the equation.
Example 1: Find the equation of a line with slope m = − 58 and y-intercept (0, 1).
Solution: The given y-intercept implies that b = 1. Substitute the slope m and the
y-value of the y-intercept b into the equation y = mx + b .
Answer: y = − 58 x + 1
Finding a linear equation is very straightforward if the slope and y-intercept are
given. This is certainly not always the case; however, the example demonstrates
502
Chapter 3 Graphing Lines
that the algebraic equation of a line depends on these two pieces of information. If
the graph is given, then we can often read it to determine the y-intercept and slope.
Solution: By reading the graph, we can see that the y-intercept is (0, 4), and thus
Furthermore, from the points (0, 4) to (4, 2), we can see that the rise is −2 units and
the run is 4 units.
Answer: y = − 12 x + 4
Often the y-intercept and slope will not be given or are not easily discernible from
the graph. For this reason, we will develop some algebraic techniques that allow us
to calculate these quantities.
Example 3: Find the equation of the line with slope m = − 23 passing through
(−6, 3).
For the ordered pair (−6, 3) to be a solution, it must solve the equation. Therefore,
we can use it to find b. Substitute the appropriate x- and y-values as follows:
After substituting the appropriate values, solve for the only remaining variable, b.
Answer: y = − 23 x − 1
Solution: Use the graph to determine the slope. From the points (−5, 2) to (−1, 0), we
can see that the rise between the points is −2 units and the run is 4 units. Therefore,
we calculate the slope as follows:
Now substitute the coordinates of one of the given points to find b. It does not
matter which one you choose. Here choose (−1, 0):
Answer: y = − 12 x − 12
As an exercise, substitute the coordinates of the point (−5, 2) to see that b will turn
out to be the same value. In fact, you can substitute any ordered pair solution of the
line to find b. We next outline an algebraic technique for finding the equation of a
nonvertical line passing through two given points.
Example 5: Find the equation of the line passing through (−4, −2) and (1, 3).
Step 1: Find the slope m. In this case, given two points, use the slope formula.
Step 2: Find b. To do this, substitute the coordinates of any given ordered pair
solution. Use (1, 3):
Step 3: Finish building the equation by substituting in the value for b. In this case,
we use b = 2.
Answer: y = x + 2
These three steps outline the process for finding the equation of any nonvertical
line in slope-intercept form. This is a completely algebraic method, but always keep
in mind the geometry behind the technique.
Note that the line has a y-intercept at (0, 2), with slope m = 1.
Example 6: Find the equation of the line passing through (−1, 3) and (5, 1).
Solution: First, find m, the slope. Given two points, use the slope formula as follows:
Finally, substitute b = 8
3
into the equation.
Answer: y = − 13 x + 83
Try this! Find the equation of the line passing through (−3, 4) and (6, −2).
Answer: y = − 23 x + 2
Video Solution
Given any point on a line and its slope, we can find the equation of that line. Begin
by applying the slope formula with a given point (x 1 , y 1 ) and a variable point
(x, y) .
Example 7: Find the equation of the line with slope m = 12 passing through (4, −1).
At this point, we must choose to present the equation of our line in either standard
form or slope-intercept form.
In this textbook, we will present our lines in slope-intercept form. This facilitates
future graphing.
Answer: y = 12 x − 3
Example 8: Find the equation of the line passing through (−5, 3) with slope
m = − 25.
Answer: y = − 25 x + 1
The geometric understanding is important because you will often be given graphs
from which you will need to determine a point on the line and the slope.
Solution: Between the points (1, 1) to (3, 0), we can see that the rise is −1 unit and
the run is 2 units. The slope of the line is m = rise −1 1
run = 2 = − .2 Use this and the
point (3, 0) to find the equation as follows:
Answer: y = − 12 x + 32
Example 10: Find the equation of the line passing through (−1, 1) and (7, −1).
Next, substitute into point-slope form using one of the given points; it does not
matter which point is used. Use m = − 14 and the point (−1, 1).
Answer: y = − 14 x + 34
Try this! Find the equation of the line passing through (4, −5) and (−4, 1).
Answer: y = − 34 x − 2
Video Solution
KEY TAKEAWAYS
• Given the graph of a line, you can determine the equation in two ways,
using slope-intercept form, y = mx + b , or point-slope form,
y − y 1 = m (x − x 1 ) .
• The slope and one point on the line is all that is needed to write the
equation of a line.
• All nonvertical lines are completely determined by their y-intercept and
slope.
• If the slope and y-intercept can be determined, then it is best to use
slope-intercept form to write the equation.
• If the slope and a point on the line can be determined, then it is best to
use point-slope form to write the equation.
TOPIC EXERCISES
1. 5x − 3y = 18
2. −6x + 2y = 12
3. x −y=5
4. −x +y=0
5. 4x − 5y = 15
6. −7x + 2y = 3
7. y =3
3
8. y =− 4
1 1
9.
5
x− 3
y = −1
5 3
10.
16
x+ 8
y=9
2 5 5
11. − x+ y=
3 2 4
1 3 1
12.
2
x− 4
y=− 2
Given the slope and y-intercept, determine the equation of the line.
18. m = 5; (0, 0)
19.
20.
21.
22.
23.
24.
27. m = 0; (−4, 3)
28. m = 3; (−2, 1)
33. m = 0; (3, 0)
59. m = 1; (0, 0)
Given the graph, use the point-slope formula to find the equation.
61.
62.
63.
64.
65.
66.
Use the point-slope formula to find the equation of the line passing through the two
points.
Part D: Applications
87. Joe has been keeping track of his cellular phone bills for the last two
months. The bill for the first month was $38.00 for 100 minutes of usage. The
bill for the second month was $45.50 for 150 minutes of usage. Find a linear
equation that gives the total monthly bill based on the minutes of usage.
88. A company in its first year of business produced 150 training manuals for
a total cost of $2,350. The following year, the company produced 50 more
manuals at a cost of $1,450. Use this information to find a linear equation
that gives the total cost of producing training manuals from the number of
manuals produced.
89. A corn farmer in California was able to produce 154 bushels of corn per
acre 2 years after starting his operation. Currently, after 7 years of
operation, he has increased his yield to 164 bushels per acre. Use this
information to write a linear equation that gives the total yield per acre
based on the number of years of operation, and use it to predict the yield for
next year.
90. A Webmaster has noticed that the number of registered users has been
steadily increasing since beginning an advertising campaign. Before starting
to advertise, he had 1,200 registered users, and after 3 months of advertising
he now has 1,590 registered users. Use this data to write a linear equation
that gives the total number of registered users, given the number of months
after starting to advertise. Use the equation to predict the number of users 7
months into the advertising campaign.
91. A car purchased new cost $22,000 and was sold 10 years later for $7,000.
Write a linear equation that gives the value of the car in terms of its age in
years.
92. An antique clock was purchased in 1985 for $1,500 and sold at auction in
1997 for $5,700. Determine a linear equation that models the value of the
clock in terms of years since 1985.
93. Discuss the merits and drawbacks of point-slope form and y-intercept
form.
ANSWERS
3: m = 1; (0, −5)
7: m = 0; (0, 3)
9: m = 3/5; (0, 3)
13: y = 1
2
x+5
15: y =− 2
3
x−4
17: y = −1
19: y = −x + 3
21: y = −1
23: y = 1
2
x
25: y = 2
3
x+8
27: y =3
29: y = −5x − 2
31: y =− 1
2
x+ 7
2
33: y =0
35: y =− 1
2
x+3
37: y = −3x + 1
2
39: y = 1
3
x−5
41: x = −6
43: y = −x
45: y = 1
2
x+1
47: y = 6x − 11
49: y = −3x + 9
51: y =− 3
4
x−3
53: y = −3x + 5
3
55: y = 10
57: y =− 3
5
x− 11
5
59: y =x
61: y = −2x + 5
63: y = 2
3
x+ 17
3
65: y =− 3
5
x− 2
5
67: y = 5
4
x+5
69: y = 2
3
x
71: y = −2x
73: y = 2
3
x+1
75: y = 2x
77: y = 1
5
x−5
79: y = −x + 7
15
81: y =− 1
4
x+ 1
2
83: y = −2x
85: y =7
LEARNING OBJECTIVES
Parallel lines28 are lines in the same plane that never intersect. Two nonvertical
lines in the same plane, with slopes m1 and m2 , are parallel if their slopes are the
same, m1 = m2 . Consider the following two lines:
529
Chapter 3 Graphing Lines
Perpendicular lines29 are lines in the same plane that intersect at right angles (90
degrees). Two nonvertical lines in the same plane, with slopes m1 and m2 , are
perpendicular if the product of their slopes is −1: m1 ⋅ m2 = −1. We can solve for
m1 and obtain m1 = − m1 . In this form, we see that perpendicular lines have
2
slopes that are negative reciprocals, or opposite reciprocals30. For example, if
given a slope
Geometrically, we note that if a line has a positive slope, then any perpendicular
line will have a negative slope. Furthermore, the rise and run between two
perpendicular lines are interchanged.
Perpendicular lines have slopes that are opposite reciprocals, so remember to find
the reciprocal and change the sign. In other words,
Solution: Since the given line is in slope-intercept form, we can see that its slope is
m = −5. Thus the slope of any line parallel to the given line must be the same,
m∥ = −5. The mathematical notation m∥ reads “m parallel.”
Answer: m∥ = −5
Solution: First, solve for y and express the line in slope-intercept form.
In this form, we can see that the slope of the given line is m = 37, and thus
m⊥ = − 73.
Answer: m⊥ = − 7
3
Answer: m⊥ = 1
3
Video Solution
We have seen that the graph of a line is completely determined by two points or one
point and its slope. Often you will be asked to find the equation of a line given some
geometric relationship—for instance, whether the line is parallel or perpendicular
to another line.
Example 3: Find the equation of the line passing through (6, −1) and parallel to
y = 12 x + 2.
Solution: Here the given line has slope m = 12 , and the slope of a line parallel is
m∥ = 12. Since you are given a point and the slope, use the point-slope form of a
line to determine the equation.
Answer: y = 12 x − 4
Through the point (6, −1) we found a parallel line, y = 12 x − 4, shown dashed.
Notice that the slope is the same as the given line, but the y-intercept is different. If
we keep in mind the geometric interpretation, then it will be easier to remember
the process needed to solve the problem.
Example 4: Find the equation of the line passing through (−1, −5) and perpendicular
to y = − 14 x + 2.
Answer: y = 4x − 1
It is not always the case that the given line is in slope-intercept form. Often you
have to perform additional steps to determine the slope. The general steps for
finding the equation of a line are outlined in the following example.
Example 5: Find the equation of the line passing through (8, −2) and perpendicular
to 6x + 3y = 1 .
Solution:
Step 1: Find the slope m. First, find the slope of the given line. To do this, solve for y
to change standard form to slope-intercept form, y = mx + b .
Step 2: Substitute the slope you found and the given point into the point-slope form
of an equation for a line. In this case, the slope is m⊥ = 12 and the given point is
(8, −2).
Answer: y = 12 x − 6
Example 6: Find the equation of the line passing through ( 72 , 1)and parallel to
2x + 14y = 7 .
The given line has the slope m = − 17 , and so m∥ = − 17. We use this and the point
( 2 , 1)in point-slope form.
7
Answer: y = − 17 x + 32
through (− 12 , 2).
Try this! Find the equation of the line perpendicular to x − 3y = 9 and passing
Answer: y = −3x + 12
Video Solution
Example 7: Find the equation of the line passing through (−3, −2) and perpendicular
to y = 4 .
If we draw the line perpendicular to the given horizontal line, the result is a vertical
line.
Equations of vertical lines look like x = k . Since it must pass through (−3, −2), we
conclude that x = −3 is the equation. All ordered pair solutions of a vertical line
must share the same x-coordinate.
Answer: x = −3
Written in this form, we see that the slope is m = 0 = 01. If we try to find the slope
of a perpendicular line by finding the opposite reciprocal, we run into a problem:
m⊥ = − 10, which is undefined. This is why we took care to restrict the definition to
two nonvertical lines. Remember that horizontal lines are perpendicular to vertical
lines.
KEY TAKEAWAYS
TOPIC EXERCISES
3
1. y =− x+8
4
1
2. y = x−3
2
3. y = 4x + 4
4. y = −3x + 7
5 1
5. y =− x−
8 2
7 3
6. y = x+
3 2
2
7. y = 9x − 5
1
8. y = −10x + 5
9. y =5
1
10. x =− 2
11. x −y=0
12. x +y=0
13. 4x + 3y = 0
14. 3x − 5y = 10
15. −2x + 7y = 14
16. −x − y = 15
1 1
17.
2
x− 3
y = −1
2 4
18. − x+ y=8
3 5
1 1
19. 2x − y=
5 10
4
20. − x − 2y = 7
5
2
y= x+3
{y =
3
21.
2
3
x−3
3
y= x−1
{y =
4
22.
4
3
x+3
y = −2x + 1
{y=
23. 1
2
x+8
{ y = 3x + 2
y = 3x − 2
24.
{ x = −2
y=5
25.
y=7
{y = −
26. 1
7
{ 5x + 3y = 9
3x − 5y = 15
27.
{ 3x + 3y = 2
x−y=7
28.
{ −x + 3y = −2
2x − 6y = 4
29.
{ 6x − 3y = −3
−4x + 2y = 3
30.
{ 2x + 3y = 6
x + 3y = 9
31.
{ x − 10 = 0
y − 10 = 0
32.
{ 2y − 10 = 0
y+2=0
33.
{ 2x + 3y = 6
3x + 2y = 6
34.
{ 10x − 8y = 16
−5x + 4y = 20
35.
1
x − 13 y = 1
{
2
36.
1
6
x + 14 y = −2
1
37. Parallel to y = x + 2 and passing through (6, −1).
2
3
38. Parallel to y =− x − 3 and passing through (−8, 2).
4
1
40. Perpendicular to y =− x + 2 and passing through (4, −3).
3
42. Perpendicular to x = 1
5
and passing through (5, −3).
1
47. Parallel to y = x − 5 and passing through (−2, 1).
4
3
48. Parallel to y =− x + 1 and passing through (4, 1/4).
4
1 1
53. Parallel to
5
x− 3
y = 2 and passing through (−15, 6).
5 1
54. Parallel to −10x − 7
y= and passing through (−1, 1/2).
2
1 1
55. Perpendicular to
2
x− 3
y = 1 and passing through (−10, 3).
62. Parallel to 6x − 3
2
y = 9 and passing through ( 13 , 3)
2
.
ANSWERS
3 4
1: m ∥ =− and m ⊥ =
4 3
1
3: m ∥ = 4 and m ⊥ = − 4
5 8
5: m ∥ =− and m ⊥ =
8 5
1
7: m ∥ = 9 and m ⊥ = − 9
9: m ∥ = 0 and m ⊥ undefined
11: m ∥ = 1 and m ⊥ = −1
4 3
13: m ∥ =− and m ⊥ =
3 4
2 7
15: m ∥ = 7
and m ⊥ =− 2
3 2
17: m ∥ = and m ⊥ =−
2 3
1
19: m ∥ = 10 and m ⊥ = − 10
21: Parallel
23: Perpendicular
25: Perpendicular
27: Perpendicular
29: Parallel
31: Neither
33: Parallel
35: Parallel
37: y = 1
2
x−4
39: y =− 1
3
x+1
41: x = −1
43: y =4
45: y = −x − 6
47: y = 1
4
x+ 3
2
49: y = 2
3
x−6
51: y =− 3
5
x+ 23
5
53: y = 3
5
x + 15
55: y =− 2
3
x− 11
3
57: y =− 1
4
x− 9
4
59: y = 1
2
x−6
61: y =− 5
2
x+ 1
4
63: x = −6
LEARNING OBJECTIVES
1. Identify a function.
2. State the domain and range of a function.
3. Use function notation.
Relationships between sets occur often in everyday life. For example, for each
month in Cape Canaveral, we can associate an average amount of rainfall. In this
case, the amount of precipitation depends on the month of the year, and the data
can be written in tabular form or as a set of ordered pairs.
548
Chapter 3 Graphing Lines
elements that compose the second column is called the range33. In this example,
the domain consists of the set of all months of the year, and the range consists of
the values that represent the average rainfall for each month.
In the context of algebra, the relations of interest are sets of ordered pairs (x, y) in
the rectangular coordinate plane. In this case, the x-values define the domain and
the y-values define the range. Of special interest are relations where every x-value
corresponds to exactly one y-value; these relations are called functions34.
Example 1: Determine the domain and range of the following relation and state
whether or not it is a function: {(−1, 4), (0, 7), (2, 3), (3, 3), (4, −2)}.
Solution: Here we separate the domain and range and depict the correspondence
between the values with arrows.
Answer: The domain is {−1, 0, 2, 3, 4}, and the range is {−2, 3, 4, 7}. The relation is a
function because each x-value corresponds to exactly one y-value.
Answer: The domain is {−4, −2, 0, 3}, and the range is {−3, 3, 5, 6, 7}. This relation is
not a function because the x-value 3 has two corresponding y-values.
In the previous example, the relation is not a function because it contains ordered
pairs with the same x-value, (3, 5) and (3, 7). We can recognize functions as relations
where no x-values are repeated.
The graph is a relation since it represents the infinite set of ordered pair solutions
to y = 34 x − 2. The domain is the set of all x-values, and in this case consists of all
real numbers. The range is the set of all possible y-values, and in this case also
consists of all real numbers. Furthermore, the graph is a function because for each
x-value there is only one corresponding y-value. In fact, any nonvertical or
nonhorizontal line is a function with domain and range consisting of all real
numbers.
Any graph is a set of ordered pairs and thus defines a relation. Consider the
following graph of a circle:
Here the graph represents a relation where many x-values in the domain
correspond to two y-values. If we draw a vertical line, as illustrated, we can see that
(3, 2) and (3, −2) are two ordered pairs with the same x-value. Therefore, the x-value
3 corresponds to two y-values; hence the graph does not represent a function. The
illustration suggests that if any vertical line intersects a graph more than once,
then the graph does not represent a function. This is called the vertical line test35.
35. If a vertical line intersects a Example 3: Given the following graph, determine the domain and range and state
graph more than once, then whether or not it is a function.
the graph does not represent a
function.
Solution: The given shape is called a parabola and extends indefinitely to the left
and right as indicated by the arrows. This suggests that if we choose any x-value,
then we will be able to find a corresponding point on the graph; therefore, the
domain consists of all real numbers. Furthermore, the graph shows that −1 is the
minimum y-value, and any y-value greater than that is represented in the relation.
Hence the range consists of all y-values greater than or equal to −1, or in interval
notation, [−1, ∞) .
Lastly, any vertical line will intersect the graph only once; therefore, it is a function.
Answer: The domain is all real numbers R = (−∞, ∞), and the range is [−1, ∞) . The
graph represents a function because it passes the vertical line test.
Try this! Given the graph, determine the domain and range and state whether or
not it is a function:
Video Solution
With the definition of a function comes special notation. If we consider each x-value
to be the input that produces exactly one output, then we can use the notation
The notation f (x) reads “f of x” and should not be confused with multiplication.
Most of our study of algebra involves functions, so the notation becomes very useful
when performing common tasks. Functions can be named with different letters;
some common names for functions are g(x), h(x), C(x), and R(x). First, consider
nonvertical lines that we know can be expressed using slope-intercept form,
y = mx + b . For any real numbers m and b, the equation defines a function, and we
can replace y with the new notation f (x) as follows:
Therefore, a linear function36 is any function that can be written in the form
f (x) = mx + b . In particular, we can write the following:
The notation also shows values to evaluate in the equation. If the value for x is given
as 8, then we know that we can find the corresponding y-value by substituting 8 in
for x and simplifying. Using function notation, this is denoted f (8) and can be
interpreted as follows:
Finally, simplify:
We have f (8) = 4. This notation tells us that when x = 8 (the input), the function
results in 4 (the output).
Answer: f (−2) = 17
Example 5: Given the linear function f (x) = −5x + 7, find x when f (x) = 10.
Solution: In this case, f (x) = 10 indicates that the function should be set equal to
10.
a. Find g(2).
Solution:
a. The notation g(2) implies that x = 2. Use the graph to determine the
corresponding y-value.
Answer: g(2) = 1
b. The notation g(x) = 3 implies that the y-value is given as 3. Use the graph to
determine the corresponding x-value.
Answer: x = 4
Example 7: Graph the linear function f (x) = − 53 x + 6 and state the domain and
range.
Solution: From the function, we see that b = 6 and thus the y-intercept is (0, 6). Also,
we can see that the slope is m = − 53 = −53 = rise. Starting from the y-intercept,
run
mark a second point down 5 units and right 3 units.
Given any coordinate on the x-axis, we can find a corresponding point on the graph;
the domain consists of all real numbers. Also, for any coordinate on the y-axis, we
can find a point on the graph; the range consists of all real numbers.
Answer: Both the domain and range consist of all real numbers R.
a. Find g (− 12 ).
Answers:
a. g (− 12 ) = 11
2
b. x = −13
Video Solution
KEY TAKEAWAYS
TOPIC EXERCISES
Part A: Functions
Determine the domain and range and state whether the relation is a function or not.
11.
12.
13.
14.
15.
16.
17.
18.
19.
20.
21.
22.
23.
24.
25.
26.
29. f (x) = 3
5
x − 4 , find f (15) .
30. f (x) = 2
5
x− 1
5
, find f (3) .
31. f (x) = 5
2
x− 1
3
, find f (− 1
3
).
35. g(x) =− 1
8
x+ 5
8
, find g ( 58 ) .
36. g(x) = 5
3
x − 5 , find g (3) .
39. f (x) =− 7
5
x − 2 , find x when f (x) = −9 .
44. g(x) =− 5
8
x + 3 , find x when g(x) = 3 .
Given f (x) = 2
3
x − 1 and g(x) = −3x + 2 calculate the following.
45. f (6)
46. f (−12)
47. f (0)
48. f (1)
49. g ( 23 )
50. g (0)
51. g (−1)
52. g (− 1
2 )
57. Given the graph of f (x) , find f (−4) , f (−1) , f (0) , and f (2) .
58. Given the graph of g(x) , find g(−3) , g(−1) , g(0) , and g(1) .
59. Given the graph of f (x) , find f (−4) , f (−1) , f (0) , and f (2) .
60. Given the graph of g(x) , find g(−4) , g(−1) , g(0) , and g(2) .
61. Given the graph of f (x) , find f (−1) , f (0) , f (1) , and f (3) .
62. Given the graph of g(x) , find g(−2) , g(0) , g(2) , and g(6) .
63. Given the graph of g(x) , find g(−4) , g(−3) , g(0) , and g(4) .
64. Given the graph of f (x) , find f (−4) , f (0) , f (1) , and f (3) .
65. Given the graph of f (x) , find x when f (x) = 3 , f (x) = 1 , and
f (x) = −3 .
66. Given the graph of g(x) , find x when g(x) = −1 , g(x) = 0 , and
g(x) = 1 .
68. Given the graph of g(x) , find x when g(x) = −2 , g(x) = 0 , and
g(x) = 4 .
69. Given the graph of f (x) , find x when f (x) = −16 , f (x) = −12 , and
f (x) = 0 .
70. Given the graph of g(x) , find x when g(x) = −3 , g(x) = 0 , and
g(x) = 1 .
71. Given the graph of f (x) , find x when f (x) = −4 , f (x) = 0 , and
f (x) = −2 .
72. Given the graph of g(x) , find x when g(x) = 5 , g(x) = 3 , and
g(x) = 2 .
73. The cost in dollars of producing pens with a company logo is given by the
function C(x) = 1.65x + 120, where x is the number of pens
produced. Use the function to calculate the cost of producing 200 pens.
74. The revenue in dollars from selling sweat shirts is given by the function
R(x) = 29.95x , where x is the number of sweat shirts sold. Use the
function to determine the revenue if 20 sweat shirts are sold.
78. The perimeter of a square depends on the length of each side s and is
modeled by the function P(s) = 4s. If the perimeter of a square measures
140 meters, then use the function to calculate the length of each side.
79. A certain cellular phone plan charges $18 per month and $0.10 per
minute of usage. The cost of the plan is modeled by the function
C(x) = 0.10x + 18 , where x represents the number of minutes of usage
per month. Determine the minutes of usage if the cost for the month was
$36.
Graph the linear function and state the domain and range.
81. f (x) =− 5
2
x + 10
82. f (x) = 3
5
x − 10
83. g(x) = 6x + 2
85. h(t) = 1
2
t−3
86. h(t) =− 3
4
t+3
89. Is a vertical line a function? What are the domain and range of a vertical
line?
90. Is a horizontal line a function? What are the domain and range of a
horizontal line?
ANSWERS
1: Yes
3: No
5: No
11: Domain: {−4, −1, 2}; range: {1, 2, 3}; function: yes
27: f (−2) = −6
29: f (15) =5
31: f (− 1
3
)=− 7
6
33: g(−4) =5
35: g ( 58 ) = 35
64
37: x =2
39: x =5
41: x = 12
43: x = 1
6
45: f (6) =3
47: f (0) = −1
49: g ( 23 ) =0
51: g (−1) =5
53: x = 3
2
55: x =1
71: f (−4)
= −4 and f (4) = −4 ; f (0) = 0; f (−2) = −2 and
f (2) = −2
73: $450
77: 15 centimeters
LEARNING OBJECTIVES
We know that a linear equation with two variables has infinitely many ordered pair
solutions that form a line when graphed. A linear inequality with two variables37,
on the other hand, has a solution set consisting of a region that defines half of the
plane.
For the inequality, the line defines one boundary of the region that is shaded. This
indicates that any ordered pair that is in the shaded region, including the boundary
line, will satisfy the inequality. To see that this is the case, choose a few test
37. An inequality relating linear points38 and substitute them into the inequality.
expressions with two variables.
The solution set is a region
defining half of the plane.
578
Chapter 3 Graphing Lines
Also, we can see that ordered pairs outside the shaded region do not solve the linear
inequality.
The graph of the solution set to a linear inequality is always a region. However, the
boundary may not always be included in that set. In the previous example, the line
was part of the solution set because of the “or equal to” part of the inclusive
inequality ≤. If we have a strict inequality <, we would then use a dashed line to
indicate that those points are not included in the solution set.
Consider the point (0, 3) on the boundary; this ordered pair satisfies the linear
equation. It is the “or equal to” part of the inclusive inequality that makes it part of
the solution set.
So far, we have seen examples of inequalities that were “less than.” Now consider
the following graphs with the same boundary:
Given the graphs above, what might we expect if we use the origin (0, 0) as a test
point?
Try this! Which of the ordered pairs (−2, −1), (0, 0), (−2, 8), (2, 1), and (4, 2) solve the
inequality y > − 12 x + 2?
Video Solution
Solution:
Step 1: Graph the boundary line. In this case, graph a dashed line y = −3x + 1
because of the strict inequality. By inspection, we see that the slope is
m = −3 = −3 1
= rise
runand the y-intercept is (0, 1).
Step 2: Test a point not on the boundary. A common test point is the origin (0, 0).
The test point helps us determine which half of the plane to shade.
Step 3: Shade the region containing the solutions. Since the test point (0, 0) was not
a solution, it does not lie in the region containing all the ordered pair solutions.
Therefore, shade the half of the plane that does not contain this test point. In this
case, shade above the boundary line.
Answer:
Consider the problem of shading above or below the boundary line when the
inequality is in slope-intercept form. If y > mx + b , then shade above the line. If
y < mx + b , then shade below the line. Use this with caution; sometimes the
boundary is given in standard form, in which case these rules do not apply.
Solution: Here the boundary is defined by the line 2x − 5y = −10 . Since the
inequality is inclusive, we graph the boundary using a solid line. In this case, graph
the boundary line using intercepts.
Since the test point is in the solution set, shade the half of the plane that contains it.
Answer:
In this example, notice that the solution set consists of all the ordered pairs below
the boundary line. This may be counterintuitive because of the original ≥ in the
inequality. This illustrates that it is a best practice to actually test a point. Solve for
y and you see that the shading is correct.
In slope-intercept form, you can see that the region below the boundary line should
be shaded. An alternate approach is to first express the boundary in slope-intercept
form, graph it, and then shade the appropriate region.
Solution: First, graph the boundary line y = 2 with a dashed line because of the
strict inequality.
In this case, shade the region that contains the test point.
Answer:
Answer:
Video Solution
KEY TAKEAWAYS
• Linear inequalities with two variables have infinitely many ordered pair
solutions, which can be graphed by shading in the appropriate half of a
rectangular coordinate plane.
• To graph the solution set of a linear inequality with two variables, first
graph the boundary with a dashed or solid line depending on the
inequality. If given a strict inequality, use a dashed line for the
boundary. If given an inclusive inequality, use a solid line. Next, choose
a test point not on the boundary. If the test point solves the inequality,
then shade the region that contains it; otherwise, shade the opposite
side.
• When graphing the solution sets of linear inequalities, it is a good
practice to test values in and out of the solution set as a check.
TOPIC EXERCISES
1. y < 5x + 1 ; (0, 0)
1
2. y >− x − 4 ; (0, −2)
2
2
3. y ≤ x + 1 ; (6, 5)
3
1
4. y ≥− x − 5 ; (−3, −8)
3
3 ( 3
5. y < 1
5
x− 1
; −
1
, − 1)
6. 4x − 3y ≤ 2 ; (−2, −1)
7. −x + 4y > 7 ; (0, 0)
10. x ≤ 0 ; (0, 7)
1
13. y ≥− x
2
4
14. y ≥ x−8
5
15. y ≤ 8x − 7
17. y > −x + 4
18. y >x−2
19. y ≥ −1
20. y < −3
21. x <2
22. x ≥2
3 1
23. y ≤ x−
4 2
3 5
24. y >− x+
2 2
26. 7x − 2y > 14
27. 5x − y < 10
28. x −y<0
29. 3x − 2y ≥ 0
30. x − 5y ≤ 0
31. −x + 2y ≤ −4
32. −x + 2y ≤ 3
33. 2x − 3y ≥ −1
34. 5x − 4y < −3
1 1
35.
2
x− 3
y<1
1 1 1
36.
2
x− 10
y≥ 2
37. x ≥ −2y
38. x < 2y + 3
39. 3x −y+2>0
40. 3 − y − 2x < 0
41. −4x ≤ 12 − 3y
42. 5x ≤ −4y − 12
43. Write an inequality that describes all points in the upper half-plane
above the x-axis.
44. Write an inequality that describes all points in the lower half-plane
below the x-axis.
45. Write an inequality that describes all points in the half-plane left of the
y-axis.
46. Write an inequality that describes all points in the half-plane right of the
y-axis.
47. Write an inequality that describes all ordered pairs whose y-coordinates
are at least 2.
48. Write an inequality that describes all ordered pairs whose x-coordinate is
at most 5.
ANSWERS
1: Yes
3: Yes
5: Yes
7: No
9: Yes
11:
13:
15:
17:
19:
21:
23:
25:
27:
29:
31:
33:
35:
37:
39:
41:
43: y >0
45: x <0
47: y ≥2
597
Chapter 3 Graphing Lines
REVIEW EXERCISES
3. Graph the points (−3, 5), (−3, −3), and (3, −3) on a rectangular coordinate
plane. Connect the points and calculate the area of the shape.
4. Graph the points (−4, 1), (0, 1), (0, −2), and (−4, −2) on a rectangular
coordinate plane. Connect the points and calculate the area of the shape.
5. Graph the points (1, 0), (4, 0), (1, −5), and (4, −5) on a rectangular
coordinate plane. Connect the points and calculate the perimeter of the
shape.
6. Graph the points (−5, 2), (−5, −3), (1, 2), and (1, −3) on a rectangular
coordinate plane. Connect the points and calculate the perimeter of the
shape.
15. Show algebraically that the points (−1, −1), (1, −3), and (2, 0) form an
isosceles triangle.
16. Show algebraically that the points (2, −1), (6, 1), and (5, 3) form a right
triangle.
18. 6x − 5y = 4 ; (−1, − 2)
19. y = 3
4
x + 1 ; (− 2
3
, 1
2 )
20. y =− 3
5
x − 2 ; (10, − 8)
21. y = −x + 2
22. y = 2x − 3
1
23. y = x−2
2
2
24. y =− x
3
25. y =3
26. x = −3
27. x − 5y = 15
28. 2x − 3y = 12
29.
30.
31.
32.
33. 3x − 4y = 12
34. 2x − y = −4
1 1
35.
2
x− 3
y=1
1 2
36. − x+ y=2
2 3
5
37. y =− x+5
3
38. y = −3x + 4
39.
40.
45. 12x − 4y = 8
46. 3x − 6y = 24
1 3
47. − x+ y=1
3 4
48. −5x + 3y = 0
49. y = −x + 3
50. y = 4x − 1
51. y = −2x
5
52. y =− x+3
2
53. 2x − 3y = 9
3
54. 2x + y=3
2
55. y =0
56. x − 4y = 0
57.
58.
59.
60.
Find the equation of a line, given the slope and a point on the line.
Find the equation of the line given two points on the line.
{ 6x − 14y = 42
−3x + 7y = 14
71.
{ 2x − 3y = 36
2x + 3y = 18
72.
{ 8x − 2y = −1
x + 4y = 2
73.
{x = 2
y=2
74.
Introduction to Functions
Determine the domain and range and state whether it is a function or not.
83.
84.
85.
86.
, find g (− ).
1 1
89. g(x) = x− 1
2 3 3
, find g ( ).
3 1 2
90. g(x) =− x+
4 3 3
3 1
94. g(x) =− x+ , find x when g(x) = −1 .
4 3
95. f (3)
102. x ≤ −5 ; (−6, 4)
103. y ≥ −2x + 1
104. y < 3x − 4
105. −x +y≤3
5 1
106.
2
x+ 2
y≤2
107. 3x − 5y > 0
108. y >0
SAMPLE EXAM
1. Graph the points (−4, −2), (−4, 1), and (0, −2) on a rectangular coordinate
plane. Connect the points and calculate the area of the shape.
Given the set of x-values {−2, −1, 0, 1, 2}, find the corresponding y-values and graph
the following.
3. y =x−1
4. y = −x + 1
5. On the same set of axes, graph y = 4 and x = −3 . Give the point where
they intersect.
Find the x- and y-intercepts and use those points to graph the following.
6. 2x −y=8
7. 12x + 5y = 15
8. Calculate the slope of the line passing through (−4, −5) and (−3, 1).
Determine the slope and y-intercept. Use them to graph the following.
3
9. y =− x+6
2
10. 5x − 2y = 6
{ 4x − 6y = 30
−2x + 3y = −12
a. y = −2
b. x = 1
3
16. Find the equation to the line passing through (−2, 3) and (4, 1).
17. Find the equation of the line parallel to 5x −y=6 passing through
(−1, −2).
4
Given a linear function f (x) =− x + 2 , determine the following.
5
19. f (10)
23. A rental car company charges $32.00 plus $0.52 per mile driven. Write an
equation that gives the cost of renting the car in terms of the number of
miles driven. Use the formula to determine the cost of renting the car and
driving it 46 miles.
24. A car was purchased new for $12,000 and was sold 5 years later for
$7,000. Write a linear equation that gives the value of the car in terms of its
age in years.
1:
5: Perimeter: 16 units
7: 10 units
⎯⎯
9: 5√ 2 units
17: No
19: Yes
21:
23:
25:
27:
33:
35:
37:
41: −7/4
43: −4/3
49:
51:
53:
55:
57: y = −2x + 1
59: y = −5
1
61: y = x + 10
2
2 8
63: y = x−
3 3
4
65: y = x−1
5
3 1
67: y =− x+
4 2
69: y = −6
71: Parallel
73: Perpendicular
75: y = 5x + 49
3
77: y =− x+5
4
79: y = −1
81: Domain: {−10, −5, 5}; range: {−1, 2}; function: yes
89: g (− 1
3 )=−
1
2
91: x = 4
9
93: x = 8
3
95: f (3) = −2
97: Yes
99: No
101: No
103:
105:
107:
3:
5: Intersection: (−3, 4)
7:
11: m ⊥ = 1
3
3
15: y =− x+7
4
17: y = 5x + 3
19: f (10) = −6
21:
25: 18 meters
619
Chapter 4 Solving Linear Systems
LEARNING OBJECTIVES
Real-world applications are often modeled using more than one variable and more
than one equation. A system of equations1 consists of a set of two or more
equations with the same variables. In this section, we will study linear systems2
consisting of two linear equations each with two variables. For example,
620
Chapter 4 Solving Linear Systems
{ −2x + 3y = 5
x−y=1
Example 1: Determine whether (1, 0) is a solution to the system .
Answer: Since (1, 0) does not satisfy both equations, it is not a solution.
{ −2x + 3y = 16
x − y = −6
Try this! Is (−2, 4) a solution to the system ?
Answer: Yes
Video Solution
First, rewrite the equations in slope-intercept form so that we may easily graph
them.
Next, replace these forms of the original equations in the system to obtain what is
called an equivalent system5. Equivalent systems share the same solution set.
If we graph both of the lines on the same set of axes, then we can see that the point
of intersection is indeed (3, 2), the solution to the system.
5. A system consisting of
equivalent equations that
share the same solution set.
{ 2x + y = 1
x − y = −4
Example 2: Solve by graphing: .
Solution:
Step 2: Write the equivalent system and graph the lines on the same set of axes.
Step 3: Use the graph to estimate the point where the lines intersect and check to
see if it solves the original system. In the above graph, the point of intersection
appears to be (−1, 3).
Answer: (−1, 3)
{ −2x + 3y = −18
2x + y = 2
Example 3: Solve by graphing: .
Solution: We first solve each equation for y to obtain an equivalent system where
the lines are in slope-intercept form.
{ y = −3
3x + y = 6
Example 4: Solve by graphing: .
Solution:
The graphing method for solving linear systems is not ideal when the solution
consists of coordinates that are not integers. There will be more accurate algebraic
methods in sections to come, but for now, the goal is to understand the geometry
involved when solving systems. It is important to remember that the solutions to a
system correspond to the point, or points, where the graphs of the equations
intersect.
{ 5x + 2y = −2
−x + y = 6
Try this! Solve by graphing: .
Answer: (−2, 4)
Video Solution
Systems with at least one solution are called consistent systems7. Up to this point,
all of the examples have been of consistent systems with exactly one ordered pair
solution. It turns out that this is not always the case. Sometimes systems consist of
two linear equations that are equivalent. If this is the case, the two lines are the
same and when graphed will coincide. Hence the solution set consists of all the
points on the line. This is a dependent system8. Given a consistent linear system
with two variables, there are two possible results:
The solutions to independent systems9 are ordered pairs (x, y). We need some way
to express the solution sets to dependent systems, since these systems have
infinitely many solutions, or points of intersection. Recall that any line can be
written in slope-intercept form, y = mx + b . Here, y depends on x. So we may
express all the ordered pair solutions (x, y) in the form (x, mx + b) , where x is
any real number.
7. A system with at least one
solution.
{ 4x − 6y = 18
solutions, denoted by −2x + 3y = −9
(x, mx + b). Example 5: Solve by graphing: .
Solution: Determine slope-intercept form for each linear equation in the system.
In slope-intercept form, we can easily see that the system consists of two lines with
the same slope and same y-intercept. They are, in fact, the same line. And the
system is dependent.
Answer: (x, 2
3
x − 3)
In this example, it is important to notice that the two lines have the same slope and
same y-intercept. This tells us that the two equations are equivalent and that the
simultaneous solutions are all the points on the line y = 23 x − 3. This is a
dependent system, and the infinitely many solutions are expressed using the form
(x, mx + b) . Other resources may express this set using set notation, {(x, y) |
y = 23 x − 3}, which reads “the set of all ordered pairs (x, y) such that y equals two-
thirds x minus 3.”
Sometimes the lines do not cross and there is no point of intersection. Such systems
have no solution, Ø, and are called inconsistent systems10.
{ −4x + 10y = 10
−2x + 5y = −15
Example 6: Solve by graphing: .
In slope-intercept form, we can easily see that the system consists of two lines with
the same slope and different y-intercepts. Therefore, they are parallel and will
never intersect.
{ −2x − 2y = 2
x + y = −1
Try this! Solve by graphing: .
Answer: (x, − x − 1)
Video Solution
KEY TAKEAWAYS
TOPIC EXERCISES
Determine whether the given ordered pair is a solution to the given system.
{ −2x − 2y = 2
x + y = −1
1. (3, −2);
{ −2x − 2y = 2
x + y = −1
2. (−5, 0);
{ 3x − y = −12
−x + y = −4
3. (−2, −6);
{ −5x − 3y = 11
3x + 2y = −8
4. (2, −7);
{ −13x + 2y = −6
5x − 5y = 15
5. (0, −3);
{ −2x − 4y = 0
x+y=− 4
6. (− 1
2
, 1
4
);
{ −4x − 8y = 5
−x − y = −1
7. ( 34 , 1
4
);
1
x + 12 y = 1
{
3
8. (−3, 4);
2
3
x − 32 y = −8
{ 5x − 10y = 5
y = −3
9. (−5, −3);
{ −7x + 4y = 8
x=4
10. (4, 2);
11.
12.
13.
14.
15.
16.
17.
18.
19.
20.
Solve by graphing.
{ y = −x + 1
y= 2
x+6
21.
3
y= x+2
{y = −
4
22.
1
4
x−2
{ y = −x + 2
y=x−4
23.
{ y = 4x − 5
y = −5x + 4
24.
y = 2 x + 1
25.
y= 3x
5
2
y=− x+6
{y =
5
26.
2
5
x + 10
{y = x + 1
y = −2
27.
{ x = −3
y=3
28.
y=0
{y =
29. 2
5
x−4
{ y = 3x
x=2
30.
y =
31.
y =
3
x−6
5
3
5
x−3
1
y=− x+1
{y = −
2
32.
1
2
x+1
{ −6x + 3y = −6
2x + 3y = 18
33.
{ 2x + 8y = 8
−3x + 4y = 20
34.
{ 2x − 3y = 9
−2x + y = 1
35.
{ 5x + 4y = −4
x + 2y = −8
36.
{ 2x − 3y = 6
4x + 6y = 36
37.
{ 6x − 3y = −6
2x − 3y = 18
38.
{ 5x − 15y = −15
−x + 3y = 3
40.
{ −x + y = 0
x−y=0
41.
{y − x = 1
y=x
42.
{
3x + 2y = 0
43.
x=2
1 2
2x + y=
{ −3x +
3 3
44.
1
2
y = −2
1 1
x+ y=2
{− 1
10 5
45.
1
5
x+ 5
y = −1
1 1
x− y=1
{
3 2
46.
1 1
3
x+ 5
y=1
1 1
x+ y=0
{
9 6
47.
1 1 1
9
x+ 4
y= 2
5 x− 1 y=5
16
48.
− 5 x + 1 y = 5
16
2
2 2
49.
−
1
x− 1
y = 92
6 2
1
18
x+ 1
6
y = − 32
1 1 1
x− y=−
{
2 4 2
50.
1 1
3
x− 2
y=3
{ x = −5
y=4
51.
{x = 2
y = −3
52.
{x = 0
y=0
53.
{ y=3
y = −2
54.
{ y = −5
y=5
55.
{y − 2 = 0
y=2
56.
{x = 1
x = −5
57.
{x = 0
y=x
58.
{ −x + y = −2
4x + 6y = 3
59.
{ 3x + 10y = −10
−2x + 20y = 20
60.
Set up a linear system of two equations and two variables and solve it using the
graphing method.
61. The sum of two numbers is 20. The larger number is 10 less than five
times the smaller.
A regional bottled water company produces and sells bottled water. The following
graph depicts the supply and demand curves of bottled water in the region. The
horizontal axis represents the weekly tonnage of product produced, Q. The vertical
axis represents the price per bottle in dollars, P.
65. Determine the price at which the quantity demanded is equal to the
quantity supplied.
66. If production of bottled water slips to 20 tons, then what price does the
demand curve predict for a bottle of water?
67. If production of bottled water increases to 40 tons, then what price does
the demand curve predict for a bottle of water?
68. If the price of bottled water is set at $2.50 dollars per bottle, what
quantity does the demand curve predict?
69. Discuss the weaknesses of the graphing method for solving systems.
70. Explain why the solution set to a dependent linear system is denoted by
(x, mx + b).
ANSWERS
1: No
3: No
5: Yes
7: No
9: Yes
11: (5, 0)
13: (2, 1)
15: (0, 0)
17: (x, 2x − 2)
19: ∅
21: (−2, 3)
25: (5, 3)
29: (10, 0)
31: ∅
33: (3, 4)
37: (6, 2)
39: ∅
41: (x, x)
45: (10, 5)
47: (−9, 6)
49: (x, 1
3
x − 9)
51: (−5, 4)
53: (0, 0)
55: ∅
57: ∅
63: (6, 6)
65: $1.25
67: $1.00
LEARNING OBJECTIVE
In this section, we will define a completely algebraic technique for solving systems.
The idea is to solve one equation for one of the variables and substitute the result
into the other equation. After performing this substitution step, we will be left with
a single equation with one variable, which can be solved using algebra. This is called
the substitution method11, and the steps are outlined in the following example.
{ 3x − 2y = −7
2x + y = 7
Example 1: Solve by substitution: .
Solution:
Step 1: Solve for either variable in either equation. If you choose the first equation,
you can isolate y in one step.
Step 2: Substitute the expression −2x + 7 for the y variable in the other equation.
644
Chapter 4 Solving Linear Systems
This leaves you with an equivalent equation with one variable, which can be solved
using the techniques learned up to this point.
Step 3: Solve for the remaining variable. To solve for x, first distribute −2:
Step 4: Back substitute12 to find the value of the other coordinate. Substitute x = 1
into either of the original equations or their equivalents. Typically, we use the
equivalent equation that we found when isolating a variable in step 1.
Step 5: Check. Verify that these coordinates solve both equations of the original
system:
The substitution method for solving systems is a completely algebraic method. Thus
graphing the lines is not required.
Answer: (1, 5)
{ x−y=3
2x − y = 12
Example 2: Solve by substitution: .
Solution: In this example, we can see that x has a coefficient of 1 in the second
equation. This indicates that it can be isolated in one step as follows:
Substitute 3 + y for x in the first equation. Use parentheses and take care to
distribute.
Use x = 3 + y to find x.
{ x = −1
3x − 5y = 17
Example 3: Solve by substitution: .
Solution: In this example, the variable x is already isolated. Hence we can substitute
x = −1 into the first equation.
Answer: (−1, −4). It is a good exercise to graph this particular system to compare the
substitution method to the graphing method for solving systems.
{ 8x + 2y = 10
3x + y = 4
Try this! Solve by substitution: .
Answer: (1, 1)
Video Solution
{ 24x − 4y = −15
2x + 8y = 5
Example 4: Solve by substitution: .
As we know, not all linear systems have only one ordered pair solution. Recall that
some systems have infinitely many ordered pair solutions and some do not have
any solutions. Next, we explore what happens when using the substitution method
to solve a dependent system.
{ 10x − 2y = 2
−5x + y = −1
Example 5: Solve by substitution: .
Solution: Since the first equation has a term with coefficient 1, we choose to solve
for that first.
This process led to a true statement; hence the equation is an identity and any real
number is a solution. This indicates that the system is dependent. The simultaneous
solutions take the form (x, mx + b), or in this case, (x, 5x − 1), where x is any real
number.
Answer: (x, 5x − 1)
We can see that both equations represent the same line, and thus the system is
dependent. Now explore what happens when solving an inconsistent system using
the substitution method.
{ 14x − 6y = −16
−7x + 3y = 3
Example 6: Solve by substitution: .
Answer: No solution, Ø
In slope-intercept form, it is easy to see that the two lines have the same slope but
different y-intercepts.
{ 4x − 10y = 6
2x − 5y = 3
Try this! Solve by substitution: .
Answer: (x, 2
5
x − 35 )
Video Solution
KEY TAKEAWAYS
TOPIC EXERCISES
Solve by substitution.
{ −3x + y = 1
y = 4x − 1
1.
{ 4x − y = 2
y = 3x − 8
2.
{ x + 3y = −8
x = 2y − 3
3.
{ 2x + 3y = 12
x = −4y + 1
4.
{ −5x + 2y = 2
y = 3x
5.
{ 2x + 3y = 10
y=x
6.
{ −4x + y = 2
y = 4x + 1
7.
{ 3x + y = 5
y = −3x + 5
8.
{ 2x − y = −3
y = 2x + 3
9.
{ x − 2y = 5
y = 5x − 1
10.
{ 3x − y = 4
y = −7x + 1
11.
{ 5x − 2y = 0
x = 6y + 2
12.
{ −2x − y = −6
y = −2
13.
{ x − 4y = −3
x = −3
14.
{ 7x − 5y = 9
y=− 5
x+3
15.
{ 6x − 9y = 0
y= 3
x−1
16.
1 1
{ x − 6y = 4
y= 2
x+ 3
17.
3 1
{ 2x + 4y = 1
y=− 8
x+ 2
18.
{ 2x + 3y = 16
x+y=6
19.
{ −2x + 3y = −2
x−y=3
20.
{ 3x − 2y = 17
2x + y = 2
21.
{ 3x + 5y = −5
x − 3y = −11
22.
{ 3x − 4y = −2
x + 2y = −3
23.
{ 9x − y = 10
5x − y = 12
24.
{ −4x − 8y = 24
x + 2y = −6
25.
{ −2x − 6y = −12
x + 3y = −6
26.
{ 6x − 2y = −2
−3x + y = −4
27.
{ 2x − 10y = −20
x − 5y = −10
28.
{ 4x + 3y = −1
3x − y = 9
29.
{ 4x + 2y = −2
2x − y = 5
30.
{ 2x − 5y = −6
−x + 4y = 0
31.
{ 5x + 2y = −8
3y − x = 5
32.
{ 4x + 10y = 2
2x − 5y = 1
33.
{ 6x + 14y = 0
3x − 7y = −3
34.
10x − y = 3
{ −5x +
35. 1
2
y=1
1 1 2
− x+ y=
{
3 6 3
36.
1 1 3
2
x− 3
y=− 2
1 2
x+ y=1
{
3 3
37.
1 1 1
4
x− 3
y=− 12
1
x − y = 12
{
7
38.
1
4
x + 12 y = 2
3 2 1
− x+ y=
{
5 5 2
39.
1 1
3
x− 12
y= − 13
1 2
x= y
{x −
2 3
40.
2
3
y=2
1 1 5
− x+ y=
{
2 2 8
41.
1 1 1
4
x+ 2
y= 4
{ −x + 2y = 3
x−y=0
42.
{ 2x − 3y = 0
y = 3x
43.
{ −6x + 3y = −6
2x + 3y = 18
44.
{ 2x + 8y = 8
−3x + 4y = 20
45.
{ 3x + 2y = 7
5x − 3y = −1
46.
{ 2x + 7y = 1
−3x + 7y = 2
47.
{ y = −3
y=3
48.
{ x = −2
x=5
49.
{y = 4
y=4
50.
51. The sum of two numbers is 19. The larger number is 1 less than three
times the smaller.
52. The sum of two numbers is 15. The larger is 3 more than twice the
smaller.
57. Describe what drives the choice of variable to solve for when beginning
the process of solving by substitution.
ANSWERS
1: (2, 7)
3: (−5, −1)
5: (2, 6)
7: ∅
9: (x, 2x + 3)
19: (2, 4)
25: (x, − 1
2
x − 3)
27: ∅
33: (1/2, 0)
35: ∅
37: (1, 1)
43: (0, 0)
45: (−4, 2)
49: ∅
LEARNING OBJECTIVES
In this section, the goal is to develop another completely algebraic method for
solving a system of linear equations. We begin by defining what it means to add
equations together. In the following example, notice that if we add the expressions
on both sides of the equal sign, we obtain another true statement.
This is true in general: if A, B, C, and D are algebraic expressions, then we have the
following addition property of equations13:
664
Chapter 4 Solving Linear Systems
The sum of y and −y is zero and that term is eliminated. This leaves us with a linear
equation with one variable that can be easily solved:
At this point, we have the x coordinate of the simultaneous solution, so all that is
left to do is back substitute to find the corresponding y-value.
Hence the solution to the system is (3, 2). This process describes the elimination
(or addition) method14 for solving linear systems. Of course, the variable is not
always so easily eliminated. Typically, we have to find an equivalent system by
applying the multiplication property of equality to one or both of the equations as a
means to line up one of the variables to eliminate. The goal is to arrange that either
the x terms or the y terms are opposites, so that when the equations are added, the
terms eliminate. The steps for the elimination method are outlined in the following
example.
{ 3x − 2y = −7
14. A means of solving a system by 2x + y = 7
Example 1: Solve by elimination: .
adding equivalent equations in
such a way as to eliminate a
variable.
Solution:
Step 1: Multiply one, or both, of the equations to set up the elimination of one of
the variables. In this example, we will eliminate the variable y by multiplying both
sides of the first equation by 2. Take care to distribute.
Step 4: Check. Remember that the solution must solve both of the original
equations.
Answer: (1, 5)
Occasionally, we will have to multiply both equations to line up one of the variables
to eliminate. We want the resulting equivalent equations to have terms with
opposite coefficients.
{ 3x + 2y = 7
5x − 3y = −1
Example 2: Solve by elimination: .
Solution: We choose to eliminate the terms with variable y because the coefficients
have different signs. To do this, we first determine the least common multiple of
the coefficients; in this case, the LCM(3, 2) is 6. Therefore, multiply both sides of
both equations by the appropriate values to obtain coefficients of −6 and 6.
Back substitute.
Answer: (1, 2)
Sometimes linear systems are not given in standard form. When this is the case, it is
best to first rearrange the equations before beginning the steps to solve by
elimination.
{ 3y = 4x + 1
5x + 12y = 11
Example 3: Solve by elimination: .
This results in the following equivalent system where like terms are aligned in
columns:
We can eliminate the term with variable y if we multiply the second equation by −4.
Back substitute.
{ −3x − 2y = 4
2x + y = −3
Try this! Solve by elimination: .
Answer: (−2, 1)
Video Solution
At this point, we explore what happens when solving dependent and inconsistent
systems using the elimination method.
{ 6x − 2y = 14
3x − y = 7
Example 4: Solve by elimination: .
Solution: To eliminate the variable x, we could multiply the first equation by −2.
A true statement indicates that this is a dependent system. The lines coincide, and
we need y in terms of x to present the solution set in the form (x, mx + b) . Choose
one of the original equations and solve for y. Since the equations are equivalent, it
does not matter which one we choose.
Answer: (x, 3x − 7)
{ 2x − 6y = 12
−x + 3y = 9
Example 5: Solve by elimination: .
A false statement indicates that the system is inconsistent. The lines are parallel
and do not intersect.
Answer: No solution, ∅
{ 2x + 10y = 30
3x + 15y = −15
Try this! Solve by elimination: .
Answer: No solution, ∅
Video Solution
Given a linear system where the equations have fractional coefficients, it is usually
best to clear the fractions before beginning the elimination method.
− 1 x+ 1 y= 4
10
Example 6: Solve:
1 x + 1 y = − 2.
7
2 5
3 21
Solution: Recall that we can clear fractions by multiplying both sides of an equation
by the least common denominator (LCD). Take care to distribute and then simplify.
This results in an equivalent system where the equations have integer coefficients,
Back substitute.
Answer: (−3, 1)
This results in an equivalent system where the equations have integer coefficients:
Back substitute.
Try this! Solve using elimination:
1 2
x− y=3
3 3
.
1 1 8
3
x− 2
y= 3
Video Solution
We have developed three methods for solving linear systems of two equations with
two variables. In this section, we summarize the strengths and weaknesses of each
method.
The graphing method is useful for understanding what a system of equations is and
what the solutions must look like. When the equations of a system are graphed on
the same set of axes, we can see that the solution is the point where the graphs
intersect. The graphing is made easy when the equations are in slope-intercept
form. For example,
The simultaneous solution (−1, 10) corresponds to the point of intersection. One
drawback of this method is that it is very inaccurate. When the coordinates of the
solution are not integers, the method is practically unusable. If we have a choice, we
typically avoid this method in favor of the more accurate algebraic techniques.
In this case, it is easy to solve for y in the first equation and then substitute the
result into the other equation. One drawback of this method is that it often leads to
equivalent equations with fractional coefficients, which are tedious to work with. If
there is not a coefficient of 1, then it usually is best to choose the elimination
method.
The elimination method is a completely algebraic method that makes use of the
addition property of equations. We multiply one or both of the equations to obtain
equivalent equations where one of the variables is eliminated if we add them
together. For example,
Here we multiply both sides of the first equation by 5 and both sides of the second
equation by −2. This results in an equivalent system where the variable x is
eliminated when we add the equations together. Of course, there are other
combinations of numbers that achieve the same result. We could even choose to
eliminate the variable y. No matter which variable is eliminated first, the solution
will be the same. Note that the substitution method, in this case, would require
tedious calculations with fractional coefficients. One weakness of the elimination
method, as we will see later in our study of algebra, is that it does not always work
for nonlinear systems.
KEY TAKEAWAYS
TOPIC EXERCISES
Solve by elimination.
{ 2x − y = 9
x+y=3
1.
{ 5x + y = −18
x − y = −6
2.
{ −x − 2y = 0
x + 3y = 5
3.
{ x − y = −7
−x + 4y = 4
4.
{ x − y = −3
−x + y = 2
5.
{ 6x + 4y = 2
3x − y = −2
6.
{ 10x − y = 4
5x + 2y = −3
7.
{ x − 7y = 21
−2x + 14y = 28
8.
{ 12x − 6y = −24
−2x + y = 4
9.
{ 3x + 12y = 6
x + 8y = 3
10.
{ 4x + 10y = 14
2x − 3y = 15
11.
{ 3x − 9y = 15
4x + 3y = −10
12.
{ 8x + 3y = −15
−4x − 5y = −3
13.
{ 4x − 2y = −112
−2x + 7y = 56
14.
{ 3x + 5y = −10
−9x − 15y = −15
15.
{ 2x + 6y = −7
6x − 7y = 4
16.
{ −5x − 3y = −7
4x + 2y = 4
17.
{ 3x + 2y = 7
5x − 3y = −1
18.
{ 2x + 5y = −14
7x + 3y = 9
19.
{ 7x + 2y = −15
9x − 3y = 3
20.
{ −7x + 6y = 11
5x − 3y = −7
21.
{ 3x + 7y = −1
2x + 9y = 8
22.
{ 3x + 3y = −5
2x + 2y = 5
23.
{ 2x − 4y = 8
−3x + 6y = −12
24.
{ 15x + 10y = −1
25x + 15y = −1
25.
{ 18x − 12y = 5
2x − 3y = 2
26.
{ −3x − 2y = 4
y = −2x − 3
27.
{ 6y = 4x − 15
28x + 6y = 9
28.
{ y = −5x + 5
y = 5x + 15
29.
{ 5x − 8y = −16
2x − 3y = 9
30.
1 1 1
x− y=
{
2 3 6
31.
5 7
2
x+y= 2
1 1
x− y=1
{ x+y=
4 9
32.
3
4
1 1 1
x− y=
{
2 4 3
33.
1
4
x+ 1
2
y= − 19
6
14
− x + 2y = 4
{−
3
34.
1
3
x + 17 y = 21
4
{ 0.5x + y = −2.75
1.3x + 0.1y = 0.35
36.
39. The sum of two numbers is 14. The larger number is 1 less than two times
the smaller.
40. The sum of two numbers is 30. The larger is 2 more than three times the
smaller.
{ 3x + y = 12
y = 2x − 3
43.
x + 3y = −5
{y=
44. 1
3
x+5
{ y=3
x = −1
45.
{x + 9 = 0
y= 2
46.
{ −x + y = 1
y=x
47.
{ y = −10
y = 5x
48.
{ 3x + 4y = 2
3y = 2x − 24
49.
{ −2y + 2 = 3x
y=− 2
x+1
50.
{ 7x = 2y + 23
7y = −2x − 1
51.
{ 3x + 2y − 5 = 0
5x + 9y − 14 = 0
52.
y = − 5 x + 10
53.
y = 5 x − 10
16
16
{
y=− 5
x + 12
54.
x=6
2 (x − 3) + y = 0
{ 3 (2x + y − 1) = 15
55.
3 − 2 (x − y) = −3
{ 4x − 3 (y + 1) = 8
56.
2 (x + 1) = 3 (2y − 1) − 21
{ 3 (x + 2) = 1 − (3y − 2)
57.
x y
− = −7
{
2 3
58. y
x
3
− 2
= −8
x y 3
− =
{
4 2 4
59. y
x 1
3
+ 6
= 6
1 2
x− y=3
{
3 3
60.
1 1 8
3
x− 2
y= 3
1 1 4
− x+ y=
{
10 2 5
61.
1 1 2
7
x+ 3
y=− 21
y=− 5 x+ 1
62.
1 x+ 1 y= 1
3
3 2
5 10
1 2
− x+y=−
{−
7 3
63.
1 1 1
14
x+ 2
y= 3
1 1 1
x− y=
{− 3
15 12 3
64.
3 3
10
x+ 8
y=− 2
{ −0.75x + 1.25y = −4
0.15x − 0.25y = −0.3
69.
71. How do we choose the best method for solving a linear system?
72. What does it mean for a system to be dependent? How can we tell if a
given system is dependent?
ANSWERS
1: (4, −1)
3: (−10, 5)
5: ∅
7: (1/5, −2)
9: (x, 2x + 4)
13: (−3, 3)
15: ∅
17: (−1, 4)
23: ∅
27: (−2, 1)
31: (1, 1)
35: (−4, 6)
43: (3, 3)
45: (−1, 3)
47: Ø
53: (32, 0)
55: (x, − 2x + 6)
57: (−4, 3)
61: (−3, 1)
63: ∅
69: Ø
LEARNING OBJECTIVES
We now have the techniques needed to solve linear systems. For this reason, we are
no longer limited to using one variable when setting up equations that model
applications. If we translate an application to a mathematical setup using two
variables, then we need to form a linear system with two equations.
Example 1: The sum of two numbers is 40 and their difference is 8. Find the
numbers.
Solution:
Identify variables.
Set up equations: When using two variables, we need to set up two equations. The
first key phrase, “the sum of the two numbers is 40,” translates as follows:
689
Chapter 4 Solving Linear Systems
And the second key phrase, “the difference is 8,” leads us to the second equation:
Solve: We can solve the resulting system using any method of our choosing. Here we
choose to solve by elimination. Adding the equations together eliminates the
variable y.
Check: The sum of the two numbers should be 42 and their difference 8.
Example 2: The sum of 9 times a larger number and twice a smaller is 6. The
difference of 3 times the larger and the smaller is 7. Find the numbers.
The first sentence describes a sum and the second sentence describes a difference.
Solve using the elimination method. Multiply the second equation by 2 and add.
Answer: The larger number is 4/3 and the smaller number is −3.
Try this! The sum of two numbers is 3. When twice the smaller number is
subtracted from 6 times the larger the result is 22. Find the numbers.
Video Solution
In this section, the interest and money problems15 should seem familiar. The
difference is that we will be making use of two variables when setting up the
algebraic equations.
Example 3: A roll of 32 bills contains only $5 bills and $10 bills. If the value of the
roll is $220, then how many of each bill are in the roll?
When using two variables, we need to set up two equations. The first equation is
created from the fact that there are 32 bills.
The second equation sums the value of each bill: the total value is $220.
Answer: There are twenty $5 bills and twelve $10 bills. The check is left to the
reader.
Example 4: A total of $6,300 was invested in two accounts. Part was invested in a CD
at a 4 12 %annual interest rate and part was invested in a money market fund at a
3 34 %annual interest rate. If the total simple interest for one year was $267.75, then
how much was invested in each account?
Solution:
To set up a second equation, use the fact that the total interest was $267.75. Recall
that the interest for one year is the interest rate times the principal
(I = prt = pr ⋅ 1 = pr ). Use this to add the interest in both accounts. Be sure to use
the decimal equivalents for the interest rates given as percentages.
Back substitute.
At this point, we should be able to solve these types of problems in two ways: with
one variable and now with two variables. Setting up word problems with two
variables often simplifies the entire process, particularly when the relationships
between the variables are not so clear.
Try this! On the first day of a two-day meeting, 10 coffees and 10 doughnuts were
purchased for a total of $20.00. Since nobody drank the coffee and all the doughnuts
were eaten, the next day only 2 coffees and 14 doughnuts were purchased for a total
of $13.00. How much did each coffee and each doughnut cost?
Video Solution
Mixture Problems
Percentage Amount
In other words, we multiply the percentage times the total to get the amount of
each part of the mixture.
Solution:
The total amount of saline solution needed is 72 ounces. This leads to one equation,
The second equation adds up the amount of salt in the correct percentages. The
amount of salt is obtained by multiplying the percentage times the amount, where
16. Applications involving a the variables x and y represent the amounts of the solutions.
mixture of amounts usually
given as a percentage of some
total.
Solve.
Back substitute.
Answer: We need 60 ounces of the 2% saline solution and 12 ounces of the 5% saline
solution.
Solution:
The second equation adds up the amount of alcohol from each solution in the
correct percentages. The amount of alcohol in the end result is 32% of 8 ounces, or
0.032(8).
Now we can form a system of two linear equations and two variables as follows:
In this example, multiply the second equation by 100 to eliminate the decimals. In
addition, multiply the first equation by −10 to line up the variable y to eliminate.
Back substitute.
Answer: To obtain 8 ounces of a 32% alcohol mixture we need to mix 4.4 ounces of
the 50% alcohol solution and 3.6 ounces of the 10% solution.
Video Solution
Recall that the distance traveled is equal to the average rate times the time traveled
at that rate, D = r ⋅ t. These uniform motion problems17 usually have a lot of
data, so it helps to first organize that data in a chart and then set up a linear
system. In this section, you are encouraged to use two variables.
Example 7: An executive traveled a total of 8 hours and 1,930 miles by car and by
plane. Driving to the airport by car, she averaged 60 miles per hour. In the air, the
plane averaged 350 miles per hour. How long did it take her to drive to the airport?
Solution: We are asked to find the time it takes her to drive to the airport; this
indicates that time is the unknown quantity.
17. Applications relating distance, Use the formula D = r ⋅ t to fill in the unknown distances.
average rate, and time.
The distance column and the time column of the chart help us to set up the
following linear system.
Solve.
Now back substitute to find the time it took to drive to the airport x:
It is not always the case that time is the unknown quantity. Read the problem
carefully and identify what you are asked to find; this defines your variables.
Example 8: Flying with the wind, an airplane traveled 1,365 miles in 3 hours. The
plane then turned against the wind and traveled another 870 miles in 2 hours. Find
the speed of the airplane and the speed of the wind.
Solution: There is no obvious relationship between the speed of the plane and the
speed of the wind. For this reason, use two variables as follows:
With the wind, the airplane’s total speed is x + w . Flying against the wind, the total
speed is x − w .
Use the rows of the chart along with the formula D = r ⋅ t to construct a linear
system that models this problem. Take care to group the quantities that represent
the rate in parentheses.
If we divide both sides of the first equation by 3 and both sides of the second
equation by 2, then we obtain the following equivalent system:
Back substitute.
Answer: The speed of the airplane is 445 miles per hour and the speed of the wind is
10 miles per hour.
Try this! A boat traveled 24 miles downstream in 2 hours. The return trip, which
was against the current, took twice as long. What are the speeds of the boat and of
the current?
Answer: The speed of the boat is 9 miles per hour and the speed of the current is 3
miles per hour.
Video Solution
KEY TAKEAWAYS
TOPIC EXERCISES
1. The sum of two integers is 54 and their difference is 10. Find the integers.
2. The sum of two integers is 50 and their difference is 24. Find the integers.
3. The sum of two positive integers is 32. When the smaller integer is
subtracted from twice the larger, the result is 40. Find the two integers.
4. The sum of two positive integers is 48. When twice the smaller integer is
subtracted from the larger, the result is 12. Find the two integers.
5. The sum of two integers is 74. The larger is 26 more than twice the
smaller. Find the two integers.
6. The sum of two integers is 45. The larger is 3 less than three times the
smaller. Find the two integers.
7. The sum of two numbers is zero. When 4 times the smaller number is
added to 8 times the larger, the result is 1. Find the two numbers.
9. The sum of 12 times the larger number and 11 times the smaller is −36.
The difference of 12 times the larger and 7 times the smaller is 36. Find the
numbers.
10. The sum of 4 times the larger number and 3 times the smaller is 7. The
difference of 8 times the larger and 6 times the smaller is 10. Find the
numbers.
12. Mary has her total savings of $12,500 in two different CD accounts. One
CD earns 4.4% interest and another earns 3.2% interest. If her total interest
for the year is $463, then how much does she have in each CD account?
13. Sally’s $1,800 savings is in two accounts. One account earns 6% annual
interest and the other earns 3%. Her total interest for the year is $93. How
much does she have in each account?
14. Joe has two savings accounts totaling $4,500. One account earns 3 34 %
annual interest and the other earns 2 58 %. If his total interest for the year
is $141.75, then how much is in each account?
15. Millicent has $10,000 invested in two accounts. For the year, she earns
$535 more in interest from her 7% mutual fund account than she does from
her 4% CD. How much does she have in each account?
16. A small business has $85,000 invested in two accounts. If the account
earning 3% annual interest earns $825 more in interest than the account
earning 4.5% annual interest, then how much is invested in each account?
17. Jerry earned a total of $284 in simple interest from two separate
accounts. In an account earning 6% interest, Jerry invested $1,000 more than
twice the amount he invested in an account earning 4%. How much did he
invest in each account?
18. James earned a total of $68.25 in simple interest from two separate
accounts. In an account earning 2.6% interest, James invested one-half as
much as he did in the other account that earned 5.2%. How much did he
invest in each account?
19. A cash register contains $10 bills and $20 bills with a total value of $340.
If there are 23 bills total, then how many of each does the register contain?
20. John was able to purchase a pizza for $10.80 with quarters and dimes. If
he uses 60 coins to buy the pizza, then how many of each did he have?
21. Dennis mowed his neighbor’s lawn for a jar of dimes and nickels. Upon
completing the job, he counted the coins and found that there were 4 less
than twice as many dimes as there were nickels. The total value of all the
coins is $6.60. How many of each coin did he have?
22. Two families bought tickets for the big football game. One family ordered
2 adult tickets and 3 children’s tickets for a total of $26.00. Another family
ordered 3 adult tickets and 4 children’s tickets for a total of $37.00. How
much did each adult ticket cost?
23. Two friends found shirts and shorts on sale at a flea market. One bought
5 shirts and 3 shorts for a total of $51.00. The other bought 3 shirts and 7
shorts for a total of $80.00. How much was each shirt and each pair of
shorts?
24. On Monday Joe bought 10 cups of coffee and 5 doughnuts for his office at
a cost of $16.50. It turns out that the doughnuts were more popular than the
coffee. Therefore, on Tuesday he bought 5 cups of coffee and 10 doughnuts
for a total of $14.25. How much was each cup of coffee?
25. A 15% acid solution is to be mixed with a 25% acid solution to produce 12
gallons of a 20% acid solution. How much of each is needed?
26. One alcohol solution contains 12% alcohol and another contains 26%
alcohol. How much of each should be mixed together to obtain 5 gallons of a
14.8% alcohol solution?
27. A nurse wishes to obtain 40 ounces of a 1.2% saline solution. How much
of a 1% saline solution must she mix with a 2% saline solution to achieve the
desired result?
30. How many pounds of pure peanuts must be combined with a 20% peanut
mix to produce 10 pounds of a 32% peanut mix?
31. How much cleaning fluid with 20% alcohol content, must be mixed with
water to obtain a 24-ounce mixture with 10% alcohol content?
32. A chemist wishes to create a 32-ounce solution with 12% acid content. He
uses two types of stock solutions, one with 30% acid content and another
with 10% acid content. How much of each does he need?
34. A 50% fruit juice concentrate can be purchased wholesale. Best taste is
achieved when water is mixed with the concentrate in such a way as to
obtain a 12% fruit juice mixture. How much water and concentrate is needed
to make a 50-ounce fruit juice drink?
36. Pure sugar is to be mixed with a fruit salad containing 10% sugar to
produce 48 ounces of a salad containing 16% sugar. How much pure sugar is
required?
37. An airplane averaged 460 miles per hour on a trip with the wind behind
it and 345 miles per hour on the return trip against the wind. If the total
round trip took 7 hours, then how long did the airplane spend on each leg of
the trip?
38. The two legs of a 330-mile trip took 5 hours. The average speed for the
first leg of the trip was 70 miles per hour and the average speed for the
second leg of the trip was 60 miles per hour. How long did each leg of the
trip take?
39. An executive traveled 1,200 miles, part by helicopter and part by private
jet. The jet averaged 320 miles per hour while the helicopter averaged 80
miles per hour. If the total trip took 4½ hours, then how long did she spend
in the private jet?
40. Joe took two buses on the 463-mile trip from San Jose to San Diego. The
first bus averaged 50 miles per hour and the second bus was able to average
64 miles per hour. If the total trip took 8 hours, then how long was spent in
each bus?
42. Two brothers drove the 2,793 miles from Los Angeles to New York. One of
the brothers, driving in the day, was able to average 70 miles per hour, and
the other, driving at night, was able to average 53 miles per hour. If the total
trip took 45 hours, then how many hours did each brother drive?
43. A boat traveled 24 miles downstream in 2 hours. The return trip took
twice as long. What was the speed of the boat and the current?
44. A helicopter flying with the wind can travel 525 miles in 5 hours. On the
return trip, against the wind, it will take 7 hours. What are the speeds of the
helicopter and of the wind?
45. A boat can travel 42 miles with the current downstream in 3 hours.
Returning upstream against the current, the boat can only travel 33 miles in
3 hours. Find the speed of the current.
46. A light aircraft flying with the wind can travel 180 miles in 1½ hours. The
aircraft can fly the same distance against the wind in 2 hours. Find the speed
of the wind.
47. Compose a number or money problem that can be solved with a system
of equations of your own and share it on the discussion board.
49. Compose a uniform motion problem that can be solved with a system of
equations of your own and share it on the discussion board.
ANSWERS
27: 32 ounces of the 1% saline solution and 8 ounces of the 2% saline solution
29: 1.5 pounds of the 10% cashew mix and 0.5 pounds of the 30% cashew mix
33: 4.4 ounces of the 50% ammonia solution and 3.6 ounces of the 10%
ammonia solution
35: 4 gallons
37: The airplane flew 3 hours with the wind and 4 hours against the wind.
LEARNING OBJECTIVES
We know that each inequality in the set contains infinitely many ordered pair
solutions defined by a region in a rectangular coordinate plane. When considering
two of these inequalities together, the intersection of these sets defines the set of
simultaneous ordered pair solutions. When we graph each of the above inequalities
separately, we have
715
Chapter 4 Solving Linear Systems
The intersection is shaded darker and the final graph of the solution set is
presented as follows:
The graph suggests that (3, 2) is a solution because it is in the intersection. To verify
this, show that it solves both of the original inequalities:
Points on the solid boundary are included in the set of simultaneous solutions and
points on the dashed boundary are not. Consider the point (−1, 0) on the solid
boundary defined by y = 2x + 2 and verify that it solves the original system:
Notice that this point satisfies both inequalities and thus is included in the solution
set. Now consider the point (2, 0) on the dashed boundary defined by y = x − 2
and verify that it does not solve the original system:
This point does not satisfy both inequalities and thus is not included in the solution
set.
Solutions to a system of linear inequalities are the ordered pairs that solve all the
inequalities in the system. Therefore, to solve these systems, graph the solution sets
of the inequalities on the same set of axes and determine where they intersect. This
intersection, or overlap, defines the region of common ordered pair solutions.
{ 3x − 6y ≥ 6
−2x + y > −4
Example 1: Graph the solution set: .
Answer:
{ 4x − 6y > 12
−2x + 3y > 6
Example 2: Graph the solution set: .
Use a dashed line for each boundary. For the first inequality, shade all points above
the boundary. For the second inequality, shade all points below the boundary.
As you can see, there is no intersection of these two shaded regions. Therefore,
there are no simultaneous solutions.
Answer: No solution, ∅
y ≥ −4
Example 3: Graph the solution set: y < x + 3 .
y ≤ −3x + 3
Solution: The intersection of all the shaded regions forms the triangular region as
pictured darkened below:
After graphing all three inequalities on the same set of axes, we determine that the
intersection lies in the triangular region pictured.
Answer:
The graphic suggests that (−1, 1) is a common point. As a check, substitute that
point into the inequalities and verify that it solves all three conditions.
KEY TAKEAWAY
TOPIC EXERCISES
Determine whether the given point is a solution to the given system of linear
equations.
{ y ≥ −x + 3
y≤x+3
1. (3, 2);
{ y ≥ 2x − 1
y < −3x + 4
2. (−3, −2);
y > −x + 5
{y ≤
3. (5, 0); 3
4
x−2
2
y< x+1
{y ≥
3
4. (0, 1);
5
2
x−2
{ 2x + 3y < 6
−4x + 3y ≥ −12
5. (−1, 8
3
);
−x + y < 0
6. (−1, −2); x + y < 0
x + y < −2
{ y ≥ −x + 3
y≤x+3
7.
{ y ≥ 2x − 1
y < −3x + 4
8.
{ y < −1
y>x
9.
2
y< x+1
{y ≥
3
10.
5
2
x−2
y > −x + 5
{y ≤
11. 3
4
x−2
y >
12.
y <
3
x+3
5
3
5
x−3
{ 2x + y ≥ 1
−x + y ≤ 6
14.
{ 4x − 3y < 15
−2x + 3y > 3
15.
{ 2x + 3y < 6
−4x + 3y ≥ −12
16.
{ −4x + 3y < −6
5x + y ≤ 4
17.
{ −x + 2y ≤ 0
3x + 5y < 15
18.
{ 5x + y > 5
x≥0
19.
{ y≥1
x ≥ −2
20.
{y + 2 ≥ 0
x−3<0
21.
{ −x + y < 1
x−y≥0
23.
{ y−x<1
−x + y ≥ 0
24.
{x ≤ 2
x > −2
25.
{y < 2
y > −1
26.
{ 3x − 6y ≥ 18
−x + 2y > 8
27.
{ 6x − 8y > −8
−3x + 4y ≤ 4
28.
2x + y < 3
{ −x ≤
29. 1
2
y
2x + 6y ≤ 6
{−
30. 1
3
x−y≤3
y<3
31. y > x
x > −4
y < 1
32. y ≥ x − 1
y < −3x + 3
−x + y < 0
34. x + y ≤ 0
x + y > −2
x+y<2
35.
−x + y ≤ 2
x<3
y+4≥0
1
36. 2 x + 3 y ≤ 1
1
1
− 2 x + 3 y ≤ 1
1
37. Construct a system of linear inequalities that describes all points in the
first quadrant.
38. Construct a system of linear inequalities that describes all points in the
second quadrant.
39. Construct a system of linear inequalities that describes all points in the
third quadrant.
40. Construct a system of linear inequalities that describes all points in the
fourth quadrant.
ANSWERS
1: Yes
3: No
5: No
7:
9:
11:
13:
15:
17:
19:
21:
23:
25:
27: No solution, ∅
29:
31:
33:
35:
{y > 0
x>0
37:
{y < 0
x<0
39:
732
Chapter 4 Solving Linear Systems
REVIEW EXERCISES
Determine whether the given ordered pair is a solution to the given system.
{ −3x + y = −6
5x − y = 8
1. (1, −3);
{ 6x − 5y = −2
4x − 2
y = −10
2. (−3, −4);
3 x− 1 y=− 2
5
3. (−1, 1/5);
− 1 x − 1 y = 1
5
3 3
2 10
3 1
x+ y=−
{
4 4
4. (1/2, −1);
2 4
3
x−y= 3
5.
6.
7.
8.
Solve by graphing.
1
y= x−3
{y = −
2
9.
3
4
x+2
y=5
{y = −
10. 4
5
x+1
{ 2x − 3y = 3
x − 2y = 0
11.
{ −4x + 2y = 16
5x − y = −11
12.
{ 5x + 4y = 12
2
x + 2y = 6
13.
{ 3x − 5y = 5
6x − 10y = −2
14.
Solve by substitution.
{x + y = 6
y = 7x − 2
15.
{ x = −2y − 1
2x − 4y = 10
16.
{ 5x − 7y = −8
x−y=0
17.
{ −x + y = 7
9x + 2y = −41
18.
{ 2x − 9y = 4
6x − 3y = 4
19.
{ 12x + 3y = 6
8x − y = 7
20.
20x − 4y = −3
{ −5x + y = −
21. 1
2
3x − y = 6
{x −
22. 1
3
y=2
{ 8x − 4y = −10
x = −1
23.
{ 14x − 4y = 0
y = −7
24.
Solve by elimination.
{ 3x − 8y = 5
x−y=5
25.
{ 9x + 4y = −30
7x + 2y = −10
26.
{ 2x − 5y = 17
9x − 6y = −6
27.
{ 3x + 7y = 14
4x − 2y = 30
28.
5 11
x − 2y = −
{
2 4
29.
1 1
6
x− 3
y= − 13
2x − 3 y = 20
30.
3 x − 1 y = 11
2
2 3
3 6
{
−1.25x − 0.45y = −12.23
32.
0.5x − 1.5y = 5.9
{ −12x + 5y = 10
6x − 2
y = −5
33.
{ 9x + 4y = 3
27x + 12y = −2
34.
{ 4x − 3y = 2
6x − 5y = 0
35.
{ 10x + 3y = 6
5x = 1
36.
{ 3x = 6y − 18
8y = −2x + 6
37.
{ 9x − 27y − 3 = 0
6y = 3x + 1
38.
39. The sum of two numbers is 74 and their difference is 38. Find the
numbers.
40. The sum of two numbers is 34. When the larger is subtracted from twice
the smaller, the result is 8. Find the numbers.
41. A jar full of 40 coins consisting of dimes and nickels has a total value of
$2.90. How many of each coin are in the jar?
42. A total of $9,600 was invested in two separate accounts earning 5.5% and
3.75% annual interest. If the total simple interest earned for the year was
$491.25, then how much was invested in each account?
46. Flying with the wind, an airplane traveled 1,065 miles in 3 hours. On the
return trip, against the wind, the airplane traveled 915 miles in 3 hours.
What is the speed of the wind?
Determine whether the given point is a solution to the system of linear inequalities.
{ −3x + y ≤ −6
5x − y > 8
47. (5, −2);
{ −5x + y > 1
2x − 3y > −10
48. (2, 3);
{x − y ≥ 0
y < −10
49. (2, −10);
1
y> x−4
{y < −
2
50. (0, −2);
3
4
x+2
{ 2x + 3y < 12
8x + 3y ≤ 24
51.
{ 4x − y ≥ 0
x+y≥7
52.
{ −2x + 6y > −6
x − 3y > −12
53.
{x − y > 0
y≤7
54.
y<4
55. y ≥ 3 x + 1
4
y > −x − 1
x − y ≥ −3
56. x − y ≤ 3
x+y<1
SAMPLE EXAM
{ −4x + y = 14
2x − 3y = −12
1. Is (−3, 2) a solution to the system ?
{ 4x − y < 0
x+y≥7
2. Is (−2, 9) a solution to the system ?
3.
4.
{ y = − 12 x + 3
y=x−3
5.
{ −x + 6y = −18
2x + 3y = 6
6.
{x + y = 3
y=2
7.
{ x = −5
y=x
8.
{ 2x − 3y = −9
5x + y = −14
9.
{ x − 2y = 2
4x − 3y = 1
10.
{ 10x + 2y = 4
5x + y = 1
11.
{ 3x − 6y = 12
x − 2y = 4
12.
{ −5x + 2y = −17
4x − y = 13
13.
{ 4x + 5y = 7
7x − 3y = −23
14.
{ x − 6y = 6
−3x + 18y = 18
15.
{ 8x − 6y = 6
−4x + 3y = −3
16.
1 3 7
x+ y=
{ 4x −
2 4 4
17.
1 4
3
y= 3
{ x − 4y < 8
3x + 4y < 24
19.
{ 3x − 8y ≤ 0
x≤8
20.
Set up a linear system of two equations and two variables and solve it using any
method.
21. The sum of two integers is 23. If the larger integer is one less than twice
the smaller, then find the two integers.
22. James has $2,400 saved in two separate accounts. One account earns 3%
annual interest and the other earns 4%. If his interest for the year totals $88,
then how much is in each account?
23. Mary drives 110 miles to her grandmother’s house in a total of 2 hours.
On the freeway, she averages 62 miles per hour. In the city she averages 34
miles per hour. How long does she spend on the freeway?
24. A 15% acid solution is to be mixed with a 35% acid solution to produce 12
ounces of a 22% acid solution. How much of each is needed?
25. Joey has bag full of 52 dimes and quarters with a total value of $8.35. How
many of each coin does Joey have?
1: Yes
3: Yes
5: (−3, 1)
7: Ø
9: (4, −1)
11: (6, 3)
13: (x, − 5
4
x + 3)
15: (1, 5)
17: (4, 4)
21: Ø
25: (7, 2)
33: (x, 12
5
x + 2)
35: (5, 6)
39: 18 and 56
43: 3.6 ounces of the 1% saline solution and 2.4 ounces of the 3% saline
solution
47: Yes
49: No
51:
53:
55:
1: Yes
3: (−1, −2)
5: (4, 1)
7: (1, 2)
9: (−3, 1)
11: Ø
15: Ø
17: (1/2, 2)
19:
21: 8 and 15
746
Chapter 5 Polynomials and Their Operations
LEARNING OBJECTIVES
For example,
Here the base is 5 and the exponent is 4. Exponents are sometimes indicated with
the caret (^) symbol found on the keyboard: 5^4 = 5*5*5*5.
Expanding the expression using the definition produces multiple factors of the
1. An equivalent expression base, which is quite cumbersome, particularly when n is large. For this reason, we
written using a rational
exponent.
747
Chapter 5 Polynomials and Their Operations
will develop some useful rules to help us simplify expressions with exponents. In
this example, notice that we could obtain the same result by adding the exponents.
In general, this describes the product rule for exponents2. If m and n are positive
integers, then
In other words, when multiplying two expressions with the same base, add the
exponents.
Solution:
Answer: 1023
In the previous example, notice that we did not multiply the base 10 times itself.
When applying the product rule, add the exponents and leave the base unchanged.
2. x m ⋅ x n = x m+n; the
product of two expressions
with the same base can be
simplified by adding the
exponents. Example 2: Simplify: x 6 ⋅ x 12 ⋅ x .
Answer: x 19
Example 3: Simplify: (x + y) (x + y) .
9 13
Answer: (x + y) 22
The commutative property of multiplication allows us to use the product rule for
exponents to simplify factors of an algebraic expression.
Example 4: Simplify: 2x 8 y ⋅ 3x 4 y 7 .
Solution: Multiply the coefficients and add the exponents of variable factors with
the same base.
Answer: 6x 12 y 8
Next, we will develop a rule for division by first looking at the quotient of 27 and
23 .
Here we can cancel factors after applying the definition of exponents. Notice that
the same result can be obtained by subtracting the exponents.
This describes the quotient rule for exponents3. If m and n are positive integers
and x ≠ 0 , then
xm In other words, when you divide two expressions with the same base, subtract the
3. x n = x m−n; the quotient of
two expressions with the same exponents.
base can be simplified by
subtracting the exponents.
12y 15
Example 5: Simplify: .
4y 7
Solution: Divide the coefficients and subtract the exponents of the variable y.
Answer: 3y 8
20x 10 (x+5)
6
10x 9 (x+5)
Example 6: Simplify: 2
.
Solution:
Answer: 2x(x + 5)
4
After writing the base 23 as a factor four times, expand to obtain 12 factors of 2. We
can obtain the same result by multiplying the exponents.
In general, this describes the power rule for exponents4. Given positive integers m
and n, then
Example 7: Simplify: (y 6 ) .
7
Solution:
n
4. (x m ) = x mn ; a power Answer: y 42
raised to a power can be
simplified by multiplying the
exponents.
To summarize, we have developed three very useful rules of exponents that are
used extensively in algebra. If given positive integers m and n, then
xm
Quotient rule: = x m−n , x ≠ 0
x n
(x ) = x
m n
m⋅n
Power rule:
Answer: y 29
Video Solution
After expanding, we have four factors of the product xy. This is equivalent to raising
each of the original factors to the fourth power. In general, this describes the
5. (xy) = x n y n; if a product
n power rule for a product5. If n is a positive integer, then
is raised to a power, then apply
that power to each factor in
the product.
Solution: We must apply the exponent 7 to all the factors, including the coefficient,
2.
If a coefficient is raised to a relatively small power, then present the real number
equivalent, as we did in this example: 27 = 128.
Answer: 128a7 b7
Solution:
Answer: 81x 4 y 12
Solution:
Answer: 64x 6 y 15 z 3
Solution:
Answer: 125(x + y)
9
Here we obtain four factors of the quotient, which is equivalent to the numerator
and the denominator both raised to the fourth power. In general, this describes the
power rule for a quotient6. If n is a positive integer and y ≠ 0 , then
In other words, given a fraction raised to a power, we can apply that exponent to
the numerator and the denominator. This rule requires that the denominator is
nonzero. We will make this assumption for the remainder of the section.
Solution: First, apply the power rule for a quotient and then the power rule for a
product.
(y ) =
n
x xn
6. y n ; if a quotient is
raised to a power, then apply
that power to the numerator
and the denominator.
3
Answer: 27a3
b
In practice, we often combine these two steps by applying the exponent to all
factors in the numerator and the denominator.
Solution: Apply the exponent 5 to all of the factors in the numerator and the
denominator.
5 10
Answer: a b15
32c
Solution:
25x 10 (2x−1) 8
Answer:
9y 14
It is a good practice to simplify within parentheses before using the power rules;
this is consistent with the order of operations.
Solution:
Answer: 16x 8 y 8 z 4
To summarize, we have developed two new rules that are useful when grouping
symbols are used in conjunction with exponents. If given a positive integer n, where
y is a nonzero number, then
(xy) = x y
n n n
Power rule for a product:
(y )
n
x xn
Power rule for a quotient: = n
y
4x 2 (x−y)
( )
3 3
Try this! Simplify: .
3yz 5
64x 6 (x−y)
9
Answer:
27y 3 z 15
Video Solution
Using the quotient rule for exponents, we can define what it means to have 0 as an
exponent. Consider the following calculation:
Eight divided by 8 is clearly equal to 1, and when the quotient rule for exponents is
applied, we see that a 0 exponent results. This leads us to the definition of zero as
an exponent7, where x ≠ 0 :
7. x 0 = 1; any nonzero base
raised to the 0 power is defined
to be 1.
It is important to note that 00 is undefined. If the base is negative, then the result is
still +1. In other words, any nonzero base raised to the 0 power is defined to be 1. In
the following examples, assume all variables are nonzero.
a. (−5)
0
b. −50
Solution:
Answers: a. 1; b. −1
Answer: 25x 6 z 4
5c d
Solution:
Answer: 1
Video Solution
KEY TAKEAWAYS
TOPIC EXERCISES
3. −10 ⋅a⋅a⋅a⋅a⋅a⋅a⋅a
4. 12 ⋅x⋅x⋅y⋅y⋅y⋅y⋅y⋅y
5. −6 ⋅ (x − 1)(x − 1)(x − 1)
Simplify.
7
7. 2 ⋅ 25
9
8. 3 ⋅3
4
9. −2
4
10. (−2)
3
11. −3
4
12. (−3)
13
13. 10 ⋅ 10 5 ⋅ 10 4
8
14. 10 ⋅ 10 7 ⋅ 10
5 12
15.
52
10 7
16.
10
10 12
17.
10 9
18. (7 )
3 5
19. (4 )
8 4
⋅ (10 5 )
6 4
20. 10
Simplify.
6
21. (−x)
22. a5 ⋅ (−a)2
23. x 3 ⋅ x5 ⋅ x
24. y 5 ⋅ y4 ⋅ y2
+ 1) 4 (y 5 ) ⋅ y 2
4
26. (x
27. (x + 1) 5 (x + 1) 8
(3x−1) 5
29.
(3x−1) 2
(a−5)
37
(a−5)
30. 13
31. xy 2 ⋅ x 2y
32. 3x 2 y 3 ⋅ 7xy 5
37. 2x 2 (x + y) ⋅ 3x 5 (x + y)
5 4
39. x 2 y ⋅ xy 3 ⋅ x 5 y 5
42. (−x 2 ) (x ) (x )
3 3 4 2 3
a10 ⋅(a6 )
3
43.
a3
10x 9 (x 3 )
5
44.
2x 5
a6 b 3
45.
a2 b 2
m 10 n 7
46.
m3 n4
20x 5 y 12 z 3
47.
10x 2 y 10 z
−24a16 b 12 c3
48.
6a6 b 11 c
16 x 4 (x+2) 3
49.
4x(x+2)
50y 2 (x+y)
20
10y(x+y)
50. 17
Simplify.
5
51. (2x)
52. (−3y)
4
53. (−xy)
3
54. (5xy)
3
55. (−4abc)
2
56. (
2x )
7 2
(− )
3
5
57.
3y
58. ( c )
3ab 3
(− )
2xy 4
59.
3z
( x )
5y(2x−1) 3
60.
61. (3x 2 )
3
62. (−2x 3 )
2
63. (xy 5 )
7
64. (x 2 y 10 )
2
65. (3x 2 y)
3
66. (2x 2 y 3 z 4 )
5
67. (−7ab c )
4 2 2
[x y (x + y) ]
5 4 4 5
68.
69. [2y(x + 1) 5 ]
3
( b3 )
3
a
70.
( 3b )
4
5a2
71.
(− )
2
2x 3
72.
3y 2
(− )
3
x2
73.
y3
( 3c3 d 2 )
4
ab 2
74.
( )
6
2x 7 y(x−1) 3
75.
z5
76. (2x 4 ) ⋅ (x 5 )
3 2
77. (x 3 y) ⋅ (xy 4 )
2 3
(2x (x + y) ) ⋅ (2x (x + y) )
4 5 2 3
3 4
80.
( xy 2 )
−3x 5 y 4 3
81.
( xy 2 )
−3x 5 y 4 2
82.
( 5x 5 y 10 )
−25x 10 y 15 3
83.
( 5xy 2 )
10x 3 y 5 2
84.
(− )
5
24ab 3
85.
6bc
( 16x 2 y )
−2x 3 y 2
86.
( 3abc )
3
30ab 3
87.
( 2s2 t )
3
3s3 t 2
88.
6xy 5 (x+y)
( 3y 2 z(x+y) 2 )
6 5
89.
(− )
4
64a5 b 12 c2 (2ab−1)14
90.
32a2 b 10 c2 (2ab−1)7
(2)
91. The probability of tossing a fair coin and obtaining n heads in a row is
n
given by the formula P = 1
. Determine the probability, as a percent, of
tossing 5 heads in a row.
93. If each side of a square measures 2x 3 units, then determine the area in
terms of the variable x.
94. If each edge of a cube measures 5x 2 units, then determine the volume
in terms of the variable x.
0
95. 7
0
96. (−7)
0
97. −10
0
98. −3 ⋅ (−7) 0
0
99. 8675309
2
100. 5 ⋅ 30 ⋅ 23
0
101. −3 ⋅ (−2) 2 ⋅ (−3) 0
102. 5x 0 y 2
2 2 0 5
103. (−3) x y z
(x ) y (z )
2 3 2 3 2 0
104. −3
105. 2x 3 y 0 z ⋅ 3x 0 y 3 z 5
c ⋅ 3a2 (b 3 c2 )
2 0 0
106. −3ab
107. (−8xy 2 )
0
( y3 )
0
2x 2
108.
9x 0 y 4
109.
3y 3
0
112. Why is 0 undefined?
4
113. Explain to a beginning student why 3 ⋅ 3 2 ≠ 9 6.
ANSWERS
5
1: (2x)
3: −10a7
5: −6(x − 1) 3
12
7: 2
9: −16
11: −27
22
13: 10
10
15: 5
3
17: 10
32
19: 4
21: x 6
23: x 9
25: a23
27: (x + 1) 13
29: (3x − 1) 3
31: x 3 y 3
2
33: −16a3 b
5 2
35: −6a3 b c
37: 6x 7 (x + y)
9
39: x 8 y 9
41: 27x 5 y 6 z 5
43: a25
45: a4 b
47: 2x 3 y 2 z 2
49: 4x 3 (x + 2) 2
51: 32x 5
53: −x 3 y 3
2 2
55: 16a2 b c
125
57: −
27y 3
16x 4 y 4
59:
81z 4
61: 27x 6
63: x 7 y 35
65: 27x 6 y 3
8 4
67: 49a2 b c
69: 8y 3 (x + 1) 15
625a8
71:
81b 4
x6
73: −
y9
64x 42 y 6 (x−1) 18
75:
z 30
77: x 9 y 14
19
79: −81a10 b
81: −27x 12 y 6
83: −125x 15 y 15
1024a5 b 10
85: −
c5
1000b 6
87:
c3
32x 5 y 15 (x+y)
20
89:
z5
91: 3 1
8
%
93: A = 4x 6
95: 1
97: −1
99: 1
101: −4
103: 9x 2 z 5
105: 6x 3 y 3 z 6
107: 1
109: 3y
LEARNING OBJECTIVES
Definitions
8. An algebraic expression
consisting of terms with real The degree of a term9 in a polynomial is defined to be the exponent of the variable,
number coefficients and or if there is more than one variable in the term, the degree is the sum of their
variables with whole number
exponents. Recall that x 0 = 1; any constant term can be written as a product of x 0
exponents.
and itself. Hence the degree of a constant term is 0.
9. The exponent of the variable; if
there is more than one variable
in the term, the degree of the
term is the sum their
exponents.
775
Chapter 5 Polynomials and Their Operations
Term Degree
3x 2 2
6x 2 y 2+1=3
7a2 b3 2+3=5
0
8
, since 8 = 8x 0
2x 1, since x = x1
Polynomial Degree
4x 5 − 3x 3 + 2x − 1 5
10. The largest degree of all of its
terms.
Polynomial Degree
1 5
x+ 1, because x = x1
2 4
In this text, we will call polynomials with four or more terms simply polynomials.
Solution: Here there are three terms. The highest variable exponent is 5. Therefore,
this is a trinomial of degree 5.
Solution: The term 4x 2 y has degree 3; −6xy 4 has degree 5; 5x 3 y 3 has degree 6;
14. An algebraic expression and the constant term 4 has degree 0. Therefore, the polynomial has 4 terms with
consisting of terms with real
number coefficients and degree 6.
variables with whole number
exponents.
Of particular interest are polynomials with one variable15, where each term is of
the form an x n . Here an is any real number and n is any whole number. Such
polynomials have the standard form
In this form, we can see that the subtraction in the original corresponds to negative
coefficients. Because addition is commutative, we can write the terms in descending
order based on the degree of each term as follows:
We can further classify polynomials with one variable by their degree as follows:
Polynomial Name
5 Constant (degree 0)
2x + 1 Linear (degree 1)
3x 2 + 5x − 3 Quadratic (degree 2)
x3 + x2 + x + 1 Cubic (degree 3)
7x 4 + 3x 3 − 7x + 8 Fourth-degree polynomial
Evaluating Polynomials
Given the values for the variables in a polynomial, we can substitute and simplify
using the order of operations.
Solution: First, replace the variable with parentheses and then substitute the given
value.
Answer: −11/2
Solution:
Answer: 0
Solution:
Answer: 41
Solution:
Answer: 9
2
πcubic meters
Answer: −50
Video Solution
Polynomial functions with one variable are functions that can be written in the
form
where an is any real number and n is any whole number. Some examples of the
different classes of polynomial functions are listed below:
Since there are no restrictions on the values for x, the domain of any polynomial
function consists of all real numbers.
17. A polynomial function with
degree 0.
20. A polynomial function with Solution: Recall that the function notation f (5) indicates we should evaluate the
degree 3. function when x = 5 . Replace every instance of the variable x with the value 5.
Answer: f (−1) = 8
Answer: g (−1) = −6
Video Solution
KEY TAKEAWAYS
• Polynomials are special algebraic expressions where the terms are the
products of real numbers and variables with whole number exponents.
• The degree of a polynomial with one variable is the largest exponent of
the variable found in any term.
• The terms of a polynomial are typically arranged in descending order
based on the degree of each term.
• When evaluating a polynomial, it is a good practice to replace all
variables with parentheses and then substitute the appropriate values.
• All polynomials are functions.
TOPIC EXERCISES
Part A: Definitions
1. 2x +1
2. x 2 + 7x + 2
3. 2 − 3x 2 + x
4. 4x
5. x 2 − x3 + x + 1
6. 5 − 10x 3
Classify the given polynomial as a monomial, binomial, or trinomial and state the
degree.
7. x 3 −1
8. x 2 y 2
9. x − x5 + 1
10. x 2 + 3x − 1
4
11. 5ab
1 1
12.
3
x− 2
13. −5x 3 + 2x + 1
14. 8x 2 −9
15. 4x 5 − 5x 3 + 6x
16. 8x 4 − x 5 + 2x − 3
17. 9x +7
18. x 5 + x4 + x3 + x2 − x + 1
19. 6x −1 + 5x 4 − 8
4
20. x − 3x 2 + 3
21. 7
22. x 2
23. 4x 2 y − 3x 3 y 3 + xy 3
2
24. a3 b − 6ab
3
25. a3 b
x2 y2
26. y − x
27. xy −3
29. −3x 10 y 2 z − xy 12 z + 9x 13 + 30
30. 7x 0
31. 1 − 6x + 7x 2
32. x − 9x 2 − 8
33. 7 − x 3 + x 7 − x 2 + x − 5x 5
34. a3 − a9 + 6a5 − a + 3 − a4
Evaluate.
37. 2x − 3 , where x = 3
38. x 2 − 3x + 5 , where x = −2
1 1 1
39. − x+ , where x =−
2 3 3
1
40. −x 2 + 5x − 1 , where x = − 2
43. y 3 − 2y + 1 , where y = −2
46. x 3 − x 2, where x = 5
3 1 3 2
47.
4
x2 − 2
x+ 6
, where x =− 3
5 1 1
48.
8
x2 − 4
x+ 2
, where x =4
57. a = 3, b = −5 , c = 0
58. a = 1, b = 0, and c = −4
1
59. a = ,b = −4 , and c = 2
4
60. a = 1, b = 5, and c = 6
The volume of a sphere in cubic units is given by the formula V = 43 πr3 , where r
is the radius. For each problem, calculate the volume of a sphere given the following
radii.
61. r = 3 centimeters
62. r = 1 centimeter
65. r = 0.15 in
The height in feet of a projectile launched vertically from the ground with an initial
velocity v 0 in feet per second is given by the formula h = −16t 2 + v 0 t, where
t represents time in seconds. For each problem, calculate the height of the projectile
given the following initial velocity and times.
The stopping distance of a car, taking into account an average reaction time, can be
estimated with the formula d = 0.05v 2 + 1.5 , where d is in feet and v is the
speed in miles per hour. For each problem, calculate the stopping distance of a car
traveling at the given speeds.
2
Given the linear function f (x) = x + 6, evaluate each of the following.
3
73. f (−6)
74. f (−3)
75. f (0)
76. f (3)
79. f (−2)
80. f (−1)
81. f (0)
82. f (2)
83. g(−2)
84. g (−1)
85. g (0)
86. g (1)
The height in feet of a projectile launched vertically from the ground with an initial
velocity of 128 feet per second is given by the function h(t) = −16t 2 + 128t ,
where t is in seconds. Calculate and interpret the following.
87. h(0)
88. h ( 12 )
89. h(1)
90. h(3)
91. h(4)
92. h(5)
93. h(7)
94. h(8)
96. Explain how to convert feet per second into miles per hour.
97. Find and share the names of fourth-degree, fifth-degree, and higher
polynomials.
ANSWERS
1: Linear
3: Quadratic
5: Cubic
7: Binomial; degree 3
9: Trinomial; degree 5
31: 7x 2 − 6x + 1
33: x 7 − 5x 5 − x 3 − x 2 + x + 7
35:
37: 3
39: 1/2
41: −5
43: −3
45: −15
47: 7/6
49: 6
51: −11
53: −35
55: 0
57: 25
59: 14
67:
Time Height
t = 0 seconds h = 0 feet
t = 1 second h = 48 feet
t = 2 seconds h = 64 feet
t = 3 seconds h = 48 feet
t = 4 seconds h = 0 feet
73: 2
75: 6
77: x =6
79: 19
81: 5
83: −15
85: −1
89: The projectile is 112 feet above the ground 1 second after launch.
91: The projectile is 256 feet above the ground 4 seconds after launch.
93: The projectile is 112 feet above the ground 7 seconds after launch.
LEARNING OBJECTIVES
1. Add polynomials.
2. Subtract polynomials.
3. Add and subtract polynomial functions.
Adding Polynomials
Recall that we combine like terms, or terms with the same variable part, as a means
to simplify expressions. To do this, add the coefficients of the terms to obtain a
single term with the same variable part. For example,
Notice that the variable part, x 2 , does not change. This, in addition to the
commutative and associative properties of addition, allows us to add
polynomials21.
797
Chapter 5 Polynomials and Their Operations
Answer: 5x 2 + 2x + 3
Solution: Remember that the variable parts have to be exactly the same before we
can add the coefficients.
Solution:
Answer: x 3 + 2x 2 + 5
Video Solution
Solution: Subtract each term within the parentheses and then combine like terms.
Answer: 7x − 5
Solution: Distribute the −1, remove the parentheses, and then combine like terms.
Answer: x 2 + 4x + 7
Solution: Distribute the −1, remove the parentheses, and then combine like terms.
Answer: −9x 3 − 9x 2 + 3x + 5
difference correctly. First, write the quantity (2x 2 + 5x − 2); from this, subtract
Solution: Since subtraction is not commutative, we must take care to set up the
Answer: −4x 2 + 8x − 1
Solution: Apply the distributive property, remove the parentheses, and then
combine like terms.
Answer: 6x 2 − 4x − 4
Answer: 7x 2 y − 7xy 2 + 7
Video Solution
When using function notation, be careful to group the entire function and add or
subtract accordingly.
Solution: The notation (f + g)(x) indicates that you should add the functions
f (x) + g(x) and collect like terms.
Answer: (f + g)(x) = 2x 2 − x + 6
Solution: The notation (f − g)(x) indicates that you should subtract the functions
f (x) − g(x):
Answer: (f − g)(x) = 2x 2 − 8
We may be asked to evaluate the sum or difference of two functions. We have the
option to first find the sum or difference and use the resulting function to evaluate
for the given variable, or to first evaluate each function and then find the sum or
difference.
Therefore,
Answer: (f − g) (5) = −7
Alternate Solution: Since (f − g)(5) = f (5) − g(5), we can find f (5) and g(5) and
then subtract the results.
Therefore, we have
Answer: (f − g) (5) = −7
KEY TAKEAWAYS
TOPIC EXERCISES
Add.
1. (2x + 1) + (−x + 7)
2. (−6x + 5) + (3x − 1)
3. ( x + 12 ) + ( 13 x − 2)
2
3
4. ( x − 34 ) + ( 56 x + 18 )
1
3
5. (2x + 1) + (x − 3) + (5x − 2)
6. (2x − 8) + (−3x 2 + 7x − 5)
7. (x 2 − 3x + 7) + (3x 2 − 8x − 5)
8. (−5x 2 − 1 + x) + (−x + 7x 2 − 9)
9. ( x + 16 ) + (− x − 1)
1 1 3 2
2
x2 − 3 2
x2 + 3
10. (− x − 6) + (2x 2 − x + 52 )
3 1 3
5
x2 + 4 8
11. (x 2 + 5) + (3x 2 − 2x + 1) + (x 2 + x − 3)
16. (3x + 5) + (x 2 − x + 1) + (x 3 + 2x 2 − 3x + 6)
18. (1 + 7x − 5x 3 + 4x 4 ) + (−3x 3 + 5 − x 2 + x)
Subtract.
33. ( x − 34 ) − ( 34 x + 18 )
1
4
34. (− x + 37 ) − ( 25 x − 32 )
3
5
35. (x 2 + 7x − 5) − (4x 2 − 5x + 1)
38. ( x − 34 ) − ( 32 x 2 − x + 12 )
1 1 1
2
x2 + 3 6
39. ( x − 13 ) − ( 13 x 2 + x + 59 )
5 1 3
9
x2 + 5 10
46. (x 2 + xy − y 2 ) − (x 2 + xy − y 2 )
55. The cost in dollars of producing customized coffee mugs with a company
logo is given by the formula C = 150 + 0.10x , where x is the number of
cups produced. The revenue from selling the cups in the company store is
given by R = 10x − 0.05x 2 , where x is the number of units sold.
b. Find the profit from producing and selling 100 mugs in the company
store.
56. The cost in dollars of producing sweat shirts is given by the formula
C = 10q + 1200 , where C is the cost and q represents the quantity
produced. The revenue generated by selling the sweat shirts for $37 each is
given by R = 37q , where q represents the quantity sold. Determine the
profit generated if 125 sweat shirts are produced and sold.
57. The outer radius of a washer is 3 times the radius of the hole.
58. Derive a formula for the surface area of the following rectangular solid.
Simplify.
69. ( 12 x2 − 3
4
x + 14 ) − ( 32 x − 34 ) + ( 54 x − 12 )
70. ( 95 x2 − 1
3
x + 2) − ( 103 x 2 − 45 ) − (x + 52 )
1 2 1
76. f (x) = x 2 − 5x + 3
and g (x) = 3
x2 − x − 2
77. (f + g) (x)
78. (g + f ) (x)
79. (f − g) (x)
80. (g − f ) (x)
81. (g + g) (x)
82. (f + g) (3)
83. (f + g) (−2)
84. (f + g) (0)
85. (f − g) (0)
86. (f − g) (−2)
87. (g − f ) (−2)
88. (g − f ) ( 12 )
89. (f + g) (x)
90. (g + f ) (x)
91. (f − g) (x)
92. (g − f ) (x)
93. (f + g) (−2)
94. (f − g) (−2)
95. (f + g) (0)
96. (f − g) (0)
ANSWERS
1: x +8
3
3: x − 2
5: 8x −4
7: 4x 2 − 11x + 2
1 5
9: −x 2 + x−
3 6
11: 5x 2 −x+3
15: 2x 2 + 12x − 17
17: 6x 5 + x 4 − 5x 3 + x 2 − 6x − 3
19: 5x 2 y 2 − 10xy − 1
21: −3x 2 + y2
25: 9x +7
27: 6x 2 − 12x + 2
29: 2x 2 y 2 −1
31: 3x −2
1 7
33: − x−
2 8
37: −3x 3 + x 2 − 18
2 1 8
39:
9
x2 − 10
x− 9
43: 5x 4 + 4x 3 + x 2 + 2x + 1
45: −x 2 y 2 − xy + 14
51: 2x − 10
53: −5x − 13
59: −2x +4
61: 6
63: 11x 2 − 6x
65: −x 2 + 5x − 1
69: 1
2
x2 − x + 1
2
75: (f
+ g) (x) = 15 x 2 − x + 12 and
(f − g) (x) = − 5 x + 2x + 6
1 2 1
77: (f + g) (x) = x 2 + 5x − 4
79: (f − g) (x) = −x 2 − x − 2
81: (g + g) (x) = 2x 2 + 6x − 2
85: (f − g) (0) = −2
87: (g − f ) (−2) = 4
89: (f + g) (x) = 7x 2 + 3x − 2
91: (f − g) (x) = 3x 2 − 9x + 6
93: (f + g) (−2) = 20
95: (f + g) (0) = −2
LEARNING OBJECTIVES
Multiplying by a Monomial
Recall the product rule for exponents: if m and n are positive integers, then
In other words, when multiplying two expressions with the same base, add the
exponents. This rule applies when multiplying a monomial by a monomial. To find
the product of monomials, multiply the coefficients and add the exponents of
variable factors with the same base. For example,
816
Chapter 5 Polynomials and Their Operations
Solution: In this case, multiply the monomial, −5x , by the binomial, 4x − 2 . Apply
the distributive property and then simplify.
Answer: 6x 4 − 10x 3 + 2x 2
Solution:
Video Solution
In the same way that we used the distributive property to find the product of a
monomial and a binomial, we will use it to to find the product of two binomials.
Here we apply the distributive property multiple times to produce the final result.
This same result is obtained in one step if we apply the distributive property to a
and b separately as follows:
This is often called the FOIL23 method. We add the products of the first terms of
each binomial ac, the outer terms ad, the inner terms bc, and finally the last terms
bd. This mnemonic device only works for products of binomials; hence it is best to
just remember that the distributive property applies.
Solution: Distribute 1
2
x and then distribute − 14.
Answer: 1
4
x2 − 1
16
Solution:
Answer: 6y 3 + 3y 2 − 2y − 1
Solution: After multiplying each term of the trinomial by x 2 and −5, simplify.
Answer: 8x 3 − 12x 2 + 6x − 1
The confusion comes from the product to a power rule of exponents, where we
apply the power to all factors. Since there are two terms within the parentheses,
that rule does not apply. Care should be taken to understand what is different in the
following two examples:
Video Solution
Solution: Multiply each term of the first trinomial by each term of the second
trinomial and then combine like terms.
Aligning like terms in columns, as we have here, aids in the simplification process.
Answer: 2x 4 − 3x 3 + 5x 2 + 11x − 15
Because the results could coincidentally be the same, a check by evaluating does not
necessarily prove that we have multiplied correctly. However, after verifying a few
values, we can be fairly confident that the product is correct.
Answer: x 4 − 4x 3 − 2x 2 + 12x + 9
Video Solution
24. We can be fairly certain that (click to see video)
we have multiplied the
polynomials correctly if we Special Products
check that a few values
evaluate to the same results in
In this section, the goal is to recognize certain special products that occur often in
the original expression and in
the answer. our study of algebra. We will develop three formulas that will be very useful as we
move along. The three should be memorized. We begin by considering the following
two calculations:
Answer: 9x 2 + 30x + 25
25. The trinomials obtained by
(a + b) = a + 2ab + b
squaring the binomials
2 2 2
Answer: x 2 − 8x + 16
Solution:
Answer: 49x 2 − 16
Video Solution
Solution: Multiply all terms of the trinomial by the monomial function f (x).
We have
Answer: (f ⋅ g) (−1) = 52
we were asked to evaluate multiple values for the function (f ⋅ g) (x), it would be
g(−1) separately and then multiply the results (try this as an exercise). However, if
best to first determine the general form, as we have in the previous example.
KEY TAKEAWAYS
TOPIC EXERCISES
Multiply.
1. 5x (−3x 2 y)
2. (−2x 3 y 2 ) (−3xy )
4
1
3.
2
(4x − 3)
( 3 x − 6)
3 2
4. −
4
5. 3x(5x − 2)
6. −4x(2x − 1)
7. x 2 (3x + 2)
8. −6x 2 (5x + 3)
9. 2ab(4a − 2b)
11. 6x 2 y 3 (−3x y + xy )
3 2
(−5ab + 6a b)
3 3 2
12. 3ab
1
13. −
2
x 2 y(4xy − 10)
17. −2 (5x 2 − 3x + 4)
(25x − 50xy + 5y )
4 2 2
18.
5
19. 3x (5x 2 − 2x + 3)
20. −x (x 2 + x − 1)
21. x 2 (3x − 5x − 7)
2
22. x 3 (−4x − 7x + 9)
2
23.
1
4
x 4 (8x 3 − 2x 2 + 1
2
x − 5)
24. −
1
3
x 3 ( 32 x 5 − 2
3
x3 + 9
2
x − 1)
27.
2
3
xy 2 (9x 3 y − 27xy + 3xy 3 )
Multiply.
33. (3x − 2) (x + 4)
34. (x + 2) (x − 3)
35. (x − 1) (x + 1)
37. (2x − 5) (x + 3)
39. (−3x + 1) (x − 1)
40. (x + 5) (−x + 1)
41. (y − 23 ) (y + 23 )
42. ( x + 13 ) ( 32 x − 23 )
1
2
43. ( x + 15 ) ( 14 x + 25 )
3
4
44. ( ) (5 x − 2)
1 3 3 5
5
x+ 10
45. (y 2 − 2) (y + 2)
46. (y 3 − 1) (y 2 + 2)
49. (x − 5) (2x 2 + 3x + 4)
50. (3x − 1) (x 2 − 4x + 7)
53. (x − 12 ) (3x 2 + 4x − 1)
54. ( x − 14 ) (3x 2 + 9x − 3)
1
3
55. (x + 3) 3
56. (x − 2) 3
57. (3x − 1) 3
58. (2x + y)
3
60. (x 2 − 2) (x 3 − 2x 2 + x + 1)
Multiply.
61. (x 2 − x + 1) (x 2 + 2x + 1)
63. (2x 2 − 3x + 5) (x 2 + 5x − 1)
64. (a + b + c) (a − b − c)
65. (a + 2b − c)
2
66. (x + y + z)
2
67. (x − 3) 4
68. (x + y)
4
70. Find the volume of a cube where each side measures x − 5 units.
Multiply.
71. (x + 2) 2
72. (x − 3) 2
73. (2x + 5)
2
74. (3x − 7) 2
75. (−x + 2) 2
76. (−9x + 1) 2
77. (a + 6)
2
79. ( x + 34 )
2 2
3
80. ( x − 35 )
1 2
2
81. (x 2 + 2)
2
82. (x 2 + y2)
2
83. (x + 4) (x − 4)
86. ( x − 13 ) ( 15 x + 13 )
1
5
87. ( x + 25 ) ( 32 x − 25 )
3
2
91. A box is made by cutting out the corners and folding up the edges of a
square piece of cardboard. A template for a cardboard box with a height of 2
inches is given. Find a formula for the volume, if the initial piece of
cardboard is a square with sides measuring x inches.
92. A template for a cardboard box with a height of x inches is given. Find a
formula for the volume, if the initial piece of cardboard is a square with
sides measuring 12 inches.
99. (f ⋅ g) (x)
100. (g ⋅ f ) (x)
101. (f ⋅ g) (0)
102. (f ⋅ g) (−1)
103. (f ⋅ g) (1)
104. (f ⋅ g) ( 12 )
105. (f ⋅ g) (x)
106. (g ⋅ f ) (x)
107. (f ⋅ g) (0)
108. (f ⋅ g) (−1)
109. (f ⋅ g) (1)
110. (f ⋅ g) ( 12 )
111. (f ⋅ f ) (x)
112. (g ⋅ g) (x)
114. Explain how to quickly multiply a binomial with its conjugate. Give an
example.
115. What are the advantages and disadvantages of using the mnemonic
device FOIL?
ANSWERS
1: −15x 3 y
3
3: 2x − 2
5: 15x 2 − 6x
7: 3x 3 + 2x 2
9: 8a2 b − 4ab 2
11: −18x 5 y 4 + 6x 3 y 5
13: −2x 3 y 2 + 5x 2 y
15: −30x 9
17: −10x 2 + 6x − 8
19: 15x 3 − 6x 2 + 9x
21: 3x 4 − 5x 3 − 7x 2
1 1 5
23: 2x 7 − x6 + x5 − x4
2 8 4
25: a4 b − 3a3 b 2 + a2 b 3
27: 6x 4 y 3 − 18x 2 y 3 + 2x 2 y 5
29: 6x 3 − 9x 2 + 15x
33: 3x 2 + 10x − 8
35: x 2 −1
37: 2x 2 + x − 15
39: −3x 2 + 4x − 1
4
41: y 2 − 9
3 7 2
43:
16
x2 + 20
x+ 25
45: y 3 + 2y 2 − 2y − 4
47: a4 − b4
49: 2x 3 − 7x 2 − 11x − 20
51: 8x 3 − 27
5 1
53: 3x 3 + x 2 − 3x +
2 2
55: x 3 + 9x 2 + 27x + 27
61: x 4 + x3 + x + 1
69: x 3 + 6x 2 + 8x
71: x 2 + 4x + 4
73: 4x 2 + 20x + 25
75: x 2 − 4x + 4
77: a2 + 12a + 36
4 9
79:
9
x2 + x + 16
81: x 4 + 4x 2 + 4
83: x 2 − 16
85: 25x 2 −9
9 4
87:
4
x2 − 25
89: 16x 2 − y2
101: (f ⋅ g) (0) = 3
103: (f ⋅ g) (1) = −2
107: (f ⋅ g) (0) = −5
109: (f ⋅ g) (1) = 12
LEARNING OBJECTIVES
1. Divide by a monomial.
2. Divide by a polynomial using the division algorithm.
3. Divide polynomial functions.
Dividing by a Monomial
Recall the quotient rule for exponents: if x is nonzero and m and n are positive
integers, then
In other words, when dividing two expressions with the same base, subtract the
exponents. This rule applies when dividing a monomial by a monomial. In this
section, we will assume that all variables in the denominator are nonzero.
28y 3
Example 1: Divide: 7y .
Solution: Divide the coefficients and subtract the exponents of the variable y.
Answer: 4y 2
841
Chapter 5 Polynomials and Their Operations
24x 7 y 5
Example 2: Divide: .
8x 3 y 2
Solution: Divide the coefficients and apply the quotient rule by subtracting the
exponents of the like bases.
Answer: 3x 4 y 3
Solution: Break up the fraction by dividing each term in the numerator by the
monomial in the denominator and then simplify each term.
Answer: −x 2 + 5x − 3
Check your division by multiplying the answer, the quotient28, by the monomial in
the denominator, the divisor29, to see if you obtain the original numerator, the
dividend30.
Solution:
Answer: −3a2 + 7
3
ab − 1. The check is optional and is left to the reader.
Answer: 8x 3 − 4x 2 + 5
2
x+1
Video Solution
The same technique outlined for dividing by a monomial does not work for
polynomials with two or more terms in the denominator. In this section, we will
outline a process called polynomial long division31, which is based on the division
algorithm for real numbers. For the sake of clarity, we will assume that all
expressions in the denominator are nonzero.
x 3 +3x 2 −8x−4
Example 5: Divide: x−2
.
Step 1: To determine the first term of the quotient, divide the leading term of the
dividend by the leading term of the divisor.
Step 2: Multiply the first term of the quotient by the divisor, remembering to
distribute, and line up like terms with the dividend.
Step 3: Subtract the resulting quantity from the dividend. Take care to subtract
both terms.
Step 4: Bring down the remaining terms and repeat the process from step 1.
Notice that the leading term is eliminated and that the result has a degree that is
one less than the dividend. The complete process is illustrated below:
Polynomial long division ends when the degree of the remainder32 is less than the
degree of the divisor. Here the remainder is 0. Therefore, the binomial divides the
polynomial evenly and the answer is the quotient shown above the division line.
To check the answer, multiply the divisor by the quotient to see if you obtain the
dividend:
Answer: x 2 + 5x + 2
Just as with real numbers, the final answer adds the fraction where the remainder is
the numerator and the divisor is the denominator to the quotient. In general, when
dividing we have
6x 2 −5x+3
Example 6: Divide: 2x−1
.
This is the quotient of the given leading terms: (6x 2 ) ÷ (2x) = 3x. Multiply 3x
To start, determine what monomial times 2x − 1 results in a leading term 6x 2 .
times the divisor 2x − 1 and line up the result with like terms of the dividend.
Subtract the result from the dividend and bring down the constant term +3.
Subtracting eliminates the leading term and −5x − (−3x) = −5x + 3x = −2x .
The quotient of −2x and 2x is −1. Multiply 2x − 1 by −1 and line up the result.
The constant term 2 has degree 0, and thus the division ends. We may write
2
Answer: 3x − 1 + 2x−1 . To check that this result is correct, we multiply as follows:
27x 3 +64
Example 7: Divide: 3x+4
.
Solution: Notice that the binomial in the numerator does not have terms with
degree 2 or 1. The division is simplified if we rewrite the expression with
placeholders:
We begin with 27x 3 ÷ 3x = 9x 2 and work the rest of the division algorithm.
Answer: 9x 2 − 12x + 16
Solution:
Begin the process by dividing the leading terms to determine the leading term of
the quotient 3x 4 ÷ x 2 = 3x 2 . Take care to distribute and line up the like terms.
Continue the process until the remainder has a degree less than 2.
x−2
Answer: 3x 2 + 4x − 1 +
x 2 −2x+5
Polynomial long division takes time and practice to master. Work lots of problems
and remember that you may check your answers by multiplying the quotient by the
divisor (and adding the remainder if present) to obtain the dividend.
7
Answer: 4x 3 − 4x 2 − x + 1 − 5x−3
Video Solution
(f /g) (x) =
f (x)
Division of functions:
g(x)
The quotient of two polynomial functions does not necessarily have a domain of all
real numbers. The values for x that make the function in the denominator 0 are
restricted from the domain. This will be discussed in more detail at a later time. For
now, assume all functions in the denominator are nonzero.
Example 10: Calculate: (f /g) (−1), given f (x) = −3x 3 + 7x 2 − 11x − 1 and
g (x) = 3x − 1.
Therefore,
KEY TAKEAWAYS
TOPIC EXERCISES
Divide.
81y 5
1.
9y 2
36y 9
2.
9y 3
52x 2 y
3.
4xy
24xy 5
4.
2xy 4
25x 2 y 5 z 3
5.
5xyz
77x 4 y 9 z
6. −
22x 3 y 3 z
125a3 b 2 c
7.
−10abc
36a2 b 3 c5
8.
−6a2 b 2 c3
9x 2 +27x−3
9.
3
9x 7 −6x 4 +12x 3 −x 2
17.
3x 2
26. Find the quotient of 64a2 bc3 − 16a5 bc7 and 4a2 bc3 .
Divide.
27. (2x 2 − 5x − 3) ÷ (x − 3)
28. (3x 2 + 5x − 2) ÷ (x + 2)
x 3 −x 2 −2x−12
31.
x−3
2x 3 +11x 2 +4x−5
32.
x+5
2x 3 −x 2 −4x+3
33.
2x+3
x 2 +8x+17
37.
x+5
2x 2 −5x+5
38.
x−2
6x 2 −13x+9
39.
−2x+1
−12x 2 +x+1
40.
3x+2
x 3 +9x 2 +19x+1
41.
x+4
2x 3 −13x 2 +17x−11
42.
x−5
9x 3 −12x 2 +16x−15
43.
3x−2
x 5 +7 x 4 −x 3 −7 x 2 −49 x+9
49.
x 2 +7x−1
x 3 −27
51.
x−3
8x 3 +125
52.
2x+5
x 5 −2x 3 +3x−1
55.
x−1
x 4 −3x 2 +5x−13
56.
x+2
a2 −4
57.
a+2
a5 +1
58.
a5 +1
a6 −1
59.
a−1
x 5 −1
60.
x−1
x 5 +x 4 +6x 3 +12x 2 −4
61.
x 2 +x−1
5x 5 −15x 3 +25x 2 −5
63.
5x
87. How do you use the distributive property when dividing a polynomial by
a monomial?
88. Compare long division of real numbers with polynomial long division.
Provide an example of each.
ANSWERS
1: 9y 3
3: 13x
5: 5xy 4 z 2
25
7: −
2
a2 b
9: 3x 2 + 9x − 1
11: 10x 2 − 5x + 15
13: 2x 4 + 3x 2 − 1
15: 2x 3 − 3x + 1
1
17: 3x 5 − 2x 2 + 4x − 3
1
19: 4a3 − 8a2 + 5a − 4
21: −4x 5 y 5 + 2y 2 + 1
23: −18xy 2
25: a2 − 6a + 9
27: 2x +1
29: 2x +3
31: x 2 + 2x + 4
33: x 2 − 2x + 1
35: 2x 3 − x 2 + 3x + 1
2
37: x +3+ x+5
4
39: −3x +5+ −2x+1
5
41: x 2 + 5x − 1 + x+4
7
43: 3x 2 − 2x + 4 − 3x−2
7
45: 2x 4 − 5x 3 + 3x 2 − 2x + 5 − 3x+1
47: 5x 3 + 2x 2 − 7x + 1
2
49: x 3 −7+ x 2 +7x−1
51: x 2 + 3x + 9
53: 3x 4 − 4x 2 + 3
1
55: x 4 + x3 − x2 − x + 2 + x−1
57: a −2
59: a5 + a4 + a3 + a2 + a + 1
2x+1
61: x 3 + 7x + 5 + x 2 +x−1
1
63: x 4 − 3x 2 + 5x − x
2
65: 15x 2 z 10 − y4 + 5z
8
67: x 5 + 9x 3 − 2x + 1 − 3x−2
73: (f /g)
5
(x) = 6x − 2 − 3x−5
83: (f /g)
5
(x) = 5x 2 − 3x + 1 + x−2
85: (f /g)
3
(0) = − 2
LEARNING OBJECTIVES
Negative Exponents
Notice that 4, 8, and 32 are all powers of 2. Hence we can write 4 = 22 , 8 = 23 , and
32 = 25 .
If the exponent of the term in the denominator is larger than the exponent of the
term in the numerator, then the application of the quotient rule for exponents
results in a negative exponent. In this case, we have the following:
We conclude that 2−3 = 13 . This is true in general and leads to the definition of
2
negative exponents34. Given any integer n and x ≠ 0 , then
1
34. x −n = x n ,given any integer
n, where x is nonzero.
864
Chapter 5 Polynomials and Their Operations
Here x ≠ 0 because 10 is undefined. For clarity, in this section, assume all variables
are nonzero.
Solution:
1
Answer: 100
Solution:
Answer: − 1
3
Solution:
Answer: y 3
If the grouped quantity is raised to a negative exponent, then apply the definition
and write the entire grouped quantity in the denominator. If there is no grouping,
then apply the definition only to the base preceding the exponent.
Solution: First, apply the definition of −3 as an exponent and then apply the power
of a product rule.
1
Answer:
8a3 b3
Solution:
1
Answer:
9x 2 y 6
−3
Example 6: Simplify: x −4 .
y
Solution:
y4
Answer:
x3
−2x −5y 3
Example 7: Simplify: .
z −2
Solution: Take care with the coefficient −2; recognize that this is the base and that
the exponent is actually +1: −2 = (−2)1 . Hence the rules of negative exponents do
not apply to this coefficient; leave it in the numerator.
x −n ym
35. y −m = x n , given any integers −2y 3 z 2
Answer:
m and n, where x ≠ 0 and x5
y ≠ 0.
(−3x )
−4 −3
Example 8: Simplify: .
y −2
Solution: Apply the power of a product rule before applying negative exponents.
x 12 y 2
Answer: − 27
(3x )
2 −4
(−2y −1z 3 )
Example 9: Simplify: −2
.
Solution:
4z 6
Answer:
81x 8 y 2
(5x y)
2 3
Solution: First, apply the power of a product rule and then the quotient rule.
Answer: 125x 11 y 6
To summarize, we have the following rules for negative integer exponents with
nonzero bases:
1
Negative exponents: x −n =
xn
x −n ym
Quotients with negative exponents: = n
y −m x
(−5xy )
−3 −2
Try this! Simplify: .
5x 4 y −4
y 10
Answer:
125x 6
Video Solution
where n is an integer and 1 ≤ a < 10. This form is particularly useful when the
numbers are very large or very small. For example,
It is cumbersome to write all the zeros in both of these cases. Scientific notation is
an alternative, compact representation of these numbers. The factor 10n indicates
the power of 10 to multiply the coefficient by to convert back to decimal form:
This is equivalent to moving the decimal in the coefficient fifteen places to the
right. A negative exponent indicates that the number is very small:
This is equivalent to moving the decimal in the coefficient eleven places to the left.
While all of these are equal, 5.63 × 10−3 is the only form considered to be
expressed in scientific notation. This is because the coefficient 5.63 is between 1 and
10 as required by the definition. Notice that we can convert 5.63 × 10−3 back to
decimal form, as a check, by moving the decimal to the left three places.
Solution: Here we count twelve decimal places to the left of the decimal point to
obtain the number 1.075.
Solution: Here we count six decimal places to the right to obtain 3.045.
Solution: Use the fact that multiplication is commutative and apply the product
rule for exponents.
Solution:
Example 15: The speed of light is approximately 6.7 × 108 miles per hour. Express
this speed in miles per second.
Solution: A unit analysis indicates that we must divide the number by 3,600.
Answer: The speed of light is approximately 1.9 × 105 miles per second.
Example 16: By what factor is the radius of the sun larger than the radius of earth?
Solution: We want to find the number that when multiplied times the radius of
earth equals the radius of the sun.
Therefore,
Answer: The radius of the sun is approximately 110 times that of earth.
Video Solution
KEY TAKEAWAYS
TOPIC EXERCISES
−1
1. 5
−2
2. 5
−1
3. (−7)
−1
4. −7
1
5.
2 −3
5
6.
3 −2
7. (
5)
3 −2
8. (
2)
1 −5
9. (−
3)
2 −4
10. (−
3)
1 −3
11. x −4
12. y −1
13. 3x −5
−5
14. (3x)
1
15.
y −3
5
16.
2x −1
x −1
17.
y −2
1
(x−y)
18. −4
x 2 y −3
19.
z −5
20. xy −3
21. (ab)
−1
1
22.
(ab)−1
23. −5x −3 y 2 z −4
−2 3 −5
24. 3 x y z
25. 3x −4 y 2 ⋅ 2x −1 y 3
3
26. −10a2 b ⋅ 2a−8 b −10
27. (2a−3 )
−2
28. (−3x 2 )
−1
29. (5a2 b c)
−3 −2
31. (−2r2 s0 t −3 )
−1
32. (2xy −3 z 2 )
−3
33. (−5a2 b c )
−3 0 4
34. (−x −2 y 3 z −4 )
−7
( 2x −3 )
−5
1
35.
( y2 )
−2
2x
36.
( 2y −1 )
−4
x
37.
( c5 )
−5
−3a2 b
38.
( 5yz −1 )
20x −3 y 2 −1
39.
( 2r3 st 0 )
−3
4r5 s−3 t 4
40.
( y 2 z3 )
2xy 3 z −1 −3
41.
(− )
2
3a2 bc
42.
ab 0 c4
(− )
xyz −4
43.
x y −2 z 3
4
( )
0
125x −3 y 4 z −5
−
5x 2 y 4 (x+y)
44. 3
−2
45. (x n )
( yn )
−2
xn
46.
The value in dollars of a new MP3 player can be estimated by using the formula
V = 100(t + 1)−1 , where t is the number of years after purchase.
52. According to the formula, will the MP3 ever be worthless? Explain.
53. 9.3 × 10 9
54. 1.004 × 10 4
55. 6.08 × 10 10
56. 3.042 × 10 7
57. 4.01 × 10 −7
59. 9.9 × 10 −3
60. 7.0011 × 10 −5
61. 500,000,000
62. 407,300,000,000,000
63. 9,740,000
64. 100,230
65. 0.0000123
66. 0.000012
67. 0.000000010034
68. 0.99071
69. (3 × 10 5 ) (9 × 10 4 )
9.12×10 −9
73.
3.2×10 10
1.15×10 9
74.
2.3×10 −11
1.004×10 −8
75.
2.008×10 −14
3.276×10 25
76.
5.2×10 15
81. The population density of earth refers to the number of people per
× 10 7
square mile of land area. If the total land area on earth is 5.751
9
square miles and the population in 2007 was estimated to be 6.67 × 10
people, then calculate the population density of earth at that time.
82. In 2008 the population of New York City was estimated to be 8.364
million people. The total land area is 305 square miles. Calculate the
population density of New York City.
83. The mass of earth is 5.97 × 10 24 kilograms and the mass of the moon
22
is 7.35 × 10 kilograms. By what factor is the mass of earth greater than
the mass of the moon?
84. The mass of the sun is 1.99 × 10 30 kilograms and the mass of earth is
24
5.97 × 10 kilograms. By what factor is the mass of the sun greater than
the mass of earth? Express your answer in scientific notation.
15
86. One light year, 9.461 × 10 meters, is the distance that light travels
in a vacuum in one year. If the distance to the nearest star to our sun,
16
Proxima Centauri, is estimated to be 3.991 × 10 meters, then calculate
the number of years it would take light to travel that distance.
87. It is estimated that there are about 1 million ants per person on the
planet. If the world population was estimated to be 6.67 billion people in
2007, then estimate the world ant population at that time.
88. The sun moves around the center of the galaxy in a nearly circular orbit.
The distance from the center of our galaxy to the sun is approximately
26,000 light years. What is the circumference of the orbit of the sun around
the galaxy in meters?
89. Water weighs approximately 18 grams per mole. If one mole is about
6 × 10 23 molecules, then approximate the weight of each molecule of
water.
9 6
90. A gigabyte is 1 × 10 bytes and a megabyte is 1 × 10 bytes. If the
average song in the MP3 format consumes about 4.5 megabytes of storage,
then how many songs will fit on a 4-gigabyte memory card?
ANSWERS
1
1:
5
1
3: − 7
5: 8
25
7:
9
81
9:
16
1
11:
x4
3
13:
x5
15: y 3
y2
17: x
x 2 z5
19:
y3
1
21:
ab
−5y 2
23:
x 3 z4
6y 5
25:
x5
a6
27:
4
b6
29:
25a4 c2
t3
31: −
2r2
625a8
33:
b 12
32
35:
x 15
16
37:
x 4y4
x3
39:
4yz
z 12
41:
8x 3 y 3
x 12 z 8
43:
y 12
1
45:
x 2n
47: $100
49: $20
51: $1
53: 9,300,000,000
55: 60,800,000,000
57: 0.000000401
59: 0.0099
61: 5 × 10 8
63: 9.74 × 10 6
65: 1.23 × 10 −5
67: 1.0034 × 10 −8
69: 2.7 × 10 10
75: 5 × 10 5
77: 1.888 × 10 9
79: 5.15 × 10 11
83: 81.2
85: 1.807
887
Chapter 5 Polynomials and Their Operations
REVIEW EXERCISES
Rules of Exponents
Simplify.
3
1. 7 ⋅ 76
59
2.
56
3. y 5 ⋅ y2 ⋅ y3
4. x 3 y 2 ⋅ xy 3
2
5. −5a3 b c ⋅ 6a2 bc2
55x 2 yz 5
6.
5xyz 2
( 2c3 )
2
−3a2 b 4
7.
( 4c4 )
3
−2a3 b
8.
9. −5x 3 y 0 (z 2 ) ⋅ 2x 4 (y 3 ) z
3 2
10. (−25x 6 y 5 z)
0
11. Each side of a square measures 5x 2 units. Find the area of the square in
terms of x.
12. Each side of a cube measures 2x 3 units. Find the volume of the cube in
terms of x.
Introduction to Polynomials
Classify the given polynomial as a monomial, binomial, or trinomial and state the
degree.
13. 8a3 −1
14. 5y 2 −y+1
2
15. −12ab
16. 10
17. 7 − x 2 − 5x
18. 5x 2 − 1 − 3x + 2x 3
Evaluate.
19. 2x 2 − x + 1 , where x = −3
1 3 1
20.
2
x− 4
, where x = 3
2 1 3
21. b − 4ac, where a = − ,b = −3 , and c = −
2 2
1 1
22. a2 − b 2, where a = − 2
and b =− 3
units, the width measures 3 units, and the height measures 5 units, then
calculate the surface area.
28. The surface area of a sphere is given by the formula SA = 4πr2 , where
r represents the radius of the sphere. If a sphere has a radius of 5 units, then
calculate the surface area.
30. ( x − 19 ) + ( 16 x + 12 )
1
3
31. (7x 2 − x + 9) + (x 2 − 5x + 6)
37. (x 5 − x 3 + x − 1) − (x 4 − x 2 + 5)
(f + g) (x) .
(x) = 3x 2 − x + 5 and g (x) = x 2 − 9, find
41. Given f
(f − g) (x) .
(x) = 3x 2 − x + 5 and g (x) = x 2 − 9, find
42. Given f
(f + g) (−2) .
(x) = 3x 2 − x + 5 and g (x) = x 2 − 9, find
43. Given f
(f − g) (−2) .
(x) = 3x 2 − x + 5 and g (x) = x 2 − 9, find
44. Given f
Multiplying Polynomials
Multiply.
45. 6x 2 (−5x )
4
(7a b)
2 2
46. 3ab
51. (x − 8) (x + 5)
53. (3x − 1) 2
54. (3x − 1) 3
56. (x 2 + 3) (x 3 − 2x − 1)
57. (5y + 7)
2
58. (y 2 − 1)
2
(f ⋅ g) (−1).
63. Given f
(x) = 7x − 2 and g (x) = x 2 − 3x + 1 , find
Dividing Polynomials
Divide.
7y 2 −14y+28
65. 7
70. (2x 3 − 5x 2 + 5x − 6) ÷ (x − 2)
x 4 −4
76.
x−4
Negative Exponents
Simplify.
−2
83. (−10)
−2
84. −10
85. 5x −3
86. (5x)
−3
1
87.
7y −3
3x −4
88.
y −2
−2a2 b −5
89.
c−8
90. (−5x 2 yz −1 )
−2
91. (−2x −3 y 0 z 2 )
−3
( 5ab2 c2 )
−1
−10a5 b 3 c2
92.
( 2a4 b−3 c )
−3
a2 b −4 c0
93.
The value in dollars of a new laptop computer can be estimated by using the formula
V = 1200(t + 1)−1 , where t represents the number of years after the
purchase.
96. 2,030,000,000
97. 0.00000004011
4×10 16
100.
8×10 −7
9×10 −30
101.
4×10 −10
103. 0.0003/120,000,000,000,000
SAMPLE EXAM
Simplify.
1. −5x 3 (2x y)
2
2. (x 2 )
4
⋅ x3 ⋅ x
(−2x y )
2 3 2
3.
x 2y
4. a. (−5) ; b. −5
0 0
Evaluate.
5. 2x 2 − x + 5 , where x = −5
6. a2 − b 2, where a = 4 and b = −3
7. (3x 2 − 4x + 5) + (−7x 2 + 9x − 2)
8. (8x 2 − 5x + 1) − (10x 2 + 2x − 1)
9. ( a − 12 ) − ( 23 a2 + a − 29 ) + ( 15 )
3 2 1 5
5 3
a− 18
10. 2x 2 (2x − 3x − 4x + 5)
3 2
11. (2x − 3) (x + 5)
12. (x − 1) 3
81x 5 y 2 z
13.
−3x 3 yz
x 3 −5x 2 +7x−2
15.
x−2
6x 4 −x 3 −13x 2 −2x−1
16.
2x−1
Simplify.
−3
17. 2
18. −5x −2
19. (2x 4 y −3 z)
−2
( ab−3 c2 )
−3
−2a3 b −5 c−2
20.
22. If each side of a cube measures 4x 4 units, calculate the volume in terms
of x.
24. The cost in dollars of producing custom t-shirts is given by the formula
C = 120 + 3.50x , where x represents the number of t-shirts produced.
The revenue generated by selling the t-shirts for $6.50 each is given by the
formula R = 6.50x , where x represents the number of t-shirts sold.
b. Use the formula to calculate the profit from producing and selling 150 t-
shirts.
25. The total volume of water in earth’s oceans, seas, and bays is estimated
× 10 19 cubic feet. By what factor is the volume of the moon,
to be 4.73
7.76 × 10 20 cubic feet, larger than the volume of earth’s oceans? Round
to the nearest tenth.
9
1: 7
3: y 10
3 3
5: −30a5 b c
9a4 b 8
7:
4c6
9: −10x 7 y 6 z 7
11: A = 25x 4
17: −x 2 − 5x + 7
19: 22
21: 6
23: −7
25: f (−2) = 24
29: 12x −5
31: 8x 2 − 6x + 15
33: 8y +8
35: x 2 y 2 − 5xy + 7
37: x 5 − x4 − x3 + x2 + x − 6
39: 7x +9
41: (f + g) (x) = 4x 2 − x − 4
43: (f + g) (−2) = 14
45: −30x 6
49: 2x 4 y 2 − 5x 3 y 3 + 2x 2 y
51: x 2 − 3x − 40
53: 9x 2 − 6x + 1
59: x 4 −1
65: y 2 − 2y + 4
67: −a + 4b + 1
69: 5x −2
71: 5x 3 + 2x 2 − 3x + 4
3
73: 5x 2 − 10x + 1 + 2x−7
1
75: x 3 −x+5+ 5x+4
77: x 2 + 5x − 10
79: (f /g)
2
(x) = x 2 + x − 3 − x−1
1
83:
100
5
85:
x3
y3
87: 7
−2a2 c8
89:
b5
x9
91: −
8z 6
3 3
93: 8a6 b c
95: $1,200
97: 4.011 × 10 −8
99: 5.796 × 10 19
1: −10x 5 y
3: 4x 2 y 5
5: 60
7: −4x 2 + 5x + 3
2 5
9: −
3
a2 − 9
11: 2x 2 + 7x − 15
13: −27x 2 y
15: x 2 − 3x + 1
1
17:
8
y6
19:
4x 8 z 2
21: 5x 2 y − 2xy 2 + 1
25: 16.4
903
Chapter 6 Factoring and Solving by Factoring
LEARNING OBJECTIVES
Recall that a prime number is defined as a natural number with exactly two natural
number factors, 1 and itself. The first ten prime numbers follow:
904
Chapter 6 Factoring and Solving by Factoring
Since the prime factorization is unique, it does not matter how we choose to
initially factor the number; the end result will be the same. The prime factorization
of 60 follows:
Recall that the greatest common factor (GCF)5 of any two natural numbers is the
product of all the common prime factors.
The numbers 7 and 15 share no common natural number factor other than 1; we say
that they are relatively prime6.
GCF of Monomials
Given two or more monomials, it will be useful to find the greatest common
monomial factor of each. For example, consider 6x 5 y 3 z and 8x 2 y 3 z 2 . The variable
part of these two monomials look very much like the prime factorization of natural
numbers and, in fact, can be treated the same way. Steps for finding the GCF of
monomials7 are outlined in the following example.
Solution:
In this case, the common variables with the smallest exponents are x 2 , y 3 , and z 1 .
Step 3: The GCF of the monomials is the product of the common variable factors
and the GCF of the coefficients. Therefore,
7. The product of the GCF of the
coefficients and all common
variable factors.
Answer: 2x 2 y 3 z
It is worth pointing out that the GCF in the previous example divides both
expressions evenly:
The factors 3x 3 and 4z share no common monomial factors other than 1; they are
relatively prime.
Example 4: Determine the GCF of the following expressions: 30x 6 y and 18x 4 y 2 z .
The variable factors in common are x 4 and y. The factor z is not in common and we
have
Answer: 6x 4 y
The GCF (12, 60, 24) = 22 ⋅ 3 = 12. Next, determine the common factors of the
variable part:
Answer: 12a2 b2 (a + b) . Note that the variable c is not common to all three
2
Video Solution
We have seen that application of the distributive property is the key to multiplying
polynomials. The process of factoring a polynomial8 involves using the
distributive property in reverse to write each polynomial as a product of
polynomial factors.
To demonstrate this idea, we multiply and factor side by side. Factoring utilizes the
GCF of the terms.
Factoring out the GCF9 involves rewriting a polynomial as a product where a factor
is the GCF of all of its terms:
The steps for factoring out the GCF of a polynomial10 are outlined in the following
example.
Solution:
Step 1: Identify the GCF of all the terms. In this case, the GCF(7, 21, 14) = 7, and the
common variable factor with the smallest exponent is x 2 . The GCF of the
polynomial is 7x 2 .
Step 2: Determine the terms of the missing factor by dividing each term of the
original expression by the GCF. (This step is usually performed mentally.)
9. The process of rewriting a
polynomial as a product using
the GCF of all of its terms.
Step 3: Apply the distributive property (in reverse) using the terms found in the
previous step.
Step 4: As a check, multiply using the distributive property to verify that the
product equals the original expression. (This step is optional and can be performed
mentally.)
Answer: 7x 2 (x 2 + 3x − 2)
Solution: There are no variable factors in common and the GCF(48, 16, 4) = 4.
Answer: 4 (12a − 4b + c)
Solution: The GCF(25, 15, 5) = 5, and the common variable factor with smallest
exponents is x 1 . The GCF of all the terms is 5x .
Answer: 5x (5x 2 + 3x + 1)
If the GCF is the same as one of the terms, then, after the GCF is factored out, a
constant term 1 will remain. In the previous example, we can see that 5x 5x
= 1. The
importance of remembering the constant term becomes clear when performing the
check using the distributive property:
The constant term 1 allows us to obtain the same original expression after we
distribute.
Solution: The GCF(10, 15, 20) = 5, and the common variables with smallest
exponent are x 1 and y 3 . Therefore, the GCF of the terms is 5xy 3 . The first term
does not have a variable factor of z and thus cannot be a part of the greatest
common factor. If we divide each term by 5xy 3 , we obtain
Solution: The GCF(24, 8) = 8, and the variable factors with smallest exponents are
a6 , b2 , and c. Therefore, the GCF of all the terms is 8a6 b2 c.
Of course, not all polynomials with integer coefficients can be factored as a product
of polynomials with integer coefficients other than 1 and itself. If this is the case,
then we say that it is a prime polynomial11.
Solution: Prime: there are no polynomial factors other than 1 and itself.
Answer: Prime
Answer: 4x 2 y (4x 2 y 2 − 2y 4 − 1)
Video Solution
In this section, we outline a technique for factoring polynomials with four terms.
First, review some preliminary examples where the terms have a common binomial
factor.
To determine the terms of the remaining factor, divide each term by (x − 3):
Answer: (x − 3) (5x + 2)
Recall that 1 is always a common factor. If the GCF is the same as a term, then the
factor 1 remains after we factor out that GCF.
Solution: Rewrite the second term −(4x + 1) as −1(4x + 1) and then factor out
the common binomial factor (4x + 1).
Remember that the goal for this section is to develop a technique that enables us to
factor polynomials with four terms into a product of binomials. The intermediate
step of this process looks like the previous two examples. For example, we wish to
factor
Begin by grouping the first two terms and the last two terms. Then factor out the
GCF of each grouping:
The steps that follow outline a technique for factoring four-term polynomials called
factor by grouping12.
Solution: Group terms in such a way as to obtain a binomial with common factors.
Step 1: Group the first two and last two terms and then factor out the GCF of each.
The GCF of the first two terms is 2x 2 , and the GCF of the second two terms is 3.
Step 2: At this point, the polynomial is a binomial. Factor out any factors common
to both terms. Here (x + 2) is a common factor.
Step 3: Optional check: multiply to verify that we obtain the original expression.
Answer: (x + 2) (2x 2 + 3)
Solution: The GCF of the first two terms is a2 and the GCF of the second two terms
is 1.
Solution: The GCF for the first group is 6x 3 . We have to choose 5 or −5 to factor out
of the second group.
Tip
The sign of the leading coefficient in the second grouping usually indicates
whether or not to factor out a negative factor. If that coefficient is positive,
factor out a positive factor. If it is negative, factor out a negative factor.
When all the terms of a polynomial have a GCF other than 1, it is a best practice to
factor that out before factoring by grouping.
Solution: Here we notice that the greatest common factor of all the terms is 3y .
Begin by factoring out the GCF and then factor the result by grouping.
Answer: 3y (y 2 + 3) (y − 2)
Sometimes we must first rearrange the terms in order to obtain a common factor.
Solution: Simply factoring the GCF out of the first group and last group does not
yield a common binomial factor.
We must rearrange the terms, searching for a grouping that produces a common
factor. In this example, we have a workable grouping if we switch the terms a3 and
ab.
Not all factorable four-term polynomials can be factored with this technique. For
example,
Answer: (x − y) (x 2 − y)
Video Solution
KEY TAKEAWAYS
TOPIC EXERCISES
Give the prime factorization of each number and determine the GCF.
1. 18, 24
2. 45, 75
3. 72, 60
4. 168, 175
5. 144, 245
6. 15, 50, 60
7. 14, 63, 70
11. 15x, 30
12. 14x, 21
13. 45x 4 , 8x 3
16. 12x 3 , 4x 2 , 6x
20. 15x (x + 2) , 9 (x + 2)
24. 12y 5 + 7y 2 = y 2 ( ? )
27. 36a5 b
7
− 60a6 b 5 = 12a5 b 5 ( ? )
29. 4x −8
30. 27x −9
31. 3x − 18
32. 5x − 10
33. 25x − 16
34. 72x − 35
36. 14a2 − 7a
38. 8x 4 − 16x 2
39. 5x 6 + x3
40. 3x 7 − 9x 5
43. 27x 3 − 6x 2 + 3x
44. 8x 3 − 12x 2 + 2x
45. 9x 4 + 18x 3 − 3x 2
49. 12x 5 y 2 − 8x 3 y
4 3
50. 125a8 b c − 25a2 b 3 c3
51. 6x 4 y 3 − 4x 3 y 2 + 8x 2 y
55. 2x (x − 3) + 5 (x − 3)
60. (x + 6) − 3x 2 (x + 6)
61. (a + b) − 3a(a + b)
2 2
65. 4x 2 − 16x = 4x (x − 4)
67. 3x 3 − 5x 6 = x 3 (3 − x 2 )
69. x 3 − x 2 + x = x (x 2 − x)
Use polynomial long division to show that the given factor divides the polynomial
evenly.
75. The height in feet of an object tossed into the air is given by the function
h (t) = −16t 2 + 32t, where t is the time in seconds after it is tossed.
Write the function in factored form.
76. The height in feet of an object dropped from a 16‑foot ladder is given by
the function h (t) = −16t 2 + 16 , where t is the time in seconds after it
is tossed. Write the function in factored form.
78. The surface area of a cone is given by the formula SA = πr2 + πrs,
where r represents the radius of the base and s represents the slant height.
Express this formula in factored form.
Factor by grouping.
79. x 2 − 10x + 2x − 20
80. x 2 − 6x − 3x + 18
81. x 3 + 2x 2 + 2x + 4
82. x 3 − 3x 2 + 5x − 15
83. x 3 + 7x 2 − 2x − 14
84. 2x 3 + 2x 2 − x − 1
85. x 3 − 5x 2 + 4x − 20
86. 6x 3 − 3x 2 + 2x − 1
87. 9x 3 − 6x 2 − 3x + 2
88. 2x 4 − x 3 − 6x + 3
89. x 5 + x 3 + 2x 2 + 2
90. 6x 5 − 4x 3 − 9x 2 + 6
94. a4 − 3a3 b 2 + ab 2 − 3b 4
4
95. 3a2 b − 6b 3 − a2 b + 2
96. 3x 3 + 2y 3 + x 3 y 3 + 6
97. −3x 3 − 5y 3 + x 3 y 3 + 15
98. 2x 3 y 3 + 2 − y 3 − 4x 3
99. 3x 2 − y 3 + xy 2 − 3xy
100. 2x 2 + y 3 − 2xy − xy 2
103. 2x 3 + 6x 2 − 10x − 30
104. 6x 3 − 3x 2 − 42x + 21
107. −12x 5 + 4x 4 + 6x 3 − 2x 2
111. Research the Euclidean algorithm for finding the GCF of two natural
numbers. Give an example that illustrates the steps.
115. Make up a factoring problem of your own and provide the answer. Post
the problem and the solution on the discussion board.
ANSWERS
1: 18 = 2 ⋅ 3 2 , 24 = 2 3 ⋅ 3, GCF = 6
3: 72 = 2 3 ⋅ 3 2, 60 = 2 2 ⋅ 3 ⋅ 5, GCF = 12
7: 14 = 2 ⋅ 7, 63 = 3 2 ⋅ 7, 70 = 2 ⋅ 5 ⋅ 7, GCF = 7
9: 60 = 2 2 ⋅ 3 ⋅ 5, 72 = 2 3 ⋅ 3 2, 900 = 2 2 ⋅ 3 2 ⋅ 5 2, GCF = 12
11: 15
13: x 3
15: 3x
17: 12xy
2
19: 3ab
21: 4(2x − 1)
23: (5x + 2)
29: 4 (x − 2)
31: 3(x − 6)
33: Prime
35: 15x (x + 2)
39: x 3 (5x + 1)
3
41: 6 (3a2 + 5a − 1)
43: 3x (9x 2 − 2x + 1)
45: 3x 2 (3x + 6x − 1)
2
(x − 3x − 2x + 4)
4 2
47: 7x
49: 4x 3 y (3x 2 y − 2)
53: 9x 2 y 5 z 2 (9x y − 2y z + 1)
5 3 2
55: (x − 3)(2x + 5)
65: Yes
67: No
69: No
77: SA = 2πr (r + h)
79: (x − 10) (x + 2)
81: (x + 2) (x 2 + 2)
83: (x + 7) (x 2 − 2)
85: (x − 5) (x 2 + 4)
89: (x 2 + 1) (x 3 + 2)
97: (x 3 − 5) (y 3 − 3)
99: (x − y) (3x + y 2 )
101: 5 (x − 7) (x − 3)
103: 2 (x + 3) (x 2 − 5)
105: 4x (x + 1) (x 2 − 3)
LEARNING OBJECTIVES
The middle term of the trinomial, 7x, is the sum of the products of the outer and
inner terms of the binomials:
935
Chapter 6 Factoring and Solving by Factoring
And the product of the last terms of each binomial is equal to the last term of the
trinomial.
This gives us
13. Describes the method of In short, if the leading coefficient of a factorable trinomial is one, then the factors
factoring a trinomial by of the last term must add up to the coefficient of the middle term. This observation
systematically checking factors is the key to factoring trinomials using the technique known as trial and error13
to see if their product is the
(or guess and check14). The steps are outlined in the following example.
original trinomial.
Solution: Note that the polynomial to be factored has three terms; it is a trinomial
with a leading coefficient of 1. Use trial and error to factor as follows:
Step 1: Write two sets of blank parentheses. If a trinomial of this form factors, then
it will factor into two linear binomial factors.
Step 2: Write the factors of the first term in the first space of each set of
parentheses. In this case, factor x 2 = x ⋅ x .
Step 3: Determine the factors of the last term whose sum equals the coefficient of
the middle term. To do this, list all of the factorizations of 12 and search for factors
whose sum equals the coefficient of the middle term, 7.
Choose 12 = 3 ⋅ 4 because 3 + 4 = 7.
Step 4: Write in the last term of each binomial using the factors determined in the
previous step.
Answer: (x + 3) (x + 4)
Since multiplication is commutative, the order of the factors does not matter.
If the last term of the trinomial is positive, then either both of the constant factors
must be negative or both must be positive. Therefore, when looking at the list of
factorizations of the last term, we are searching for sums that are equal to the
coefficient of the middle term.
In this case, choose −4 and −5 because (−4) (−5) = +20and −4 + (−5) = −9.
Check.
Answer: (x − 4) (x − 5)
If the last term of the trinomial is negative, then one of its factors must be negative.
In this case, search the list of factorizations of the last term for differences that
equal the coefficient of the middle term.
The factors of 12 are listed below. In this example, we are looking for factors whose
difference is −4.
Multiply to check.
Answer: (x + 2) (x − 6)
Often our first guess will not produce a correct factorization. This process may
require repeated trials. For this reason, the check is very important and is not
optional.
Example 4: Factor: x 2 + 5x − 6.
Suppose we choose the factors 2 and 3 because 2 + 3 = 5, the coefficient of the middle
term. Then we have the following incorrect factorization:
In this case, the middle term is correct but the last term is not. Since the last term
in the original expression is negative, we need to choose factors that are opposite in
sign. Therefore, we must try again. This time we choose the factors −1 and 6 because
−1 + 6 = 5.
Answer: (x − 1) (x + 6)
If we choose the factors wisely, then we can reduce much of the guesswork in this
process. However, if a guess is not correct, do not get discouraged; just try a
different set of factors.
Solution:
Here there are no factors of 20 whose sum is 3. Therefore, the original trinomial
cannot be factored as a product of two binomials. This trinomial is prime.
Answer: Prime
Answer: (x + 2) (x − 15)
Video Solution
The techniques described can also be used to factor trinomials with more than one
variable.
Next, look for factors of the coefficient of the last term, 72, whose sum is −14.
Therefore, the coefficient of the last term can be factored −72 = 4 (−18), where
Multiply to check.
Video Solution
An alternate technique for factoring trinomials, called the AC method15, makes use
of the grouping method for factoring four-term polynomials. If a trinomial in the
form ax 2 + bx + c can be factored, then the middle term, bx, can be replaced with
two terms with coefficients whose sum is b and product ac. This substitution results
in an equivalent expression with four terms that can be factored by grouping. The
steps are outlined in the following example.
15. Method for factoring Solution: In this example a = 1, b = −1, and c = −30.
trinomials by replacing the
middle term with two terms
that allow us to factor the Step 1: Determine the product ac.
resulting four-term polynomial
by grouping.
Step 2: Find factors of ac whose sum equals the coefficient of the middle term, b.
We can see that the sum of the factors 5 and −6 is equal to the coefficient of the
middle term, −1.
Step 3: Use the factors as coefficients for the terms that replace the middle term.
Here −x = −6x + 5x . Write
Answer: (x − 6) (x + 5)
Notice that the AC method is consistent with the trial and error method. Both
methods require that b = m + n, where c = mn. In the example above,
Therefore, −14x = −6x − 8x . Substitute the new terms and factor by grouping.
At this point, it is recommended that the reader stop and factor as many trinomials
of the form x 2 + bx + c as time allows before moving on to the next section.
Factoring trinomials is one of the more important skills that we learn in this course
and should be mastered.
KEY TAKEAWAYS
TOPIC EXERCISES
1. x 2 + 5x − 6 = (x + 2) (x + 3)
2. x 2 + 6x + 16 = (x + 8) (x − 2)
3. y 2 + 2y − 8 = (y + 4) (y − 2)
4. y 2 − 10y + 21 = (y − 3) (y − 7)
− 10a + 25 = (a − 5)
2
5. a2
6. a2 + 6a + 9 = (a − 3)2
7. x 2 + 10x − 25 = (x + 5) (x − 5)
8. x 2 + 5x + 14 = (x − 2) (x + 7)
10. y 2 − 3y + 2 = (y − 2) (y − 1)
Factor.
11. x 2 + 6x + 8
12. x 2 + 4x + 3
13. x 2 + 3x + 2
14. x 2 +x−2
15. x 2 + 3x − 10
16. x 2 − 2x − 35
17. x 2 − 13x + 12
18. x 2 − 15x + 36
19. x 2 − 12x + 36
20. x 2 + 18x + 81
21. x 2 − 2x + 1
22. x 2 − 18x + 81
23. x 2 + 5x + 5
24. x 2 − 4x + 6
25. x 2 − 20x + 91
26. x 2 + 20x + 91
27. x 2 − 2x − 48
28. x 2 + 16x + 48
29. x 2 + 22x + 48
30. x 2 + 22x − 48
31. y 2 + 7y + 12
32. y 2 + 8y − 20
33. y 2 − 16y + 60
34. y 2 − 31y − 32
35. a2 − 11a − 42
36. a2 − 14a − 51
37. a2 + 26a + 25
39. a2 + 4a − 1
40. a2 − 6a + 2
41. y 2 − 14x + 40
42. y 2 − 4y − 96
43. x 2 − 2xy + y 2
44. x 2 + 2xy + y 2
49. x 2 y 2 − 6xy + 9
50. x 2 y 2 + 25xy − 26
51. a2 + 4ab + 4b 2
53. a2 − ab − 12b 2
59. x 2 + 5x − 14
60. x 2 + 2x − 48
61. x 2 − 9x + 8
62. x 2 − 14x + 24
63. x 2 − x − 72
64. x 2 − x − 90
65. y 2 − 8y + 16
66. y 2 + 16y + 64
67. x 2 + 4x + 12
68. x 2 + 5x − 8
71. Create your own trinomial of the form x 2 + bx + c that factors. Share
it along with the solution on the discussion board.
72. Write out your own list of steps for factoring a trinomial of the form
x 2 + bx + c and share your steps on the discussion board.
73. Create a trinomial that does not factor and share it along with an
explanation of why it does not factor.
ANSWERS
1: No
3: Yes
5: Yes
7: No
9: Yes
11: (x + 2) (x + 4)
13: (x + 1) (x + 2)
15: (x − 2) (x + 5)
17: (x − 1) (x − 12)
19: (x − 6)
2
21: (x − 1) 2
23: Prime
25: (x − 7) (x − 13)
27: (x + 6) (x − 8)
29: Prime
31: (y + 3) (y + 4)
33: (y − 6) (y − 10)
35: (a + 3) (a − 14)
37: (a + 1) (a + 25)
39: Prime
41: (y − 10) (y − 4)
43: (x − y)
2
45: (x − 15y) (x − y)
49: (xy − 3)
2
51: (a + 2b)
2
57: A(x) = (x − 7) 2
59: (x + 7) (x − 2)
61: (x − 8) (x − 1)
63: (x − 9) (x + 8)
65: (y − 4)
2
67: Prime
LEARNING OBJECTIVES
As we have seen before, the product of the first terms of each binomial is equal to
the first term of the trinomial. The middle term of the trinomial is the sum of the
products of the outer and inner terms of the binomials. The product of the last
terms of each binomial is equal to the last term of the trinomial. Visually, we have
the following:
955
Chapter 6 Factoring and Solving by Factoring
In general,
obtained. Multiply (2x + 5) (3x + 7) and carefully follow the formation of the
error method. The key lies in the understanding of how the middle term is
middle term.
If we think of the FOIL method for multiplying binomials, then the middle term
results from the sum of the inner product and the outer product. In this case,
14x + 15x = 29x , as illustrated below:
For this reason, we need to look for products of the factors of the first and last
terms whose sum is equal to the coefficient of the middle term. For example, to
factor 6x 2 + 29x + 35, look at the factors of 6 and 35.
Example 1: Factor: 3x 2 + 7x + 2.
Solution: Since the leading coefficient and the last term are both prime, there is
only one way to factor each.
The middle and last term are both positive; therefore, the factors of 2 are chosen as
positive numbers. In this case, the only choice is in which grouping to place these
factors.
Notice that these products differ only in their middle terms. Also, notice that the
middle term is the sum of the inner and outer product, as illustrated below:
Answer: (x + 2) (3x + 1)
Solution: First, consider the factors of the first and last terms.
We search for products of factors whose sum equals the coefficient of the middle
term, 38. For brevity, the thought process is illustrated starting with the factors 2
and 6. Factoring begins at this point with the first term.
We search for factors of 20 that along with the factors of 12 produce a middle term
of 38x.
Solution: First, consider the factors of the first and last terms.
We are searching for products of factors whose sum equals the coefficient of the
middle term, −23. Factoring begins at this point with two sets of blank parentheses:
Since the last term is positive and the middle term is negative, we know that both
factors of the last term must be negative. Here we list all possible combinations
with the factors of 10x 2 = 2x ⋅ 5x .
Since the leading coefficient is prime, we can begin with the following:
We look for products of the factors of 5 and 16 that could possibly add to 38.
Since the last term is negative, we must look for factors with opposite signs. Here
we can see that the products 2 and 40 add up to 38 if they have opposite signs:
Therefore, use −2 and 8 as the factors of 16, making sure that the inner and outer
products are −2x and 40x :
After lots of practice, the process described in the previous example can be
performed mentally.
Video Solution
Solution: Search for factors of the first and last terms such that the sum of the
inner and outer products equals the middle term.
Video Solution
It is a good practice to first factor out the GCF, if there is one. Doing this produces a
trinomial factor with smaller coefficients. As we have seen, trinomials with smaller
coefficients require much less effort to factor. This commonly overlooked step is
worth identifying early.
After factoring out 3, the coefficients of the resulting trinomial are smaller and
have fewer factors.
After some thought, we can see that the combination that gives the coefficient of
the middle term is 4 (−2) + 1 (−1) = −8 − 1 = −9.
Check.
The factor 3 is part of the factored form of the original expression; be sure to
include it in the answer.
Answer: 3 (4x − 1) (x − 2)
Solution: In this example, the leading coefficient is −1. Before beginning the
factoring process, factor out the −1:
At this point, factor the remaining trinomial as usual, remembering to write the −1
as a factor in your final answer. Because 3 + (−5) = −2, use 3 and 5 as the factors of 15.
Solution: The GCF of all the terms is 5. However, in this case factor out −5 because
this produces a trinomial factor where the leading coefficient is positive.
Focus on the factors of 12 and 6 that combine to give the middle coefficient, 1.
Answer: −1 (x − 6) (x + 4)
Video Solution
In this case, the sum of the factors −6 and −15 equals the middle coefficient, −21.
Therefore, −21x = −6x − 15x , and we can write
The sum of factors 2 and −63 equals the middle coefficient, −61. Replace −61x with
2x − 63x :
KEY TAKEAWAYS
TOPIC EXERCISES
Factor.
1. 3x 2 − 14x − 5
2. 5x 2 + 7x + 2
3. 2x 2 + 5x − 3
4. 2x 2 + 13x − 7
5. 2x 2 + 9x − 5
6. 7x 2 + 20x − 3
7. 7x 2 − 46x − 21
8. 3x 2 +x−2
9. 5x 2 + 34x − 7
10. 5x 2 − 28x − 12
11. 9x 2 − 12x + 4
12. 4x 2 − 20x + 25
15. 2x 2 + 7x + 16
16. 6x 2 − 19x − 10
19. 12y 2 − 8y + 1
21. 9x 2 − 12xy + 4y 2
25. 4x 2 y 2 + 16xy − 9
Factor.
29. 6x 2 − 20x − 16
32. 3x 2 + 39x − 90
39. 6x 2 y 2 + 46xy + 28
45. −x 2 − 4x + 21
46. −x 2 + x + 12
47. −x 2 + 15x − 56
48. −x 2 + x + 72
49. −y 2 + 10y − 25
50. −y 2 − 16y − 64
51. 36 − 9a − a2
52. 72 − 6a − a2
53. 32 + 4x − x 2
Factor out a negative common factor first and then factor further if possible.
55. −8x 2 + 6x + 9
57. −18x 2 − 6x + 4
58. 2 + 4x − 30x 2
65. The height in feet of a projectile launched from a tower is given by the
function h(t) = −16t 2 + 64t + 80 , where t represents the number of
seconds after launch. Rewrite the given function in factored form.
66. The height in feet of a projectile launched from a tower is given by the
function h(t) = −16t 2 + 64t + 192 , where t represents the number of
seconds after launch. Rewrite the given function in factored form.
67. 2x 2 + 5x − 7
68. 3x 2 + 7x − 10
69. 4x 2 − 25x + 6
71. 6x 2 + 23x − 18
72. 8x 2 + 10x − 25
73. 4x 2 + 28x + 40
74. −6x 2 − 3x + 30
78. Write out your own list of steps for factoring a trinomial of the form
ax 2 + bx + c and share it on the discussion board.
79. Create a trinomial of the form ax 2 + bx + c that does not factor and
share it along with the reason why it does not factor.
ANSWERS
1: (x − 5) (3x + 1)
3: (x + 3) (2x − 1)
5: (x + 5) (2x − 1)
7: (x − 7) (7x + 3)
9: (x + 7) (5x − 1)
11: (3x − 2) 2
13: (7x + 1) 2
15: Prime
29: 2 (x − 4) (3x + 2)
31: 5(2x − 1) 2
41: 5x (x − 12) (x − 1)
45: −1 (x − 3) (x + 7)
47: −1 (x − 7) (x − 8)
49: −1(y − 5)
2
51: −1 (a − 3) (a + 12)
53: −1 (x − 8) (x + 4)
61: −2 (x − 14) (x + 1)
63: −3y 2 (x − 4) (x − 2)
67: (x − 1) (2x + 7)
69: (x − 6) (4x − 1)
73: 4 (x + 2) (x + 5)
LEARNING OBJECTIVES
Difference of Squares
A binomial is a polynomial with two terms. We begin with our first special binomial
called difference of squares16:
Solution:
Step 1: Identify the binomial as difference of squares and determine the square
16. a2 − b = (a + b) (a − b) ,
factors of each term.
2
978
Chapter 6 Factoring and Solving by Factoring
Answer: (x + 4) (x − 4)
It is worth taking some extra time at this point to review all of the squares of
integers from 1 to 12.
Recognizing these perfect square integers helps speed the factoring process.
In this case, a = 3x and b = 11. Substitute into the formula for difference of
squares.
It may be the case that the terms of the binomial have a common factor. If so, it will
be difficult to identify the perfect squares until we first factor out the GCF.
Solution: The terms are not perfect squares. However, notice that they do have a
common factor. First, factor out the GCF, 3.
Solution: Here we have a binomial with two variables and recognize that it is a
difference of squares.
Video Solution
Therefore,
When the degree of the special binomial is greater than two, we may need to apply
the difference of squares formula multiple times. A polynomial is completely
factored when none of the factors can be factored any further.
2
17. a2 + b does not have a
general factored equivalent.
To do this, recall the power rule for exponents, (x m )n = x mn . When exponents are
raised to a power, multiply them. With this in mind, determine that (x 2 ) = x 4
2
and write
At this point, notice that the factor (x 2 − 4) is itself a difference of two squares
and thus can be further factored using a = x and b = 2. The factor (x 2 + 4) is a
sum of squares, which cannot be factored using real numbers.
Answer: (x 2 + 4) (x + 2) (x − 2)
Video Solution
18. a3 + b3 = (a + b) (a2 − ab + b2 ),
(click to see video)
where a and b represent
algebraic expressions. Sum and Difference of Cubes
19. a3 − b3 = (a − b) (a2 + ab + b2 ),
Two other special binomials of interest are the sum18 and difference of cubes19:
where a and b represent
algebraic expressions.
The process for factoring the sum and difference of cubes is very similar to that for
the difference of squares. We first identify a and b and then substitute into the
appropriate formula. The separate formulas for sum and difference of cubes allow
us to always choose a and b to be positive.
Example 6: Factor: x 3 + 8.
Solution: The plus sign and the fact that the terms are cubes indicate to us that this
is a sum of cubes.
The resulting trinomial is prime and the factoring is complete. We can check this
factorization by multiplying.
Answer: (x + 2) (x 2 − 2x + 4)
It is helpful to review the perfect cubes of integers from 1 to 12. This will aid you in
identifying sums and differences of cubes.
We can write
Answer: (y − 5) (y 2 + 5y + 25)
Always look for common factors when factoring. If the terms of the binomial have a
GCF other than 1, then factor that out first.
Example 9: Factor: x 3 y 3 − 1.
Solution: This binomial is a difference of cubes with two variables. Identify what is
being cubed.
Answer: (xy − 1) (x 2 y 2 + xy + 1)
Video Solution
Solution: When confronted with a binomial that is both a difference of squares and
cubes, as this is, make it a rule to factor using difference of squares first.
The resulting two binomial factors are a sum and a difference of cubes. Each can be
factored further.
Therefore, we have
The trinomial factors are prime and the expression is completely factored.
Answer: (x + 2) (x 2 − 2x + 4) (x − 2) (x 2 + 2x + 4)
As an exercise, factor the previous example as a difference of cubes first and then
compare the results. Why do you think we make it a rule to factor using difference
of squares first?
Answer: (x + y) (x 2 − xy + y 2 ) (x − y) (x 2 + xy + y 2 )
Video Solution
KEY TAKEAWAYS
TOPIC EXERCISES
Factor completely.
1. x 2 −9
2. x 2 − 100
3. y 2 − 36
4. y 2 − 144
5. x 2 +4
6. x 2 −5
7. m 2 − 81
8. m 2 − 64
9. 16x 2 −9
10. 25x 2 −4
11. 144x 2 −1
12. 9x 2 − 121
13. 4y 2 − 81
14. 100y 2 − 49
15. 9 − 4x 2
16. 100 − x2
17. 1 − y2
18. 25 − 9y 2
19. −3x 2 + 75
20. −16x 2 + 25
21. 2x 2 − 72
25. x 2 − y2
26. 25x 2 − 9y 2
27. a2 − 4b 2
2
28. a2 b − 36
29. 4x 2 y 2 −1
30. x 2 y 2 − 25
33. −100xy 3 + 4x 3 y
35. (x + 1) 2 − y 2
− (y − 2)
2
36. x 2
− 3) 2 − (y + 3)
2
37. (x
38. (x 2 + 2) − (x − 1) 2
2
39. (x 2 − 1) − (2x + 3) 2
2
40. x 4 −1
41. x 4 − y4
42. 16x 4 − 81
4
43. a4 b − 16
44. a4 − 16b 4
45. x 8 −1
46. 25x 8 −1
47. a8 − b2
48. a4 −9
49. x 8 − y8
50. 81x 8 −1
51. The height of a projectile dropped from a 64-foot tower is given by the
function h(t) = −16t 2 + 64 , where t represents the time in seconds
after it is dropped. Rewrite this function in factored form. (Hint: Factor out
−16 first.)
52. The height of a projectile dropped from a 36-foot tower is given by the
function h(t) = −16t 2 + 36 , where t represents the time in seconds
after it is dropped. Rewrite this function in factored form.
Factor completely.
53. x 3 −1
54. x 3 +1
55. y 3 − 27
56. y 3 −8
57. 8y 3 +1
58. 27y 3 −1
60. 8a3 + 27
61. a3 + 216
62. a3 − 125
63. x 3 − 1000
64. 343m 3 −1
65. 512n 3 +1
66. 8x 3 + 343
69. 27y 3 − 64
70. x 3 +3
71. 5x 3 +1
72. 1 − y3
73. 27 − 1,000y 3
75. x 3 − y3
76. x 3 + y3
77. x 3 y 3 + 125
78. 8x 3 y 3 − 27
79. 27a3 − 8b 3
81. 128x 3 + 2y 3
82. 54x 3 − 2y 3
85. (x + 1) 3 − 8
− (x − 5)
3
86. 8x 3
87. (x − 2) 3 + (x + 2) 3
+ 3)3 + (b − 3)
3
88. (a
89. x 6 −1
90. x 6 +1
91. 64a6 −1
92. a6 − 81b 6
93. x 6 − y6
94. x 6 + y6
96. What can be said about the degrees of the factors of a polynomial? Give
an example.
97. Make up your own difference of squares factoring exercise and provide
the answer. Explain how you solved it.
98. Make up your own sum or difference of cubes factoring exercise and
provide the answer. Explain how you solved it.
ANSWERS
1: (x + 3) (x − 3)
3: (y + 6) (y − 6)
5: Prime
7: (m + 9) (m − 9)
9: (4x + 3) (4x − 3)
17: (1 + y) (1 − y)
19: −3 (x + 5) (x − 5)
21: 2 (x + 6) (x − 6)
25: (x + y) (x − y)
35: (x + 1 + y) (x + 1 − y)
37: (x + y) (x − y − 6)
39: (x 2 + 2x + 2) (x 2 − 2x − 4)
41: (x 2 + y 2 ) (x + y) (x − y)
45: (x 4 + 1) (x 2 + 1) (x + 1) (x − 1)
49: (x 4 + y 4 ) (x 2 + y 2 ) (x + y) (x − y)
53: (x − 1) (x 2 + x + 1)
55: (y − 3) (y 2 + 3y + 9)
71: Prime
75: (x − y) (x 2 + xy + y 2 )
85: (x − 1) (x 2 + 4x + 7)
87: 2x (x 2 + 12)
89: (x + 1) (x 2 − x + 1) (x − 1) (x 2 + x + 1)
93: (x + y) (x 2 − xy + y 2 ) (x − y) (x 2 + xy + y 2 )
LEARNING OBJECTIVE
1. Check for common factors. If the terms have common factors, then
factor out the greatest common factor (GCF) and look at the resulting
polynomial factors to factor further.
2. Determine the number of terms in the polynomial.
a2 − b2 = (a + b) (a − b)
Difference of
squares:
a3 − b3 = (a − b) (a2 + ab + b2 )
Difference of
cubes:
999
Chapter 6 Factoring and Solving by Factoring
Note
If an expression has a GCF, then factor this out first. Doing so is often overlooked
and typically results in factors that are easier to work with. Also, look for the
resulting factors to factor further; many factoring problems require more than one
step. A polynomial is completely factored when none of the factors can be factored
further.
Solution: This four-term polynomial has a GCF of 3x . Factor this out first.
Answer: 3x (2x − 1) (x + 2) (x − 2)
Solution: This trinomial has a GCF of 2xy . Factor this out first.
correct coefficient for the middle term: 3 (−5) + 3 (−5) = −15 − 15 = −30.
Check.
Answer: 2xy(3x − 5)
2
Solution: This trinomial has a GCF of 5ab2. Factor this out first.
Video Solution
Solution: This binomial does not have a GCF. Therefore, begin factoring by
identifying it as a difference of squares.
We can write
Therefore,
Answer: (x + 2) (x + 3) (x 2 − 3x + 9)
Video Solution
KEY TAKEAWAYS
• Use the polynomial type to determine the method used to factor it.
• It is a best practice to look for and factor out the greatest common factor
(GCF) first. This will facilitate further factoring and simplify the process.
Be sure to include the GCF as a factor in the final answer.
• Look for resulting factors to factor further. It is often the case that
factoring requires more than one step.
• If a binomial can be considered as both a difference of squares and a
difference of cubes, then first factor it as a difference of squares. This
results in a more complete factorization.
TOPIC EXERCISES
Factor completely.
1. 2x 5 y 2 − 12x 4 y 3
2. 18x 5 y 3 − 6x 4 y 5
3. 5x 2 + 20x − 25
4. 4x 2 + 10x − 6
5. 24x 3 − 30x 2 − 9x
7. 6x 3 + 27x 2 − 9x
9. 5x 3 − 30x 2 − 15x + 90
10. 6x 4 + 24x 3 − 2x 2 − 8x
11. x 4 − 6x 3 + 8x − 48
13. 4x 3 − 4x 2 − 9x + 9
15. 2x 3 + 250
16. 3x 5 − 81x 2
17. 2x 5 − 162x 3
18. 4x 4 − 36
19. x 4 + 16
20. x 3 +9
21. 72 − 2x 2
22. 5x 4 − 25x 2
23. 7x 3 − 14x
29. a4 − 16
31. y 5 + y4 − y − 1
32. 4y 5 + 2y 4 − 4y 2 − 2y
33. 3x 8 − 192x 2
34. 4x 7 + 4x
37. 5x 5 − 3x 4 − 5x 3 + 3x 2
2
38. 4a2 b − 4a2 − 9b 2 + 9
41. 6x 2 + 5xy + 6y 2
43. (3x − 1) 2 − 64
44. (x − 5) − (x − 2) 2
2
45. (x + 1) 3 + 8
46. (x − 4) 3 − 27
48. (x − 4) 2 + 5 (x − 4) + 6
54. 9y 6 − 13y 4 + 4y 2
57. x 4 −1
58. 16x 4 − 64
59. x 8 −1
60. 81x 8 −1
61. x 16 −1
62. x 12 −1
68. 5m 2 n 2 + 10mn − 15
70. The volume of a certain right circular cylinder is given by the function
V(x) = 4πx 3 − 4πx 2 + πx . Write the function in its factored form.
71. First, factor the trinomial 24x 2 − 28x − 40 . Then factor out the
GCF. Discuss the significance of factoring out the GCF first. Do you obtain the
same result?
ANSWERS
1: 2x 4 y 2 (x − 6y)
3: 5 (x − 1) (x + 5)
5: 3x (2x − 3) (4x + 1)
7: 3x (2x 2 + 9x − 3)
9: 5 (x − 6) (x 2 − 3)
11: (x − 6) (x + 2) (x 2 − 2x + 4)
15: 2 (x + 5) (x 2 − 5x + 25)
17: 2x 3 (x + 9) (x − 9)
19: Prime
21: 2 (6 + x) (6 − x)
23: 7x (x 2 − 2)
25: (5x + 1)
2
27: − (x − 3) (7x + 2)
29: (a2 + 4) (a + 2) (a − 2)
31: (y 2 + 1) (y − 1) (y + 1)
2
33: 3x 2 (x + 2) (x 2 − 2x + 4) (x − 2) (x 2 + 2x + 4)
37: x 2 (5x − 3) (x + 1) (x − 1)
41: Prime
43: 3 (x − 3) (3x + 7)
45: (x + 3) (x 2 + 3)
47: 2 (x + 1) (2x − 5)
55: −3 (a + 4) (2a − 3)
57: (x 2 + 1) (x + 1) (x − 1)
59: (x 4 + 1) (x 2 + 1) (x + 1) (x − 1)
61: (x 8 + 1) (x 4 + 1) (x 2 + 1) (x + 1) (x − 1)
69: V(x) = x (x + 1) (x − 3)
LEARNING OBJECTIVES
Learning how to solve equations is one of our main goals in algebra. Up to this
point, we have solved linear equations, which are of degree 1. In this section, we
will learn a technique that can be used to solve certain equations of degree 2. A
quadratic equation20 is any equation that can be written in the standard form21
where a, b, and c are real numbers and a ≠ 0. The following are some examples of
quadratic equations, all of which will be solved in this section:
1013
Chapter 6 Factoring and Solving by Factoring
Solution: To verify solutions, substitute the values for x and then simplify to see if a
true statement results.
Answer: Both values produce true statements. Therefore, they are both solutions to
the equation.
In other words, if any product is equal to zero, then one or both of the variable
factors must be equal to zero.
Example 2: Solve: (x − 8) (x + 7) = 0.
To verify that these are solutions, substitute them for the variable x.
Example 3: Solve: x 2 + 3x − 10 = 0.
Solution: The goal is to produce a product that is equal to zero. We can do that by
factoring the trinomial on the left side of the equation.
Next, apply the zero-product property and set each factor equal to zero.
This leaves us with two linear equations, each of which can be solved for x .
Check the solutions by substituting into the original equation to verify that we
obtain true statements.
Solution:
Step 1: Express the quadratic equation in standard form. For the zero-product
property to apply, the quadratic expression must be equal to zero. Use the addition
and subtraction properties of equality to combine opposite-side like terms and
obtain zero on one side of the equation.
Step 3: Apply the zero-product property and set each variable factor equal to zero.
This is a perfect square trinomial. Hence setting each factor equal to zero results in
a repeated solution.
A repeated solution is called a double root25 and does not have to be written twice.
Video Solution
Not all quadratic equations in standard form are trinomials. We often encounter
binomials.
Example 6: Solve: x 2 − 9 = 0.
Solution: This quadratic equation is given in standard form, where the binomial on
the left side is a difference of squares. Factor as follows:
Answer: The solutions are 3 and −3, which can also be written as ±3.
Next, factor the expression. Notice that the binomial on the left has a GCF of 5x .
Once it is in standard form, we can factor and then set each factor equal to zero.
Solution: We begin by factoring out the GCF of 5. Then factor the resulting
trinomial.
Next, we set each variable factor equal to zero and solve for x.
Notice that the factor 5 is not a variable factor and thus did not contribute to the
solution set.
Solution: Clear the fractions by multiplying both sides of the equation by the LCD,
which is equal to 6.
At this point, we have an equivalent equation with integer coefficients and can
factor as usual. Begin with the factors of 15 and 2.
Video Solution
We use this property to find equations, given the solutions. To do this, the steps for
solving by factoring are performed in reverse.
Example 12: Find a quadratic equation with integer coefficients, given solutions 1/
2 and −3/4.
Answer: 8x 2 + 2x − 3 = 0
Try this! Find a quadratic equation with integer coefficients, given solutions −1 and
2/3.
Answer: 3x 2 + x − 2 = 0
Video Solution
The zero-product property is true for any number of factors that make up an
equation. If an expression is equal to zero and can be factored into linear factors,
then we will be able to set each factor equal to zero and solve for each equation.
Notice that the degree of the polynomial is 3 and we obtained three solutions. In
general, for any polynomial equation with one variable of degree n, the
fundamental theorem of algebra26 guarantees n real solutions or fewer. We have
seen that many polynomials do not factor. This does not imply that equations
involving these unfactorable polynomials do not have real solutions. In fact, many
polynomial equations that do not factor do have real solutions. We will learn how to
solve these types of equations as we continue in our study of algebra.
Video Solution
KEY TAKEAWAYS
TOPIC EXERCISES
Determine whether the given set of values are solutions to the quadratic equation.
1. {−3, 5}; x 2 − 2x − 15 = 0
2. {7, −1}; x 2 − 6x − 7 = 0
1
3. {−1/2, 1/2}; x 2 − =0
4
9
4. {−3/4, 3/4}; x 2 − =0
16
6. {−5, 1}; x 2 − 4x − 5 = 0
Solve.
7. (x − 3) (x + 2) = 0
8. (x + 5) (x + 1) = 0
9. (2x − 1) (x − 4) = 0
11. (x − 2) 2 = 0
12. (5x + 3) = 0
2
13. 7x (x − 5) = 0
15. (x − 12 ) (x + 34 ) = 0
16. (x + 58 ) (x − 38 ) = 0
17. ( x + 12 ) ( 16 x − 23 ) = 0
1
4
18. ( x − 3) = 0
1 2
5
19. −5 (x + 1) (x − 2) = 0
20. 12 (x − 7) (x − 6) = 0
21. (x + 5) (x − 1) = 0
22. (x + 5) (x + 1) = 0
24. 5(7x − 8) 2 = 0
Solve.
25. x 2 −x−6=0
26. x 2 + 3x − 10 = 0
27. y 2 − 10y + 24 = 0
28. y 2 + 6y − 27 = 0
29. x 2 − 14x + 40 = 0
30. x 2 + 14x + 49 = 0
31. x 2 − 10x + 25 = 0
32. 3x 2 + 2x − 1 = 0
33. 5x 2 − 9x − 2 = 0
34. 7y 2 + 20y − 3 = 0
35. 9x 2 − 42x + 49 = 0
37. 2y 2 +y−3=0
38. 7x 2 − 11x − 6 = 0
39. 2x 2 = −15x + 8
40. 8x − 5 = 3x 2
41. x 2 − 36 = 0
42. x 2 − 100 = 0
43. 4x 2 − 81 = 0
45. x 2 =4
46. 9y 2 =1
47. 16y 2 = 25
48. 36x 2 = 25
49. 4x 2 − 36 = 0
50. 2x 2 − 18 = 0
52. −3x 2 + 6x = 0
54. x 2 =0
55. (x + 1) 2 − 25 = 0
56. (x − 2) 2 − 36 = 0
57. 5x (x − 4) = −4 + x
58. (x − 1) (x − 10) = 22
59. (x − 3) (x − 5) = 24
60. −2x (x − 9) = x + 21
61. (x + 1) (6x + 1) = 2x
Clear the fractions by first multiplying both sides by the LCD and then solve.
65. 1
15
x2 + 1
3
x+ 2
5
=0
66. 1
14
x2 − 1
2
x+ 3
7
=0
67. 3
2
x2 − 2
3
=0
68. 5
2
x2 − 1
10
=0
69. 3
14
x2 − 21
2
=0
70. 1
3
x2 − 1
5
x=0
71. 1
32
x2 − 1
2
x+2=0
72. 1
3
x2 + 5
6
x− 1
2
=0
73. The sides of a square measure x + 3 units. If the area is 25 square units,
then find x.
74. The height of a triangle is 2 units more than its base. If the area is 40
square units, then find the length of the base.
75. The sides of a right triangle have measures that are consecutive integers.
Find the length of the hypotenuse. (Hint: The hypotenuse is the longest side.
Apply the Pythagorean theorem.)
76. The profit in dollars generated by producing and selling x custom lamps
is given by the function P(x) = −10x 2 + 800x − 12000 . How many
lamps must be sold and produced to break even? (Hint: We break even when
the profit is zero.)
Assuming dry road conditions and average reaction times, the safe stopping
distance, d in feet of an average car is given using the formula d = 201 v 2 + v,
where v represents the speed of the car in miles per hour. For each problem below,
given the stopping distance, determine the safe speed.
77. 15 feet
78. 40 feet
79. 75 feet
Find a quadratic equation with integer coefficients, given the following solutions.
81. −3, 1
82. −5, 3
83. −10, −3
84. −7, −4
85. −1, 0
86. 0, 3/5
87. −2, 2
Solve.
95. 7x (x + 5) (x − 9) = 0
96. (x − 1) (x − 2) (x − 3) = 0
98. 8x(x − 4) 2 = 0
99. 4 (x + 3) (x − 2) (x + 1) = 0
101. x 3 − x 2 − 2x = 0
102. 2x 3 + 5x 2 − 3x = 0
105. 3x 3 − 27x = 0
106. −2x 3 + 8x = 0
107. x 3 + x2 − x − 1 = 0
108. x 3 + 2x 2 − 16x − 32 = 0
109. 8x 3 − 4x 2 − 18x + 9 = 0
112. Make up your own quadratic equation and post it and the solutions on
the discussion board.
ANSWERS
1: Yes
3: Yes
5: No
7: −2, 3
9: 1/2, 4
11: 2
13: 0, 5
17: −2, 4
19: −1, 2
21: −5, 1
25: −2, 3
27: 4, 6
29: 4, 10
31: 5
33: −1/5, 2
35: 7/3
37: −3/2, 1
39: −8, ½
41: −6, 6
45: −2, 2
49: −3, 3
51: −2, 0
53: 0, 2
55: −6, 4
57: 1/5, 4
59: −1, 9
63: −6, 5
65: −3, −2
69: ±7
71: 8
73: 2 units
75: 5 units
81: x 2 + 2x − 3 = 0
83: x 2 + 13x + 30 = 0
85: x 2 +x=0
87: x 2 −4=0
89: 3x 2 + 11x − 4 = 0
93: x 2 − 6x + 9 = 0
95: −5, 0, 9
97: 0, 1, 10
101: −1, 0, 2
103: 0, 1, 2
105: −3, 0, 3
107: −1, 1
LEARNING OBJECTIVES
Number Problems
The algebraic setups of the word problems that we have previously encountered led
to linear equations. When we translate the applications to algebraic setups in this
section, the setups lead to quadratic equations. Just as before, we want to avoid
relying on the “guess and check” method for solving applications. Using algebra to
solve problems simplifies the process and is more reliable.
Example 1: One integer is 4 less than twice another integer, and their product is 96.
Set up an algebraic equation and solve it to find the two integers.
Solution: First, identify the variables. Avoid two variables by using the relationship
between the two unknowns.
The key phrase, “their product is 96,” indicates that we should multiply and set the
product equal to 96.
1038
Chapter 6 Factoring and Solving by Factoring
Next, factor completely and set each variable factor equal to zero.
The problem calls for two integers whose product is +96. The product of two
positive numbers is positive and the product of two negative numbers is positive.
Hence we can have two sets of solutions. Use 2n − 4 to determine the other
integers.
Answer: Two sets of integers solve this problem: {8, 12} and {−6, −16}. Notice that
(8)(12) = 96 and (−6)(−16) = 96; our solutions check out.
With quadratic equations, we often obtain two solutions for the identified
unknown. Although it may be the case that both are solutions to the equation, they
may not be solutions to the problem. If a solution does not solve the original
application, then we disregard it.
Recall that consecutive odd and even integers both are separated by two units.
Example 2: The product of two consecutive positive odd integers is 99. Find the
integers.
Solution:
The key phase, “product…is 99,” indicates that we should multiply and set the
product equal to 99.
Because the problem asks for positive integers, n = 9 is the only solution. Back
substitute to determine the next odd integer.
Example 3: Given two consecutive positive odd integers, the product of the larger
and twice the smaller is equal to 70. Find the integers.
Solution:
The key phrase “twice the smaller” can be translated to 2n. The phrase “product…is
70” indicates that we should multiply this by the larger odd integer and set the
product equal to 70.
Solve by factoring.
Because the problem asks for positive integers, n = 5 is the only solution. Back
substitute into n + 2 to determine the next odd integer.
Try this! The product of two consecutive positive even integers is 168. Find the
integers.
Video Solution
When working with geometry problems, it is helpful to draw a picture. Below are
some area formulas that you are expected to know. (Recall that π ≈ 3.14.)
Area of a rectangle27: A = l ⋅ w
Area of a square28: A = s2
Area of a triangle29: A= 1
2
bh
Example 4: The floor of a rectangular room has a length that is 4 feet more than
twice its width. If the total area of the floor is 240 square feet, then find the
dimensions of the floor.
27. A = lw , where l represents
the length and w represents Solution:
the width.
Use the formula A = l ⋅ w and the fact that the area is 240 square feet to set up an
algebraic equation.
Solve by factoring.
At this point we have two possibilities for the width of the rectangle. However,
since a negative width is not defined, choose the positive solution, w = 10 . Back
substitute to find the length.
It is important to include the correct units in the final presentation of the answer.
In the previous example, it would not make much sense to say the width is 10. Make
sure to indicate that the width is 10 feet.
Example 5: The height of a triangle is 3 inches less than twice the length of its base.
If the total area of the triangle is 7 square inches, then find the lengths of the base
and height.
Solution:
To avoid fractional coefficients, multiply both sides by 2 and then rewrite the
quadratic equation in standard form.
In this case, disregard the negative answer; the length of the base is 7/2 inches long.
Use 2b − 3 to determine the height of the triangle.
Answer: The base measures 7/2 = 3½ inches and the height is 4 inches.
Try this! The base of a triangle is 5 units less than twice the height. If the area is 75
square units, then what is the length of the base and height?
Video Solution
Recall that a right triangle is a triangle where one of the angles measures 90°. The
side opposite of the right angle is the longest side of the triangle and is called the
hypotenuse. The Pythagorean theorem31 gives us a relationship between the legs
and hypotenuse of any right triangle, where a and b are the lengths of the legs and c
is the length of the hypotenuse:
Given certain relationships, we use this theorem when determining the lengths of
sides of right triangles.
Solution:
Once it is in standard form, factor and set each variable factor equal to zero.
Because lengths cannot be negative, disregard the negative answer. In this case, the
long leg measures 8 inches. Use x − 2 to determine the length of the short leg.
Answer: The short leg measures 6 inches and the long leg measures 8 inches.
Solution:
Solve by factoring.
Try this! The hypotenuse of a right triangle measures 13 units. If one leg is 2 units
more than twice that of the other, then find the length of each leg.
Video Solution
The height of an object launched upward, ignoring the effects of air resistance, can
be modeled with the following formula:
With this formula, the height can be calculated at any given time t after the object is
launched. The coefficients represent the following:
1
− g The letter g represents the acceleration due to gravity.
2
We consider only problems where the acceleration due to gravity can be expressed
as g = 32 ft/sec2. Therefore, in this section time will be measured in seconds and
the height in feet. Certainly though, the formula is valid using units other than
these.
Solution: An inefficient method for finding the time to hit the ground is to simply
start guessing at times and evaluating. To do this, construct a chart.
Use the table to sketch the height of the projectile over time.
We see that at 4 seconds, the projectile hits the ground. Note that when this occurs,
the height is equal to 0. Now we need to solve this problem algebraically. To find
the solution algebraically, use the fact that the height is 0 when the object hits the
ground. We need to find the time, t, when h (t) = 0.
Example 9: The height of a certain book dropped from the top of a 144-foot
building is given by h (t) = −16t2 + 144. How long does it take to hit the ground?
Answer: The book takes 3 seconds to hit the ground when dropped from the top of a
144-foot building.
Try this! The height of a projectile, shot straight up into the air from the ground, is
given by h (t) = −16t2 + 80t. How long does it take to come back down to the
ground?
Video Solution
KEY TAKEAWAYS
TOPIC EXERCISES
1. One integer is five times another. If the product of the two integers is 80,
then find the integers.
2. One integer is four times another. If the product of the two integers is 36,
then find the integers.
3. An integer is one more than four times another. If the product of the two
integers is 39, then find the integers.
5. An integer is 2 less than twice another. If the product of the two integers
is 220, then find the integers.
6. An integer is 3 more than twice another. If the product of the two integers
is 90, then find the integers.
7. One integer is 2 units more than another. If the product of the two
integers is equal to five times the larger, then find the two integers.
8. A positive integer is 1 less than twice another. If the product of the two
integers is equal to fifteen times the smaller, then find the two integers.
10. One positive integer is 3 more than another. If the product of the two
integers is equal to twelve times the smaller, then find the integers.
11. An integer is 3 more than another. If the product of the two integers is
equal to 2 more than four times their sum, then find the integers.
12. An integer is 5 more than another. If the product of the two integers is
equal to 2 more than twice their sum, then find the integers.
13. The product of two consecutive positive even integers is 120. Find the
integers.
14. The product of two consecutive positive odd integers is 99. Find the
integers.
15. The product of two consecutive positive integers is 110. Find the
integers.
16. The product of two consecutive positive integers is 42. Find the integers.
17. The product of two consecutive positive odd integers is equal to 1 less
than seven times the sum of the integers. Find the integers.
18. The product of two consecutive positive even integers is equal to 22 more
than eleven times the sum of the integers. Find the integers.
19. The sum of the squares of two consecutive positive odd integers is 74.
Find the integers.
20. The sum of the squares of two consecutive positive even integers is 100.
Find the integers.
21. The sum of the squares of two consecutive positive integers is 265. Find
the integers.
22. The sum of the squares of two consecutive positive integers is 181. Find
the integers.
23. For two consecutive positive odd integers, the product of twice the
smaller and the larger is 126. Find the integers.
24. For two consecutive positive even integers, the product of the smaller
and twice the larger is 160. Find the integers.
25. The width of a rectangle is 7 feet less than its length. If the area of the
rectangle is 170 square feet, then find the length and width.
26. The length of a rectangle is 2 feet more than its width. If the area of the
rectangle is 48 square feet, then find the length and width.
27. The width of a rectangle is 3 units less than the length. If the area is 70
square units, then find the dimensions of the rectangle.
28. The width of a rectangle measures one half of the length. If the area is 72
square feet, then find the dimensions of the rectangle.
29. The length of a rectangle is twice that of its width. If the area of the
rectangle is 72 square inches, then find the length and width.
30. The length of a rectangle is three times that of its width. If the area of
the rectangle is 75 square centimeters, then find the length and width.
31. The length of a rectangle is 2 inches more than its width. The area of the
rectangle is equal to 12 inches more than three times the perimeter. Find
the length and width of the rectangle.
32. The length of a rectangle is 3 meters more than twice the width. The area
of the rectangle is equal to 10 meters less than three times the perimeter.
Find the length and width of the rectangle.
34. A 2-foot brick border is constructed around a square cement slab. If the
total area, including the border, is 121 square feet, then what are the
dimensions of the slab?
35. The area of a picture frame including a 2-inch wide border is 99 square
inches. If the width of the inner area is 2 inches more than its length, then
find the dimensions of the inner area.
36. A box can be made by cutting out the corners and folding up the edges of
a square sheet of cardboard. A template for a cardboard box with a height of
2 inches is given. What is the length of each side of the cardboard sheet if
the volume of the box is to be 50 cubic inches?
37. The height of a triangle is 3 inches more than the length of its base. If the
area of the triangle is 44 square inches, then find the length of its base and
height.
38. The height of a triangle is 4 units less than the length of the base. If the
area of the triangle is 48 square units, then find the length of its base and
height.
39. The base of a triangle is twice that of its height. If the area is 36 square
centimeters, then find the length of its base and height.
40. The height of a triangle is three times the length of its base. If the area is
73½ square feet, then find the length of the base and height.
41. The height of a triangle is 1 unit more than the length of its base. If the
area is 5 units more than four times the height, then find the length of the
base and height of the triangle.
42. The base of a triangle is 4 times that of its height. If the area is 3 units
more than five times the height, then find the length of the base and height
of the triangle.
45. If the sides of a right triangle are consecutive even integers, then what
are their measures?
46. The hypotenuse of a right triangle is 13 units. If the length of one leg is 2
more than twice the other, then what are their lengths?
47. The shortest leg of a right triangle measures 9 centimeters and the
hypotenuse measures 3 centimeters more than the longer leg. Find the
length of the hypotenuse.
48. The long leg of a right triangle measures 24 centimeters and the
hypotenuse measures 4 centimeters more three times the short leg. Find the
length of the hypotenuse.
53. The height of an object dropped from the top of a 64-foot building is
given by h (t) = −16t 2 + 64. How long will it take the object to hit the
ground?
54. The height of an object dropped from an airplane at 1,600 feet is given by
h (t) = −16t 2 + 1,600 . How long will it take the object to hit the
ground?
55. An object is dropped from a ladder at a height of 16 feet. How long will it
take to hit the ground?
56. An object is dropped from a 144-foot building. How long will it take to hit
the ground?
57. The height of a projectile, shot straight up into the air from the ground
at 128 feet/second, is given by h (t) = −16t 2 + 128t . How long does it
take to come back down to the ground?
58. A baseball, tossed up into the air from the ground at 32 feet/second, is
given by h (t) = −16t 2 + 32t. How long does it take to come back down
to the ground?
59. How long will it take a baseball thrown into the air at 48 feet/second to
come back down to the ground?
60. A football is kicked up into the air at 80 feet/second. Calculate how long
will it hang in the air.
62. If the sides of a square are doubled, then by what factor is the area
increased? Why?
63. Design your own geometry problem involving the area of a rectangle or
triangle. Post the question and a complete solution on the discussion board.
64. Write down your strategy for setting up and solving word problems.
Share your strategy on the discussion board.
ANSWERS
3: 3, 13
7: {5, 7} or {−2, 0}
9: 6, 15
13: 10, 12
15: 10, 11
17: 13, 15
19: 5, 7
21: 11, 12
23: 7, 9
33: 3 inches
47: 15 centimeters
49: 3 seconds
51: 5 seconds
53: 2 seconds
55: 1 second
57: 8 seconds
59: 3 seconds
1063
Chapter 6 Factoring and Solving by Factoring
REVIEW EXERCISES
Introduction to Factoring
1. 12x 3 − 24x 2 + 4x = 4x ( ? )
2. 10y 4 − 35y 3 − 5y 2 = 5y 2 ( ? )
5. 22x 2 + 11x
6. 15y 4 − 5y 3
8. 12a5 + 20a3 − 4a
9. 9x 3 y 2 − 18x 2 y 2 + 27xy 2
5
10. 16a5 b c − 8a3 b 6 + 24a3 b 2 c
Factor by grouping.
11. x 2 + 2x − 5x − 10
12. 2x 2 − 2x − 3x + 3
13. x 3 + 5x 2 − 3x − 15
14. x 3 − 6x 2 + x − 6
15. x 3 − x 2 y − 2x + 2y
2
16. a2 b − 2a3 + 6ab − 3b 3
17. x 2 + 5x + 6 = (x + 6) (x − 1)
18. x 2 + 3x − 10 = (x + 5) (x − 2)
19. x 2 + 6x + 9 = (x + 3) 2
20. x 2 − 6x − 9 = (x − 3) (x + 3)
Factor.
21. x 2 − 13x − 14
22. x 2 + 13x + 12
23. y 2 + 10y + 25
25. a2 − 8a − 48
2
26. b − 18b + 45
27. x 2 + 2x + 24
28. x 2 − 10x − 16
29. a2 + ab − 2b 2
2
30. a2 b + 5ab − 50
Factor.
31. 5x 2 − 27x − 18
32. 3x 2 − 14x + 8
33. 4x 2 − 28x + 49
34. 9x 2 + 48x + 64
35. 6x 2 − 29x − 9
36. 8x 2 + 6x + 9
41. −y 2 + 9y + 36
42. −a2 − 7a + 98
Factor completely.
45. x 2 − 81
46. 25x 2 − 36
47. 4x 2 − 49
48. 81x 2 −1
49. x 2 − 64y 2
50. 100x 2 y 2 −1
51. 16x 4 − y4
52. x 4 − 81y 4
53. 8x 3 − 125
54. 27 + y3
56. 3x 4 y 2 + 24xy 5
57. 64x 6 − y6
58. x 6 +1
Factor completely.
59. 8x 3 − 4x 2 + 20x
4
60. 50a4 b c + 5a3 b 5 c2
61. x 3 − 12x 2 − x + 12
63. −y 2 − 15y + 16
64. x 2 − 18x + 72
65. 144x 2 − 25
66. 3x 4 − 48
69. a4 b − 343ab 4
Solve.
71. (x − 9) (x + 10) = 0
72. −3x (x + 8) = 0
73. 6 (x + 1) (x − 1) = 0
75. x 2 + 5x − 50 = 0
76. 3x 2 − 13x + 4 = 0
77. 3x 2 − 12 = 0
79. (x − 2) (x + 6) = 20
80. 2 (x − 2) (x + 3) = 7x − 9
81. 5
2
x2 − 20
3
x=0
82. 2
3
x2 − 5
12
x+ 1
24
=0
Find a quadratic equation with integer coefficients, given the following solutions.
83. −7, 6
84. 0, −10
86. ±3/2
87. An integer is 4 less than twice another. If the product of the two integers
is 96, then find the integers.
88. The sum of the squares of two consecutive positive even integers is 52.
Find the integers.
89. A 20-foot ladder leaning against a wall reaches a height that is 4 feet
more than the distance from the wall to the base of the ladder. How high
does the ladder reach?
90. The height of an object dropped from the top of a 196-foot building is
given by h(t) = −16t 2 + 196 , where t represents the number of
seconds after the object has been released. How long will it take the object to
hit the ground?
91. The length of a rectangle is 1 centimeter less than three times the width.
If the area is 70 square centimeters, then find the dimensions of the
rectangle.
92. The base of a triangle is 4 centimeters more than twice the height. If the
area of the triangle is 80 square centimeters, then find the measure of the
base.
SAMPLE EXAM
2
1. Determine the GCF of the terms 25a2 b c , 50ab 4 , and 35a3 b 3 c2 .
24x 2 y 3 − 16x 3 y 2 + 8x 2 y = 8x 2 y ( )
2. Determine the missing factor:
? .
Factor.
3. 12x 5 − 15x 4 + 3x 2
4. x 3 − 4x 2 − 2x + 8
5. x 2 − 7x + 12
6. 9x 2 − 12x + 4
7. x 2 − 81
8. x 3 + 27y 3
Factor completely.
9. x 3 + 2x 2 − 4x − 8
10. x 4 −1
12. x 6 −1
Solve.
13. (2x + 1) (x − 7) = 0
14. 3x (4x − 3) (x + 1) = 0
15. x 2 − 64 = 0
16. x 2 + 4x − 12 = 0
17. 2
3
x2 + 8
9
x− 1
6
=0
18. (x − 5) (x − 3) = −1
19. 3x (x + 3) = 14x + 2
21. An integer is 4 less than twice another. If the product of the two integers
is 70, then find the integers.
22. The sum of the squares of two consecutive positive odd integers is 130.
Find the integers.
23. The length of a rectangle is 4 feet more than twice its width. If the area is
160 square feet, then find the dimensions of the rectangle.
24. The height of a triangle is 6 centimeters less than four times the length
of its base. If the area measures 27 square centimeters, then what is the
height of the triangle?
1: (3x 2 − 6x + 1)
3: (2a2 − a + 3)
5: 11x (2x + 1)
7: 6a (3a2 − 2a + 5)
9: 9xy 2 (x − 2x + 3)
2
11: (x + 2) (x − 5)
13: (x + 5) (x 2 − 3)
15: (x − y) (x 2 − 2)
17: No
19: Yes
21: (x − 14) (x + 1)
23: (y + 5)
2
25: (a − 12) (a + 4)
27: Prime
29: (a − b) (a + 2b)
31: (5x + 3) (x − 6)
33: (2x − 7) 2
35: Prime
41: −1 (y − 12) (y + 3)
43: −2 (9x + 1) (x − 8)
45: (x + 9) (x − 9)
59: 4x (2x 2 − x + 5)
61: (x − 12) (x + 1) (x − 1)
63: −1 (y + 16) (y − 1)
71: 9, −10
73: −1, 1
75: −10, 5
77: ±2
79: −8, 4
81: 0, 8/3
83: x 2 + x − 42 = 0
85: 18x 2 − 7x − 1 = 0
89: 16 feet
2
1: 5ab
3: 3x 2 (4x − 5x + 1)
3 2
5: (x − 4) (x − 3)
7: (x + 9) (x − 9)
9: (x + 2) 2 (x − 2)
13: −1/2, 7
15: ±8
19: −1/3, 2
25: 4½ sec
1077
Chapter 7 Rational Expressions and Equations
LEARNING OBJECTIVES
x+3
The example x−5 consists of linear expressions in both the numerator and
denominator. Because the denominator contains a variable, this expression is not
defined for all values of x.
x+3
Example 1: Evaluate x−5 for the set of x-values {−3, 4, 5}.
1078
Chapter 7 Rational Expressions and Equations
This example illustrates that variables are restricted to values that do not make the
denominator equal to 0. The domain of a rational expression3 is the set of real
numbers for which it is defined, and restrictions4 are the real numbers for which
the expression is not defined. We often express the domain of a rational expression
in terms of its restrictions.
x+7
Example 2: Find the domain of the following: .
2x 2 +x−6
3. The set of real numbers for Because rational expressions are undefined when the denominator is 0, we wish to
which the rational expression
is defined.
find the values for x that make it 0. To do this, apply the zero-product property. Set
each factor in the denominator equal to 0 and solve.
4. The set of real numbers for
which a rational expression is
not defined.
We conclude that the original expression is defined for any real number except 3/2
and −2. These two values are the restrictions to the domain.
We can express the domain of the previous example using notation as follows:
x 4 +x 3 −2x 2 −x
Example 3: Determine the domain: .
x 2 −1
Solution: To find the restrictions to the domain, set the denominator equal to 0 and
solve:
These two values cause the denominator to be 0. Hence they are restricted from the
domain.
2
Example 4: Determine the domain: x −25
4
.
When simplifying fractions, look for common factors that cancel. For example,
We say that the fraction 12/60 is equivalent to 1/5. Fractions are in simplest form if
the numerator and denominator share no common factor other than 1. Similarly,
when working with rational expressions, look for factors to cancel. For example,
x+4 1
In words, (x−3)(x+4) is equivalent to x−3 , if x ≠ 3 and x ≠ −4 . We can verify this
by choosing a few values with which to evaluate both expressions to see if the
results are the same. Here we choose x = 7 and evaluate as follows:
2
Example 5: Simplify and state the restriction: 25x 3 .
15x
Therefore, the domain consists of all real numbers x, where x ≠ 0 . With this
understanding, we can cancel common factors.
5
Answer: 3x , where x ≠ 0
3x(x−5)
(2x+1)(x−5)
Example 6: State the restrictions and simplify: .
Solution: To determine the restrictions, set the denominator equal to 0 and solve.
The domain consists of all real numbers except for −1/2 and 5. Next, we find an
equivalent expression by canceling common factors.
3x
Answer: 2x+1 , where x ≠ − 12 and x ≠ 5
Typically, rational expressions are not given in factored form. If this is the case,
factor first and then cancel. The steps are outlined in the following example.
3x+6
Example 7: State the restrictions and simplify: .
x 2 +x−2
Solution:
Step 2: Determine the restrictions to the domain. To do this, set the denominator
equal to 0 and solve.
3
Answer: x−1 , where x ≠ 1 and x ≠ −2
x 2 +7x−30
Example 8: State the restrictions and simplify: .
x 2 −7x+12
x+10
Answer: x−4 , where x ≠ 3 and x ≠ 4
2
Try this! State the restrictions and simplify: x 2 −16 .
5x −20x
x+4
Answer: 5x , where x ≠ 0 and x ≠ 4
Video Solution
xy+y 2 −3x−3y
Example 9: Simplify: . (Assume all denominators are nonzero.)
x 2 −y 2
Solution: Factor the numerator by grouping. Factor the denominator using the
formula for a difference of squares.
y−3
Answer: x−y
Recall that the opposite of the real number a is −a. Similarly, we can define the
opposite of a polynomial P to be −P. We first consider the opposite of the binomial
a − b:
− (a − b) = b − a.
the opposite is
Answer: 3−x
x−3
= −1, where x ≠ 3
or
Take care not to confuse this with the opposite binomial property. Also, it is
important to recall that
In other words, show a negative fraction by placing the negative sign in the
numerator, in front of the fraction bar, or in the denominator. Generally, negative
denominators are avoided.
4−x 2
Example 11: Simplify and state the restrictions: .
x 2 +3x−10
x+2
Answer: − x+5 , where x ≠ 2 and x ≠ −5
2
Try this! Simplify and state the restrictions: 2x −7x−15 .
25−x
2
2x+3
Answer: − x+5 , where x ≠ ±5
Video Solution
where p(x) and q(x) are polynomials and q(x) ≠ 0. The domain of a rational
function consists of all real numbers x such that the denominator q(x) ≠ 0.
Example 12:
2x 2 +5x−3
a. Simplify: r (x) = .
6x 2 +18x
c. Calculate r(−2).
Solution:
b. To determine the restrictions, set the denominator of the original function equal
to 0 and solve.
c. Since −2 is not a restriction, substitute it for the variable x using the simplified
form.
Answers:
a. r (x) = 2x−1
6x
c. r (−2) = 5
12
If a cost function6 C(x) represents the cost of producing x units, then the average
cost7 c(x) is the cost divided by the number of units produced.
Example 13: The cost in dollars of producing t-shirts with a company logo is given
by C (x) = 7x + 200, where x represents the number of shirts produced.
Determine the average cost of producing
a. 40 t-shirts
b. 250 t-shirts
c. 1,000 t-shirts
Answers:
a. If 40 t-shirts are produced, then the average cost per t-shirt is $12.00.
b. If 250 t-shirts are produced, then the average cost per t-shirt is $7.80.
c. If 1,000 t-shirts are produced, then the average cost per t-shirt is $7.20.
KEY TAKEAWAYS
• Rational expressions usually are not defined for all real numbers. The
real numbers that give a value of 0 in the denominator are not part of
the domain. These values are called restrictions.
• Simplifying rational expressions is similar to simplifying fractions. First,
factor the numerator and denominator and then cancel the common
factors. Rational expressions are simplified if there are no common
factors other than 1 in the numerator and the denominator.
• Simplified rational expressions are equivalent for values in the domain
of the original expression. Be sure to state the restrictions if the
denominators are not assumed to be nonzero.
such as (a + b) = (b + a).
will then cancel. Do not confuse this with factors that involve addition,
TOPIC EXERCISES
5
1. x ; {−1, 0, 1}
4x
2. ; {−1, 0, 1}
3x 2
1
3. ; {−10, −9, 0}
x+9
x+6
4. ; {−6, 0, 5}
x−5
3x(x−2)
5. ; {0, 1/2, 2}
2x−1
9x 2 −1
6. x−7 ; {0, 1/3, 7}
5
7. ; {−3, 0, 3}
x 2 −9
x 2 −25
8. ; {−5, −4, 5}
x 2 −3x−10
An object’s weight depends on its height above the surface of earth. If an object
weighs 120 pounds on the surface of earth, then its weight in pounds, W, x miles
above the surface is approximated by the formula
120 ⋅ 4000 2
W=
(4000 + x) 2
For each problem below, approximate the weight of a 120-pound object at the given
height above the surface of earth. (1 mile = 5,280 feet)
The price to earnings ratio (P/E) is a metric used to compare the valuations of
similar publicly traded companies. The P/E ratio is calculated using the stock price
and the earnings per share (EPS) over the previous 12‑month period as follows:
If each share of a company stock is priced at $22.40, then calculate the P/E ratio
given the following values for the earnings per share.
17. $1.40
18. $1.21
1
21.
3x
3x 2
22.
7x 5
3x(x+1)
23.
x+4
2x 2 (x−3)
24.
x−1
1
25.
5x−1
x−2
26.
3x−2
x−9
27.
5x(x−2)
(x−3)(x+6)
1
28.
x
29.
1−x 2
x 2 −9
30.
x 2 −36
1
31.
2x(x+3)(2x−1)
x−3
32.
(3x−1)(2x+3)
4x(2x+1)
33.
12x 2 +x−1
x−5
34.
3x 2 −15x
5x 2
35.
20x 3
12x 6
36.
60x
3x 2 (x−2)
37.
9x(x−2)
20(x−3)(x−5)
38.
6(x−3)(x+1)
6x 2 (x−8)
39.
36x(x+9)(x−8)
16x 2 −1
40.
(4x+1) 2
9x 2 −6x+1
41.
(3x−1) 2
x−7
42.
x 2 −49
x 2 −64
43.
x 2 +8x
x+10
44.
x 2 −100
2x 3 −12x 2
45.
5x 2 −30x
30x 5 +60x 4
46.
2x 3 −8x
2x−1
47.
2x 2 +x−6
x 2 −x−6
48.
3x 2 −8x−3
6x 2 −25x+25
49.
3x 2 +16x−35
3x 2 +4x−15
50.
x 2 −9
x 2 −10x+21
51.
x 2 −4x−21
x 3 −1
52.
x 2 −1
x 3 +8
53.
x 2 −4
x 4 −16
54.
x 2 −4
x−9
55.
9−x
3x−2
56.
2−3x
x+6
57.
6+x
3x+1
58.
1+3x
(2x−5)(x−7)
59.
(7−x)(2x−1)
(3x+2)(x+5)
(x−5)(2+3x)
60.
x 2 −4
61.
(2−x) 2
16−9x 2
62.
(3x+4) 2
4x 2 (10−x)
63.
3x 3 −300x
−2x+14
64.
x 3 −49x
2x 2 −7x−4
65.
1−4x 2
9x 2 −4
66.
4x−6x 2
x 2 −5x−14
67.
7−15x+2x 2
2x 3 +x 2 −2x−1
68.
1+x−2x 2
x 3 +2x−3x 2 −6
69.
2+x 2
27+x 3
70.
x 2 +6x+9
64−x 3
71.
x 2 −8x+16
x 2 +4
72.
4−x 2
−15x 3 y 2
5xy 2 (x+y)
73.
14x 7 y 2 (x−2y)
4
7x 8 y(x−2y)
74. 2
y+x
75.
x 2 −y 2
y−x
76.
x 2 −y 2
x 2 −y 2
(x−y)
77. 2
a2 −ab−6b 2
78.
a2 −6ab+9b 2
2a2 −11a+12
79.
−32+2a2
a2 b−3a2
80.
3a2 −3ab
xy 2 −x+y 3 −y
81.
x−xy 2
x 3 −xy 2 −x 2 y+y 3
82.
x 2 −2xy+y 2
x 3 −27
83.
x 2 +3x+9
x 2 −x+1
84.
x 3 +1
5x
85. f (x) = ;f (0), f (2), f (4)
x−3
x+7
86. f (x) = ;f (−1) , f (0), f (1)
x 2 +1
x3
87. g (x) = ; g (0) , g (2) , g (−2)
(x−2) 2
x 2 −9
88. g (x) = ; g (−2) , g (0) , g (2)
9−x 2
x3
89. g (x) = ; g (−1) , g (0) , g (1)
x 2 +1
−3x 2 −6x
91. f (x) = x 2 +4x+4
x 2 +6x+9
92. f (x) = 2x 2 +5x−3
9−x
93. g (x) = x 2 −81
x 3 −27
94. g (x) = 3−x
3x−15
95. g (x) = 10−2x
25−5x
96. g (x) = 4x−20
97. The cost in dollars of producing coffee mugs with a company logo is
given by C (x) = x + 40 , where x represents the number of mugs
produced. Calculate the average cost of producing 100 mugs and the average
cost of producing 500 mugs.
98. The cost in dollars of renting a moving truck for the day is given by
C (x) = 0.45x + 90 , where x represents the number of miles driven.
Calculate the average cost per mile if the truck is driven 250 miles in one
day.
99. The cost in dollars of producing sweat shirts with a custom design on the
back is given by C(x) = 1200 + (12 − 0.05x)x , where x represents
the number of sweat shirts produced. Calculate the average cost of
producing 150 custom sweat shirts.
100. The cost in dollars of producing a custom injected molded part is given
by C(x) = 500 + (3 − 0.001x)x , where x represents the number of
parts produced. Calculate the average cost of producing 1,000 custom parts.
b−a
101. Explain why
a−b
= −1 and illustrate this fact by substituting some
numbers for the variables.
b+a
102. Explain why
a+b
= 1 and illustrate this fact by substituting some
numbers for the variables.
x
103. Explain why we cannot cancel x in the expression .
x+1
ANSWERS
1: −5, undefined, 5
5: 0, undefined, 0
9:
11:
17: 16
21: x ≠0
23: x ≠ −4
25: x ≠ 1
5
27: x ≠ 0 and x ≠ 2
29: x ≠ ±1
31: x ≠ 0 , x ≠ −3 , and x ≠ 1
2
33: x ≠− 1
3
and x ≠ 1
4
1
35: ;x ≠0
4x
x
37: ;x ≠ 0, 2
3
x
39: ;x ≠ 0, −9, 8
6(x+9)
41: 1 ; x ≠ 1
3
x−8
43: x ;x ≠ 0, −8
2x
45: ;x ≠ 0, 6
5
2x−1
47: ;x ≠ −2, 3
2x 2 +x−6 2
2x−5
49: x+7 ; x ≠ −7, 5
3
x−3
51: ;x ≠ −3, 7
x+3
x 2 −2x+4
53: ;x ≠ ±2
x−2
55: −1; x ≠9
57: 1; x ≠ −6
2x−5
59: − ;x ≠ 1
,7
2x−1 2
x+2
61: ;x ≠2
x−2
4x
63: − ;x ≠ ±10, 0
3(x+10)
x−4
65: ;x ≠± 1
1−2x 2
x+2
67: ;x ≠ 1
,7
2x−1 2
69: x − 3 ; none
16+4x+x 2
71: − ;x ≠4
x−4
3x 2
73: − x+y
1
75: x−y
x+y
77: x−y
2a−3
79:
2(4+a)
x+y
81: − x
83: x −3
1
93: g(x) =− ;x ≠ ±9
x+9
3
95: g(x) =− ;x ≠5
2
97: The average cost of producing 100 mugs is $1.40 per mug. The average
cost of producing 500 mugs is $1.08 per mug.
99: $12.50
LEARNING OBJECTIVES
In this section, assume that all variable expressions in the denominator are nonzero
unless otherwise stated.
1107
Chapter 7 Rational Expressions and Equations
2 20y 4
Example 1: Multiply: 12x3 ⋅ .
5y 3 6x
Solution: Multiply numerators and denominators and then cancel common factors.
8y
Answer: x
Solution: Leave the product in factored form and cancel the common factors.
Answer: x−3
x+7
15x 2 y 3 x(2x−1)
Example 3: Multiply: (2x−1) ⋅ 2 .
3x y(x+3)
5xy 2
Answer: x+3
Typically, rational expressions will not be given in factored form. In this case, first
factor all numerators and denominators completely. Next, multiply and cancel any
common factors, if there are any.
x+5
Example 4: Multiply: x−5 ⋅ x−5 .
x 2 −25
Keep in mind that 1 is always a factor; so when the entire numerator cancels out,
make sure to write the factor 1.
1
Answer: x−5
x 2 +3x+2 x 2 −7x+12
Example 5: Multiply:
x 2 −5x+6
⋅ x 2 +8x+7
.
Solution:
(x+2)(x−4)
Answer: (x−2)(x+7)
3(2x−3)
Answer: − x(x+4)
2
Example 7: Multiply: 7−x
2
⋅ x +10x+21
x +3x 2
.
x −49
Answer: − 1x
2 2
Try this! Multiply: x8−x
−64
⋅ x 2x+x
+9x+8
.
Answer: −x
Video Solution
8x 5 y 20xy 4
Example 8: Divide:
25z 6
÷ 15z 3
.
Solution: First, multiply by the reciprocal of the divisor and then cancel.
4
Answer: 6x3 3
25y z
x+2 x+3
Example 9: Divide:
x 2 −4
÷ x−2
.
Solution: After multiplying by the reciprocal of the divisor, factor and cancel.
1
Answer: x+3
2 x 2 +9x+14
Example 10: Divide: x 2 −6x−16 ÷ 2 .
x +4x−21 x −8x+15
Solution: Begin by multiplying by the reciprocal of the divisor. After doing so,
factor and cancel.
(x−8)(x−5)
Answer:
(x+7) 2
9−4x 2
Example 11: Divide: x+2 ÷ (2x − 3).
2x+3
Answer: − x+2
4x 2 +7x−2 1−4x
Try this! Divide:
25x 2
÷ 100x 4
.
Answer: −4x 2 (x + 2)
Video Solution
The product and quotient of two rational functions can be simplified using the
techniques described in this section. The restrictions to the domain of a product
consist of the restrictions of each function.
Example 12: Calculate (f ⋅ g) (x) and determine the restrictions to the domain.
Solution: In this case, the domain of f (x) consists of all real numbers except 0, and
the domain of g(x) consists of all real numbers except 1/4. Therefore, the domain of
the product consists of all real numbers except 0 and 1/4. Multiply the functions
and then simplify the result.
The restrictions to the domain of a quotient will consist of the restrictions of each
function as well as the restrictions on the reciprocal of the divisor.
Solution:
In this case, the domain of f (x) consists of all real numbers except 3 and 8, and the
domain of g(x) consists all real numbers except 3. In addition, the reciprocal of g(x)
has a restriction of −8. Therefore, the domain of this quotient consists of all real
numbers except 3, 8, and −8.
KEY TAKEAWAYS
TOPIC EXERCISES
2x 9
1.
3
⋅ 4x 2
−5x 3 y2
2. y ⋅ 25x
5x 2 4y 2
3.
2y
⋅ 15x 3
16a4 49b
4. ⋅ 32a3
7b 2
x−6 24x 2
5.
12x 3
⋅ x−6
x+10 x−2
6.
2x−1
⋅ x+10
(y−1)
2
1
7.
y+1
⋅ y−1
y 2 −9 2y−3
8.
y+3
⋅ y−3
2a−5 2a+5
9.
a−5
⋅ 4a2 −25
2a2 −9a+4
10.
a2 −16
⋅ (a2 + 4a)
2x 2 +3x−2 2x
11. ⋅ x+2
(2x−1) 2
9x 2 +19x+2 x 2 −4x+4
12.
4−x 2
⋅ 9x 2 −8x−1
x 2 +8x+16 x 2 −3x−4
13.
16−x 2
⋅ x 2 +5x+4
x 2 −x−2 x 2 +2x−15
14.
x 2 +8x+7
⋅ x 2 −5x+6
x+1 3−x
15.
x−3
⋅ x+5
2x−1 x+6
16.
x−1
⋅ 1−2x
9+x 3
17.
3x+1
⋅ x+9
1 5x+2
18.
2+5x
⋅ 5x
100−y 2 25y 2
19.
y−10
⋅ y+10
3y 3 36y 2 −25
20.
6y−5
⋅ 5+6y
23.
x+9
−x 2 +14x−45
⋅ (x 2 − 81)
24.
1
2+5x
⋅ (25x 2 + 20x + 4)
x 2 +x−6 2x 2 −8
25.
3x 2 +15x+18
⋅ x 2 −4x+4
5x 15x 2
27.
8
÷ 4
3 −15
28.
8y
÷ 2y 2
5x 9
3y 3
29.
25x 10
9y 5
12x 4 y 2
21z 5
30.
6x 3 y 2
7z 3
(x−4) 2 x−4
31.
30x 4
÷ 15x
5y 4 10y 5
÷
10(3y−5) 2(3y−5)
32. 2 3
x 2 −9
33.
5x
÷ (x − 3)
÷ (8 + y)
y 2 −64
34.
8y
(a−8)2 a−8
35.
2a2 +10a
÷ a
3 12ab(a−2b)
36. 24a2 b (a − 2b) ÷ 5
x 2 +7x+10 1
37.
x 2 +4x+4
÷ x 2 −4
2x 2 −x−1 1
38.
2x 2 −3x+1
÷ 4x 2 −1
y+1 y 2 −1
39.
y 2 −3y
÷ y 2 −6y+9
a2 −3a−18 a2 +a−6
41.
2a2 −11a−6
÷ 2a2 −a−1
y 2 −7y+10
y 2 +5y−14
42.
2y 2 −9y−5
y 2 +14y+49
6y 2 +y−1
4y 2 +4y+1
43.
3y 2 +2y−1
2y 2 −7y−4
x 2 −7x−18 x 2 −81
44.
x 2 +8x+12
÷ x 2 +12x+36
÷ (b − 2a)
4a2 −b 2 2
45.
b+2a
÷ (y − x)
x 2 −y 2 2
46. y+x
5y 2 (y−3) 25y(3−y)
47.
4x 3
÷ 2x 2
15x 3 25x 6
3(y+7)
÷
9(7+y)
48. 2
3x+4 7x
49.
x−8
÷ 8−x
3x−2 2−3x
50.
2x+1
÷ 3x
4x 2−x
52. ÷ x 2 −4
(x+2) 2
÷ (b − a)
a2 −b 2 2
53. a
÷ (2b 2 + ab − a2 )
(a−2b)2
54.
2b
x 2 −6x+9 9−x 2
55.
x 2 +7x+12
÷ x 2 +8x+16
2x 2 −9x−5 1−4x+4x 2
56.
25−x 2
÷ −2x 2 −9x+5
3x 2 −16x+5
100−4x 2
57.
9x 2 −6x+1
3x 2 +14x−5
10x 2 −25x−15
x 2 −6x+9
58.
9−x 2
2
x +6x+9
Recall that multiplication and division are to be performed in the order they appear
from left to right. Simplify the following.
1 x−1 x−1
59.
x2
⋅ x+3
÷ x3
x−7 1 x−7
60.
x+9
⋅ x3
÷ x
x+1 x x2
61.
x−2
÷ x−5
⋅ x+1
4x 2 −1 2x−1 3x+2
64.
3x+2
÷ x+5
⋅ 2x+1
1 1
65. f (x) = x and g(x) = x−1
x+1
66. f (x) = x−1
and g(x) = x2 − 1
3x+2 x 2 −4
67. f (x) = and g(x) =
x+2 (3x+2) 2
(x−6)
2
(1−3x) 2
68. f (x) = and g(x) =
x−6 9x 2 −1
25x 2 −1 x 2 −9
69. f (x) = and g(x) =
x 2 +6x+9 5x+1
x 2 −49 4x 2 −4x+1
70. f (x) = and g(x) = 7−x
2x 2 +13x−7
1 x−2
71. f (x) = x and g(x) = x−1
(5x+3)
2
5x+3
72. f (x) = and g(x) =
x2 6−x
5−x x 2 −25
73. f (x) = and g(x) =
(x−8) 2 x−8
x 2 −2x−15 2x 2 −5x−3
74. f (x) = and g(x) =
x 2 −3x−10 x 2 −7x+12
3x 2 +11x−4 x 2 −2x+1
75. f (x) = and g(x) =
9x 2 −6x+1 3x 2 −4x+1
36−x 2 x 2 −12x+36
76. f (x) = and g(x) =
x 2 +12x+36 x 2 +4x−12
77. In the history of fractions, who is credited for the first use of the fraction
bar?
1 x−7
79. Explain why x = 7 is a restriction to x ÷ .
x−2
ANSWERS
3
1:
2x
2y
3:
3x
2
5: x
y−1
7:
y+1
1
9:
a−5
2x
11:
2x−1
13: −1
x+1
15: −
x+5
3
17:
3x+1
19: −25y 2
a+5
21: −
a2 +1
(x+9) 2
23: −
x−5
25: 2/3
1
27:
6x
3y 2
29:
5x
x−4
31:
2x 3
x+3
33:
5x
2(a+5)
a−8
35:
37: (x + 5) (x − 2)
y−3
y(y−1)
39:
a−1
41:
a−2
y−4
43:
y+1
1
45:
2a−b
y
47: −
10x
3x+4
49: − 7x
7x−1
51: −
4x+1
a+b
53: −
a(b−a)
(x−3)(x+4)
55: −
(x+3) 2
57: −1/4
x
59:
x+3
x(x−5)
61:
x−2
x 2 +1
63:
x+1
65: (f ⋅ g) (x) = 1
;x ≠ 0, 1
x(x−1)
69: (f ⋅ g) (x) =
(x−3)(5x−1)
;x ≠ −3, − 1
x+3 5
LEARNING OBJECTIVES
In this section, assume that all variable factors in the denominator are nonzero.
1127
Chapter 7 Rational Expressions and Equations
Solution: Add the numerators 3 and 7, and write the result over the common
denominator, y.
Answer: 10
y
x−5 1
Example 2: Subtract: 2x−1 − 2x−1.
Solution: Subtract the numerators x − 5 and 1, and write the result over the
common denominator, 2x − 1 .
x−6
Answer: 2x−1
2x+7 x+10
( )(x−3) ( )(x−3)
Example 3: Subtract: − .
x+5 x+5
1
Answer: x+5
Solution: Subtract and add the numerators. Make use of parentheses and write the
result over the common denominator, x 2 − 36.
Answer: x−1
x−6
x 2 +1 x 2 −2x
Try this! Subtract:
2x 2 −7x−4
− 2x 2 −7x−4
.
1
Answer: x−4
Video Solution
In this section, assume that all variable factors in the denominator are nonzero.
Solution: In this example, the LCD = xy . To obtain equivalent terms with this
y
common denominator, multiply the first term by y and the second term by xx .
y+x
Answer: xy
Solution: Since the LCD = y(y − 3) , multiply the first term by 1 in the form of
(y−3) y
and the second term by y .
(y−3)
3
y(y−3)
Answer: −
It is not always the case that the LCD is the product of the given denominators.
Typically, the denominators are not relatively prime; thus determining the LCD
requires some thought. Begin by factoring all denominators. The LCD is the product
of all factors with the highest power. For example, given
there are three base factors in the denominator: x , (x + 2), and (x − 3). The
highest powers of these factors are x 3 , (x + 2)2 , and (x − 3)1 . Therefore,
The general steps for adding or subtracting rational expressions are illustrated in
the following example.
x 3
Example 7: Subtract:
x 2 +4x+3
− x 2 −4x−5
.
Solution:
Step 3: Add or subtract the numerators and place the result over the common
denominator.
(x−9)
(x+3)(x−5)
Answer:
x 2 −9x+18 x
Example 8: Subtract:
x 2 −13x+36
− x−4
.
18
Answer: (x−4)(x−9)
1 1
Example 9: Subtract:
x 2 −4
− 2−x
.
Solution: First, factor the denominators and determine the LCD. Notice how the
opposite binomial property is applied to obtain a more workable denominator.
Now that we have equivalent terms with a common denominator, add the
numerators and write the result over the common denominator.
x+3
Answer: (x+2)(x−2)
y−1 y+1 y 2 −5
Example 10: Simplify: y+1 − y−1 + 2 .
y −1
We can see that the LCD is (y + 1) (y − 1). Find equivalent fractions with this
denominator.
Next, subtract and add the numerators and place the result over the common
denominator.
y−5
Answer: y−1
x 5
Try this! Simplify: −2 + 1+x − 1−x.
x 2 −1
x+3
Answer: x−1
Video Solution
y 2 +y−1
y 2 (y−1)
Answer:
Solution:
Here the domain of f consists of all real numbers except −3, and the domain of g
consists of all real numbers except 2. Therefore, the domain of f + g consists of all
real numbers except −3 and 2.
2x+1
Answer: (x+3)(x−2) , where x ≠ −3, 2
Solution:
The domain of f consists of all real numbers except 5 and −5, and the domain of g
consists of all real numbers except 5. Therefore, the domain of f − g consists of all
real numbers except −5 and 5.
3
Answer: − x+5 , where x ≠ ±5
KEY TAKEAWAYS
TOPIC EXERCISES
3 7
1. x + x
9 10
2. x − x
x 3
3. y − y
4 6
4.
x−3
+ x−3
7 x
5.
2x−1
− 2x−1
8 3x
6.
3x−8
− 3x−8
2 x−11
7.
x−9
+ x−9
y+2 y+3
8.
2y+3
− 2y+3
2x−3 x−4
9.
4x−1
− 4x−1
2x 3x+4 x−2
10.
x−1
− x−1
+ x−1
1 2y−9 13−5y
11.
3y
− 3y
− 3y
x 3
13.
(x+1)(x−3)
− (x+1)(x−3)
3x+5 x+6
(2x−1)(x−6) (2x−1)(x−6)
14. −
x 6
15.
x 2 −36
+ x 2 −36
x 9
16.
x 2 −81
− x 2 −81
x 2 +2 x−22
17.
x 2 +3x−28
+ x 2 +3x−28
x 3−x
18.
2x 2 −x−3
− 2x 2 −x−3
1 1
19.
2
+ 3x
1 1
20.
5x 2
− x
1 3
21.
12y 2
+ 10y 3
1 1
22. x − 2y
1
23. y −2
3
24.
y+2
−4
2
25.
x+4
+2
2 1
26. y − y2
3 1
27.
x+1
+ x
1 2
28.
x−1
− x
1 1
29.
x−3
+ x+5
1 1
30.
x+2
− x−3
x 2
31.
x+1
− x−2
2x−3 x
32.
x+5
− x−3
y+1 y−1
33.
y−1
+ y+1
3y−1 y+4
34.
3y
− y−2
2x−5 2x+5
35.
2x+5
− 2x−5
2 2x+1
36.
2x−1
− 1−2x
3x+4 2
37.
x−8
− 8−x
1 1
38.
y−1
+ 1−y
2x 2 x+15
39.
x 2 −9
+ 9−x 2
x 1 15−x
40.
x+3
+ x−3
− (x+3)(x−3)
2x 1 2(x−1)
41.
3x−1
− 3x+1
+ (3x−1)(3x+1)
(2x+1)(x−5)
4x x 16x−3
42.
2x+1
− x−5
+
x 2 4
43.
3x
+ x−2
+ 3x(x−2)
18(x−2)
(x+6)(x−6)
−2x 3x
44.
x+6
− 6−x
−
(x+5)(x−7)
x 1 25−7x
45.
x+5
− x−7
−
x 2
46.
x 2 −2x−3
+ x−3
1 x2
47.
x+5
− x 2 −25
5x−2 2
48.
x 2 −4
− x−2
1 6x−3
49.
x+1
− x 2 −7x−8
3x 1
50.
9x 2 −16
− 3x+4
2x 1
51.
x 2 −1
+ x 2 +x
x(4x−1) x
52.
2x 2 +7x−4
− 4+x
3x 2 2x
53.
3x 2 +5x−2
− 3x−1
2x 11x+4
54.
x−4
− x 2 −2x−8
x 6x−24
55.
2x+1
+ 2x 2 −7x−4
1 1
56.
x 2 −x−6
+ x 2 −3x−10
x 3
57.
x 2 +4x+3
− x 2 −4x−5
y+1 y
58.
2y 2 +5y−3
− 4y 2 −1
y−1 2
59.
y 2 −25
− y 2 −10y+25
3x 2 +24 12
60.
x 2 −2x−8
− x−4
4x 2 +28 28
61.
x 2 −6x−7
− x−7
a a2 −9a+18
62.
4−a
+ a2 −13a+36
3a−12 a+2
63.
a2 −8a+16
− 4−a
a2 −14 5
64.
2a2 −7a−4
− 1+2a
1 x 3
65.
x+3
− x 2 −6x+9
+ x 2 −9
3x 2x 23x−10
66. x+7 − x−2
+ x 2 +5x−14
−2x 4 4(x+5)
68.
3x+1
− x−2
+ 3x 2 −5x−2
3x 2 6x 2 −5x−9
70.
2x−3
− 2x+3
− 4x 2 −9
1 1 2
71.
y+1
+ y + y 2 −1
1 1 1
72. y − y+1
+ y−1
−2
73. 5 + 2 −1
−1
74. 6 + 4 −2
75. x −1 + y −1
76. x −2 − y −1
77. (2x − 1) −1 − x −2
78. (x − 4) −1 − (x + 1) −1
79. 3x 2 (x − 1) −1 − 2x
80. 2(y − 1) − (y − 1)
−2 −1
1 1
81. f (x) = and g (x) =
3x x−2
1 1
82. f (x) = and g (x) =
x−1 x+5
x 1
83. f (x) = and g (x) =
x−4 4−x
x 1
84. f (x) = and g (x) =
x−5 2x−3
x−1 4
85. f (x) = and g (x) =
x 2 −4 x 2 −6x−16
5
86. f (x) = and g (x) = 3x + 4
x+2
1
87. f (x) = x
1
88. f (x) = 2x
x
89. f (x) = 2x−1
1
90. f (x) = x+2
1 2 3
91. Explain to a classmate why this is incorrect:
x2
+ x2
= 2x 2
.
ANSWERS
10
1: x
x−3
3: y
7−x
5:
2x−1
7: 1
x+1
9:
4x−1
y−1
11: y
1
13:
x+1
1
15:
x−6
x+5
17: x+7
3x+2
19:
6x
5y+18
21:
60y 3
1−2y
23: y
2(x+5)
25:
x+4
4x+1
27:
x(x+1)
2(x+1)
(x−3)(x+5)
29:
x 2 −4x−2
31:
(x−2)(x+1)
2(y 2 +1)
(y+1)(y−1)
33:
(2x+5)(2x−5)
40x
35: −
3(x+2)
37:
x−8
2x+5
39:
x+3
2x+1
41:
3x+1
x 2 +4x+4
43:
3x(x−2)
x−6
45: x−7
−x 2 +x−5
(x+5)(x−5)
47:
5
49: −
x−8
2x−1
51:
x(x−1)
x(x−4)
53:
(x+2)(3x−1)
x+6
55:
2x+1
x−9
(x−5)(x+3)
57:
y 2 −8y−5
(y+5)(y−5)
59: 2
4x
61:
x+1
a+5
63:
a−4
6x
65: −
(x+3)(x−3) 2
x−7
67:
x+2
−x−5
69:
4x−1
2y−1
y(y−1)
71:
27
73:
50
x+y
75: xy
(x−1) 2
77:
x 2 (2x−1)
x(x+2)
79:
x−1
85: (f
+ g) (x) = (x+2)(x−2)(x−8) (
− g) (x) =
x(x−5) x 2 −13x+16
; f ;
(x+2)(x−2)(x−8)
x ≠ −2, 2, 8
87: (f + f ) (x) = 2
x;x ≠0
89: (f + f ) (x) = 2x
;x ≠ 1
2x−1 2
LEARNING OBJECTIVES
Definitions
1152
Chapter 7 Rational Expressions and Equations
1
2
+ 1x
Example 1: Simplify: 1
.
4
− 12
x
Solution:
9. A rational expression where
the numerator or denominator Step 1: Simplify the numerator and denominator. The goal is to obtain a single
consists of one or more
rational expressions.
algebraic fraction divided by another single algebraic fraction. In this example, find
2x
Answer: x−2
1 1
x − x−2
Example 2: Simplify: 4
.
x 2 −2x
Solution:
Answer: − 12
1− 4x − 21
x2
Example 3: Simplify 15
.
1− 2x −
x2
Solution: The LCD of the rational expressions in both the numerator and
denominator is x 2 . Multiply by the appropriate factors to obtain equivalent terms
with this as the denominator and then subtract.
Answer: x−7
x−5
1
1−
x2
Example 4: Simplify: 1
.
x −1
Solution:
x+1
Answer: − x
1 1
81
−
x2
Try this! Simplify: 1
.
9
+ 1x
x−9
Answer: 9x
Video Solution
1
2
+ 1x
Example 5: Simplify: 1
.
4
− 12
x
Solution:
Step 1: Determine the LCD of all the fractions in the numerator and denominator.
In this case, the denominators of the given fractions are 2, x , 4, and x 2 . Therefore,
the LCD is 4x 2 .
Step 2: Multiply the numerator and denominator by the LCD. This step should clear
the fractions in both the numerator and denominator.
This leaves us with a single algebraic fraction with a polynomial in the numerator
and in the denominator.
2x
Answer: x−2
Note
This was the same problem that we began this section with, and the results
here are the same. It is worth taking the time to compare the steps involved
using both methods on the same problem.
15
1− 2x −
x2
Example 6: Simplify: 14 5
.
3− x − x2
At this point, we have a rational expression that can be simplified by factoring and
then canceling the common factors.
x+3
Answer: 3x+1
It is important to point out that multiplying the numerator and denominator by the
same nonzero factor is equivalent to multiplying by 1 and does not change the
problem. Because x 2 = 1, we can multiply the numerator and denominator by x 2
2
x
in the previous example and obtain an equivalent expression.
1 3
x+1
+ x−3
Example 7: Simplify: 2 1
.
x−3
− x+1
4x
Answer: x+5
1
y − 14
Try this! Simplify: 1 1
.
−
16 y2
4y
Answer: − y+4
Video Solution
KEY TAKEAWAYS
TOPIC EXERCISES
1
2
1. 5
4
7
8
2. 5
4
10
3
3. 20
9
4
− 21
4. 8
7
2
3
5. 5
6
7
4
6. 14
3
1− 32
7. 5
4
− 13
1
2
−5
8. 1
2
+ 13
1+ 32
9.
1− 14
2− 12
10.
1+ 34
5x 2
x+1
11. 25x
x+1
7+x
7x
12. x+7
14x 2
3y
x
13.
y2
x−1
5a2
b−1
14.
15a3
(b−1)2
1+ 1x
15.
2− 1x
2
x +1
16.
3− 1x
2
3y
−4
17.
6− 1y
5
y − 12
18. 10−y
y2
1
5
− 1x
19. 1 1
−
25 x2
1
x + 15
20. 1 1
−
25 x2
1 1
x −3
21. 1
9
− 12
x
1
4
+ 1x
22. 1 1
− 16
x2
1
16−
x2
23. 1
x −4
2− 1y
24. 1
1−
4y 2
1
x + 1y
25. 1 1
−
y2 x2
1
2x
− 43
26. 1
− 16
4x 2 9
2 1
−
25 2x 2
27. 1 1
5
− 2x
4 1
−
25 4x 2
28. 1 1
5
+ 4x
1
y − 1x
29.
4− xy2
1
ab
+2
30. 1 1
a+b
1
y + 1x
31. xy
3x
32. 1
3
− 1x
1− 4x − 21
x2
33. 15
1− 2x −
x2
1− 3x − 4
x2
34. 16
1−
x2
3− 2x1 − 1
2x 2
35.
2− 2x + 1
2x 2
1
2
− 5x + 12
x2
36. 1
2
− 6x + 18
x2
1 4
x −
3x 2
37. 8 16
3− x + 3x 2
3 1
1+ 10x −
10x 2
38. 3 1
5
− 10x
− 12
5x
x−1
39. 5
1+ 4x −
x2
2− 2x5 − 3
x2
40.
4x+3
1
x−3
+ 2x
41. 1 3
x − x−3
1 1
+
4x−5 x2
42. 1 1
+
x2 3x−10
1 4
x+5
+ x−2
43. 2 1
x−2
− x+5
3 2
x−1
− x+3
44. 2 1
x+3
+ x−3
x 2
x+1
− x+3
45. x 1
3x+4
+ x+1
x 2
x−9
+ x+1
46. x 1
7x−9
− x+1
x 1
3x+2
− x+2
47. x 2
x+2
− x+2
x 1
x−4
+ x+2
48. x 1
3x+4
+ x+2
a3 −8b 3
27
49.
a−2b
27a3 +b 3
ab
50.
3a+b
1 1
+
b3 a3
51. 1 1
b
+ a
1 1
−
b3 a3
52. 1 1
a − b
x 2 +y 2
xy +2
53.
x 2 −y 2
2xy
x 4y
y +4+ x
54. x 2y
y +3+ x
1
55. 1 +
1+ 12
1
56. 2 −
1+ 13
1
57. 1
1+ 1+x
x+1
x
58. 1
1− x+1
1− 1x
59.
x− 1x
1
x −x
60. x−1
x2
61. Choose a problem from this exercise set and clearly work it out on paper,
explaining each step in words. Scan your page and post it on the discussion
board.
63. Two methods for simplifying complex rational expressions have been
presented in this section. Which of the two methods do you feel is more
efficient, and why?
ANSWERS
2
1:
5
3
3:
2
5: 4/5
7: −6/11
10
9:
3
x
11:
5
3(x−1)
13: xy
x+1
15:
2x−1
2
17: −
3
5x
19:
x+5
3x
21: −
x+3
4x+1
23: − x
xy
25: x−y
2x+5
27:
5x
x−y
29:
4xy−2
x+y
31:
x 2y2
x−7
33:
x−5
3x+1
35:
2x−1
1
37:
3x−4
x2
39:
x+5
3(x−2)
41: −
2x+3
5x+18
43:
x+12
(x−1)(3x+4)
45:
(x+2)(x+3)
x+1
47:
3x+2
a2 +2ab+4b 2
49:
27
a2 −ab+b 2
51:
a2 b 2
2(x+y)
53: x−y
5
55:
3
x+1
57:
x+2
1
59:
x+1
LEARNING OBJECTIVES
Solve rational equations by clearing the fractions by multiplying both sides of the
equation by the least common denominator (LCD).
Solution: We first make a note that x ≠ 0 and then multiply both sides by the LCD,
3x:
Check your answer by substituting 12 for x to see if you obtain a true statement.
10. An equation containing at least
one rational expression.
1172
Chapter 7 Rational Expressions and Equations
After multiplying both sides of the previous example by the LCD, we were left with a
linear equation to solve. This is not always the case; sometimes we will be left with a
quadratic equation.
1 3
Example 2: Solve: 2 − x(x+1) = x+1.
After distributing and dividing out the common factors, a quadratic equation
remains. To solve it, rewrite it in standard form, factor, and then set each factor
equal to 0.
Up to this point, all of the possible solutions have solved the original equation.
However, this may not always be the case. Multiplying both sides of an equation by
variable factors may lead to extraneous solutions11, which are solutions that do
not solve the original equation. A complete list of steps for solving a rational
equation is outlined in the following example.
x 2 5
Example 3: Solve: x+2 + x 2 +5x+6
= x+3
.
Solution:
Step 3: Multiply both sides of the equation by the LCD. Distribute carefully and then
simplify.
Step 4: Solve the resulting equation. Here the result is a quadratic equation.
Rewrite it in standard form, factor, and then set each factor equal to 0.
Step 5: Check for extraneous solutions. Always substitute into the original
equation, or the factored equivalent. In this case, choose the factored equivalent to
check:
x 3 7x
Try this! Solve: x−5 + x+2 = x 2 −3x−10
.
Answer: −3
Video Solution
Sometimes all potential solutions are extraneous, in which case we say that there is
no solution to the original equation. In the next two examples, we demonstrate two
ways in which a rational equation can have no solutions.
Example 4: Solve: 3x − 2
x+2
= 1
x+2
.
2 x −4
Answer: No solution, ∅
x 4 36
Example 5: Solve: x−4 − x+5 = x 2 +x−20
.
Both of these values are restrictions of the original equation; hence both are
extraneous.
Answer: No solution, ∅
1 x 4x
Try this! Solve: x+1 + x−3 = x 2 −2x−3
.
Answer: ∅
Video Solution
It is important to point out that this technique for clearing algebraic fractions only
works for equations. Do not try to clear algebraic fractions when simplifying expressions.
As a reminder, we have
Literal Equations
Literal equations, or formulas, are often rational equations. Hence the techniques
described in this section can be used to solve for particular variables. Assume that
all variable expressions in the denominator are nonzero.
Solution: The goal is to isolate x. Assuming that y is nonzero, multiply both sides by
y and then add 5 to both sides.
Answer: x = yz + 5
ab
Answer: c = b+a
y+1
Try this! Solve for y: x = y−1 .
x+1
Answer: y = x−1
Video Solution
KEY TAKEAWAYS
TOPIC EXERCISES
Solve.
1 1 1
1.
2
+ x = 8
1 1 2
2.
3
− x = 9
1 2 1
3.
3x
− 3
= x
2 1 3
4.
5x
− x = 10
1
5.
2x+1
=5
3
6.
3x−1
+4=5
2x−3 2
7.
x+5
= x+5
5x x−1
8.
2x−1
= 2x−1
5 6
9. x−7 = x−9
5 3
10.
x+5
= x+1
x 6
11.
6
− x =0
5 x
12. x + 5
= −2
x 2
13.
x+12
= x
2x 1
14.
x+5
= 6−x
1 x
15. x + 2x+1
=0
9x 4
16.
3x−1
− x =0
2 48
17. 1 − x = x2
9 5
18. 2 − x = x2
12 12
19. 1 + x = x−2
3x−5 1
20. 1 − =− x
x(3x−4)
x 14
21.
2
= x+3
3x x+1
22.
2
= 3−x
−3x+3
23. 6 = x−1
12 6(4−x)
24.
x−2
=2+ x−2
2x 3(x−1)
25. 2 + =
x−3 x−3
(x−1)(6x−1)
x 1 x
26.
x−1
+ 6x−1
=
12 1 2
27.
x 2 −81
= x+9
− x−9
14 2 3
28.
x 2 −49
= x−7
− x+7
6x 4 3x
29.
x+3
+ x−3
= x 2 −9
3x 17 48
30.
x+2
− x−2
=− x 2 −4
31. x −1 +3=0
32. 4 − y −1 = 0
33. y −2 −4=0
34. 9x −2 −1=0
35. 3(x − 1) −1 + 5 = 0
36. 5 − 2(3x + 1) −1 = 0
2 2
37. 3 + =
x−3 x−3
1 1
38. x = x+1
x x+1
39.
x+1
= x
3x−1 x
40.
3x
= x+3
4x−7 3x−2
41.
x−5
= x−5
x 1
42.
x 2 −9
= x−3
3x+4 2
43.
x−8
− 8−x
=1
1 6
44. x = x(x+3)
3 1 13
45. x = x+1
+ x(x+1)
x 3 9x
46.
x−1
− 4x−1
= (4x−1)(x−1)
1 x 2
47.
x−4
+ x−2
= x 2 −6x+8
x x−1 5
48.
x−5
+ x 2 −11x+30
= x−6
x 6 5
49.
x+1
− 5x 2 +4x−1
=− 5x−1
−8 2(x+2) 1
50.
x 2 −4x−12
+ x 2 +4x−60
= x+2
x 20 4
51.
x+2
− x 2 −x−6
=− x−3
x+7 x−1 4
52.
x−1
+ x+1
= x 2 −1
x+7 81 9
55.
x−2
− x 2 +5x−14
= x+7
x 5x+30
56.
x−6
+1= 36−x 2
2x 4 −7
57.
x+1
− 4x−3
= 4x 2 +x−3
x−5 5 5x
58.
x−10
+ x−5
=− x 2 −15x+50
5 x+1 5
59.
x 2 +5x+4
+ x 2 +3x−4
= x 2 −1
1 x−9 1
60.
x 2 −2x−63
+ x 2 +10x+21
= x 2 −6x−27
4 2(x−2) x+2
61.
x 2 −4
+ x 2 −4x−12
= x 2 −8x+12
6x 11x+1 6x
63.
x−1
− 2x 2 −x−1
= 2x+1
8x 4x 1
64.
2x−3
+ 2x 2 −7x+6
= x−2
D
65. Solve for r: t= r.
2A
66. Solve for b: h= b
.
I
67. Solve for P: t= Pr
.
C
68. Solve for π : r= .
2π
1 1 1
69. Solve for c: a = b
+ c.
y−y 1
70. Solve for y: m= x−x 1 .
S
74. Solve for S: h= 2πr
− r.
x
75. Solve for x: y= x+2
.
2x+1
76. Solve for x: y= 5x
.
1 1 1
77. Solve for R: R = R1
+ R2
.
1 1 1
78. Solve for S 1 : = S1
+ S2
.
f
79. Explain why multiplying both sides of an equation by the LCD sometimes
produces extraneous solutions.
81. Explain how we can tell the difference between a rational expression and
a rational equation. How do we treat them differently?
ANSWERS
1: −8/3
3: −1
5: −2/5
7: 5/2
9: −3
11: −6, 6
13: −4, 6
15: −1
17: −6, 8
19: −4, 6
21: −7, 4
23: ∅
25: ∅
27: −39
31: −1/3
35: 2/5
37: ∅
39: −1/2
41: ∅
43: −7
45: 5
47: −1
49: ∅
51: −4
53: 5/3
55: ∅
57: 1/2
59: −6, 4
61: 10
63: 1/3
D
65: r = t
I
67: P = tr
ab
69: c = b−a
P−2l
71: w = 2
1−sn
73: m = s
2y
75: x = 1−y
R1 R2
77: R = R 1 +R 2
LEARNING OBJECTIVES
Number Problems
Recall that the reciprocal12 of a nonzero number n is 1/n. For example, the
reciprocal of 5 is 1/5 and 5 ⋅ 1/5 = 1. In this section, the applications will often
involve the key word “reciprocal.” When this is the case, we will see that the
algebraic setup results in a rational equation.
Example 1: A positive integer is 4 less than another. The sum of the reciprocals of
the two positive integers is 10/21. Find the two integers.
1192
Chapter 7 Rational Expressions and Equations
We can solve this rational expression by multiplying both sides of the equation by
the least common denominator (LCD). In this case, the LCD is 21n(n − 4).
The question calls for integers and the only integer solution is n = 7. Hence
disregard 6/5. Use the expression n − 4 to find the smaller integer.
Answer: The two positive integers are 3 and 7. The check is left to the reader.
Example 2: A positive integer is 4 less than another. If the reciprocal of the smaller
integer is subtracted from twice the reciprocal of the larger, then the result is 1/30.
Find the two integers.
Solution:
Solve this rational expression by multiplying both sides by the LCD. The LCD is
30n(n − 4).
Here we have two viable possibilities for the larger integer. For this reason, we will
we have two solutions to this problem.
Answer: Two sets of positive integers solve this problem: {6, 10} and {20, 24}.
Try this! The difference between the reciprocals of two consecutive positive odd
integers is 2/15. Find the integers.
Video Solution
where the distance, D, is given as the product of the average rate, r, and the time, t,
traveled at that rate. If we divide both sides by the average rate, r, then we obtain
13. Described by the formula the formula
D = rt, where the distance, D,
is given as the product of the
average rate, r, and the time, t,
traveled at that rate.
For this reason, when the unknown quantity is time, the algebraic setup for
distance problems often results in a rational equation. Similarly, when the
unknown quantity is the rate, the setup also may result in a rational equation.
We begin any uniform motion problem by first organizing our data with a chart.
Use this information to set up an algebraic equation that models the application.
Example 5: Mary spent the first 120 miles of her road trip in traffic. When the
traffic cleared, she was able to drive twice as fast for the remaining 300 miles. If the
total trip took 9 hours, then how fast was she moving in traffic?
Solution: First, identify the unknown quantity and organize the data.
To avoid introducing two more variables for the time column, use the formula
t = Dr. Here the time for each leg of the trip is calculated as follows:
The algebraic setup is defined by the time column. Add the times for each leg of the
trip to obtain a total of 9 hours:
We begin solving this equation by first multiplying both sides by the LCD, 2x.
Example 6: A passenger train can travel, on average, 20 miles per hour faster than a
freight train. If the passenger train covers 390 miles in the same time it takes the
freight train to cover 270 miles, then how fast is each train?
Solution: First, identify the unknown quantities and organize the data.
Use the formula t = Dr to fill in the time column for each train.
Because the trains travel the same amount of time, finish the algebraic setup by
equating the expressions that represent the times:
Solve this equation by first multiplying both sides by the LCD, x(x + 20) .
Answer: The speed of the passenger train is 65 miles per hour and the speed of the
freight train is 45 miles per hour.
Example 7: Brett lives on the river 8 miles upstream from town. When the current
is 2 miles per hour, he can row his boat downstream to town for supplies and back
in 3 hours. What is his average rowing speed in still water?
Solution:
Rowing downstream, the current increases his speed, and his rate is x + 2 miles per
hour. Rowing upstream, the current decreases his speed, and his rate is x − 2 miles
per hour. Begin by organizing the data in the following chart:
Use the formula t = Dr to fill in the time column for each leg of the trip.
The algebraic setup is defined by the time column. Add the times for each leg of the
trip to obtain a total of 3 hours:
Solve this equation by first multiplying both sides by the LCD, (x + 2) (x − 2).
Try this! Dwayne drove 18 miles to the airport to pick up his father and then
returned home. On the return trip he was able to drive an average of 15 miles per
hour faster than he did on the trip there. If the total driving time was 1 hour, then
what was his average speed driving to the airport?
Answer: His average speed driving to the airport was 30 miles per hour.
Video Solution
The rate at which a task can be performed is called a work rate14. For example, if a
painter can paint a room in 8 hours, then the task is to paint the room, and we can
write
In other words, the painter can complete 18 of the task per hour. If he works for less
than 8 hours, then he will perform a fraction of the task. For example,
Obtain the amount of the task completed by multiplying the work rate by the
amount of time the painter works. Typically, work-rate problems involve people
14. The rate at which a task can be working together to complete tasks. When this is the case, we can organize the data
performed. in a chart, just as we have done with distance problems.
Suppose an apprentice painter can paint the same room by himself in 10 hours.
Then we say that he can complete 101 of the task per hour. Let t represent the time it
takes both of the painters, working together, to paint the room.
To complete the chart, multiply the work rate by the time for each person. The
portion of the room each can paint adds to a total of 1 task completed. This is
represented by the equation obtained from the first column of the chart:
This setup results in a rational equation that can be solved for t by multiplying both
sides by the LCD, 40.
Therefore, the two painters, working together, complete the task in 4 49 hours.
1 1
Here t1
and t2
are the individual work rates and t is the time it takes to complete
one task working together. If we factor out the time, t, and then divide both sides by
t, we obtain an equivalent work-rate formula:
1 1
15. t ⋅ t + t ⋅ t = 1, where t1
1 2 1
and t1 are the individual work
2
rates and t is the time it takes In summary, we have the following equivalent work-rate formulas:
to complete the task working
together.
Example 3: Working alone, Billy’s dad can complete the yard work in 3 hours. If
Billy helps his dad, then the yard work takes 2 hours. How long would it take Billy
working alone to complete the yard work?
Solution: The given information tells us that Billy’s dad has an individual work rate
of 13 task per hour. If we let x represent the time it takes Billy working alone to
complete the yard work, then Billy’s individual work rate is 1x , and we can write
Working together, they can complete the task in 2 hours. Multiply the individual
work rates by 2 hours to fill in the chart.
The amount of the task each completes will total 1 completed task. To solve for x,
we first multiply both sides by the LCD, 3x.
Of course, the unit of time for the work rate need not always be in hours.
Example 4: Working together, two construction crews can build a shed in 5 days.
Working separately, the less experienced crew takes twice as long to build a shed
than the more experienced crew. Working separately, how long does it take each
crew to build a shed?
Solution:
Working together, the job is completed in 5 days. This gives the following setup:
The first column in the chart gives us an algebraic equation that models the
problem:
To determine the time it takes the less experienced crew, we use 2x:
Answer: Working separately, the experienced crew takes 7½ days to build a shed,
and the less experienced crew takes 15 days to build a shed.
Try this! Joe’s garden hose fills the pool in 12 hours. His neighbor has a thinner
hose that fills the pool in 15 hours. How long will it take to fill the pool using both
hoses?
Video Solution
KEY TAKEAWAYS
• In this section, all of the steps outlined for solving general word
problems apply. Look for the new key word “reciprocal,” which
indicates that you should write the quantity in the denominator of a
fraction with numerator 1.
• When solving distance problems where the time element is unknown,
D
use the equivalent form of the uniform motion formula, t = r , to avoid
introducing more variables.
• When solving work-rate problems, multiply the individual work rate by
the time to obtain the portion of the task completed. The sum of the
portions of the task results in the total amount of work completed.
TOPIC EXERCISES
1. A positive integer is twice another. The sum of the reciprocals of the two
positive integers is 3/10. Find the two integers.
2. A positive integer is twice another. The sum of the reciprocals of the two
positive integers is 3/12. Find the two integers.
5. A positive integer is 2 less than another. If the sum of the reciprocal of the
smaller and twice the reciprocal of the larger is 5/12, then find the two
integers.
7. The sum of the reciprocals of two consecutive positive even integers is 11/
60. Find the two even integers.
8. The sum of the reciprocals of two consecutive positive odd integers is 16/
63. Find the integers.
10. The difference of the reciprocals of two consecutive positive odd integers
is 2/99. Find the integers.
13. A positive integer is 5 less than another. If the reciprocal of the smaller
integer is subtracted from 3 times the reciprocal of the larger, then the
result is 1/12. Find the two integers.
14. A positive integer is 6 less than another. If the reciprocal of the smaller
integer is subtracted from 10 times the reciprocal of the larger, then the
result is 3/7. Find the two integers.
15. James can jog twice as fast as he can walk. He was able to jog the first 9
miles to his grandmother’s house, but then he tired and walked the
remaining 1.5 miles. If the total trip took 2 hours, then what was his average
jogging speed?
16. On a business trip, an executive traveled 720 miles by jet aircraft and
then another 80 miles by helicopter. If the jet averaged 3 times the speed of
the helicopter and the total trip took 4 hours, then what was the average
speed of the jet?
17. Sally was able to drive an average of 20 miles per hour faster in her car
after the traffic cleared. She drove 23 miles in traffic before it cleared and
then drove another 99 miles. If the total trip took 2 hours, then what was
her average speed in traffic?
18. Harry traveled 15 miles on the bus and then another 72 miles on a train.
If the train was 18 miles per hour faster than the bus and the total trip took
2 hours, then what was the average speed of the train?
19. A bus averages 6 miles per hour faster than a trolley. If the bus travels 90
miles in the same time it takes the trolley to travel 75 miles, then what is the
speed of each?
20. A passenger car averages 16 miles per hour faster than the bus. If the bus
travels 56 miles in the same time it takes the passenger car to travel 84
miles, then what is the speed of each?
21. A light aircraft travels 2 miles per hour less than twice as fast as a
passenger car. If the passenger car can travel 231 miles in the same time it
takes the aircraft to travel 455 miles, then what is the average speed of
each?
22. Mary can run 1 mile per hour more than twice as fast as Bill can walk. If
Bill can walk 3 miles in the same time it takes Mary to run 7.2 miles, then
what is Bill’s average walking speed?
24. A jet airliner traveling with a 30-mile-per-hour tailwind covers 525 miles
in the same amount of time it is able to travel 495 miles after the tailwind
eases to 10 miles per hour. What is the speed of the airliner in still air?
25. A boat averages 16 miles per hour in still water. With the current, the
boat can travel 95 miles in the same time it travels 65 miles against it. What
is the speed of the current?
26. A river tour boat averages 7 miles per hour in still water. If the total
24-mile tour downriver and 24 miles back takes 7 hours, then how fast is the
river current?
27. If the river current flows at an average 3 miles per hour, then a tour boat
makes the 9-mile tour downstream with the current and back the 9 miles
against the current in 4 hours. What is the average speed of the boat in still
water?
29. Jose drove 15 miles to pick up his sister and then returned home. On the
return trip, he was able to average 15 miles per hour faster than he did on
the trip to pick her up. If the total trip took 1 hour, then what was Jose’s
average speed on the return trip?
30. Barry drove the 24 miles to town and then back in 1 hour. On the return
trip, he was able to average 14 miles per hour faster than he averaged on the
trip to town. What was his average speed on the trip to town?
31. Jerry paddled his kayak upstream against a 1-mile-per-hour current for
12 miles. The return trip downstream with the 1-mile-per-hour current took
1 hour less time. How fast can Jerry paddle the kayak in still water?
32. It takes a light aircraft 1 hour more time to fly 360 miles against a
30-mile-per-hour headwind than it does to fly the same distance with it.
What is the speed of the aircraft in calm air?
33. James can paint the office by himself in 7 hours. Manny paints the office
in 10 hours. How long will it take them to paint the office working together?
34. Barry can lay a brick driveway by himself in 12 hours. Robert does the
same job in 10 hours. How long will it take them to lay the brick driveway
working together?
35. Jerry can detail a car by himself in 50 minutes. Sally does the same job in
1 hour. How long will it take them to detail a car working together?
36. Jose can build a small shed by himself in 26 hours. Alex builds the same
small shed in 2 days. How long would it take them to build the shed working
together?
37. Allison can complete a sales route by herself in 6 hours. Working with an
associate, she completes the route in 4 hours. How long would it take her
associate to complete the route by herself?
38. James can prepare and paint a house by himself in 5 days. Working with
his brother, Bryan, they can do it in 3 days. How long would it take Bryan to
prepare and paint the house by himself?
40. The teacher’s assistant can grade class homework assignments by herself
in 1 hour. If the teacher helps, then the grading can be completed in 20
minutes. How long would it take the teacher to grade the papers working
alone?
41. A larger pipe fills a water tank twice as fast as a smaller pipe. When both
pipes are used, they fill the tank in 5 hours. If the larger pipe is left off, then
how long would it take the smaller pipe to fill the tank?
42. A newer printer can print twice as fast as an older printer. If both
printers working together can print a batch of flyers in 45 minutes, then
how long would it take the newer printer to print the batch working alone?
43. Working alone, Henry takes 9 hours longer than Mary to clean the
carpets in the entire office. Working together, they clean the carpets in 6
hours. How long would it take Mary to clean the office carpets if Henry were
not there to help?
44. Working alone, Monique takes 4 hours longer than Audrey to record the
inventory of the entire shop. Working together, they take inventory in 1.5
hours. How long would it take Audrey to record the inventory working
alone?
45. Jerry can lay a tile floor in 3 hours less time than Jake. If they work
together, the floor takes 2 hours. How long would it take Jerry to lay the
floor by himself?
46. Jeremy can build a model airplane in 5 hours less time than his brother.
Working together, they need 6 hours to build the plane. How long would it
take Jeremy to build the model airplane working alone?
47. Harry can paint a shed by himself in 6 hours. Jeremy can paint the same
shed by himself in 8 hours. How long will it take them to paint two sheds
working together?
49. Jerry can lay a tile floor in 3 hours, and his assistant can do the same job
in 4 hours. If Jerry starts the job and his assistant joins him 1 hour later,
then how long will it take to lay the floor?
50. Working alone, Monique takes 6 hours to record the inventory of the
entire shop, while it takes Audrey only 4 hours to do the same job. How long
will it take them working together if Monique leaves 2 hours early?
ANSWERS
1: {5, 10}
3: {4, 8}
5: {6, 8}
7: {10, 12}
9: {6, 8}
21: Passenger car: 66 miles per hour; aircraft: 130 miles per hour
33: 4 2
17
hours
35: 27 3
11
minutes
37: 12 hours
39: 2 hours
41: 15 hours
43: 9 hours
45: 3 hours
47: 6 6
7
hours
49: 2 1
7
hours
7.7 Variation
LEARNING OBJECTIVES
Direct Variation
Consider a freight train moving at a constant speed of 30 miles per hour. The
equation that expresses the distance traveled at that speed in terms of time is given
by
After 1 hour the train has traveled 30 miles, after 2 hours the train has traveled 60
miles, and so on. We can construct a chart and graph this relation.
1219
Chapter 7 Rational Expressions and Equations
In this example, we can see that the distance varies over time as the product of the
constant rate, 30 miles per hour, and the variable, t. This relationship is described
as direct variation16 and 30 is called the variation constant. In addition, if we
divide both sides of D = 30t by t we have
“y varies directly as x”
y = kx
“y is directly proportional17 to x”
16. Describes two quantities x and
y that are constant multiples of
each other: y = kx .
“y is proportional to x”
Solution:
Use the fact that “the circumference is directly proportional to the diameter” to
write an equation that relates the two variables.
18. The nonzero multiple k, when Now use this formula to find d when the circumference is 20 inches.
quantities vary directly or
inversely.
Typically, we will not be given the constant of variation. Instead, we will be given
information from which it can be determined.
Example 2: An object’s weight on earth varies directly to its weight on the moon. If
a man weighs 180 pounds on earth, then he will weigh 30 pounds on the moon. Set
up an algebraic equation that expresses the weight on earth in terms of the weight
on the moon and use it to determine the weight of a woman on the moon if she
weighs 120 pounds on earth.
Solution:
We are given that the “weight on earth varies directly to the weight on the moon.”
To find the constant of variation k, use the given information. A 180-pound man on
earth weighs 30 pounds on the moon, or y = 180 when x = 30 .
Solve for k.
This implies that a person’s weight on earth is 6 times her weight on the moon. To
answer the question, use the woman’s weight on earth, y = 120 pounds, and solve
for x.
Inverse Variation
If we wish to travel a fixed distance, then we can determine the average speed
required to travel that distance in a given amount of time. For example, if we wish
to drive 240 miles in 4 hours, we can determine the required average speed as
follows:
The average speed required to drive 240 miles in 4 hours is 60 miles per hour. If we
wish to drive the 240 miles in 5 hours, then determine the required speed using a
similar equation:
In this case, we would only have to average 48 miles per hour. We can make a chart
and view this relationship on a graph.
“y varies inversely20 as x”
k
y= x
“y is inversely proportional21 to x”
Solve for k.
Example 4: The weight of an object varies inversely as the square of its distance
from the center of earth. If an object weighs 100 pounds on the surface of earth
(approximately 4,000 miles from the center), then how much will it weigh at 1,000
miles above earth’s surface?
Solution:
Use the given information to find k. An object weighs 100 pounds on the surface of
earth, approximately 4,000 miles from the center. In other words, w = 100 when d =
4,000:
Solve for k.
To use the formula to find the weight, we need the distance from the center of
earth. Since the object is 1,000 miles above the surface, find the distance from the
center of earth by adding 4,000 miles:
Answer: The object will weigh 64 pounds at a distance 1,000 miles above the surface
of earth.
Joint Variation
Vocabulary Translation
y = kxz
Example 5: The area of an ellipse varies jointly as a, half of the ellipse’s major axis,
and b, half of the ellipse’s minor axis. If the area of an ellipse is 300π cm2 , where
22. Describes a quantity y that a = 10 cm and b = 30 cm, then what is the constant of proportionality? Give a
varies directly as the product
of two other quantities x and z: formula for the area of an ellipse.
y = kxz .
23. Used when referring to joint
variation.
Solution: If we let A represent the area of an ellipse, then we can use the statement
“area varies jointly as a and b” to write
To find the constant of variation, k, use the fact that the area is 300π when a = 10
and b = 30.
Answer: The constant of proportionality is π, and the formula for the area is
A = abπ .
Try this! Given that y varies directly as the square of x and inversely to z, where y =
2 when x = 3 and z = 27, find y when x = 2 and z = 16.
Answer: 3/2
Video Solution
KEY TAKEAWAY
TOPIC EXERCISES
7. Every particle of matter in the universe attracts every other particle with
a force, F, that is directly proportional to the product of the masses, m 1 and
m 2 , of the particles and inversely proportional to the square of the
distance, d, between them.
10. The time, t, it takes an object to fall is directly proportional to the square
root of the distance, d, it falls.
27. y varies jointly as x and z, where y = 2/3 when x = 1/2 and z = 12.
34. y varies jointly as x and the square of z, where y = 6 when x = 1/4 and z =
2/3.
38. y varies directly as the square root of x and inversely as the square of z,
where y = 15 when x = 25 and z = 2.
39. y varies directly as the square of x and inversely as z and the square of w,
where y = 14 when x = 4, w = 2, and z = 2.
40. y varies directly as the square root of x and inversely as z and the square
of w, where y = 27 when x = 9, w = 1/2, and z = 4.
42. The sales tax on the purchase of a new car varies directly as the price of
the car. If an $18,000 new car is purchased, then the sales tax is $1,350. How
much sales tax is charged if the new car is priced at $22,000?
published to be $1.10, then determine the value of the stock if the EPS
increases by $0.20.
44. The distance traveled on a road trip varies directly with the time spent
on the road. If a 126-mile trip can be made in 3 hours, then what distance
can be traveled in 4 hours?
46. The area of circle varies directly as the square of its radius. If the area of
a circle with radius 7 centimeters is determined to be 49π square
centimeters, then find the constant of proportionality.
47. The surface area of a sphere varies directly as the square of its radius.
When the radius of a sphere measures 2 meters, the surface area measures
16π square meters. Find the surface area of a sphere with radius 3 meters.
48. The volume of a sphere varies directly as the cube of its radius. When the
radius of a sphere measures 3 meters, the volume is 36π cubic meters. Find
the volume of a sphere with radius 1 meter.
49. With a fixed height, the volume of a cone is directly proportional to the
square of the radius at the base. When the radius at the base measures 10
centimeters, the volume is 200 cubic centimeters. Determine the volume of
the cone if the radius of the base is halved.
50. The distance, d, an object in free fall drops varies directly with the square
of the time, t, that it has been falling. If an object in free fall drops 36 feet in
1.5 seconds, then how far will it have fallen in 3 seconds?
Hooke’s law suggests that the extension of a hanging spring is directly proportional
to the weight attached to it. The constant of variation is called the spring constant.
Figure 7.1
Robert Hooke
(1635–1703)
56. After an accident, it was determined that it took a driver 80 feet to stop
his car. In an experiment under similar conditions, it takes 45 feet to stop
the car moving at a speed of 30 miles per hour. Estimate how fast the driver
was moving before the accident.
Boyle’s law states that if the temperature remains constant, the volume, V, of a given
mass of gas is inversely proportional to the pressure, p, exerted on it.
Figure 7.2
Robert Boyle
(1627–1691)
Portrait of Robert
Boyle, from
http://commons.wikim
edia.org/wiki/
File:Robert_boyle.jpg.
57. A balloon is filled to a volume of 216 cubic inches on a diving boat under
1 atmosphere of pressure. If the balloon is taken underwater approximately
33 feet, where the pressure measures 2 atmospheres, then what is the
volume of the balloon?
59. To balance a seesaw, the distance from the fulcrum that a person must
sit is inversely proportional to his weight. If a 72-pound boy is sitting 3 feet
from the fulcrum, then how far from the fulcrum must a 54-pound boy sit to
balance the seesaw?
many men will be required to lay 2,400 square feet of cobblestone given 6
hours?
62. The volume of a right circular cylinder varies jointly as the square of its
radius and its height. A right circular cylinder with a 3-centimeter radius
and a height of 4 centimeters has a volume of 36π cubic centimeters. Find a
formula for the volume of a right circular cylinder in terms of its radius and
height.
64. The time, t, it takes an object to fall is directly proportional to the square
root of the distance, d, it falls. An object dropped from 4 feet will take 1/2
second to hit the ground. How long will it take an object dropped from 16
feet to hit the ground?
Newton’s universal law of gravitation states that every particle of matter in the
universe attracts every other particle with a force, F, that is directly proportional to
the product of the masses, m 1 and m 2 , of the particles and inversely proportional
to the square of the distance, d, between them. The constant of proportionality is
called the gravitational constant.
Figure 7.3
Sir Isaac Newton
(1643–1724)
Source: Portrait of
Isaac Newton by Sir
Godfrey Kneller, from
http://commons.wikim
edia.org/wiki/
File:GodfreyKneller-
IsaacNewton-1689.jpg.
65. If two objects with masses 50 kilograms and 100 kilograms are 1/2 meter
apart, then they produce approximately 1.34 × 10 −6 newtons (N) of
force. Calculate the gravitational constant.
66. Use the gravitational constant from the previous exercise to write a
formula that approximates the force, F, in newtons between two masses m 1
and m 2 , expressed in kilograms, given the distance d between them in
meters.
67. Calculate the force in newtons between earth and the moon, given that
the mass of the moon is approximately 7.3 × 10 22 kilograms, the mass of
24
earth is approximately 6.0× 10 kilograms, and the distance between
11
them is on average 1.5 × 10 meters.
68. Calculate the force in newtons between earth and the sun, given that the
mass of the sun is approximately 2.0 × 10 30 kilograms, the mass of earth
24
is approximately 6.0
× 10 kilograms, and the distance between them is
8
on average 3.85 × 10 meters.
71. If y varies directly as the square of x and inversely as the square of t, then
how does y change if both x and t are doubled?
ANSWERS
1: D = kt
3: d = kv 2
k
5: V = p
m 1 ⋅m 2
7: F =k
d2
⎯⎯
⎯
9: T = k√ L
11: y = 5x
13: y = 4x
15: y = 7
5
x
35
17: y = x
27
19: y = x
1
21: y = 4x
23: y = 4xz
25: y = 2
3
xz
27: y = 1
9
xz
29: y = 5x 2
3
31: y = x2
33: y = 3xz 2
5xz
35: y = w2
20√x
37: y = z
7x 2
39: y = w2 z
41: $382.50
43: $26.65
45: 2π
51: 1/4
55: 49 feet
59: 4 feet
61: 4 men
67: 1.98 × 10 20 N
1241
Chapter 7 Rational Expressions and Equations
REVIEW EXERCISES
25
1. ; {−5, 0, 5}
2x 2
x−4
2. ; {1/2, 2, 4}
2x−1
1
3. ; {−3, 0, 3}
x 2 +9
x+3
4. ; {−3, 0, 3}
x 2 −9
5
5. x
1
6.
x(3x+1)
x+2
7.
x 2 −25
x−1
8.
(x−1)(2x−3)
x−8
9.
x 2 −64
3x 2 +9x
10.
2x 3 −18x
x 2 −5x−24
11.
x 2 −3x−40
2x 2 +9x−5
12.
4x 2 −1
x 2 −144
13.
12−x
8x 2 −10x−3
14.
9−4x 2
x−3
15. Given f (x) = , find f (−3) , f (0), and f (3).
x 2 +9
x 2 −2x−24
16. Simplify g (x) = and state the restrictions.
2x 2 −9x−18
3x 5 x−3
17.
x−3
⋅ 9x 2
12y 2 (2y−1)
(2y−1)
18. ⋅ 3y
y3
3x 2 x 2 −4x+4
19.
x−2
⋅ 5x 3
x 2 −8x+15 12x 2
20.
9x 5
⋅ x−3
x 2 −36 2x 2 +10x
21.
x 2 −x−30
⋅ x 2 +5x−6
9x 2 +11x+2 9x−2
22.
4−81x 2
⋅
(x+1) 2
9x 2 −25 3x+5
23.
5x 3
÷ 15x 4
4x 2 2x
24.
4x 2 −1
÷ 2x−1
3x 2 −13x−10 9x 2 +12x+4
25.
x 2 −x−20
÷ x 2 +8x+16
2x 2 +xy−y 2 4x 2 −y 2
26.
x 2 +2xy+y 2
÷ 3x 2 +2xy−y 2
27.
2x 2 −6x−20
8x 2 +17x+2
÷ (8x 2 − 39x − 5)
28.
12x 2 −27x 4
15x 4 +10x 3
÷ (3x 2 + x − 2)
25y 2 −1 10y 2
5y 4 (y−2)
1
⋅ ÷
(y−2)
29.
5y−1 2
10x 4 5x 2 x−1
30.
1−36x 2
÷ 6x 2 −7x+1
⋅ 2x
, calculate (f ⋅ g) (x)
16x 2 −9 x 2 +3x−10
31. Given f (x) = and g (x) =
x+5 4x 2 +5x−6
and state the restrictions.
, calculate (f /g)
x+7 x 2 −49
32. Given f (x) = and g (x) = (x) and
5x−1 25x 2 −5x
state the restrictions.
5x 3
33. y − y
x 3
34.
x 2 −x−6
− x 2 −x−6
2x 1
35.
2x+1
+ x−5
3 1−2x
36. x−7 + x2
7x 2
37.
4x 2 −9x+2
− x−2
5 20−9x
38.
x−5
+ 2x 2 −15x+25
x 2 5(x−3)
39.
x−5
− x−3
− x 2 −8x+15
3x x−4 12(2−x)
40.
2x−1
− x+4
+ 2x 2 +7x−4
1 1
41.
x 2 +8x−9
− x 2 +11x+18
4 3
42.
x 2 +13x+36
+ x 2 +6x−27
y+1 1 2y
43.
y+2
− 2−y
+ y 2 −4
1 1 2
44. y − 1−y
− y 2 −1
Complex Fractions
Simplify.
4− 2x
47. 2x−1
3x
1
3
− 3y1
48. 1
5
− 5y1
1
6
+ 1x
49. 1 1
−
36 x2
1 1
100
−
x2
50. 1 1
10
− x
x 2
x+3
− x+1
51. x 1
x+4
+ x+3
3 1
x − x−5
52. 5
x+2
− 2x
35
1− 12
x + x2
53. 25
1−
x2
2− 15
x +
25
x2
54.
2x−5
Solve.
6 2
55.
x−6
= 2x−1
x x+2
56.
x−6
= x−2
1 2 1
57.
3x
− 9
= x
2 3 1
58.
x−5
+ 5
= x−5
x 4 10
59.
x−5
+ x+5
=− x 2 −25
2 12 2−3x 2
60. x − 2x+3
= 2x 2 +3x
x+1 x−6
61.
2(x−2)
+ x =1
5x+2 x
62.
x+1
− x+4
=4
x 1 4x−7
63.
x+5
+ x−4
= x 2 +x−20
2 x 2(3−4x)
64.
3x−1
+ 2x+1
= 6x 2 +x−1
x 1 2x
65.
x−1
+ x+1
= x 2 −1
2x 1 4−7x
66.
x+5
− 2x−3
= 2x 2 +7x−15
1 1 1
67. Solve for a: a = b
+ c.
2y−1
68. Solve for y: x= 3y
.
69. A positive integer is twice another. The sum of the reciprocals of the two
positive integers is 1/4. Find the two integers.
71. Mary can jog, on average, 2 miles per hour faster than her husband,
James. James can jog 6.6 miles in the same amount of time it takes Mary to
jog 9 miles. How fast, on average, can Mary jog?
72. Billy traveled 140 miles to visit his grandmother on the bus and then
drove the 140 miles back in a rental car. The bus averages 14 miles per hour
slower than the car. If the total time spent traveling was 4.5 hours, then
what was the average speed of the bus?
73. Jerry takes twice as long as Manny to assemble a skateboard. If they work
together, they can assemble a skateboard in 6 minutes. How long would it
take Manny to assemble the skateboard without Jerry’s help?
74. Working alone, Joe completes the yard work in 30 minutes. It takes Mike
45 minutes to complete work on the same yard. How long would it take them
working together?
Variation
79. The distance an object in free fall drops varies directly with the square of
the time that it has been falling. It is observed that an object falls 16 feet in 1
second. Find an equation that models the distance an object will fall and use
it to determine how far it will fall in 2 seconds.
80. The weight of an object varies inversely as the square of its distance from
the center of earth. If an object weighs 180 pounds on the surface of earth
(approximately 4,000 miles from the center), then how much will it weigh at
2,000 miles above earth’s surface?
SAMPLE EXAM
15x 3 (3x−1) 2
1.
3x(3x−1)
x 2 −144
2.
x 2 +12x
x 2 +x−12
3.
2x 2 +7x−4
9−x 2
4.
(x−3) 2
5x x−5
5.
x 2 −25
⋅ 25x 2
x 2 +x−6 3x 2 −5x−2
6.
x 2 −4x+4
⋅ x 2 −9
x 2 −4x−12 x−6
7.
12x 2
÷ 6x
2x 2 −7x−4 2x 2 +7x+3
8.
6x 2 −24x
÷ 10x 2 +30x
1 1
9.
x−5
+ x+5
x 8 12x
10.
x+1
− 2−x
− x 2 −x−2
1
y + 1x
11. 1 1
−
y2 x2
9
1− 6x +
x2
12.
2− 5x − 3
x2
Solve.
1 1
15.
3
+ x =2
1 3
16.
x−5
= 2x−3
9 20
17. 1 − x + =0
x2
x+2 1 4(x+1)
18.
x−2
+ x+2
= x 2 −4
x 1 3x−10
19.
x−2
− x−3
= x 2 −5x+6
5 x 9x−4
20.
x+4
− 4−x
= x 2 −16
120
21. Solve for r: P= 1+3r
.
22. An integer is three times another. The sum of the reciprocals of the two
integers is 1/3. Find the two integers.
23. Working alone, Joe can paint the room in 6 hours. If Manny helps, then
together they can paint the room in 2 hours. How long would it take Manny
to paint the room by himself?
24. A river tour boat averages 6 miles per hour in still water. With the
current, the boat can travel 17 miles in the same time it can travel 7 miles
against the current. What is the speed of the current?
at 35 miles per hour can break to a stop in 25 feet. Find an equation that
models the breaking distance under optimal conditions and use it to
determine the breaking distance if the automobile is moving 28 miles per
hour.
5: x ≠0
7: x ≠ ±5
1
9: ;x ≠ ±8
x+8
x+3
11: ;x ≠ −5, 8
x+5
13: − (x + 12) ; x ≠ 12
15: f (−3) = − 1
3
, f (0) = − 1
3
, f (3) = 0
x3
17:
3
3(x−2)
19:
5x
2x
21:
x−1
23: 3x (3x − 5)
x+4
25:
3x+2
2
27:
(8x+1) 2
(5y+1)(y−2)
29:
50y 6
31: (f ⋅ g) (x) =
(4x+3)(x−2)
;x ≠ −5, − 2, 3
x+2 4
5x−3
33: y
2x 2 −8x+1
(2x+1)(x−5)
35:
1
37: −
4x−1
x−5
39:
x−3
3
41:
(x−1)(x+2)(x+9)
y
43:
y−2
47: 6
6x
49:
x−6
(x−3)(x+4)
51:
(x+1)(x+2)
x−7
53:
x+5
55: −3/5
57: −3
59: −10, 1
61: 3, 8
63: 3
65: Ø
bc
67: a = b+c
69: 6, 12
73: 9 minutes
75: y = 3x
77: y = 3xz
1: 5x 2 (3x − 1) ; x ≠ 0, 1
3
x−3
3: ;x ≠ −4, 1
2x−1 2
5x(x+5)
1
5:
x+2
7:
2x
( )(x+5)
2x
9:
x−5
xy
11: x−y
15: 3/5
17: 4, 5
19: 4
40 1
21: r = P
− 3
23: 3 hours
25: y = 1
49
x 2 ; 16 feet
1256
Chapter 8 Radical Expressions and Equations
8.1 Radicals
LEARNING OBJECTIVES
Square Roots
The square root1 of a number is that number that when multiplied by itself yields
the original number. For example, 4 is a square root of 16, because 42 = 16. Since
(−4)2 = 16, we can say that −4 is a square root of 16 as well. Every positive real
number has two square roots, one positive and one negative. For this reason, we use
the radical sign √ to denote the principal (nonnegative) square root2 and a
negative sign in front of the radical −√ to denote the negative square root.
If the radicand3, the number inside the radical sign, is nonnegative and can be
1. The number that, when
multiplied by itself, yields the factored as the square of another nonnegative number, then the square root of the
original number. number is apparent. In this case, we have the following property:
2. The positive square root of a
real number, denoted with the
symbol √ .
1257
Chapter 8 Radical Expressions and Equations
⎯⎯⎯⎯
a. √36
⎯⎯⎯⎯⎯⎯
b. √144
⎯⎯⎯⎯⎯⎯⎯
c. √0.04
d. √ 19
⎯⎯⎯
Solution:
⎯⎯⎯⎯ ⎯⎯⎯⎯
a. √36 = √62 = 6
⎯⎯⎯⎯⎯⎯ ⎯⎯⎯⎯⎯⎯
b. √144 = √122 = 12
⎯⎯⎯⎯⎯⎯⎯ ⎯⎯⎯⎯⎯⎯⎯⎯⎯
c. √0.04 = √(0.2)2 = 0.2
d. √ 19 = √( 13 ) = 13
⎯⎯⎯ ⎯⎯⎯⎯⎯⎯⎯2⎯
⎯⎯
a. −√4
⎯⎯
b. −√1
Solution:
⎯⎯ ⎯⎯⎯⎯
a. −√4 = −√22 = −2
⎯⎯ ⎯⎯⎯⎯
b. −√1 = −√12 = −1
The radicand may not always be a perfect square. If a positive integer is not a
⎯⎯
perfect square, then its square root will be irrational. For example, √2 is an
irrational number and can be approximated on most calculators using the square
root button.
Next, consider the square root of a negative number. To determine the square root
of −9, you must find a number that when squared results in −9:
The square root of a negative number is currently left undefined. For now, we will
⎯⎯⎯⎯⎯
state that √−9 is not a real a number.
Cube Roots
The cube root4 of a number is that number that when multiplied by itself three
times yields the original number. Furthermore, we denote a cube root using the
symbol √ 3 , where 3 is called the index5. For example,
one real cube root. Hence the technicalities associated with the principal root do
not apply. For example,
When simplifying cube roots, look for factors that are perfect cubes.
⎯⎯⎯⎯
a. √27
3
⎯⎯⎯⎯
b. √64
3
⎯⎯
c. √0
3
d. √
⎯⎯⎯
3 1
8
Solution:
⎯⎯⎯⎯ ⎯⎯⎯⎯
a. √27 = √33 = 3
3 3
⎯⎯⎯⎯ ⎯⎯⎯⎯
b. √64 = √43 = 4
3 3
⎯⎯ ⎯⎯⎯⎯
c. √0 = √03 = 0
3 3
d. √ = √( 12 ) = 12
⎯⎯⎯ ⎯⎯⎯⎯⎯⎯⎯3⎯
3 1 3
8
⎯⎯⎯⎯⎯
a. √−8
3
⎯⎯⎯⎯⎯
b. √−1
3
c. √
⎯⎯⎯⎯⎯⎯⎯
1
3
− 27
Solution:
⎯⎯⎯⎯⎯ ⎯⎯⎯⎯⎯⎯⎯⎯⎯
a. √−8 = √(−2)3 = −2
3 3
⎯⎯⎯⎯⎯ ⎯⎯⎯⎯⎯⎯⎯⎯⎯
b. √−1 = √(−1)3 = −1
3 3
c. √ =√ (− 3 ) = − 3
⎯⎯⎯⎯⎯⎯⎯ ⎯⎯⎯⎯⎯⎯⎯⎯⎯⎯⎯
1 1 3 1
3
− 27 3
It may be the case that the radicand is not a perfect cube. If an integer is not a
⎯⎯
perfect cube, then its cube root will be irrational. For example, √2 is an irrational
3
number which can be approximated on most calculators using the root button.
Depending on the calculator, we typically type in the index prior to pushing the
button and then the radicand as follows:
Therefore, we have
nth Roots
For any integer n ≥ 2, we define the nth root6 of a positive real number as that
number that when raised to the nth power yields the original number. Given any
nonnegative real number a, we have the following property:
Here n is called the index and an is called the radicand. Furthermore, we can refer
n ⎯⎯
to the entire expression √ a as a radical7. When the index is an integer greater
than 3, we say “fourth root”, “fifth root”, and so on. The nth root of any number is
apparent if we can write the radicand with an exponent equal to the index.
⎯⎯⎯⎯
a. √81
4
⎯⎯⎯⎯
b. √32
5
⎯⎯
c. √1
7
d. √
⎯⎯⎯⎯
4 1
16
Solution:
⎯⎯⎯⎯ ⎯⎯⎯⎯
a. √81 = √34 = 3
4 4
⎯⎯⎯⎯ ⎯⎯⎯⎯
b. √32 = √25 = 2
5 5
d. √ = √( 12 ) = 12
⎯⎯⎯⎯ ⎯⎯⎯⎯⎯⎯⎯4⎯
4 1 4
16
If the index is n = 2, then the radical indicates a square root and it is customary to
write the radical without the index, as illustrated below:
We have already taken care to define the principal square root of a number. At this
point, we extend this idea to nth roots when n is even. For example, 3 is a fourth
root of 81, because 34 = 81. And since (−3)4 = 81, we can say that −3 is a fourth
root of 81 as well. Hence we use the radical sign √
n to denote the principal
(nonnegative) nth root8 when n is even. In this case, for any real number a, we use
the following property:
For example,
The negative nth root, when n is even, will be denoted using a negative sign in front
of the radical −√
n .
We have seen that the square root of a negative number is not real because any real
number, when squared, will result in a positive number. In fact, a similar problem
arises for any even index:
8. The positive nth root when n is
even.
Here the fourth root of −81 is not a real number because the fourth power of any
real number is always positive.
Example 6: Simplify.
⎯⎯⎯⎯⎯⎯⎯
a. √−16
4
⎯⎯⎯⎯
b. −√16
4
Solution:
a. The radicand is negative and the index is even. Therefore, there is no real
number that when raised to the fourth power is −16.
When n is odd, the same problems do not occur. The product of an odd number of
positive factors is positive and the product of an odd number of negative factors is
negative. Hence when the index n is odd, there is only one real nth root for any real
number a. And we have the following property:
⎯⎯⎯⎯⎯⎯⎯
a. √−32
5
⎯⎯⎯⎯⎯
b. √−1
7
Solution:
a. √−32 = √(−2)5 = −2
⎯⎯⎯⎯⎯⎯⎯ ⎯⎯⎯⎯⎯⎯⎯⎯⎯
5 5
⎯⎯⎯⎯⎯ ⎯⎯⎯⎯⎯⎯⎯⎯⎯
b. √−1 = √(−1)7 = −1
7 7
⎯⎯⎯⎯⎯⎯
Try this! Find the fourth root: √625.
4
Answer: 5
Video Solution
Summary: When n is odd, the nth root is positive or negative depending on the
sign of the radicand.
When n is even, the nth root is positive or not real depending on the sign of the
radicand.
It will not always be the case that the radicand is a perfect power of the given index.
If not, we use the following two properties to simplify them. If a and b represent
positive real numbers, then we have
n ⎯⎯⎯⎯⎯⎯⎯ n ⎯⎯ n ⎯⎯
Product rule for radicals9: √ a⋅b=√ a⋅√ b
√b =
⎯⎯a⎯ n a
√
Quotient rule for radicals10: n
n
√ b
A radical is simplified11 if it does not contain any factor that can be written as a
perfect power of the index.
⎯⎯⎯⎯⎯⎯⎯ ⎯⎯ ⎯⎯
9. √a ⋅ b = √ a ⋅ √b, where
n n n
√b
⎯a⎯⎯ n a
√ ⎯⎯⎯⎯
10. n
= n , where a and b Example 8: Simplify: √12.
√ b
represent positive real
numbers.
Solution: Here 12 can be written as 4 ⋅ 3, where 4 is a perfect square.
11. A radical where the radicand
does not consist of any factor
that can be written as a perfect
power of the index.
⎯⎯
Answer: 2√3
⎯⎯⎯⎯⎯⎯
Example 9: Simplify: √135.
Therefore,
⎯⎯⎯⎯
Answer: 3√15
Solution: Begin by finding the prime factorizations of both 50 and 121. This will
enable us to easily determine the largest perfect square factors.
Therefore,
5√2
Answer: 11
⎯⎯⎯⎯⎯⎯
Example 11: Simplify: √162.
3
Solution: Use the prime factorization of 162 to find the largest perfect cube factor:
Replace the radicand with this factorization and then apply the product rule for
radicals.
⎯⎯
Answer: 3 √6
3
⎯⎯⎯⎯
Try this! Simplify: 2 √96.
3
⎯⎯⎯⎯
Answer: 4 √12
3
Video Solution
⎯⎯⎯⎯⎯⎯⎯
Example 12: Simplify: √−96.
5
Solution: Here we note that the index is odd and the radicand is negative; hence
the result will be negative. We can factor the radicand as follows:
Then simplify:
⎯⎯
Answer: −2 √3
5
Solution: In this case, consider the equivalent fraction with −8 = (−2)3 in the
numerator and then simplify.
Answer: −1/2
⎯⎯⎯⎯⎯⎯⎯⎯
Try this! Simplify √−108.
3
⎯⎯
Answer: −3 √4
3
Video Solution
KEY TAKEAWAYS
• When working with nth roots, n determines the definition that applies.
⎯⎯⎯⎯ ⎯⎯⎯⎯
We use √ an = awhen n is odd and √ an = |a|when n is even.
n n
When n is even, the negative nth root is denoted with a negative sign in
front of the radical sign.
• To simplify square roots, look for the largest perfect square factor of the
radicand and then apply the product or quotient rule for radicals.
• To simplify cube roots, look for the largest perfect cube factor of the
radicand and then apply the product or quotient rule for radicals.
• To simplify nth roots, look for the factors that have a power that is equal
to the index n and then apply the product or quotient rule for radicals.
Typically, the process is streamlined if you work with the prime
factorization of the radicand.
TOPIC EXERCISES
Part A: Radicals
Simplify.
⎯⎯⎯⎯
1. √ 81
⎯⎯⎯⎯⎯⎯
2. √ 100
⎯⎯⎯⎯
3. √ 64
⎯⎯⎯⎯⎯⎯
4. √ 121
⎯⎯
5. √ 0
⎯⎯
6. √ 1
⎯⎯⎯⎯⎯⎯⎯
7. √ 0.25
⎯⎯⎯⎯⎯⎯⎯
8. √ 0.01
⎯⎯⎯⎯⎯⎯⎯
9. √ 1.21
⎯⎯⎯⎯⎯⎯⎯
10. √ 2.25
11. √
⎯1⎯⎯
4
12. √
⎯⎯⎯⎯
1
⎯
36
13. √
⎯25
⎯⎯⎯⎯
16
14. √
⎯⎯⎯⎯
9
⎯
25
⎯⎯⎯⎯⎯⎯⎯
15. √ −25
⎯⎯⎯⎯⎯
16. √ −9
⎯⎯⎯⎯
17. −√ 36
⎯⎯⎯⎯
18. −√ 81
⎯⎯⎯⎯⎯⎯
19. −√ 100
⎯⎯
20. −√ 1
⎯⎯⎯⎯
21. √ 27
3
⎯⎯⎯⎯⎯⎯
22. √ 125
3
⎯⎯⎯⎯
23. √ 64
3
⎯⎯
24. √ 8
3
25. √
⎯1⎯⎯
3
8
26. √
⎯⎯⎯⎯
1
⎯
3
64
27. √
⎯⎯⎯⎯
8
⎯
3
27
28. √
⎯⎯⎯⎯⎯
64
⎯
3
125
⎯⎯⎯⎯⎯⎯⎯⎯⎯
29. √ 0.001
3
⎯⎯⎯⎯⎯⎯⎯⎯⎯
30. √ 1,000
3
⎯⎯⎯⎯⎯
31. √ −1
3
⎯⎯⎯⎯⎯
32. √ −8
3
⎯⎯⎯⎯⎯⎯⎯
33. √ −27
3
⎯⎯⎯⎯⎯⎯⎯
34. √ −64
3
35. √
⎯⎯⎯⎯⎯1⎯
3
− 8
36. √
⎯⎯⎯⎯⎯⎯⎯
27
⎯
3
− 64
37. √
⎯⎯⎯⎯⎯⎯⎯8 ⎯
3
− 27
38. √
⎯⎯⎯⎯⎯⎯⎯⎯
1
⎯
3
− 125
⎯⎯⎯⎯
39. √ 81
4
⎯⎯⎯⎯⎯⎯
40. √ 625
4
⎯⎯⎯⎯
41. √ 16
4
⎯⎯⎯⎯⎯⎯⎯⎯⎯⎯⎯⎯
42. √ 10,000
4
⎯⎯⎯⎯
43. √ 32
5
⎯⎯
44. √ 1
5
⎯⎯⎯⎯⎯⎯
45. √ 243
5
⎯⎯⎯⎯⎯⎯⎯⎯⎯⎯⎯⎯⎯⎯
46. √ 100,000
5
⎯⎯⎯⎯
47. −√ 16
4
⎯⎯
48. −√ 1
6
⎯⎯⎯⎯⎯⎯⎯
49. √ −32
5
⎯⎯⎯⎯⎯
50. √ −1
5
⎯⎯⎯⎯⎯
51. √ −1
⎯⎯⎯⎯⎯⎯⎯
52. √ −16
4
⎯⎯⎯⎯⎯⎯⎯
53. −5 √ −27
3
⎯⎯⎯⎯⎯
54. −2 √ −8
3
⎯⎯⎯⎯⎯⎯⎯⎯⎯⎯⎯⎯
55. 5 √ −1,000
3
⎯⎯⎯⎯⎯⎯⎯⎯⎯
56. 3 √ −243
5
⎯⎯⎯⎯⎯⎯⎯
57. 10 √ −16
4
⎯⎯⎯⎯⎯⎯⎯
58. 2 √ −64
6
⎯⎯⎯⎯
59. 3√ 25
⎯⎯
60. 6√ 4
⎯⎯⎯⎯
61. 2 √ 27
3
⎯⎯⎯⎯⎯⎯
62. 8 √ 243
5
⎯⎯
63. −7 √ 8
3
⎯⎯⎯⎯⎯⎯
64. −4 √ 625
4
⎯⎯⎯⎯⎯⎯⎯⎯⎯⎯⎯⎯⎯⎯
65. 6 √ 100,000
5
⎯⎯⎯⎯⎯⎯
66. 5 √ 128
7
Simplify.
⎯⎯⎯⎯
67. √ 32
⎯⎯⎯⎯⎯⎯
68. √ 250
⎯⎯⎯⎯
69. √ 80
⎯⎯⎯⎯⎯⎯
70. √ 150
⎯⎯⎯⎯⎯⎯
71. √ 160
⎯⎯⎯⎯
72. √ 60
⎯⎯⎯⎯⎯⎯
73. √ 175
⎯⎯⎯⎯⎯⎯
74. √ 216
⎯⎯⎯⎯⎯⎯
75. 5√ 112
⎯⎯⎯⎯⎯⎯
76. 10√ 135
77. √
⎯50
⎯⎯⎯⎯
49
⎯⎯⎯⎯⎯⎯
78. −2√ 120
⎯⎯⎯⎯⎯⎯
79. −3√ 162
80. √
⎯8⎯⎯
9
81. √
⎯⎯⎯⎯⎯
45
⎯
121
82. √
⎯96
⎯⎯⎯⎯
81
⎯⎯⎯⎯
83. √ 54
3
⎯⎯⎯⎯
84. √ 24
3
⎯⎯⎯⎯
85. √ 48
3
⎯⎯⎯⎯
86. √ 81
3
⎯⎯⎯⎯
87. √ 40
3
⎯⎯⎯⎯⎯⎯
88. √ 120
3
⎯⎯⎯⎯⎯⎯
89. √ 162
3
⎯⎯⎯⎯⎯⎯
90. √ 500
3
91. √
⎯⎯⎯⎯⎯
54
⎯
3
125
92. √
⎯⎯⎯⎯⎯
40
⎯
3
343
⎯⎯⎯⎯⎯⎯⎯
93. 5 √ −48
3
⎯⎯⎯⎯⎯⎯⎯⎯⎯
94. 2 √ −108
3
⎯⎯⎯⎯
95. 8 √ 96
4
⎯⎯⎯⎯⎯⎯
96. 7 √ 162
4
⎯⎯⎯⎯⎯⎯
97. √ 160
5
⎯⎯⎯⎯⎯⎯
98. √ 486
5
99. √
⎯224
⎯⎯⎯⎯⎯
5
243
100. √
⎯⎯⎯⎯
5
⎯
5
32
Simplify. Give the exact answer and the approximate answer rounded to the nearest
hundredth.
⎯⎯
101. √ 8
⎯⎯⎯⎯⎯⎯
102. √ 200
⎯⎯⎯⎯
103. √ 45
⎯⎯⎯⎯
104. √ 72
105. √
⎯3⎯⎯
4
106. √
⎯5⎯⎯
9
107. √
⎯32
⎯⎯⎯⎯
25
108. √
⎯48
⎯⎯⎯⎯
49
⎯⎯⎯⎯
109. √ 80
3
⎯⎯⎯⎯⎯⎯
110. √ 320
3
⎯⎯⎯⎯
111. √ 48
3
⎯⎯⎯⎯⎯⎯
112. √ 270
3
⎯⎯⎯⎯
113. 2√ 15
⎯⎯
114. 3√ 7
⎯⎯⎯⎯
115. 5√ 10
⎯⎯
116. 10√ 3
⎯⎯
117. 2 √ 7
3
⎯⎯
118. 3 √ 6
3
⎯⎯
119. 2 √ 5
4
⎯⎯
120. 3 √ 2
4
125. Explain why there are two square roots for any nonzero real number.
126. Explain why there is only one cube root for any real number.
127. What is the square root of 1, and what is the cube root of 1? Explain
why.
⎯⎯⎯⎯⎯ ⎯⎯⎯⎯⎯
128. Explain why √ −1 is not a real number and why √ −1 is a real
3
number.
ANSWERS
1: 9
3: 8
5: 0
7: 0.5
9: 1.1
11: 1/2
13: 5/4
17: −6
19: −10
21: 3
23: 4
25: 1/2
27: 2/3
29: 0.1
31: −1
33: −3
35: −1/2
37: −2/3
39: 3
41: 2
43: 2
45: 3
47: −2
49: −2
53: 15
55: −50
59: 15
61: 6
63: −14
65: 60
⎯⎯
67: 4√ 2
⎯⎯
69: 4√ 5
⎯⎯⎯⎯
71: 4√ 10
⎯⎯
73: 5√ 7
⎯⎯
75: 20√ 7
5√2
77: 7
⎯⎯
79: −27√ 2
3√5
81:
11
⎯⎯
83: 3 √ 2
3
⎯⎯
85: 2 √ 6
3
⎯⎯
87: 2 √ 5
3
⎯⎯
89: 3 √ 6
3
3
3√ 2
91:
5
⎯⎯
93: −10 √ 6
3
⎯⎯
95: 16 √ 6
4
⎯⎯
97: 2 √ 5
5
5
2√ 7
99:
3
⎯⎯
101: 2√ 2 ≈ 2.83
⎯⎯
103: 3√ 5 ≈ 6.71
√3
105:
2
≈ 0.87
4√2
107:
5
≈ 1.13
⎯⎯⎯⎯
109: 2 √ 10 ≈ 4.31
3
⎯⎯
111: 2 √ 6 ≈ 3.63
3
⎯⎯⎯⎯
113: √ 60
⎯⎯⎯⎯⎯⎯
115: √ 250
⎯⎯⎯⎯
117: √ 56
3
⎯⎯⎯⎯
119: √ 80
4
⎯⎯
121: 3√ 2 units
⎯⎯⎯⎯
123: 2 √ 14 units
3
LEARNING OBJECTIVES
1. Simplify radical expressions using the product and quotient rule for
radicals.
2. Use formulas involving radicals.
3. Evaluate given square root and cube root functions.
⎯⎯⎯⎯⎯⎯
Example 1: Simplify: √
3
8y 3 .
⎯⎯⎯⎯
a = a when n is odd.
n n
Solution: Use the fact that √
Answer: 2y
⎯⎯⎯⎯⎯⎯
Example 2: Simplify: √9x 2 .
⎯⎯⎯⎯
a = |a| when n is even.
n n
Solution: The square root has index 2; use the fact that √
1285
Chapter 8 Radical Expressions and Equations
Answer: 3 |x|
Important Note
Typically, at this point beginning algebra texts note that all variables are
assumed to be positive. If this is the case, then x in the previous example is
positive and the absolute value operator is not needed. The example can be
simplified as follows:
⎯⎯⎯⎯⎯⎯ ⎯⎯⎯⎯⎯⎯⎯⎯
√9x 2 = √32 x 2
⎯⎯⎯⎯ ⎯⎯⎯⎯
= √32 ⋅ √x 2
= 3x
In this section, we will assume that all variables are positive. This allows us to
focus on calculating nth roots without the technicalities associated with the
principal nth root problem. For this reason, we will use the following property
for the rest of the section:
n ⎯⎯⎯⎯
√ an = a, if a≥0 nth root
When simplifying radical expressions, look for factors with powers that match the
index.
⎯⎯⎯⎯⎯⎯⎯⎯⎯⎯⎯
Example 3: Simplify: √18x 3 y 4 .
Make these substitutions and then apply the product rule for radicals and simplify.
⎯⎯⎯⎯
Answer: 3xy 2 √2x
Make these substitutions and then apply the product rule for radicals and simplify.
2a2 √a
Answer:
b3
⎯⎯⎯⎯⎯⎯⎯⎯⎯⎯⎯
Example 5: Simplify: √
3
80x 5 y 7 .
Make these substitutions and then apply the product rule for radicals and simplify.
⎯⎯⎯⎯⎯⎯⎯⎯⎯⎯
Answer: 2xy 2 ⋅ √
3
10x 2 y
√y z .
⎯⎯⎯⎯⎯
9x 6
⎯
3
Example 6: Simplify 3 9
Solution: The coefficient 9 = 32 and thus does not have any perfect cube factors. It
will be left as the only remaining radicand because all of the other factors are cubes,
as illustrated below:
Replace the variables with these equivalents, apply the product and quotient rule
for radicals, and then simplify.
3
x 2 ⋅√ 9
Answer:
yz 3
⎯⎯⎯⎯⎯⎯⎯⎯⎯⎯⎯
Example 7: Simplify: √81a4 b5 .
4
Solution: Determine all factors that can be written as perfect powers of 4. Here it is
important to see that b5 = b4 ⋅ b. Hence the factor b will be left inside the radical.
⎯⎯
Answer: 3ab ⋅ √b
4
⎯⎯⎯⎯⎯⎯⎯⎯⎯⎯⎯⎯⎯⎯⎯⎯⎯
Example 8: Simplify: √
5
−32x 3 y 6 z 5 .
Solution: Notice that the variable factor x cannot be written as a power of 5 and
thus will be left inside the radical. In addition, for y 6 = y 5 ⋅ y ; the factor y will be
left inside the radical as well.
⎯⎯⎯⎯⎯⎯
Answer: −2yz ⋅ √
5
x3y
⎯⎯⎯⎯⎯⎯⎯⎯⎯⎯⎯⎯⎯⎯⎯⎯⎯⎯
Try this! Simplify: √192x 6 y 7 z 12 . (Assume all variables are positive.)
⎯⎯⎯⎯
Answer: 8x 3 y 3 z 6 √3y
Video Solution
Tip
To easily simplify an nth root, we can divide the powers by the index.
⎯⎯⎯⎯
√a6 = a3 , which is a6÷2 = a3
3 ⎯⎯⎯6⎯
√ b = b2 , which is b6÷3 = b2
6 ⎯⎯⎯⎯
√ c6 = c , which is c6÷6 = c1
If the index does not divide into the power evenly, then we can use the quotient
and remainder to simplify. For example,
⎯⎯⎯⎯
√a5 = a2 ⋅ √⎯⎯a, which is a5÷2 = a2 r 1
3 ⎯⎯⎯5⎯ 3 ⎯⎯⎯⎯
√ b = b ⋅ √b2 , which is b5÷3 = b1 r 2
5 ⎯⎯⎯⎯ ⎯ 5 ⎯⎯⎯⎯
√ c14 = c2 ⋅ √c4 , which is c14÷5 = c2 r 4
The quotient is the exponent of the factor outside of the radical, and the
remainder is the exponent of the factor left inside the radical.
Recall that this formula was derived from the Pythagorean theorem.
It is a good practice to include the formula in its general form before substituting
values for the variables; this improves readability and reduces the probability of
making errors.
√(x 2 − x 1 ) + (y 2 − y 1 ) .
the formula d =
⎯⎯⎯⎯⎯⎯⎯⎯⎯⎯⎯⎯⎯⎯⎯⎯⎯⎯⎯⎯⎯⎯⎯⎯⎯⎯⎯⎯⎯⎯⎯⎯⎯⎯⎯⎯2⎯
2
⎯⎯
Answer: 6√2 units
where L represents the length of the pendulum in feet. If the length of a pendulum
measures 6 feet, then calculate the period rounded off to the nearest tenth of a
second.
We know that the square root is not a real number when the radicand x is negative.
Therefore, we conclude that the domain consists of all real numbers greater than or
equal to 0. Here we choose 0 and some positive values for x, calculate the
corresponding y-values, and plot the resulting ordered pairs.
After plotting the points, we can then sketch the graph of the square root function.
⎯⎯
14. The function f (x) = √x .
⎯⎯⎯⎯⎯⎯⎯⎯
Example 11: Given the function f (x) = √x + 2 , find f (−2), f (2), and f (6).
⎯⎯
Answer: f (−2) = 0, f (2) = 2, and f (6) = 2√2
Since the cube root could be either negative or positive, we conclude that the
domain consists of all real numbers. For completeness, choose some positive and
negative values for x, as well as 0, and then calculate the corresponding y-values.
Plot the points and sketch the graph of the cube root function.
Example 12: Given the function g(x) = √x − 1, find g (−7), g (0), and g (55).
3 ⎯⎯⎯⎯⎯⎯⎯⎯
KEY TAKEAWAYS
TOPIC EXERCISES
⎯⎯⎯⎯⎯⎯⎯⎯
1. √ 36a2
⎯⎯⎯⎯⎯⎯⎯⎯⎯2⎯
2. √ 121b
⎯⎯⎯⎯⎯⎯⎯⎯
3. √ x 2 y 2
⎯⎯⎯⎯⎯⎯⎯⎯⎯⎯⎯⎯⎯⎯⎯
4. √ 25x 2 y 2 z 2
⎯⎯⎯⎯⎯⎯⎯⎯⎯⎯
5. √ 180x 3
⎯⎯⎯⎯⎯⎯⎯⎯⎯⎯
6. √ 150y 3
⎯⎯⎯⎯⎯⎯⎯⎯⎯⎯2⎯
7. √ 49a3 b
⎯⎯⎯⎯⎯⎯⎯⎯⎯⎯
3
⎯
8. √ 4a4 b c
⎯⎯⎯⎯⎯⎯⎯⎯⎯⎯⎯⎯
9. √ 45x 5 y 3
⎯⎯⎯⎯⎯⎯⎯⎯⎯⎯⎯⎯
10. √ 50x 6 y 4
⎯⎯⎯⎯⎯⎯⎯⎯⎯⎯⎯⎯⎯
11. √ 64r2 s6 t 5
⎯⎯⎯⎯⎯⎯⎯⎯⎯⎯⎯⎯⎯⎯⎯
12. √ 144r8 s6 t 2
⎯⎯⎯⎯⎯⎯⎯⎯⎯⎯⎯⎯2⎯
13. √ (x + 1)
⎯⎯⎯⎯⎯⎯⎯⎯⎯⎯⎯⎯⎯⎯2⎯
14. √ (2x + 3)
⎯⎯⎯⎯⎯⎯⎯⎯⎯⎯⎯⎯⎯⎯⎯⎯2⎯
15. √ 4(3x − 1)
⎯⎯⎯⎯⎯⎯⎯⎯⎯⎯⎯⎯⎯⎯⎯⎯2⎯
16. √ 9(2x + 3)
√ 25y 2
⎯⎯⎯⎯⎯⎯
9x 3
⎯
17.
√ 9y 4
⎯4x
⎯⎯⎯⎯5⎯
18.
19. √
⎯⎯⎯⎯⎯⎯
m7
⎯
36n 4
20. √
⎯147m
⎯⎯⎯⎯⎯⎯⎯9 ⎯
n6
21. √
⎯2r
⎯⎯⎯⎯⎯
2 s5
⎯
25t 4
22. √
⎯36r
⎯⎯⎯⎯⎯5⎯
s2 t 6
⎯⎯⎯⎯⎯⎯⎯⎯
23. √ 27a3
3
⎯⎯⎯⎯⎯⎯⎯⎯⎯3⎯
24. √ 125b
3
⎯⎯⎯⎯⎯⎯⎯⎯⎯⎯⎯⎯⎯⎯
25. √ 250x 4 y 3
3
⎯⎯⎯⎯⎯⎯⎯⎯⎯⎯⎯⎯5⎯
26. √ 162a3 b
3
⎯⎯⎯⎯⎯⎯⎯⎯⎯⎯⎯⎯⎯⎯⎯
27. √ 64x 3 y 6 z 9
3
⎯⎯⎯⎯⎯⎯⎯⎯⎯⎯⎯⎯⎯⎯⎯
28. √ 216x 12 y 3
3
⎯⎯⎯⎯⎯⎯⎯⎯⎯⎯
29. √ 8x 3 y 4
3
⎯⎯⎯⎯⎯⎯⎯⎯⎯⎯⎯⎯
30. √ 27x 5 y 3
3
⎯⎯⎯⎯⎯⎯⎯⎯⎯⎯
5 6
⎯
31. √ a4 b
3
c
⎯⎯⎯⎯⎯⎯⎯⎯⎯⎯
5 3
⎯
32. √ a7 b
3
c
√ 27y 3
⎯⎯⎯⎯⎯⎯
8x 4
⎯
3
33.
√ 125y 6
⎯⎯⎯⎯⎯⎯⎯⎯
x5
⎯
3
34.
⎯⎯⎯⎯⎯⎯⎯⎯⎯⎯⎯⎯⎯⎯⎯⎯⎯⎯
35. √ 360r5 s12 t 13
3
⎯⎯⎯⎯⎯⎯⎯⎯⎯⎯⎯⎯⎯⎯⎯
36. √ 540r3 s2 t 9
3
⎯⎯⎯⎯⎯⎯⎯⎯
37. √ 81x 4
4
⎯⎯⎯⎯⎯⎯⎯⎯
38. √ x 4 y 4
4
⎯⎯⎯⎯⎯⎯⎯⎯⎯⎯⎯⎯
39. √ 16x 4 y 8
4
⎯⎯⎯⎯⎯⎯⎯⎯⎯⎯⎯⎯⎯
40. √ 81x 12 y 4
4
⎯⎯⎯⎯⎯⎯⎯⎯⎯⎯
5 6
⎯
41. √ a4 b
4
c
⎯⎯⎯⎯⎯⎯⎯⎯⎯⎯
4 6 8
⎯
42. √ 5
4
ac
⎯⎯⎯⎯⎯⎯⎯⎯⎯⎯
43. √ 128x 6
4
⎯⎯⎯⎯⎯⎯⎯⎯⎯⎯
44. √ 243y 7
4
45. √
⎯32m
⎯⎯⎯⎯⎯⎯⎯
10
⎯
5
n5
46. √
⎯⎯⎯⎯⎯⎯⎯
7 9⎯
5 3 m
n 10
⎯⎯⎯⎯⎯⎯
47. −3√ 4x 2
⎯⎯⎯⎯⎯⎯
48. 7√ 9y 2
⎯⎯⎯⎯⎯⎯⎯⎯
49. −5x√ 4x 2 y
⎯⎯⎯⎯⎯⎯⎯⎯⎯⎯⎯⎯
50. −3y√ 16x 3 y 2
⎯⎯⎯⎯⎯⎯3⎯
51. 12ab√ a5 b
⎯⎯⎯⎯⎯⎯⎯⎯2⎯
52. 6a2 b√ 9a7 b
3 ⎯⎯⎯⎯⎯⎯
53. 2x ⋅ √ 8x 6
3 ⎯⎯⎯⎯⎯⎯⎯⎯
54. −5x 2 ⋅ √ 27x 3
3 ⎯⎯⎯⎯⎯⎯⎯⎯⎯⎯⎯⎯
55. 2ab ⋅ √ −8a4 b 5
3 ⎯⎯⎯⎯⎯⎯⎯⎯⎯⎯⎯⎯⎯⎯
56. 5a2 b ⋅ √ −27a3 b 3
⎯⎯⎯⎯
57. 5√ 2x
⎯⎯⎯⎯
58. 2√ 3y
⎯⎯
59. 2x√ 3
⎯⎯
60. 3y√ 2
⎯⎯⎯⎯⎯⎯
61. ab√ 10a
2 ⎯⎯
62. 2ab √a
⎯⎯⎯⎯⎯
63. m 2 n√ mn
⎯⎯⎯⎯
64. 2m 2 n 3 √ 3n
⎯⎯⎯⎯
65. 5 √ 2x
3
⎯⎯⎯⎯
66. 3 √
3
5y
3 ⎯⎯
67. 2x ⋅√ 3
3 ⎯⎯
68. 3y ⋅√ 2
Assume that the variable could represent any real number and then simplify.
⎯⎯⎯⎯⎯⎯
69. √ 4x 2
⎯⎯⎯⎯⎯⎯⎯⎯
70. √ 25y 2
⎯⎯⎯⎯⎯⎯
71. √ 8y 3
3
⎯⎯⎯⎯⎯⎯⎯⎯⎯⎯
72. √ 125a3
3
⎯⎯⎯⎯⎯⎯⎯⎯
73. √ 64x 4
4
⎯⎯⎯⎯⎯⎯⎯⎯
74. √ 81y 4
4
⎯⎯⎯⎯⎯⎯⎯⎯
75. √ 36a4
⎯⎯⎯⎯⎯⎯⎯⎯⎯⎯
76. √ 100a8
⎯⎯⎯⎯⎯⎯
77. √ 4a6
⎯⎯⎯⎯⎯
78. √ a10
⎯⎯⎯⎯⎯⎯⎯⎯⎯⎯5⎯
79. √ 18a4 b
⎯⎯⎯⎯⎯⎯⎯⎯⎯⎯3⎯
80. √ 48a5 b
⎯⎯⎯⎯⎯⎯⎯⎯⎯⎯⎯⎯⎯⎯
81. √ 128x 6 y 8
6
⎯⎯⎯⎯⎯⎯⎯⎯⎯⎯
7 8
⎯
82. √ a6 b
6
c
The y-intercepts for any graph will have the form (0, y), where y is a real number.
Therefore, to find y-intercepts, set x = 0 and solve for y. Find the y-intercepts for the
following.
⎯⎯⎯⎯⎯⎯⎯⎯⎯
83. y = √x + 4 − 1
⎯⎯⎯⎯⎯⎯⎯⎯⎯
84. y = √x + 1 − 3
3 ⎯⎯⎯⎯⎯⎯⎯⎯⎯
85. y =√ x−1 +2
3 ⎯⎯⎯⎯⎯⎯⎯⎯⎯
86. y =√ x+1 −3
Use the distance formula to calculate the distance between the given two points.
Factor the radicand and then simplify. (Assume that all expressions are positive.)
⎯⎯⎯⎯⎯⎯⎯⎯⎯⎯⎯⎯⎯⎯⎯⎯⎯⎯⎯
93. √ x 2 − 6x + 9
⎯⎯⎯⎯⎯⎯⎯⎯⎯⎯⎯⎯⎯⎯⎯⎯⎯⎯⎯⎯⎯⎯⎯
94. √ x 2 − 10x + 25
⎯⎯⎯⎯⎯⎯⎯⎯⎯⎯⎯⎯⎯⎯⎯⎯⎯⎯⎯⎯⎯⎯⎯
95. √ 4x 2 + 12x + 9
⎯⎯⎯⎯⎯⎯⎯⎯⎯⎯⎯⎯⎯⎯⎯⎯⎯⎯⎯⎯⎯
96. √ 9x 2 + 6x + 1
97. The speed of a vehicle before the brakes were applied can be estimated
by the length of the skid marks left on the road. On dry pavement, the speed,
⎯⎯
v, in miles per hour can be estimated by the formula v = 5√ d , where d
represents the length of the skid marks in feet. Estimate the speed of a
vehicle before applying the brakes on dry pavement if the skid marks left
behind measure 36 feet.
⎯⎯⎯⎯⎯⎯
√ 32
L
T = 2π
where L represents the length in feet. Calculate the period, given the following
lengths. Give the exact value and the approximate value rounded off to the nearest
tenth of a second.
99. 8 feet
100. 32 feet
The time, t, in seconds that an object is in free fall is given by the formula
⎯
√s
t=
4
where s represents the distance it has fallen in feet. Calculate the time it takes an
object to fall, given the following distances. Give the exact value and the
approximate value rounded off to the nearest tenth of a second.
103. 48 feet
104. 80 feet
⎯⎯⎯⎯⎯⎯⎯⎯⎯
107. f (x) = √ x − 1 , find f (1) , f (2) , and f (5)
⎯⎯⎯⎯⎯⎯⎯⎯⎯
108. f (x) = √ x + 5 , find f (−5) , f (−1) , and f (20)
⎯⎯
109. f (x) = √ x + 3, find f (0) , f (1) , and f (16)
⎯⎯
110. f (x) = √ x − 5, find f (0) , f (1) , and f (25)
3 ⎯⎯
111. g(x) =√ x , find g(−1) , g(0) , and g(1)
3 ⎯⎯⎯⎯⎯⎯⎯⎯⎯
112. g(x) =√ x + 7 , find g(−15) , g(−7) , and g(20)
3 ⎯⎯
113. g(x) =√ x − 2, find g(−1) , g(0) , and g(8)
3 ⎯⎯⎯⎯⎯⎯⎯⎯⎯
114. g(x) =√ x − 1 + 2, find g(0) , g(2) , and g(9)
⎯⎯⎯⎯⎯⎯⎯⎯⎯
115. f (x) = √x + 1
⎯⎯⎯⎯⎯⎯⎯⎯⎯
116. f (x) = √x − 2
3 ⎯⎯
117. f (x) =√ x +1
3 ⎯⎯⎯⎯⎯⎯⎯⎯⎯
118. f (x) =√ x+2
⎯⎯⎯⎯
119. Give a value for x such that √ x 2 ≠ x. Explain why it is important to
assume that the variables represent positive numbers.
121. Research and discuss the methods used for calculating square roots
before the common use of electronic calculators.
122. What is a surd, and where does the word come from?
ANSWERS
1: 6a
3: xy
⎯⎯⎯⎯
5: 6x√ 5x
⎯⎯
7: 7ab√ a
⎯⎯⎯⎯⎯⎯
9: 3x 2 y√ 5xy
11: 8rs3 t 2 √ t
13: x +1
15: 2 (3x − 1)
3x√x
17:
5y
m 3 √m
19:
6n 2
rs2 √2s
21:
5t 2
23: 3a
3 ⎯⎯⎯⎯
25: 5xy ⋅√ 2x
27: 4xy 2 z 3
3 ⎯⎯
29: 2xy ⋅√ y
3 ⎯⎯⎯⎯⎯⎯
31: abc2 ⋅ √ ab 2
3 x
2x⋅√
33:
3y
3 ⎯⎯⎯⎯⎯⎯⎯⎯
35: 2rs4 t 4 ⋅ √ 45r2 t
37: 3x
39: 2xy 2
4 ⎯⎯⎯⎯⎯⎯
41: abc ⋅ √ bc2
4 ⎯⎯⎯⎯⎯⎯
43: 2x ⋅ √ 8x 2
2m 2
45: n
47: −6x
49: −10x 2 √ y
⎯⎯
2 ⎯⎯⎯⎯
51: 12a3 b √ ab
53: 4x 3
2 3 ⎯⎯⎯⎯⎯⎯
55: −4a2 b ⋅ √ ab 2
⎯⎯⎯⎯⎯⎯
57: √ 50x
⎯⎯⎯⎯⎯⎯⎯⎯
59: √ 12x 2
⎯⎯⎯⎯⎯⎯⎯⎯⎯⎯2⎯
61: √ 10a3 b
⎯⎯⎯⎯⎯⎯⎯⎯⎯
63: √ m 5 n 3
⎯⎯⎯⎯⎯⎯⎯⎯
65: √ 250x
3
⎯⎯⎯⎯⎯⎯⎯⎯
67: √ 24x 3
3
69: 2 |x|
71: 2y
73: 2 |x|
75: 6a2
77: 2 ||a3 ||
2 ⎯⎯⎯⎯
79: 3a2 b √ 2b
6 ⎯⎯⎯⎯⎯2⎯
81: 2 ||xy|| ⋅√ 2y
83: (0, 1)
85: (0, 1)
⎯⎯
87: √ 5
⎯⎯⎯⎯
89: 2√ 10
⎯⎯⎯⎯
91: 3√ 10
93: x −3
95: 2x +3
⎯⎯
105: 2√ 3 ≈ 3.5 seconds
115:
117:
LEARNING OBJECTIVES
Adding and subtracting radical expressions is similar to adding and subtracting like
terms. Radicals are considered to be like radicals16, or similar radicals17, when
⎯⎯ ⎯⎯
they share the same index and radicand. For example, the terms 3√5 and 4√5
contain like radicals and can be added using the distributive property as follows:
Typically, we do not show the step involving the distributive property and simply
write
When adding terms with like radicals, add only the coefficients; the radical part
remains the same.
⎯⎯ ⎯⎯
Example 1: Add: 3√2 + 2√2.
16. Radicals that share the same Solution: The terms contain like radicals; therefore, add the coefficients.
index and radicand.
1313
Chapter 8 Radical Expressions and Equations
⎯⎯
Answer: 5√2
⎯⎯ ⎯⎯
Example 2: Subtract: 2√7 − 3√7.
Solution:
⎯⎯
Answer: −√7
If the radicand and the index are not exactly the same, then the radicals are not
similar and we cannot combine them.
⎯⎯ ⎯⎯ ⎯⎯ ⎯⎯
Example 3: Simplify: 10√5 + 6√2 − 9√5 − 7√2.
Solution:
⎯⎯ ⎯⎯
We cannot simplify any further because √5 and √2 are not like radicals; the
radicands are not the same.
⎯⎯ ⎯⎯
Answer: √5 − √2
Caution
⎯⎯ ⎯⎯ ⎯⎯⎯⎯⎯⎯⎯⎯
It is important to point out that √5 − √2 ≠ √5 − 2. We can verify this by
calculating the value of each side with a calculator.
⎯⎯ ⎯⎯ ⎯⎯⎯⎯⎯⎯⎯⎯
In general, note that √
n
a ± √b ≠ √
n
a ± b.
n
⎯⎯ ⎯⎯ ⎯⎯ ⎯⎯
Example 4: Simplify: 3 √6 + 2√6 − √6 − 3√6.
3 3
Solution:
⎯⎯ ⎯⎯
We cannot simplify any further because √6 and √6 are not like radicals; the
3
⎯⎯ ⎯⎯
Answer: 2 √6 − √6
3
Often we will have to simplify before we can identify the like radicals within the
terms.
⎯⎯⎯⎯ ⎯⎯⎯⎯
Example 5: Subtract: √12 − √48.
Solution: At first glance, the radicals do not appear to be similar. However, after
simplifying completely, we will see that we can combine them.
⎯⎯
Answer: −2√3
Solution:
⎯⎯ ⎯⎯
Answer: −√5 − √3
⎯⎯⎯⎯ ⎯⎯
Try this! Subtract: 2√50 − 6√8.
⎯⎯
Answer: −2√2
Video Solution
Next, we work with radical expressions involving variables. In this section, assume
all radicands containing variable expressions are not negative.
Solution:
We cannot combine any further because the remaining radical expressions do not
share the same radicand; they are not like radicals. Note that
3 ⎯⎯⎯⎯ 3 ⎯⎯⎯⎯ 3 ⎯⎯⎯⎯⎯⎯⎯⎯⎯⎯⎯⎯
√ 2x − √ 3x ≠ √ 2x − 3x.
⎯⎯⎯⎯ ⎯⎯⎯⎯
Answer: √2x − √3x
3 3
We will often find the need to subtract a radical expression with multiple terms. If
this is the case, remember to apply the distributive property before combining like
terms.
Solution:
⎯⎯ ⎯⎯
Answer: −√x − 9√y
Until we simplify, it is often unclear which terms involving radicals are similar.
2y − (√ 54y − √16).
⎯⎯⎯⎯ ⎯⎯⎯⎯⎯⎯ ⎯⎯⎯⎯
Example 9: Simplify: 5 √
3 3 3
Solution:
⎯⎯⎯⎯ ⎯⎯
Answer: 2 √ 2y + 2 √2
3 3
Solution:
⎯⎯⎯⎯
Answer: 14a2 √5b
⎯⎯⎯⎯ ⎯⎯⎯⎯
Answer: x√5x + 4√5x
Video Solution
Tip
Take careful note of the differences between products and sums within a
radical.
Products Sums
⎯⎯⎯⎯⎯⎯⎯⎯ ⎯⎯⎯⎯⎯⎯⎯⎯⎯⎯⎯⎯
√ x 2 y 2 = xy √x 2 + y 2 ≠ x + y
3 ⎯⎯⎯⎯⎯⎯⎯⎯ 3 ⎯⎯⎯⎯⎯⎯⎯⎯⎯⎯⎯⎯
√ x 3 y 3 = xy √ x 3 + y3 ≠ x + y
⎯⎯⎯⎯⎯⎯⎯ ⎯⎯ ⎯⎯
The property √a ⋅ b = √ a ⋅ √b says that we can simplify radicals when the
n n n
KEY TAKEAWAYS
• Add and subtract terms that contain like radicals just as you do like
terms. If the index and radicand are exactly the same, then the radicals
are similar and can be combined. This involves adding or subtracting
only the coefficients; the radical part remains the same.
• Simplify each radical completely before combining like terms.
TOPIC EXERCISES
Simplify.
⎯⎯ ⎯⎯
1. 9√ 3 + 5√ 3
⎯⎯ ⎯⎯
2. 12√ 6 + 3√ 6
⎯⎯ ⎯⎯
3. 4√ 5 − 7√ 5
⎯⎯⎯⎯ ⎯⎯⎯⎯
4. 3√ 10 − 8√ 10
⎯⎯ ⎯⎯ ⎯⎯
5. √ 6 − 4√ 6 + 2√ 6
⎯⎯⎯⎯ ⎯⎯⎯⎯ ⎯⎯⎯⎯
6. 5√ 10 − 15√ 10 − 2√ 10
⎯⎯ ⎯⎯ ⎯⎯ ⎯⎯
7. 13√ 7 − 6√ 2 − 5√ 7 + 5√ 2
⎯⎯⎯⎯ ⎯⎯⎯⎯ ⎯⎯⎯⎯ ⎯⎯⎯⎯
8. 10√ 13 − 12√ 15 + 5√ 13 − 18√ 15
− (4√ 3 − 3√ 5 )
⎯⎯ ⎯⎯ ⎯⎯
9. 6√ 5
− (6√ 6 + √ 2 )
⎯⎯ ⎯⎯ ⎯⎯
10. −12√ 2
( ) ( 5)
⎯⎯ ⎯⎯⎯⎯ ⎯⎯⎯⎯ ⎯⎯
11. 2 √ 5 − 3 √ 10 − √ 10 + 3 √
( ) ( 15 )
⎯⎯ ⎯⎯⎯⎯ ⎯⎯ ⎯⎯⎯⎯
12. −8 √ 3 + 6 √ 15 − √ 3 − √
⎯⎯ 3 ⎯⎯ 3 ⎯⎯
13. 4 √ 6 − 3√ 5 + 6√ 6
3
(7 √ 9 − 4 √ 3 ) − (√ 9 − 3 √ 3 )
3 ⎯⎯ 3 ⎯⎯ 3 ⎯⎯ 3 ⎯⎯
15.
( ) ( 25 )
3 ⎯⎯ 3 ⎯⎯⎯⎯ 3 ⎯⎯ 3 ⎯⎯⎯⎯
16. −8 √ 5 + √ 25 − 2 √ 5 + 6 √
⎯⎯ ⎯⎯
17. 9√ x + 7√ x
18. −8√ y
⎯⎯ + 4 ⎯⎯
√y
19. 7x√ y
⎯⎯ − 3x ⎯⎯ ⎯⎯
√ y + x√ y
⎯⎯ ⎯⎯ ⎯⎯
20. 10y 2 √ x − 12y 2 √ x − 2y 2 √ x
⎯⎯⎯⎯ ⎯⎯ ⎯⎯⎯⎯ ⎯⎯
21. 2√ ab − 5√ a + 6√ ab − 10√ a
22. −3x√ y
⎯⎯ + 6 ⎯⎯ ⎯⎯ ⎯⎯
√ y − 4x√ y − 7√ y
(3√ xy − 7√ xy )
23. 5√ xy
⎯⎯⎯⎯ − ⎯⎯⎯⎯ ⎯⎯⎯⎯
− (2a√ b − 4√ ab)
⎯⎯ ⎯⎯ ⎯⎯⎯⎯
24. −8a√ b
( ) ( 3x )
⎯⎯⎯⎯ ⎯⎯⎯⎯ ⎯⎯⎯⎯ ⎯⎯⎯⎯
25. 3 √ 2x − √ 3x − √ 2x − 7 √
( ) ( 2y )
⎯⎯ ⎯⎯⎯⎯ ⎯⎯ ⎯⎯⎯⎯
26. √ y − 4 √ 2y − √ y − 5 √
⎯⎯ 3 ⎯⎯
27. 5 √
3
x − 12 √ x
28. −2 √ y ⎯⎯ − 3 3 ⎯⎯
y
3
√
(√ 4xy − √ xy ) − (2 √ 4xy − √ xy )
4 ⎯⎯⎯⎯⎯⎯ 3 ⎯⎯⎯⎯ 4 ⎯⎯⎯⎯⎯⎯ 3 ⎯⎯⎯⎯
33.
(5 √ 6y − 5√ y ) − (2 √ 6y + 3√ y )
6 ⎯⎯⎯⎯ ⎯⎯ 6 ⎯⎯⎯⎯ ⎯⎯
34.
Simplify.
⎯⎯⎯⎯ ⎯⎯⎯⎯
35. √ 75 − √ 12
⎯⎯⎯⎯ ⎯⎯⎯⎯
36. √ 24 − √ 54
⎯⎯⎯⎯ ⎯⎯⎯⎯ ⎯⎯
37. √ 32 + √ 27 − √ 8
⎯⎯⎯⎯ ⎯⎯⎯⎯ ⎯⎯⎯⎯
38. √ 20 + √ 48 − √ 45
⎯⎯⎯⎯ ⎯⎯⎯⎯ ⎯⎯⎯⎯ ⎯⎯⎯⎯
39. √ 28 − √ 27 + √ 63 − √ 12
⎯⎯⎯⎯ ⎯⎯⎯⎯ ⎯⎯⎯⎯ ⎯⎯⎯⎯
40. √ 90 + √ 24 − √ 40 − √ 54
⎯⎯⎯⎯ ⎯⎯⎯⎯ ⎯⎯⎯⎯⎯⎯ ⎯⎯
41. √ 45 − √ 80 + √ 245 − √ 5
⎯⎯⎯⎯⎯⎯ ⎯⎯⎯⎯ ⎯⎯⎯⎯ ⎯⎯
42. √ 108 + √ 48 − √ 75 − √ 3
− (√ 27 − √ 72 )
⎯⎯ ⎯⎯⎯⎯ ⎯⎯⎯⎯
43. 4√ 2
− (√ 20 − √ 50 )
⎯⎯ ⎯⎯⎯⎯ ⎯⎯⎯⎯
44. −3√ 5
⎯⎯⎯⎯ 3 ⎯⎯⎯⎯
45. √ 16 −√ 54
3
⎯⎯⎯⎯ 3 ⎯⎯⎯⎯
46. √ 81 −√ 24
3
⎯⎯⎯⎯⎯⎯ 3 ⎯⎯⎯⎯ 3 ⎯⎯
47. √ 135 +√ 40 − √ 5
3
⎯⎯⎯⎯⎯⎯ 3 ⎯⎯⎯⎯ 3 ⎯⎯
48. √ 108 −√ 32 − √ 4
3
⎯⎯⎯⎯ ⎯⎯⎯⎯
49. 2√ 27 − 2√ 12
⎯⎯⎯⎯ ⎯⎯⎯⎯
50. 3√ 50 − 4√ 32
⎯⎯⎯⎯⎯⎯ ⎯⎯⎯⎯ ⎯⎯⎯⎯
51. 3√ 243 − 2√ 18 − √ 48
⎯⎯⎯⎯⎯⎯ ⎯⎯⎯⎯ ⎯⎯⎯⎯
52. 6√ 216 − 2√ 24 − 2√ 96
⎯⎯⎯⎯ ⎯⎯⎯⎯ ⎯⎯⎯⎯ ⎯⎯⎯⎯
53. 2√ 18 − 3√ 75 − 2√ 98 + 4√ 48
⎯⎯⎯⎯ ⎯⎯⎯⎯ ⎯⎯⎯⎯ ⎯⎯⎯⎯⎯⎯
54. 2√ 45 − √ 12 + 2√ 20 − √ 108
( ) ( 54 )
⎯⎯⎯⎯⎯⎯ ⎯⎯⎯⎯ ⎯⎯⎯⎯ ⎯⎯⎯⎯
55. 2 √ 363 − 3 √ 96 − 7 √ 12 − 2 √
( ) ( 40 )
⎯⎯⎯⎯⎯⎯ ⎯⎯⎯⎯⎯⎯ ⎯⎯⎯⎯ ⎯⎯⎯⎯
56. 2 √ 288 + 3 √ 360 − 2 √ 72 − 7 √
⎯⎯⎯⎯⎯⎯ ⎯⎯⎯⎯
59. √ 81b + √ 4b
⎯⎯⎯⎯⎯⎯⎯⎯ ⎯⎯
60. √ 100a + √a
⎯⎯⎯⎯⎯⎯⎯⎯ ⎯⎯⎯⎯⎯⎯⎯⎯⎯⎯
61. √ 9a2 b − √ 36a2 b
⎯⎯⎯⎯⎯⎯⎯⎯ ⎯⎯⎯⎯⎯⎯⎯⎯
62. √ 50a2 − √ 18a2
⎯⎯⎯⎯⎯⎯ ⎯⎯⎯⎯ ⎯⎯ ⎯⎯⎯⎯
63. √ 49x − √ 9y + √ x − √ 4y
⎯⎯⎯⎯ ⎯⎯⎯⎯⎯⎯ ⎯⎯⎯⎯⎯⎯
64. √ 9x + √ 64y − √ 25x − √ ⎯⎯
y
⎯⎯⎯⎯⎯⎯⎯⎯⎯⎯⎯⎯ 3 ⎯⎯⎯⎯⎯⎯⎯⎯
78. √ 1000a2 − √ 64a2
3
3 ⎯⎯⎯⎯⎯⎯⎯⎯ 3 ⎯⎯⎯⎯⎯⎯⎯⎯⎯⎯ 3 ⎯⎯
80. x ⋅ √ 54x 3 − √ 250x 6 + x 2 ⋅ √ 2
⎯⎯⎯⎯⎯⎯⎯⎯ 4 ⎯⎯⎯⎯⎯⎯⎯⎯
81. √ 16y 2 +√ 81y 2
4
⎯⎯⎯⎯⎯⎯⎯⎯ 5 ⎯⎯⎯4⎯
82. √ 32y 4 −√ y
5
91. Choose values for x and y and use a calculator to show that
⎯⎯⎯⎯⎯⎯⎯⎯⎯ ⎯⎯ ⎯⎯
√ x + y ≠ √ x + √ y.
92. Choose values for x and y and use a calculator to show that
⎯⎯⎯⎯⎯⎯⎯⎯⎯⎯⎯⎯
√x 2 + y 2 ≠ x + y .
ANSWERS
⎯⎯
1: 14√ 3
⎯⎯
3: −2√ 5
⎯⎯
5: −√ 6
⎯⎯ ⎯⎯
7: 8√ 7 − √2
⎯⎯ ⎯⎯
9: 9√ 5 − 4√ 3
⎯⎯ ⎯⎯⎯⎯
11: −√ 5 − 4√ 10
⎯⎯ 3 ⎯⎯
13: 10 √ 6 − 3√ 5
3
⎯⎯ 3 ⎯⎯
15: 6 √ 9 −√ 3
3
⎯⎯
17: 16√ x
19: 5x√ y
⎯⎯
⎯⎯⎯⎯ ⎯⎯
21: 8√ ab − 15√ a
23: 9√ xy
⎯⎯⎯⎯
⎯⎯⎯⎯ ⎯⎯⎯⎯
25: 2√ 2x + 6√ 3x
⎯⎯
27: −7 √
3
x
5 ⎯⎯⎯⎯
29: 4a ⋅√ 3b
⎯⎯⎯⎯ 3 ⎯⎯⎯⎯
31: 13√ 2a − 5√ 2a
⎯⎯⎯⎯⎯⎯
33: −√
4
4xy
⎯⎯
35: 3√ 3
⎯⎯ ⎯⎯
37: 2√ 2 + 3√ 3
⎯⎯ ⎯⎯
39: 5√ 7 − 5√ 3
⎯⎯
41: 5√ 5
⎯⎯ ⎯⎯
43: 10√ 2 − 3√ 3
⎯⎯
45: −√ 2
3
⎯⎯
47: 4 √ 5
3
⎯⎯
49: 2√ 3
⎯⎯ ⎯⎯
51: 23√ 3 − 6√ 2
⎯⎯ ⎯⎯
53: −8√ 2 + √3
⎯⎯ ⎯⎯
55: 8√ 3 − 6√ 6
⎯⎯
57: 26 √ 2
3
⎯⎯
59: 11√ b
⎯⎯
61: −3a√ b
⎯⎯
63: 8√ x − 5√ ⎯⎯
y
⎯⎯⎯⎯
65: 20√ 2x − 12√ ⎯⎯
y
⎯⎯
67: −8m√ n
69: −2x√ y
⎯⎯ − 2y ⎯⎯
√x
71: −4x√ y
⎯⎯
⎯⎯⎯⎯
73: 3m 2 √ 3n
⎯⎯⎯⎯⎯⎯ ⎯⎯⎯⎯
75: 2a√ 3ab − 12a2 √ ab
⎯⎯
77: 2 √
3
a
3 ⎯⎯⎯⎯
79: 7x ⋅√ 2x
⎯⎯⎯⎯
81: 5 √ y 2
4
⎯⎯⎯⎯⎯⎯
83: 4 √ 2a3
4
3 ⎯⎯
85: −2x + 2√ x
3 ⎯⎯⎯⎯ 3 ⎯⎯
87: 7x ⋅√ xy − 3y ⋅ √ x
3 ⎯⎯⎯⎯⎯⎯ 3 ⎯⎯⎯⎯⎯⎯⎯⎯
89: 7x ⋅√ 6xy − 6x ⋅ √ 2xy 2
LEARNING OBJECTIVES
When multiplying radical expressions with the same index, we use the product rule
for radicals. If a and b represent positive real numbers,
⎯⎯ ⎯⎯
Example 1: Multiply: √2 ⋅ √6.
Solution: This problem is a product of two square roots. Apply the product rule for
radicals and then simplify.
⎯⎯
Answer: 2√3
1330
Chapter 8 Radical Expressions and Equations
⎯⎯ ⎯⎯
Example 2: Multiply: √9 ⋅ √6.
3 3
Solution: This problem is a product of cube roots. Apply the product rule for
radicals and then simplify.
⎯⎯
Answer: 3 √2
3
⎯⎯ ⎯⎯
Example 3: Multiply: 2√3 ⋅ 5√2.
Solution: Using the product rule for radicals and the fact that multiplication is
commutative, we can multiply the coefficients and the radicands as follows.
Typically, the first step involving the application of the commutative property is
not shown.
⎯⎯
Answer: 10√6
⎯⎯⎯⎯ ⎯⎯⎯⎯⎯⎯⎯⎯
Example 4: Multiply: −2 √5x ⋅ 3 √25x 2 .
3 3
Solution:
Answer: −30x
Use the distributive property when multiplying rational expressions with more
than one term.
⎯⎯
Solution: Apply the distributive property and multiply each term by 4√3.
⎯⎯
Answer: 24 − 36√2
Solution: Apply the distributive property and then simplify the result.
⎯⎯⎯⎯
Answer: 2x − 10x ⋅ √2x
3
The process for multiplying radical expressions with multiple terms is the same
process used when multiplying polynomials. Apply the distributive property,
simplify each radical, and then combine like terms.
⎯⎯
Answer: −3 − 2√5
Solution:
⎯⎯⎯⎯
Answer: 9x − 6√xy + y
⎯⎯ ⎯⎯ ⎯⎯
Answer: 6 − 12√2 + 5√6 − 20√3
Video Solution
Solution: Apply the distributive property and then combine like terms.
Answer: −3
Therefore, for nonnegative real numbers a and b, we have the following property:
To divide radical expressions with the same index, we use the quotient rule for
radicals. If a and b represent nonnegative numbers, where b ≠ 0, then we have
√80
Example 10: Divide: .
√10
Solution: In this case, we can see that 10 and 80 have common factors. If we apply
the quotient rule for radicals and write it as a single square root, we will be able to
reduce the fractional radicand.
⎯⎯
Answer: 2√2
√16x 5 y 4
Example 11: Divide: .
√2xy
Solution:
⎯⎯⎯⎯
Answer: 2x 2 y√2y
3
√ 54a3 b5
Example 12: Divide: 3
.
√ 16a2 b2
Solution:
3b⋅√3a
Answer: 2
To do this, multiply the fraction by a special form of 1 so that the radicand in the
denominator can be written with a power that matches the index. After doing this,
simplify and eliminate the radical in the denominator. For example,
√3
Example 13: Rationalize the denominator: .
√2
√6
Answer: 2
1
Example 14: Rationalize the denominator: .
2√3x
Solution: The radicand in the denominator determines the factors that you need to
√3x
use to rationalize it. In this example, multiply by 1 in the form .
√3x
√3x
Answer: 6x
Typically, we will find the need to reduce, or cancel, after rationalizing the
denominator.
5√2
Example 15: Rationalize the denominator: .
√5ab
√5ab
Solution: In this example, we will multiply by 1 in the form .
√5ab
Notice that a and b do not cancel in this example. Do not cancel factors inside a
radical with those that are outside.
√10ab
Answer: ab
2√3ab
Answer: 3b
Video Solution
Note that multiplying by the same factor in the denominator does not rationalize it.
√x 2
3
In this case, if we multiply by 1 in the form of , then we can write the radicand in
√x 2
3
1
Example 16: Rationalize the denominator: 3
.
√25
⎯⎯⎯⎯
Solution: The radical in the denominator is equivalent to √52 . To rationalize the
3
⎯⎯⎯⎯
denominator, it should be √53 . To obtain this, we need one more factor of 5.
3
35
√
Therefore, multiply by 1 in the form of 35
.
√
3
√ 5
Answer: 5
√ 2b2 .
⎯27a
⎯⎯⎯⎯⎯
3
Example 17: Rationalize the denominator:
√22 b
3
Solution: In this example, we will multiply by 1 in the form .
√22 b
3
3
3√ 4ab
Answer: 2b
1
Example 18: Rationalize the denominator: 5
.
√ 4x 3
√23 x 2
5
Solution: In this example, we will multiply by 1 in the form .
√23 x 2
5
5
√ 8x 2
Answer: 2x
When two terms involving square roots appear in the denominator, we can
rationalize it using a very special technique. This technique involves multiplying
the numerator and the denominator of the fraction by the conjugate of the
denominator. Recall that multiplying a radical expression by its conjugate produces
a rational number.
1
Example 19: Rationalize the denominator: .
√ √2
3 −
⎯⎯ ⎯⎯
Solution: In this example, the conjugate of the denominator is √3 + √2.
(√3+√2)
(√3+√2)
Therefore, multiply by 1 in the form .
⎯⎯ ⎯⎯
Answer: √3 + √2
Notice that the terms involving the square root in the denominator are eliminated
by multiplying by the conjugate. We can use the property
(√a + √b) (√a − √b) = a − bto expedite the process of multiplying the
⎯⎯ ⎯⎯ ⎯⎯ ⎯⎯
√2−√6
Example 20: Rationalize the denominator: .
√2+√6
√2−√6
Solution: Multiply by 1 in the form .
√2−√6
⎯⎯
Answer: −2 + √3
√x +√y
Example 21: Rationalize the denominator: .
√x −√y
√x −√y
Solution: In this example, we will multiply by 1 in the form .
√x −√y
x−2√xy +y
Answer: x−y
3√5+5
Try this! Rationalize the denominator: .
2√5−3
19√5+45
Answer: 11
Video Solution
KEY TAKEAWAYS
TOPIC EXERCISES
⎯⎯ ⎯⎯
1. √ 3 ⋅ √5
⎯⎯ ⎯⎯
2. √ 7 ⋅ √3
⎯⎯ ⎯⎯
3. √ 2 ⋅ √6
⎯⎯ ⎯⎯⎯⎯
4. √ 5 ⋅ √ 15
⎯⎯ ⎯⎯
5. √ 7 ⋅ √7
⎯⎯⎯⎯ ⎯⎯⎯⎯
6. √ 12 ⋅ √ 12
⎯⎯ ⎯⎯⎯⎯
7. 2√ 5 ⋅ 7√ 10
⎯⎯⎯⎯ ⎯⎯
8. 3√ 15 ⋅ 2√ 6
( 5)
⎯⎯ 2
9. 2 √
( )
⎯⎯ 2
10. 6√ 2
⎯⎯⎯⎯ ⎯⎯⎯⎯
11. √ 2x ⋅ √ 2x
⎯⎯⎯⎯ ⎯⎯⎯⎯
12. √ 5y ⋅ √ 5y
⎯⎯⎯⎯ ⎯⎯⎯⎯
13. √ 3a ⋅ √ 12
⎯⎯⎯⎯ ⎯⎯⎯⎯
14. √ 3a ⋅ √ 2a
⎯⎯⎯⎯ ⎯⎯⎯⎯
15. 4√ 2x ⋅ 3√ 6x
⎯⎯⎯⎯⎯⎯ ⎯⎯⎯⎯
16. 5√ 10y ⋅ 2√ 2y
⎯⎯ 3 ⎯⎯⎯⎯
17. √ 5 ⋅√ 25
3
⎯⎯ 3 ⎯⎯
18. √ 4 ⋅√ 2
3
⎯⎯ 3 ⎯⎯⎯⎯
19. √ 4 ⋅√ 10
3
⎯⎯⎯⎯ 3 ⎯⎯
20. √ 18 ⋅√ 6
3
( )( 6)
3 ⎯⎯ 3 ⎯⎯
21. 5 √ 9 2 √
( )( 4)
3 ⎯⎯ 3 ⎯⎯
22. 2 √ 4 3 √
( )
3 ⎯⎯
3
23. 2 √ 2
( )
⎯⎯ 3
24. 3 √ 4
3
⎯⎯⎯⎯⎯⎯ 3 ⎯⎯⎯⎯
25. √ 3a2
3
⋅√ 9a
⎯⎯⎯⎯ 3 ⎯⎯⎯⎯⎯⎯⎯⎯
26. √ 7b
3
⋅ √ 49b 2
⎯⎯⎯⎯⎯⎯ 3 ⎯⎯⎯⎯⎯⎯
27. √ 6x 2 ⋅ √ 4x 2
3
⎯⎯⎯⎯⎯⎯ 3 ⎯⎯⎯⎯⎯2⎯
28. √
3
12y ⋅√ 9y
⎯⎯⎯⎯⎯⎯⎯⎯⎯⎯ 3 ⎯⎯⎯⎯⎯⎯⎯⎯⎯⎯⎯⎯
29. √ 20x 2 y ⋅√ 10x 2 y 2
3
⎯⎯⎯⎯⎯⎯⎯⎯ 3 ⎯⎯⎯⎯⎯⎯⎯⎯⎯⎯⎯⎯
30. √
3
63xy ⋅√ 12x 4 y 2
(3 − √ 5 )
⎯⎯ ⎯⎯
31. √ 5
( 2)
⎯⎯ ⎯⎯ ⎯⎯
32. √ 2 √ 3 − √
( 3)
⎯⎯ ⎯⎯ ⎯⎯
33. 3√ 7 2 √ 7 − √
( 10 )
⎯⎯ ⎯⎯⎯⎯
34. 2√ 5 6 − 3√
( 2)
⎯⎯ ⎯⎯ ⎯⎯
35. √ 6 √ 3 − √
( 3)
⎯⎯⎯⎯ ⎯⎯ ⎯⎯
36. √ 15 √ 5 + √
(√ x + √ xy )
⎯⎯ ⎯⎯ ⎯⎯⎯⎯
37. √ x
(√ xy + √ y )
38. √ y
⎯⎯ ⎯⎯⎯⎯ ⎯⎯
( 10b )
⎯⎯⎯⎯⎯⎯ ⎯⎯⎯⎯⎯⎯ ⎯⎯⎯⎯⎯⎯
39. √ 2ab √ 14a − 2 √
(5√ 2a − √ 3b )
⎯⎯⎯⎯⎯⎯ ⎯⎯⎯⎯ ⎯⎯⎯⎯
40. √ 6ab
(√ 2 − √ 5 ) (√ 3 + √ 7 )
⎯⎯ ⎯⎯ ⎯⎯ ⎯⎯
41.
( )( 7)
⎯⎯ ⎯⎯ ⎯⎯ ⎯⎯
42. √ 3 + √ 2 √ 5 − √
( ) ( 6 + 1)
⎯⎯ ⎯⎯
43. 2 √ 3 − 4 3√
( )( 3)
⎯⎯ ⎯⎯
44. 5 − 2 √ 6 7 − 2 √
( 3)
⎯⎯ ⎯⎯ 2
45. √ 5 − √
( )
⎯⎯ ⎯⎯ 2
46. √ 7 − √ 2
( )( 2)
⎯⎯ ⎯⎯ ⎯⎯ ⎯⎯
47. 2 √ 3 + √ 2 2 √ 3 − √
( )( 7)
⎯⎯ ⎯⎯ ⎯⎯ ⎯⎯
48. √ 2 + 3 √ 7 √ 2 − 3 √
( )
⎯⎯ ⎯⎯⎯⎯ 2
49. √ a − √ 2b
(√ )
⎯⎯⎯⎯ 2
50. ab + 1
⎯⎯
51. What are the perimeter and area of a rectangle with length of 5√ 3
⎯⎯
centimeters and width of 3√ 2 centimeters?
⎯⎯
52. What are the perimeter and area of a rectangle with length of 2√ 6
⎯⎯
centimeters and width of √ 3 centimeters?
⎯⎯
53. If the base of a triangle measures 6√ 2 meters and the height measures
⎯⎯
3√ 2 meters, then what is the area?
⎯⎯
54. If the base of a triangle measures 6√ 3 meters and the height measures
⎯⎯
3√ 6 meters, then what is the area?
Divide.
√75
55.
√3
√360
56.
√10
√72
57.
√75
√90
58.
√98
√90x 5
59.
√2x
√96y 3
60.
√3y
√162x 7 y 5
61.
√2xy
√363x 4 y 9
62.
√3xy
3
√ 16a5 b 2
63. 3
√ 2a2 b 2
3
√ 192a2 b 7
64. 3
√ 2a2 b 2
1
65.
√5
1
66.
√6
√2
67.
√3
√3
68.
√7
5
69.
2√10
3
70.
5√6
√3 −√5
71.
√3
√6 −√2
72.
√2
1
73.
√7x
1
74.
√3y
a
75.
5√ab
3b 2
76.
2√3ab
2
77. 3
√ 36
14
78. 3
√ 7
1
79. 3
√ 4x
1
80. 3
√3y 2
3
9x⋅√ 2
81. 3
√ 9xy 2
5y 2 ⋅√
3 x
82. 3
√ 5x 2 y
3a
83. 3
2 √3a2 b 2
25n
84. 3
3 √25m 2 n
3
85. 5
√ 27x 2 y
2
86. 5
√16xy 2
ab
87. 5
√ 9a3 b
abc
88. 5
√ab 2 c3
3
89.
√ −3
10
2
90.
√6 −2
1
91.
√ √3
5 +
1
92.
√7 −√2
√3
93.
√3 +√6
√5
94.
√5 +√15
10
95.
5−3√5
−2√2
96.
4−3√2
√3 +√5
97.
√3 −√5
√10 −√2
98.
√10 +√2
2√3 −3√2
99.
4√3 +√2
6√5 +2
100.
2√5 −√2
x+√y
101.
x−√y
x−√y
102.
x+√y
√a−√b
103.
√a+√b
√ab+√2
104.
√ab−√2
√x
105.
5−2√x
1
106.
√x −y
Part C: Discussion
107. Research and discuss some of the reasons why it is a common practice
to rationalize the denominator.
ANSWERS
⎯⎯⎯⎯
1: √ 15
⎯⎯
3: 2√ 3
5: 7
⎯⎯
7: 70√ 2
9: 20
11: 2x
⎯⎯
13: 6√ a
⎯⎯
15: 24x√ 3
17: 5
⎯⎯
19: 2 √ 5
3
⎯⎯
21: 30 √ 2
3
23: 16
25: 3a
3 ⎯⎯⎯⎯
27: 2x ⋅√ 3x
3 ⎯⎯⎯⎯⎯⎯
29: 2xy ⋅√ 25x
⎯⎯
31: 3√ 5 −5
⎯⎯⎯⎯
33: 42 − 3√ 21
⎯⎯ ⎯⎯
35: 3√ 2 − 2√ 3
37: x + x√ ⎯⎯
y
⎯⎯⎯⎯ ⎯⎯⎯⎯
39: 2a√ 7b − 4b√ 5a
⎯⎯ ⎯⎯⎯⎯ ⎯⎯⎯⎯ ⎯⎯⎯⎯
41: √ 6 + √ 14 − √ 15 − √ 35
⎯⎯ ⎯⎯ ⎯⎯
43: 18√ 2 + 2√ 3 − 12√ 6 − 4
⎯⎯⎯⎯
45: 8 − 2√ 15
47: 10
⎯⎯⎯⎯⎯⎯
49: a − 2√ 2ab + 2b
( )
⎯⎯ ⎯⎯ ⎯⎯
51: Perimeter: 10 √ 3 + 6 √ 2 centimeters; area: 15√ 6 square
centimeters
55: 5
2√6
57:
5
⎯⎯
59: 3x 2 √ 5
61: 9x 3 y 2
63: 2a
√5
65:
5
√6
67:
3
√10
69:
4
3−√15
71:
3
√7x
73: 7x
√ab
75:
5b
3
√ 6
77:
3
3
√ 2x 2
79:
2x
3
3√ 6x 2 y
81: y
3
√ 9ab
83:
2b
5
√ 9x 3 y 4
85: xy
5
√ 27a2 b 4
87:
3
⎯⎯⎯⎯
89: 3√ 10 +9
√5 −√3
91:
2
⎯⎯
93: −1 + √2
−5−3√5
95:
2
⎯⎯⎯⎯
97: −4 − √ 15
15−7√6
99:
23
x 2 +2x√y +y
101:
x 2 −y
a−2√ab+b
103:
a−b
5√x +2x
105:
25−4x
LEARNING OBJECTIVES
So far, exponents have been limited to integers. In this section, we will define what
rational (or fractional) exponents20 mean and how to work with them. All of the
rules for exponents developed up to this point apply. In particular, recall the
product rule for exponents. Given any rational numbers m and n, then
For example, if we have an exponent of 12, then the product rule for exponents
implies the following:
Here 51/2 is one of two equal factors of 5; hence it is a square root of 5, and we can
write
1360
Chapter 8 Radical Expressions and Equations
a. 71/2
b. 71/3
Solution:
⎯⎯
a. 71/2 = √7
⎯⎯
b. 71/3 = √7
3
a. 811/2
b. 811/4
Solution:
⎯⎯⎯⎯
a. 811/2 = √81 = 9
⎯⎯⎯⎯ ⎯⎯⎯⎯
b. 811/4 = √81 = √34 = 3
4 4
a. (125x 3 )
1/3
b. (−32y 10 )
1/5
Solution:
a.
b.
Next, consider fractional exponents where the numerator is an integer other than
1. For example, consider the following:
This shows that 52/3 is one of three equal factors of 52 . In other words, 52/3 is the
cube root of 52 and we can write:
a. 72/5
b. 23/4
21. An equivalent expression
written using a rational
exponent. Solution:
⎯⎯⎯⎯ ⎯⎯⎯⎯
a. 72/5 = √72 = √49
5 5
⎯⎯⎯⎯ ⎯⎯
b. 23/4 = √23 = √8
4 4
a. 82/3
b. (32)3/5
Solution:
a.
b. We can often avoid very large integers by working with their prime factorization.
⎯⎯⎯⎯
Example 6: Rewrite using rational exponents: √x 2 .
3
Answer: x 2/3
⎯⎯⎯⎯
Example 7: Rewrite using rational exponents: √
6
y3 .
Answer: y 1/2
In other words, it does not matter if we apply the power first or the root first. For
example, we can apply the power before the root:
Answer: 4
Answer: 125
Video Solution
3 ⎯⎯⎯⎯
To calculate √22 = 22/3 = 2 ^ (2/3) ≈ 1.587, we would type
In this section, we review all of the rules of exponents, which extend to include
rational exponents. If given any rational numbers m and n, then we have
xm
Quotient rule: = x m−n , x ≠ 0
x n
(x ) = x
m n m⋅n
Power rule:
(xy) = x y
n n n
Power rule for a product:
(y )
n
x xn
Power rule for a quotient: = n ,y≠0
y
1
Negative exponents: x −n =
xn
Zero exponent: x 0 = 1, x ≠ 0
Solution:
Answer: 25/6
1/2
Example 10: Simplify: x 1/3 .
x
Solution:
Answer: x 1/6
Solution:
Answer: y 1/2
Solution:
Answer: 8a3 b6
Solution:
Answer: 1/125
(8a b )
3/4 3 2/3
Answer: 4a1/6 b2
Video Solution
To apply the product or quotient rule for radicals, the indices of the radicals
involved must be the same. If the indices are different, then first rewrite the
radicals in exponential form and then apply the rules for exponents.
⎯⎯ ⎯⎯
Example 14: Multiply: √2 ⋅ √2.
3
Solution: In this example, the index of each radical factor is different. Hence the
product rule for radicals does not apply. Begin by converting the radicals into an
equivalent form using rational exponents. Then apply the product rule for
exponents.
⎯⎯⎯⎯
Answer: √25
6
3
√ 4
Example 15: Divide: 5
.
√ 2
Solution: In this example, the index of the radical in the numerator is different
from the index of the radical in the denominator. Hence the quotient rule for
radicals does not apply. Begin by converting the radicals into an equivalent form
using rational exponents and then apply the quotient rule for exponents.
⎯⎯⎯⎯
Answer: √27
15
Solution: Here the radicand of the square root is a cube root. After rewriting this
expression using rational exponents, we will see that the power rule for exponents
applies.
⎯⎯
Answer: √2
3
KEY TAKEAWAYS
TOPIC EXERCISES
⎯⎯
1. √ 6
⎯⎯⎯⎯
2. √ 10
⎯⎯⎯⎯
3. √ 11
3
⎯⎯
4. √ 2
4
⎯⎯⎯2⎯
5. √ 5
3
⎯⎯⎯3⎯
6. √ 2
4
⎯⎯
7. √
5
x
⎯⎯
8. √
6
x
⎯⎯⎯⎯
9. √ x 7
6
⎯⎯⎯⎯
10. √ x 4
5
1/2
11. 2
1/3
12. 5
2/3
13. 7
3/5
14. 2
15. x 3/4
16. x 5/6
17. x −1/2
18. x −3/4
19. ( x )
1 −1/3
20. ( x )
1 −3/5
1/2
21. 25
1/2
22. 36
1/2
23. 121
1/2
24. 144
25. (
4)
1 1/2
26. (
9)
4 1/2
−1/2
27. 4
−1/2
28. 9
29. (
4)
1 −1/2
30. (
16 )
1 −1/2
1/3
31. 8
1/3
32. 125
33. (
27 )
1 1/3
34. (
125 )
8 1/3
1/3
35. (−27)
36. (−64)
1/3
1/4
37. 16
1/4
38. 625
−1/4
39. 81
−1/4
40. 16
1/5
41. 100, 000
1/5
42. (−32)
43. (
32 )
1 1/5
44. (
243 )
1 1/5
3/2
45. 9
3/2
46. 4
5/3
47. 8
2/3
48. 27
3/2
49. 16
2/5
50. 32
51. (
16 )
1 3/4
52. (
81 )
1 3/4
2/3
53. (−27)
4/3
54. (−27)
3/5
55. (−32)
4/5
56. (−32)
3/4
57. 2
2/3
58. 3
1/5
59. 5
1/7
60. 7
3/2
61. (−9)
3/2
62. −9
63. Explain why (−4)^(3/2) gives an error on a calculator and −4^(3/2) gives
an answer of −8.
64. Marcy received a text message from Mark asking her how old she was. In
response, Marcy texted back “125^(2/3) years old.” Help Mark determine
how old Marcy is.
2/3
65. 2 ⋅ 2 4/3
3/2
66. 3 ⋅ 3 1/2
1/2
67. 5 ⋅ 5 1/3
1/6
68. 2 ⋅ 2 3/4
5 7/3
71.
5 1/3
2 9/2
72.
2 1/2
2a2/3
73.
a1/6
3b 1/2
74.
b 1/3
75. (8 )
1/2 2/3
76. (3 )
6 2/3
77. (x 2/3 )
1/2
78. (y 3/4 )
4/5
79. (4x 2 y 4 )
1/2
80. (9x 6 y 2 )
1/2
( a1/2 )
4/3
a3/4
83.
( b1/10 )
10/3
b 4/5
84.
( y4 )
1/2
4x 2/3
85.
( y9 )
1/3
27x 3/4
86.
y 1/2 ⋅y 2/3
87.
y 1/6
x 2/5 ⋅x 1/2
88.
x 1/10
xy
89.
x 1/2 y 1/3
x 5/4 y
90.
xy 2/5
49a5/7 b 3/2
91.
7a3/7 b 1/4
16a5/6 b 5/4
92.
8a1/2 b 2/3
(9x y )
2/3 6 3/2
93.
x 1/2 y
(125x y )
3 3/5 2/3
94.
xy 1/3
(27a b )
1/4 3/2 2/3
95.
a1/6 b 1/2
(25a b )
2/3 4/3 3/2
96.
a1/6 b 1/3
⎯⎯ 5 ⎯⎯
97. √ 9 ⋅√ 3
3
⎯⎯ 5 ⎯⎯⎯⎯
98. √ 5 ⋅√ 25
⎯⎯ 3 ⎯⎯
99. √ x ⋅√ x
100. √ y
⎯⎯ ⋅ 4 ⎯⎯
y
√
⎯⎯⎯⎯ 4 ⎯⎯
101. √ x 2
3
⋅√ x
⎯⎯⎯⎯ 3 ⎯⎯
102. √ x 3
5
⋅√ x
3
√ 100
103.
√10
5
√ 16
104. 3
√ 4
3 2
√a
105.
√a
5 4
√b
106. 3
√ b
3 2
√x
107. 5 3
√x
4 3
√x
108. 3 2
√x
109. √ √ 16
⎯⎯⎯⎯⎯⎯⎯⎯
⎯⎯⎯⎯⎯
5
110. √ √ 9
⎯⎯⎯⎯⎯⎯⎯⎯⎯
3
111. √ √ 2
⎯⎯⎯⎯⎯⎯⎯⎯⎯
35
112. √ √ 5
⎯⎯⎯⎯⎯⎯⎯⎯⎯
35
113. √ √ 7
⎯⎯⎯⎯⎯⎯⎯⎯⎯
3
114. √ √ 3
⎯⎯⎯⎯⎯⎯⎯⎯⎯
3
115. Who is credited for devising the notation for rational exponents? What
are some of his other accomplishments?
116. When using text, it is best to communicate nth roots using rational
exponents. Give an example.
ANSWERS
1/2
1: 6
1/3
3: 11
2/3
5: 5
7: x 1/5
9: x 7/6
⎯⎯
11: √ 2
⎯⎯⎯2⎯
13: √ 7
3
⎯⎯⎯⎯
15: √ x 3
4
1
17:
√x
⎯⎯
19: √
3
x
21: 5
23: 11
25: 1/2
27: 1/2
29: 2
31: 2
33: 1/3
35: −3
37: 2
39: 1/3
41: 10
43: 1/2
45: 27
47: 32
49: 64
51: 1/8
53: 9
55: −8
57: 1.68
59: 1.38
63: In the first expression, the square root of a negative number creates an
error condition on the calculator. The square root of a negative number is
not real. In the second expression, because of the order of operations, the
negative sign is applied to the answer after 4 is raised to the (3/2) power.
65: 4
5/6
67: 5
69: y 13/20
71: 25
73: 2a1/2
75: 2
77: x 1/3
79: 2xy 2
81: 8xy 2
83: a1/3
2x 1/3
85:
y2
87: y
1/2
95: 9b
⎯⎯⎯⎯
13
⎯
97: √ 3
15
⎯⎯⎯⎯
99: √ x 5
6
⎯⎯⎯⎯⎯⎯
101: √ x 11
12
⎯⎯⎯⎯
103: √ 10
6
⎯⎯
105: √
6
a
⎯⎯
107: √
15
x
⎯⎯
109: √ 4
5
⎯⎯
111: √ 2
15
⎯⎯
113: √ 7
6
LEARNING OBJECTIVES
Radical Equations
A radical equation22 is any equation that contains one or more radicals with a
variable in the radicand. Following are some examples of radical equations, all of
which will be solved in this section:
We begin with the squaring property of equality23; given real numbers a and b, we
have the following:
1385
Chapter 8 Radical Expressions and Equations
This is important because we will use this property to solve radical equations.
Consider a very simple radical equation that can be solved by inspection:
use of the squaring property of equality and the fact that (√a) = √a2 = a when
Here we can see that x = 9 is a solution. To solve this equation algebraically, make
⎯⎯ 2 ⎯⎯⎯⎯
a is positive. Eliminate the square root by squaring both sides of the equation as
follows:
⎯⎯
As a check, we can see that √9 = 3 as expected. Because the converse of the
squaring property of equality is not necessarily true, solutions to the squared
equation may not be solutions to the original. Hence squaring both sides of an
equation introduces the possibility of extraneous solutions24, or solutions that do
not solve the original equation. For this reason, we must check the answers that
result from squaring both sides of an equation.
⎯⎯⎯⎯⎯⎯⎯⎯
Example 1: Solve: √x − 1 = 5.
Solution: We can eliminate the square root by applying the squaring property of
equality.
⎯⎯⎯⎯⎯⎯⎯⎯⎯⎯
Example 2: Solve: √5 − 4x = x.
You are left with a quadratic equation that can be solved by factoring.
Since you squared both sides, you must check your solutions.
After checking, you can see that x = −5 was extraneous; it did not solve the
original radical equation. Disregard that answer. This leaves x = 1 as the only
solution.
In the previous two examples, notice that the radical is isolated on one side of the
equation. Typically, this is not the case. The steps for solving radical equations
involving square roots are outlined in the following example.
⎯⎯⎯⎯⎯⎯⎯⎯⎯⎯
Example 3: Solve: √2x − 5 + 4 = x.
Solution:
Step 1: Isolate the square root. Begin by subtracting 4 from both sides of the
equation.
Step 2: Square both sides. Squaring both sides eliminates the square root.
Step 3: Solve the resulting equation. Here you are left with a quadratic equation
that can be solved by factoring.
Step 4: Check the solutions in the original equation. Squaring both sides introduces
the possibility of extraneous solutions; hence the check is required.
After checking, we can see that x = 3 is an extraneous root; it does not solve the
original radical equation. This leaves x = 7 as the only solution.
⎯⎯⎯⎯⎯⎯⎯⎯
Example 4: Solve: 3√x + 1 − 2x = 0.
Despite the fact that the term on the left side has a coefficient, it is still considered
isolated. Recall that terms are separated by addition or subtraction operators.
⎯⎯⎯⎯⎯⎯⎯⎯⎯⎯⎯⎯
Example 5: Solve: √4 − 11x − x + 2 = 0.
Since both possible solutions are extraneous, the equation has no solution.
Answer: No solution, Ø
The squaring property of equality extends to any positive integer power n. Given
real numbers a and b, we have the following:
⎯⎯⎯⎯⎯⎯⎯⎯⎯⎯
Example 6: Solve: √x 2 + 4 − 2 = 0.
3
Solution: Isolate the radical and then cube both sides of the equation.
Check.
⎯⎯⎯⎯⎯⎯⎯⎯⎯⎯
Try this! Solve: √2x − 1 + 2 = x.
Answer: x = 5 (x = 1 is extraneous)
Video Solution
It may be the case that the equation has two radical expressions.
⎯⎯⎯⎯⎯⎯⎯⎯⎯⎯ ⎯⎯⎯⎯⎯⎯⎯⎯⎯⎯
Example 7: Solve: √3x − 4 = √2x + 9.
Solution: Both radicals are considered isolated on separate sides of the equation.
Check x = 13 .
⎯⎯⎯⎯⎯⎯⎯⎯⎯⎯⎯⎯⎯⎯⎯⎯⎯⎯ ⎯⎯⎯⎯⎯⎯⎯⎯⎯⎯
Example 8: Solve: √x 2 + x − 14 = √x + 50.
3 3
Check.
We will learn how to solve some of the more advanced radical equations in the next
course, Intermediate Algebra.
⎯⎯⎯⎯⎯⎯⎯⎯⎯⎯ ⎯⎯⎯⎯⎯⎯⎯⎯
Try this! Solve: √3x + 1 = 2√x − 3.
Answer: 13
Video Solution
KEY TAKEAWAYS
• Solve equations involving square roots by first isolating the radical and
then squaring both sides. Squaring a square root eliminates the radical,
leaving us with an equation that can be solved using the techniques
learned earlier in our study of algebra. However, squaring both sides of
an equation introduces the possibility of extraneous solutions, so check
your answers in the original equation.
• Solve equations involving cube roots by first isolating the radical and
then cubing both sides. This eliminates the radical and results in an
equation that may be solved with techniques you have already
mastered.
TOPIC EXERCISES
Solve.
⎯⎯
1. √ x =2
⎯⎯
2. √ x =7
⎯⎯
3. √ x +7=8
⎯⎯
4. √ x +4=9
⎯⎯
5. √ x +6=3
⎯⎯
6. √ x +2=1
⎯⎯
7. 5√ x −1=0
⎯⎯
8. 3√ x −2=0
⎯⎯⎯⎯⎯⎯⎯⎯⎯
9. √ x −3 =3
⎯⎯⎯⎯⎯⎯⎯⎯⎯
10. √ x +5 =6
⎯⎯⎯⎯⎯⎯⎯⎯⎯⎯⎯
11. √ 3x +1 =2
⎯⎯⎯⎯⎯⎯⎯⎯⎯⎯⎯
12. √ 5x −4 =4
⎯⎯⎯⎯⎯⎯⎯⎯⎯⎯⎯
13. √ 7x + 4 + 6 = 11
⎯⎯⎯⎯⎯⎯⎯⎯⎯⎯⎯
14. √ 3x − 5 + 9 = 14
⎯⎯⎯⎯⎯⎯⎯⎯⎯
15. 2√ x −1 −3=0
⎯⎯⎯⎯⎯⎯⎯⎯⎯
16. 3√ x +1 −2=0
⎯⎯
17. √
3
x =2
⎯⎯
18. √
3
x =5
⎯⎯⎯⎯⎯⎯⎯⎯⎯⎯⎯
19. √ 2x +9 =3
3
⎯⎯⎯⎯⎯⎯⎯⎯⎯⎯⎯⎯⎯
20. √ 4x − 11 = 1
3
⎯⎯⎯⎯⎯⎯⎯⎯⎯⎯⎯
21. √ 5x +7 +3=1
3
⎯⎯⎯⎯⎯⎯⎯⎯⎯⎯⎯
22. √ 3x −6 +5=2
3
⎯⎯⎯⎯⎯⎯⎯⎯⎯
23. 2 √ x +2 −1=0
3
⎯⎯⎯⎯⎯⎯⎯⎯⎯⎯⎯
24. 2 √ 2x −3 −1=0
3
⎯⎯⎯⎯⎯⎯⎯⎯⎯⎯⎯⎯⎯ ⎯⎯⎯⎯⎯⎯⎯⎯⎯
25. √ 8x + 11 = 3√ x + 1
⎯⎯⎯⎯⎯⎯⎯⎯⎯⎯⎯ ⎯⎯⎯⎯⎯⎯⎯⎯⎯⎯⎯⎯⎯⎯⎯⎯
26. 2√ 3x − 4 = √ 2 (3x + 1)
⎯⎯⎯⎯⎯⎯⎯⎯⎯⎯⎯⎯⎯⎯⎯⎯ ⎯⎯⎯⎯⎯⎯⎯⎯⎯⎯⎯⎯⎯
27. √ 2 (x + 10) = √ 7x − 15
⎯⎯⎯⎯⎯⎯⎯⎯⎯⎯⎯⎯⎯⎯ ⎯⎯⎯⎯⎯⎯⎯⎯⎯
28. √ 5 (x − 4) = √ x + 4
⎯⎯⎯⎯⎯⎯⎯⎯⎯⎯⎯ 3 ⎯⎯⎯⎯
29. √ 5x −2 =√ 4x
3
⎯⎯⎯⎯⎯⎯⎯⎯⎯⎯⎯⎯⎯⎯ 3 ⎯⎯⎯⎯⎯⎯⎯⎯⎯⎯⎯⎯⎯⎯
30. √ 9 (x − 1) = √ 3 (x + 7)
3
⎯⎯⎯⎯⎯⎯⎯⎯⎯⎯⎯ 3 ⎯⎯⎯⎯⎯⎯⎯⎯⎯⎯⎯⎯⎯
31. √ 3x +1 =√ 2(x − 1)
3
⎯⎯⎯⎯ 3 ⎯⎯⎯⎯⎯⎯⎯⎯⎯⎯⎯⎯⎯
32. √ 9x =√ 3(x − 6)
3
⎯⎯⎯⎯⎯⎯⎯⎯⎯⎯⎯⎯⎯
33. √ 4x + 21 = x
⎯⎯⎯⎯⎯⎯⎯⎯⎯⎯⎯
34. √ 8x +9 =x
⎯⎯⎯⎯⎯⎯⎯⎯⎯⎯⎯⎯⎯⎯⎯⎯
35. √ 4 (2x − 3) = x
⎯⎯⎯⎯⎯⎯⎯⎯⎯⎯⎯⎯⎯⎯⎯⎯
36. √ 3 (4x − 9) = x
⎯⎯⎯⎯⎯⎯⎯⎯⎯
37. 2√ x −1 =x
⎯⎯⎯⎯⎯⎯⎯⎯⎯⎯⎯
38. 3√ 2x −9 =x
⎯⎯⎯⎯⎯⎯⎯⎯⎯⎯⎯
39. √ 9x +9 =x+1
⎯⎯⎯⎯⎯⎯⎯⎯⎯⎯⎯⎯⎯
40. √ 3x + 10 = x + 4
⎯⎯⎯⎯⎯⎯⎯⎯⎯
41. √ x −1 =x−3
⎯⎯⎯⎯⎯⎯⎯⎯⎯⎯⎯
42. √ 2x −5 =x−4
⎯⎯⎯⎯⎯⎯⎯⎯⎯⎯⎯⎯⎯
43. √ 16 − 3x = x − 6
⎯⎯⎯⎯⎯⎯⎯⎯⎯⎯⎯
44. √ 7 − 3x = x − 3
⎯⎯⎯⎯⎯⎯⎯⎯⎯⎯⎯⎯⎯
45. 3√ 2x + 10 = x + 9
⎯⎯⎯⎯⎯⎯⎯⎯⎯⎯⎯
46. 2√ 2x +5 =x+4
⎯⎯⎯⎯⎯⎯⎯⎯⎯
47. 3√ x −1 −1=x
⎯⎯⎯⎯⎯⎯⎯⎯⎯⎯⎯
48. 2√ 2x +2 −1=x
⎯⎯⎯⎯⎯⎯⎯⎯⎯⎯⎯⎯⎯⎯⎯
49. √ 10x + 41 − 5 = x
⎯⎯⎯⎯⎯⎯⎯⎯⎯⎯⎯⎯⎯⎯
50. √ 6 (x + 3) − 3 = x
⎯⎯⎯⎯⎯⎯⎯⎯⎯⎯⎯⎯⎯⎯⎯⎯⎯⎯⎯⎯⎯
51. √ 8x 2 − 4x + 1 = 2x
⎯⎯⎯⎯⎯⎯⎯⎯⎯⎯⎯⎯⎯⎯⎯⎯⎯⎯⎯⎯⎯⎯⎯
52. √ 18x 2 − 6x + 1 = 3x
⎯⎯⎯⎯⎯⎯⎯⎯⎯
53. 5√ x +2 =x+8
⎯⎯⎯⎯⎯⎯⎯⎯⎯⎯⎯⎯⎯⎯
54. 4√ 2 (x + 1) = x + 7
⎯⎯⎯⎯⎯⎯⎯⎯⎯⎯⎯⎯⎯
55. √ x 2 − 25 = x
⎯⎯⎯⎯⎯⎯⎯⎯⎯⎯⎯
56. √ x 2 +9=x
⎯⎯⎯⎯⎯⎯⎯⎯⎯⎯⎯⎯⎯
57. 3 + √ 6x − 11 = x
⎯⎯⎯⎯⎯⎯⎯⎯⎯⎯⎯
58. 2 + √ 9x − 8 = x
⎯⎯⎯⎯⎯⎯⎯⎯⎯⎯⎯⎯⎯
59. √ 4x + 25 − x = 7
⎯⎯⎯⎯⎯⎯⎯⎯⎯⎯⎯⎯⎯
60. √ 8x + 73 − x = 10
⎯⎯⎯⎯⎯⎯⎯⎯⎯⎯⎯
61. 2√ 4x + 3 − 3 = 2x
⎯⎯⎯⎯⎯⎯⎯⎯⎯⎯⎯
62. 2√ 6x + 3 − 3 = 3x
⎯⎯⎯⎯⎯⎯⎯⎯⎯⎯⎯⎯⎯⎯⎯
63. 2x − 4 = √ 14 − 10x
⎯⎯⎯⎯⎯⎯⎯⎯⎯⎯⎯⎯⎯⎯⎯
64. 3x − 6 = √ 33 − 24x
⎯⎯⎯⎯⎯⎯⎯⎯⎯⎯⎯⎯⎯
65. √ x 2
3
− 24 = 1
⎯⎯⎯⎯⎯⎯⎯⎯⎯⎯⎯⎯⎯
66. √ x 2
3
− 54 = 3
⎯⎯⎯⎯⎯⎯⎯⎯⎯⎯⎯⎯⎯
67. √ x 2 + 6x + 1 = 4
3
⎯⎯⎯⎯⎯⎯⎯⎯⎯⎯⎯⎯⎯
68. √ x 2 + 2x + 5 = 7
3
⎯⎯⎯⎯⎯⎯⎯⎯⎯⎯⎯⎯⎯⎯⎯⎯⎯⎯⎯⎯⎯⎯⎯⎯⎯
69. √ 25x 2
3
− 10x − 7 = −2
⎯⎯⎯⎯⎯⎯⎯⎯⎯⎯⎯⎯⎯⎯⎯⎯⎯⎯⎯⎯⎯⎯⎯⎯⎯
70. √ 9x 2
3
− 12x − 23 = −3
− 15x + 25 = √(x + 5) (x − 5)
⎯⎯⎯⎯⎯⎯⎯⎯⎯⎯⎯⎯⎯⎯⎯⎯⎯⎯⎯⎯⎯⎯⎯⎯⎯ ⎯⎯⎯⎯⎯⎯⎯⎯⎯⎯⎯⎯⎯⎯⎯⎯⎯⎯⎯⎯⎯⎯⎯⎯⎯⎯
71. √ 2x 2
− 4x + 4 = √x (5 − x)
⎯⎯⎯⎯⎯⎯⎯⎯⎯⎯⎯⎯⎯⎯⎯⎯⎯⎯⎯ ⎯⎯⎯⎯⎯⎯⎯⎯⎯⎯⎯⎯⎯⎯⎯
72. √ x 2
73. √ 2 (x 2 + 3x − 20) = √
⎯⎯⎯⎯⎯⎯⎯⎯⎯⎯⎯⎯⎯⎯⎯⎯⎯⎯⎯⎯⎯⎯⎯⎯⎯⎯⎯ 3 ⎯⎯⎯⎯⎯⎯⎯⎯⎯⎯⎯⎯2⎯
3
(x + 3)
74. √ 3x + 3x + 40 = √ (x − 5)
⎯⎯⎯⎯⎯⎯⎯⎯⎯⎯⎯⎯⎯⎯⎯⎯⎯⎯⎯⎯⎯⎯⎯ ⎯⎯⎯⎯⎯⎯⎯⎯⎯⎯⎯⎯⎯2⎯
3 2 3
75. x 1/2 − 10 = 0
79. (x − 1) 1/2 − 3 = 0
80. (x + 2) 1/2 − 6 = 0
89. The square root of 1 less than twice a number is equal to 2 less than the
number. Find the number.
90. The square root of 4 less than twice a number is equal to 6 less than the
number. Find the number.
91. The square root of twice a number is equal to one-half of that number.
Find the number.
92. The square root of twice a number is equal to one-third of that number.
Find the number.
93. The distance, d, measured in miles, a person can see an object is given by
the formula
⎯⎯⎯⎯
√ 6h
d=
2
where h represents the person’s height above sea level, measured in feet.
How high must a person be to see an object 5 miles away?
⎯⎯⎯⎯
√R
P
I=
⎯⎯⎯⎯⎯⎯
√ 32
L
T = 2π
where L represents the length in feet. For each problem below, calculate the length of
a pendulum, given the period. Give the exact value and the approximate value
rounded off to the nearest tenth of a foot.
95. 1 second
96. 2 seconds
⎯
√s
t=
4
where s represents the distance in feet the object has fallen. For each problem below,
calculate the distance an object falls, given the amount of time.
99. 1 second
100. 2 seconds
The x-intercepts for any graph have the form (x, 0), where x is a real number.
Therefore, to find x-intercepts, set y = 0 and solve for x. Find the x-intercepts for each
of the following.
⎯⎯⎯⎯⎯⎯⎯⎯⎯
103. y = √x − 3 − 1
⎯⎯⎯⎯⎯⎯⎯⎯⎯
104. y = √x + 2 − 3
3 ⎯⎯⎯⎯⎯⎯⎯⎯⎯
105. y =√ x−1 +2
3 ⎯⎯⎯⎯⎯⎯⎯⎯⎯
106. y =√ x+1 −3
ANSWERS
1: 4
3: 1
5: Ø
7: 1/25
9: 12
11: 1
13: 3
15: 13/4
17: 8
19: 9
21: −3
23: −15/8
25: 2
27: 7
29: 2
31: −3
33: 7
35: 2, 6
37: 2
39: −1, 8
41: 5
43: Ø
45: −3, 3
47: 2, 5
49: −4, −4
51: 1/2
53: 2, 7
55: Ø
57: 10
59: −6, −4
63: Ø
65: −5, 5
67: −9, 3
69: 1/5
71: 5, 10
73: −7, 7
75: 100
77: −8
79: 10
81: −13
83: 5/2
85: −6, −4
87: −2, 2
89: 5
91: 8
93: 16 2
3
feet
99: 16 feet
101: 4 feet
103: (4, 0)
105: (−7, 0)
1409
Chapter 8 Radical Expressions and Equations
REVIEW EXERCISES
Radicals
Simplify.
⎯⎯⎯⎯
1. √ 36
2. √
⎯⎯⎯⎯
4
⎯
25
⎯⎯⎯⎯⎯⎯⎯
3. √ −16
⎯⎯
4. −√ 9
⎯⎯⎯⎯⎯⎯
5. √ 125
3
⎯⎯⎯⎯⎯
6. 3 √ −8
3
7. √
⎯⎯⎯⎯
1
⎯
3
64
⎯⎯⎯⎯⎯⎯⎯
8. −5 √ −27
3
⎯⎯⎯⎯
9. √ 40
⎯⎯⎯⎯
10. −3√ 50
11. √
⎯98
⎯⎯⎯⎯
81
12. √
⎯⎯⎯⎯⎯
1
⎯
121
⎯⎯⎯⎯⎯⎯
13. 5 √ 192
3
⎯⎯⎯⎯⎯⎯⎯
14. 2 √ −54
3
Simplify.
⎯⎯⎯⎯⎯⎯⎯⎯
15. √ 49x 2
⎯⎯⎯⎯⎯⎯⎯⎯⎯⎯2⎯
16. √ 25a2 b
⎯⎯⎯⎯⎯⎯⎯⎯⎯⎯⎯⎯
17. √ 75x 3 y 2
⎯⎯⎯⎯⎯⎯⎯⎯⎯⎯⎯⎯⎯⎯⎯
18. √ 200m 4 n 3
√ 25y 2
⎯18x
⎯⎯⎯⎯⎯3⎯
19.
√
⎯108x
⎯⎯⎯⎯⎯⎯⎯3⎯
20.
49y 4
⎯⎯⎯⎯⎯⎯⎯⎯⎯⎯
21. √ 216x 3
3
⎯⎯⎯⎯⎯⎯⎯⎯⎯⎯⎯⎯⎯⎯⎯⎯⎯
22. √ −125x 6 y 3
3
⎯⎯⎯⎯⎯⎯⎯⎯⎯⎯⎯⎯⎯⎯
5 3
⎯
23. √ 27a7 b
3
c
⎯⎯⎯⎯⎯⎯⎯⎯⎯⎯⎯⎯⎯⎯
24. √ 120x 9 y 4
3
Use the distance formula to calculate the distance between the given two points.
Simplify.
⎯⎯ ⎯⎯
29. 8√ 3 + 3√ 3
⎯⎯⎯⎯ ⎯⎯⎯⎯
30. 12√ 10 − 2√ 10
⎯⎯ ⎯⎯ ⎯⎯ ⎯⎯
31. 14√ 3 + 5√ 2 − 5√ 3 − 6√ 2
⎯⎯⎯⎯ ⎯⎯ ⎯⎯⎯⎯ ⎯⎯
32. 22√ ab − 5a√ b + 7√ ab − 2a√ b
− (3√ x + 2√ ⎯⎯
y)
⎯⎯ ⎯⎯
33. 7√ x
( ) ( x)
⎯⎯ ⎯⎯ ⎯⎯ ⎯⎯
34. 8y √ x − 7x √ y − 5x √ y − 12y √
− (x√ 16y 3 − 2√ 9x 2 y 3 )
⎯⎯⎯⎯⎯⎯⎯⎯ ⎯⎯⎯⎯⎯⎯⎯⎯ ⎯⎯⎯⎯⎯⎯⎯⎯⎯⎯
39. 5y√ 4x 2 y
( ) ( )
⎯⎯⎯⎯⎯⎯⎯ ⎯⎯⎯⎯⎯⎯⎯⎯⎯⎯
2 c − 3a√ 16b 2 c −
⎯⎯⎯⎯⎯⎯⎯⎯⎯⎯⎯⎯⎯ ⎯⎯⎯⎯⎯
2 b 2 c − 9b√ a2 c
40. 2b√ 9a √ 64a
Multiply.
⎯⎯ ⎯⎯
45. √ 3 ⋅ √6
( 5)
⎯⎯ 2
46. 3 √
(√ 3 − √ 6 )
⎯⎯ ⎯⎯ ⎯⎯
47. √ 2
( 6)
⎯⎯ ⎯⎯ 2
48. √ 2 − √
( )( 5)
⎯⎯ ⎯⎯
49. 1 − √ 5 1 + √
(2√ 3 + √ 5 ) (3√ 2 − 2√ 5 )
⎯⎯ ⎯⎯ ⎯⎯ ⎯⎯
50.
⎯⎯⎯⎯⎯⎯ 3 ⎯⎯⎯⎯
51. √ 2a2
3
⋅√ 4a
⎯⎯⎯⎯⎯⎯⎯⎯⎯⎯ 3 ⎯⎯⎯⎯⎯⎯⎯⎯⎯
52. √ 25a2 b ⋅ √ 5a2 b 2
3
Divide.
√72
53.
√4
10√48
54.
√64
√98x 4 y 2
55.
√36x 2
3
√ 81x 6 y 7
56. 3
√ 8y 3
2
57.
√7
√6
58.
√3
14
59.
√2x
1
60.
2√15
1
61. 3
√ 2x 2
5a2 b
62. 3
√5ab 2
1
63.
√3 −√2
√2 −√6
64.
√2 +√6
Rational Exponents
1/2
65. 7
2/3
66. 3
67. x 4/5
68. y −3/4
1/2
69. 4
1/2
70. 50
2/3
71. 4
1/3
72. 81
73. (
4)
1 3/2
74. (
216 )
1 −1/3
1/2
75. 3 ⋅ 3 3/2
1/2
76. 2 ⋅ 2 1/3
4 3/2
77.
4 1/2
9 3/4
78.
9 1/4
79. (36x 4 y 2 )
1/2
80. (8x 6 y 9 )
1/3
( a1/2 )
2/5
a4/3
81.
( y2 )
1/2
16x 4/3
82.
Solve.
⎯⎯
83. √ x =5
⎯⎯⎯⎯⎯⎯⎯⎯⎯⎯⎯
84. √ 2x −1 =3
⎯⎯⎯⎯⎯⎯⎯⎯⎯
85. √ x −8 +2=5
⎯⎯⎯⎯⎯⎯⎯⎯⎯
86. 3√ x − 5 − 1 = 11
⎯⎯⎯⎯⎯⎯⎯⎯⎯⎯⎯ ⎯⎯⎯⎯⎯⎯⎯⎯⎯⎯⎯⎯⎯
87. √ 5x − 3 = √ 2x + 15
⎯⎯⎯⎯⎯⎯⎯⎯⎯⎯⎯⎯⎯
88. √ 8x − 15 = x
⎯⎯⎯⎯⎯⎯⎯⎯⎯⎯⎯
89. √ x + 41 = x − 1
⎯⎯⎯⎯⎯⎯⎯⎯⎯⎯⎯
90. √ 7 − 3x = x − 3
⎯⎯⎯⎯⎯⎯⎯⎯⎯⎯⎯⎯⎯⎯
91. 2 (x + 1) = √ 2 (x + 1)
92. √ x (x + 6) = 4
⎯⎯⎯⎯⎯⎯⎯⎯⎯⎯⎯⎯⎯⎯⎯
⎯⎯⎯⎯⎯⎯⎯⎯⎯⎯⎯⎯⎯⎯⎯⎯⎯⎯
93. √ x (3x + 10) = 2
3
⎯⎯⎯⎯⎯⎯⎯⎯⎯⎯⎯⎯⎯
94. √ 2x 2
3
−x +4=5
⎯⎯⎯⎯⎯⎯⎯⎯⎯⎯⎯⎯⎯⎯⎯⎯⎯⎯⎯⎯⎯⎯⎯⎯⎯⎯ 3 ⎯⎯⎯⎯⎯⎯⎯⎯⎯⎯⎯⎯⎯
95. √ 3 (x + 4) (x + 1) = √ 5x + 37
3
⎯⎯⎯⎯⎯⎯⎯⎯⎯⎯⎯⎯⎯⎯⎯⎯⎯⎯⎯⎯⎯⎯⎯ 3 ⎯⎯⎯⎯⎯⎯⎯⎯⎯⎯⎯⎯2⎯
96. √ 3x 2 − 9x + 24 = √ (x + 2)
3
99. (x − 5)
1/2
−2=0
SAMPLE EXAM
1. Simplify.
⎯⎯⎯⎯⎯⎯
a. √ 100
⎯⎯⎯⎯⎯⎯⎯⎯⎯
b. √ −100
⎯⎯⎯⎯⎯⎯
c. −√ 100
2. Simplify.
3 ⎯⎯⎯⎯
a. √ 27
3 ⎯⎯⎯⎯⎯⎯⎯
b. √ −27
3 ⎯⎯⎯⎯
c. −√ 27
3. √
⎯128
⎯⎯⎯⎯⎯
25
4. √
⎯192
⎯⎯⎯⎯⎯
3
125
⎯⎯⎯⎯⎯⎯⎯⎯⎯⎯⎯⎯⎯⎯
5. 5√ 12x 2 y 3 z
⎯⎯⎯⎯⎯⎯⎯⎯⎯⎯⎯⎯⎯⎯⎯⎯⎯
6. √ 250x 2 y 3 z 5
3
( b)
⎯⎯⎯⎯ ⎯⎯⎯⎯ ⎯⎯
9. 2√ ab 3√ 2a − √
(√ √ )
⎯⎯ 2
10. x − 2 ⎯⎯
y
10
11.
√2x
1
12. 3
√ 4xy 2
1
13.
√x +5
√2 −√3
14.
√2 +√3
2/3
15. 2 ⋅ 2 1/6
10 4/5
16.
10 1/3
17. (121a4 b )
2 1/2
(9y x )
1/3 6 1/2
18.
y 1/6
Solve.
⎯⎯
19. √ x −7=0
⎯⎯⎯⎯⎯⎯⎯⎯⎯⎯⎯
20. √ 3x +5 =1
⎯⎯⎯⎯⎯⎯⎯⎯⎯⎯⎯
21. √ 2x −1 +2=x
⎯⎯⎯⎯⎯⎯⎯⎯⎯⎯⎯⎯⎯⎯⎯
22. √ 31 − 10x = x − 4
⎯⎯⎯⎯⎯⎯⎯⎯⎯⎯⎯⎯⎯⎯⎯⎯⎯⎯⎯⎯⎯⎯⎯⎯⎯⎯⎯ ⎯⎯⎯⎯⎯⎯⎯⎯⎯⎯⎯⎯⎯⎯⎯⎯
23. √ (2x + 1) (3x + 2) = √ 3 (2x + 1)
T = 2π √ 32
25. The period, T, of a pendulum in seconds is given the formula
⎯⎯⎯⎯
L
⎯
, where L represents the length in feet. Calculate the length
of a pendulum if the period is 1½ seconds. Round off to the nearest tenth.
1: 6
5: 5
7: 1/4
⎯⎯⎯⎯
9: 2√ 10
7√2
11:
9
⎯⎯
13: 20 √ 3
3
15: 7x
⎯⎯⎯⎯
17: 5xy√ 3x
3x√2x
19:
5y
21: 6x
3 ⎯⎯⎯⎯⎯⎯
23: 3a2 bc ⋅ √ ab 2
⎯⎯⎯⎯
25: √ 13
⎯⎯⎯⎯
27: 2√ 29
⎯⎯
29: 11√ 3
⎯⎯ ⎯⎯
31: 9√ 3 − √2
⎯⎯
33: 4√ x − 2√ ⎯⎯
y
⎯⎯ ⎯⎯
35: √ 5 − 3√ 3
⎯⎯ ⎯⎯⎯⎯
37: −x√ 3 + 5√ 5x
39: 12xy√ y
⎯⎯
⎯⎯ 3 ⎯⎯⎯⎯
41: 4 √
3
x − 5√ xy
3 ⎯⎯
43: 2x ⋅√ y
⎯⎯
45: 3√ 2
⎯⎯ ⎯⎯
47: √ 6 − 2√ 3
49: −4
51: 2a
⎯⎯
53: 3√ 2
7xy√2
55:
6
2√7
57: 7
7√2x
59: x
3
√ 4x
61:
2x
⎯⎯ ⎯⎯
63: √ 3 + √2
⎯⎯
65: √ 7
⎯⎯⎯⎯
67: √ x 4
5
69: 2
⎯⎯
71: 2 √ 2
3
73: 1/8
75: 9
77: 4
79: 6x 2 y
81: a1/3
83: 25
85: 17
87: 6
89: 8
91: −1/2, −1
93: 2/3, −4
97: 9
99: 9
1:
a. 10
b. Not a real number
c. −10
8√2
3:
5
⎯⎯⎯⎯⎯⎯
5: 10xy√ 3yz
⎯⎯ ⎯⎯
7: 14√ 6 − 15√ 3
⎯⎯⎯⎯ ⎯⎯
9: 6a√ 2b − 2b√ a
5√2x
11: x
√x −5
13:
x−25
5/6
15: 2
17: 11a2 b
19: 49
21: 5
1425
Chapter 9 Solving Quadratic Equations and Graphing Parabolas
LEARNING OBJECTIVE
Step 3: Apply the zero-product property and set each variable factor equal to 0.
2. A solution to a quadratic
equation in standard form.
1426
Chapter 9 Solving Quadratic Equations and Graphing Parabolas
The two solutions are −2 and 2. The goal in this section is to develop an alternative
method that can be used to easily solve equations where b = 0, giving the form
If we take the square root of both sides of this equation, we obtain the following:
The notation “±” is read “plus or minus” and is used as compact notation that
⎯⎯ ⎯⎯
indicates two solutions. Hence the statement x = ±√k indicates that x = −√k or
⎯⎯
x = √k . Applying the square root property as a means of solving a quadratic
equation is called extracting the roots4.
Answer: The solutions are −5 and 5. The check is left to the reader.
Certainly, the previous example could have been solved just as easily by factoring.
However, it demonstrates a technique that can be used to solve equations in this
form that do not factor.
Example 2: Solve: x 2 − 5 = 0.
Solution: Notice that the quadratic expression on the left does not factor. We can
extract the roots if we first isolate the leading term, x 2 .
For completeness, check that these two real solutions solve the original quadratic
equation. Generally, the check is optional.
⎯⎯ ⎯⎯
Answer: The solutions are −√5 and √5.
Example 3: Solve: 4x 2 − 45 = 0.
3√5 3√5
Answer: The solutions are − 2
and 2
.
Example 4: Solve: x 2 + 9 = 0.
After applying the square root property, we are left with the square root of a
negative number. Therefore, there is no real solution to this equation.
Reverse this process to find equations with given solutions of the form ±k.
⎯⎯ ⎯⎯
Example 5: Find an equation with solutions −2√3 and 2√3.
Lastly, subtract 12 from both sides and present the equation in standard form.
Answer: x 2 − 12 = 0
2√2 2√2
Answer: x = − 3
or x = 3
Video Solution
To solve this equation by factoring, first square x + 2 and then put it in standard
form, equal to zero, by subtracting 25 from both sides.
When an equation is in this form, we can obtain the solutions in fewer steps by
extracting the roots.
At this point, separate the “plus or minus” into two equations and simplify each
individually.
In addition to fewer steps, this method allows us to solve equations that do not
factor.
Solve for x.
⎯⎯ ⎯⎯
Answer: The solutions are −1 − √3 and −1 + √3.
3−2√2 3+2√2
Answer: The solutions are 6
and 6
.
15±√6
Answer: 3
Video Solution
Example 9: The length of a rectangle is twice its width. If the diagonal measures 2
feet, then find the dimensions of the rectangle.
Solution:
The diagonal of any rectangle forms two right triangles. Thus the Pythagorean
theorem applies. The sum of the squares of the legs of a right triangle is equal to the
square of the hypotenuse:
Solve.
4√5 2√5
Answer: The length of the rectangle is 5
feet and the width is 5
feet.
KEY TAKEAWAYS
TOPIC EXERCISES
1. x 2 − 36 = 0
2. x 2 − 81 = 0
3. 4y 2 −9=0
4. 9y 2 − 25 = 0
5. (x − 2) 2 − 1 = 0
6. (x + 1) 2 − 4 = 0
7. 4(y − 2) − 9 = 0
2
8. 9(y + 1) − 4 = 0
2
9. −3(t − 1)2 + 12 = 0
11. (x − 5) − 25 = 0
2
12. (x + 2) 2 − 4 = 0
13. x 2 = 16
14. x 2 =1
15. y 2 =9
16. y 2 = 64
1
17. x 2 = 4
1
18. x 2 = 9
19. y 2 = 0.25
20. y 2 = 0.04
21. x 2 = 12
22. x 2 = 18
23. 16x 2 =9
24. 4x 2 = 25
25. 2t 2 =1
26. 3t 2 =2
27. x 2 − 100 = 0
28. x 2 − 121 = 0
29. y 2 +4=0
30. y 2 +1=0
4
31. x 2 − =0
9
9
32. x 2 − =0
25
33. y 2 − 0.09 = 0
34. y 2 − 0.81 = 0
35. x 2 −7=0
36. x 2 −2=0
37. x 2 −8=0
38. t 2 − 18 = 0
39. x 2 +8=0
40. x 2 + 125 = 0
41. 16x 2 − 27 = 0
42. 9x 2 −8=0
43. 2y 2 −3=0
44. 5y 2 −2=0
45. 3x 2 −1=0
46. 6x 2 −3=0
47. (x + 7) 2 − 4 = 0
48. (x + 9) 2 − 36 = 0
49. (2y − 3) − 81 = 0
2
50. (2y + 1) − 25 = 0
2
51. (x − 5) − 20 = 0
2
52. (x + 1) 2 − 28 = 0
54. (3t − 5) − 10 = 0
2
55. 4(y + 2) − 3 = 0
2
56. 9(y − 7) − 5 = 0
2
57. 4(3x + 1) 2 − 27 = 0
58. 9(2x − 3) 2 − 8 = 0
59. 2(3x − 1) 2 + 3 = 0
60. 5(2x − 1) 2 − 3 = 0
61. 3(y − 23 ) −
2 3
2
=0
62. 2(3y − 13 ) −
2 5
2
=0
63. ±7
64. ±13
⎯⎯
65. ±√ 7
⎯⎯
66. ±√ 3
⎯⎯
67. ±3√ 5
⎯⎯
68. ±5√ 2
⎯⎯
69. 1 ± √2
⎯⎯
70. 2 ± √3
71. 9x (x + 2) = 18x + 1
72. x 2 = 10 (x 2 − 2) − 5
73. (x + 3) (x − 7) = 11 − 4x
74. (x − 4) (x − 3) = 66 − 7x
75. (x − 2) 2 = 67 − 4x
76. (x + 3) 2 = 6x + 59
77. (2x + 1) (x + 3) − (x + 7) = (x + 3) 2
78. (3x − 1) (x + 4) = 2x (x + 6) − (x − 3)
79. If 9 is subtracted from 4 times the square of a number, then the result is
3. Find the number.
80. If 20 is subtracted from the square of a number, then the result is 4. Find
the number.
81. If 1 is added to 3 times the square of a number, then the result is 2. Find
the number.
82. If 3 is added to 2 times the square of a number, then the result is 12. Find
the number.
83. If a square has an area of 8 square centimeters, then find the length of
each side.
84. If a circle has an area of 32π square centimeters, then find the length of
the radius.
85. The volume of a right circular cone is 36π cubic centimeters when the
height is 6 centimeters. Find the radius of the cone. (The volume of a right
circular cone is given by V = 1
3
πr2 h.)
86. The surface area of a sphere is 75π square centimeters. Find the radius
of the sphere. (The surface area of a sphere is given by SA = 4πr2 .)
87. The length of a rectangle is 6 times its width. If the area is 96 square
inches, then find the dimensions of the rectangle.
88. The base of a triangle is twice its height. If the area is 16 square
centimeters, then find the length of its base.
89. A square has an area of 36 square units. By what equal amount will the
sides have to be increased to create a square with double the given area?
90. A circle has an area of 25π square units. By what amount will the radius
have to be increased to create a circle with double the given area?
91. If the sides of a square measure 1 unit, then find the length of the
diagonal.
92. If the sides of a square measure 2 units, then find the length of the
diagonal.
93. The diagonal of a square measures 5 inches. Find the length of each side.
94. The diagonal of a square measures 3 inches. Find the length of each side.
95. The length of a rectangle is twice its width. If the diagonal measures 10
feet, then find the dimensions of the rectangle.
96. The length of a rectangle is twice its width. If the diagonal measures 8
feet, then find the dimensions of the rectangle.
97. The length of a rectangle is 3 times its width. If the diagonal measures 5
meters, then find the dimensions of the rectangle.
98. The length of a rectangle is 3 times its width. If the diagonal measures 2
feet, then find the dimensions of the rectangle.
99. The height in feet of an object dropped from a 9‑foot ladder is given by
h(t) = −16t 2 + 9, where t represents the time in seconds after the
object has been dropped. How long does it take the object to hit the ground?
(Hint: The height is 0 when the object hits the ground.)
100. The height in feet of an object dropped from a 20‑foot platform is given
by h(t) = −16t 2 + 20 , where t represents the time in seconds after the
object has been dropped. How long does it take the object to hit the ground?
101. The height in feet of an object dropped from the top of a 144-foot
building is given by h(t) = −16t 2 + 144 , where t is measured in
seconds.
a. How long will it take to reach half of the distance to the ground, 72 feet?
b. How long will it take to travel the rest of the distance to the ground?
102. The height in feet of an object dropped from an airplane at 1,600 feet is
given by h(t) = −16t 2 + 1,600 , where t is in seconds.
a. How long will it take to reach half of the distance to the ground?
b. How long will it take to travel the rest of the distance to the ground?
103. Create an equation of your own that can be solved by extracting the
root. Share it, along with the solution, on the discussion board.
104. Explain why the technique of extracting roots greatly expands our
ability to solve quadratic equations.
105. Explain in your own words how to solve by extracting the roots.
106. Derive a formula for the diagonal of a square in terms of its sides.
ANSWERS
1: −6, 6
3: −3/2, 3/2
5: 1, 3
7: 1/2, 7/2
9: −1, 3
11: 0, 10
13: ±4
15: ±3
17: ±1/2
19: ±0.5
⎯⎯
21: ±2√ 3
23: ±3/4
√2
25: ±
2
27: ±10
31: ±2/3
33: ±0.3
⎯⎯
35: ±√ 7
⎯⎯
37: ±2√ 2
3√3
41: ±
4
√6
43: ±
2
√3
45: ±
3
47: −9, −5
49: −3, 6
⎯⎯
51: 5 ± 2√ 5
−2±√6
53:
3
−4±√3
55:
2
−2±3√3
57:
6
4±3√2
61:
6
63: x 2 − 49 = 0
65: x 2 −7=0
67: x 2 − 45 = 0
69: x 2 − 2x − 1 = 0
71: ±0.33
73: ±5.66
75: ±7.94
77: ±3.61
⎯⎯ ⎯⎯
79: −√ 3 or √ 3
81: −
√3 √3
3
or 3
⎯⎯
83: 2√ 2 centimeters
⎯⎯
85: 3√ 2 centimeters
⎯⎯
89: −6 + 6√ 2 ≈ 2.49 units
⎯⎯
91: √ 2 units
5√2
93: 2
inches
⎯⎯ ⎯⎯
95: Length: 4√ 5 feet; width: 2√ 5 feet
3√10 √10
97: Length: 2
meters; width: 2
meters
LEARNING OBJECTIVE
In this section, we will devise a method for rewriting any quadratic equation of the
form
in the form
The last term, 9, is the square of one-half of the coefficient of x. In general, this is
true for any perfect square trinomial of the form x 2 + bx + c.
(x − p) = q.
quadratic equation in the form
2
1449
Chapter 9 Solving Quadratic Equations and Graphing Parabolas
Note
It is important to point out that the leading coefficient must be equal to 1 for
this to be true.
Solution: In this example, the coefficient of the middle term b = 8, so find the value
that completes the square as follows:
Answer: x 2 + 8x + 16 = (x + 4)2
Solution: Here b = 3, so find the value that will complete the square as follows:
= (x + 32 )
2
Answer: x 2 + 3x + 9
4
We can use this technique to solve quadratic equations. The idea is to take any
quadratic equation in standard form and complete the square so that we can solve
it by extracting roots. The following are general steps for solving a quadratic
equation with a leading coefficient of 1 in standard form by completing the square.
Solution:
Step 1: Add or subtract the constant term to obtain the equation in the form
x 2 + bx = c . In this example, subtract 46 to move it to the right side of the
equation.
Step 2: Use ( b2 ) to determine the value that completes the square. Here b = 14:
2
Step 3: Add ( b2 ) to both sides of the equation and complete the square.
2
⎯⎯ ⎯⎯
Answer: The solutions are −7 − √3 or −7 + √3. The check is optional.
Next, find the value that completes the square using b = −18.
To complete the square, add 81 to both sides, complete the square, and then solve
by extracting the roots.
At this point, separate the “plus or minus” into two equations and solve each.
Note that in the previous example the solutions are integers. If this is the case, then
the original equation will factor.
If it factors, we can solve it by factoring. However, not all quadratic equations will
factor.
Here b = 10, and we determine the value that completes the square as follows:
⎯⎯ ⎯⎯
Answer: The solutions are −5 − 2√6 and −5 + 2√6.
Therefore,
At this point we see that extracting the root leads to the square root of a negative
number.
⎯⎯
Answer: x = 1 ± 2√7
Video Solution
−3±√17
Answer: The solutions are 2
.
So far, all of the examples have had a leading coefficient of 1. The formula ( b2 )
2
determines the value that completes the square only if the leading coefficient is 1. If
this is not the case, then simply divide both sides by the leading coefficient.
Solution: Notice that the leading coefficient is 2. Therefore, divide both sides by 2
before beginning the steps required to solve by completing the square.
Here b = 5/2, and we can find the value that completes the square as follows:
−5±√33
Answer: The solutions are 4
.
1±√7
Answer: x = 2
Video Solution
KEY TAKEAWAYS
TOPIC EXERCISES
+ 6x + ? = (x + ? )
2
1. x 2
+ 8x + ? = (x + ? )
2
2. x 2
− 2x + ? = (x − ? )
2
3. x 2
− 4x + ? = (x − ? )
2
4. x 2
+ 7x + ? = (x + ? )
2
5. x 2
+ 3x + ? = (x + ? )
2
6. x 2
x + ? = (x + ? )
2 2
7. x 2 + 3
x + ? = (x + ? )
4 2
8. x 2 + 5
x + ? = (x + ? )
3 2
9. x 2 + 4
x + ? = (x + ? )
5 2
10. x 2 + 3
Solve by factoring and then solve by completing the square. Check answers.
11. x 2 + 2x − 8 = 0
12. x 2 − 8x + 15 = 0
13. y 2 + 2y − 24 = 0
14. y 2 − 12y + 11 = 0
15. t 2 + 3t − 28 = 0
16. t 2 − 7t + 10 = 0
17. 2x 2 + 3x − 2 = 0
18. 3x 2 −x−2=0
19. 2y 2 −y−1=0
20. 2y 2 + 7y − 4 = 0
21. x 2 + 6x − 1 = 0
22. x 2 + 8x + 10 = 0
23. x 2 − 2x − 7 = 0
24. x 2 − 6x − 3 = 0
25. x 2 − 2x + 4 = 0
26. x 2 − 4x + 9 = 0
27. t 2 + 10t − 75 = 0
29. x 2 − 4x − 1 = 15
35. x 2 − 2
3
x− 1
3
=0
36. x 2 − 4
5
x− 1
5
=0
37. x 2 +x−1=0
38. x 2 +x−3=0
39. y 2 + 3y − 2 = 0
40. y 2 + 5y − 3 = 0
41. x 2 + 3x + 5 = 0
42. x 2 +x+1=0
11
43. x 2 − 7x + =0
2
3
44. x 2 − 9x + =0
2
45. t 2 − 1
2
t−1=0
46. t 2 − 1
3
t−2=0
49. 4x 2 − 8x − 1 = 0
50. 2x 2 − 4x − 3 = 0
51. 3x 2 + 6x + 1 = 0
52. 5x 2 + 10x + 2 = 0
53. 3x 2 + 2x − 3 = 0
54. 5x 2 + 2x − 5 = 0
55. 4x 2 − 12x − 15 = 0
56. 2x 2 + 4x − 43 = 0
57. 2x 2 − 4x + 10 = 0
58. 6x 2 − 24x + 42 = 0
59. 2x 2 −x−2=0
60. 2x 2 + 3x − 1 = 0
61. 3x 2 + 2x − 2 = 0
62. 3x 2 −x−1=0
63. x (x + 1) − 11 (x − 2) = 0
64. (x + 1) (x + 7) − 4 (3x + 2) = 0
65. y 2 = (2y + 3) (y − 1) − 2 (y − 1)
Solve by completing the square and round off the solutions to the nearest hundredth.
69. (2x − 1) 2 = 2x
75. Research and discuss the Hindu method for completing the square.
76. Explain why the technique for completing the square described in this
section requires that the leading coefficient be equal to 1.
ANSWERS
+ 6x + 9 = (x + 3)
2
1: x 2
− 2x + 1 = (x − 1)
2
3: x 2
5: x 2 + 7x + 49
4
= (x + 2)
7 2
7: x 2 + 2
3
x+ 1
9
= (x + 3)
1 2
9: x 2 + 3
4
x+ 9
64
= (x + 8)
3 2
11: −4, 2
13: −6, 4
15: −7, 4
17: 1/2, −2
19: −1/2, 1
⎯⎯⎯⎯
21: −3 ± √ 10
⎯⎯
23: 1 ± 2√ 2
27: −15, 5
⎯⎯
29: 2 ± 2√ 5
⎯⎯
31: 10 ± 5√ 3
35: −1/3, 1
−1±√5
37:
2
−3±√17
39:
2
7±3√3
43:
2
1±√17
45:
4
2±√5
49:
2
−3±√6
51:
3
−1±√10
53:
3
3±2√6
55:
2
1±√17
59:
4
−1±√7
61:
3
⎯⎯
63: 5 ± √3
1±√5
65:
2
5±√21
67:
2
LEARNING OBJECTIVE
In this section, we will develop a formula that gives the solutions to any quadratic
equation in standard form. To do this, we begin with a general quadratic equation
in standard form and solve for x by completing the square. Here a, b, and c are real
numbers and a ≠ 0:
Determine the constant that completes the square: take the coefficient of x, divide it
by 2, and then square it.
1469
Chapter 9 Solving Quadratic Equations and Graphing Parabolas
This derivation gives us a formula that solves any quadratic equation in standard
form. Given ax 2 + bx + c = 0, where a, b, and c are real numbers and a ≠ 0, then
the solutions can be calculated using the quadratic formula6:
6. The formula
−b±√b 2 −4ac
x= 2a
, which
gives the solutions to any
quadratic equation in the form
ax 2 + bx + c = 0, where a, Consider the quadratic equation 2x 2 − 7x + 3 = 0. It can be solved by factoring as
b, and c are real numbers and
a ≠ 0. follows:
The solutions are 1/2 and 3. The following example shows that we can obtain the
same results using the quadratic formula.
Substitute these values into the quadratic formula and then simplify.
Separate the “plus or minus” into two equations and simplify each individually.
−1±√6
Answer: The solutions are 5
.
Often terms are missing. When this is the case, use 0 as the coefficient.
Here
⎯⎯
Answer: The solutions are ±3√2.
Since the coefficient of x was 0, we could have solved the equation by extracting the
roots. As an exercise, solve the previous example using this method and verify that
the results are the same.
Substitute these values into the quadratic formula and then simplify.
In this example, notice that the radicand of the square root is 0. This results in only
one solution to this quadratic equation. Normally, we expect two solutions. When
we find only one solution, the solution is called a double root. If we solve this
equation by factoring, then the solution appears twice.
The solution involves the square root of a negative number; hence the solutions are
not real. This quadratic equation has two nonreal solutions and will be discussed in
further detail as we continue in our study of algebra. For now, simply state that the
equation does not have real solutions.
⎯⎯
Answer: 1 ± √3
Video Solution
It is important to place the quadratic equation in standard form before using the
quadratic formula.
Solution: Begin by using the distributive property to expand the left side and
combining like terms to obtain an equation in standard form, equal to 0.
Substitute these values into the quadratic formula and then simplify.
6±3√5
Answer: The solutions are 2
.
3±2√3
Answer: 3
Video Solution
KEY TAKEAWAYS
TOPIC EXERCISES
Identify the coefficients a, b, and c used in the quadratic formula. Do not solve.
1. x 2 −x+5=0
2. x 2 − 3x − 1 = 0
3. 3x 2 − 10 = 0
4. −y 2 +5=0
5. 5t 2 − 7t = 0
6. −y 2 +y=0
7. −x 2 + x = −6
8. −2x 2 − x = −15
Solve by factoring and then solve using the quadratic formula. Check answers.
11. x 2 − 10x + 24 = 0
12. x 2 − 3x − 18 = 0
13. t 2 + 6t + 5 = 0
14. t 2 + 9t + 14 = 0
15. 2x 2 − 7x − 4 = 0
16. 3x 2 −x−2=0
19. y 2 − 2y + 1 = 0
20. y 2 −1=0
21. x 2 − 6x + 4 = 0
22. x 2 − 4x + 1 = 0
23. x 2 + 2x − 5 = 0
24. x 2 + 4x − 6 = 0
25. t 2 − 4t − 1 = 0
26. t 2 − 8t − 2 = 0
27. −y 2 +y+1=0
28. −y 2 − 3y + 2 = 0
29. −x 2 + 16x − 62 = 0
30. −x 2 + 14x − 46 = 0
31. 2t 2 − 4t − 3 = 0
32. 4t 2 − 8t − 1 = 0
35. 3x 2 + 6x + 2 = 0
36. 5x 2 + 10x + 2 = 0
37. 9t 2 + 6t − 11 = 0
38. 8t 2 + 8t + 1 = 0
39. x 2 −2=0
40. x 2 − 18 = 0
41. 9x 2 −3=0
42. 2x 2 −5=0
43. y 2 +9=0
44. y 2 +1=0
45. 2x 2 =0
46. x 2 =0
47. −2y 2 + 5y = 0
48. −3y 2 + 7y = 0
49. t 2 −t =0
50. t 2 + 2t = 0
55. 1
2
t 2 + 5t + 3
2
=0
56. −t 2 + 3t − 3
4
=0
57. 3y 2 + 1
2
y− 1
3
=0
58. −2y 2 + 1
3
y+ 1
2
=0
59. 2x 2 − 10x + 3 = 4
60. 3x 2 + 6x + 1 = 8
62. 3y 2 = 5(2y − 1)
63. (t + 1)2 = 2t + 7
65. (x + 5) (x − 1) = 2x + 1
66. (x + 7) (x − 2) = 3 (x + 1)
67. x (x + 5) = 3 (x − 1)
68. x (x + 4) = −7
73. (x + 2) 2 − 2 (x + 7) = 4 (x + 1)
74. (x + 3) 2 − 10 (x + 5) = −2 (x + 1)
76. Research and discuss the history of the quadratic formula and solutions
to quadratic equations.
ANSWERS
1: a = 1, b = −1 , and c = 5
3: a = 3, b = 0, and c = −10
5: a = 5, b = −7 , and c = 0
7: a = −1, b = 1, and c = 6
9: a = 6, b = −2 , and c = 1
11: 4, 6
13: −5, −1
15: −1/2, 4
17: −3/2, 1
⎯⎯
21: 3 ± √5
⎯⎯
23: −1 ± √6
⎯⎯
25: 2 ± √5
1±√5
27:
2
⎯⎯
29: 8 ± √2
2±√10
31:
2
−3±√3
35:
3
−1±2√3
37:
3
⎯⎯
39: ±√ 2
√3
41: ±
3
47: 0, 5/2
49: 0, 1
⎯⎯⎯⎯
55: −5 ± √ 22
−1±√17
57:
12
5±3√3
59:
2
−3±√33
61:
4
⎯⎯
63: ±√ 6
⎯⎯
65: −1 ± √7
⎯⎯⎯⎯
73: 1 ± √ 15
LEARNING OBJECTIVES
Discriminant
The solutions are rational, irrational, or not real. We can determine the type and
number of solutions by studying the discriminant7, the expression inside the
radical, b2 − 4ac. If the value of this expression is negative, then the equation has
no real solutions. If the discriminant is positive, then we have two real solutions.
And if the discriminant is 0, then we have one real solution.
1488
Chapter 9 Solving Quadratic Equations and Graphing Parabolas
Since the discriminant is negative, we conclude that there are no real solutions.
If we use the quadratic formula in the previous example, we find that a negative
radicand stops the process of simplification and shows that there is no real solution.
Note
Solution: Here
Since the discriminant is positive, we can conclude that there are two real solutions.
If we use the quadratic formula in the previous example, we find that a positive
radicand in the quadratic formula leads to two real solutions.
5−3√2 5+3√2
The two real solutions are 7 and 7 . Note that these solutions are irrational;
we can approximate the values on a calculator.
Since the discriminant is positive, we conclude that there are two real solutions.
Furthermore, since the discriminant is a perfect square, we obtain two rational
solutions.
We could solve the previous quadratic equation using the quadratic formula as
follows:
Note that if the discriminant is a perfect square, then we could have factored the
original equation.
Given the special condition where the discriminant is 0, we obtain only one
solution, a double root.
Since the discriminant is 0, we conclude that there is only one real solution, a
double root.
As we will see, knowing the number and type of solutions ahead of time helps us
determine which method is best for solving a quadratic equation.
Video Solution
Use the coefficients of a quadratic equation to help decide which method is most
appropriate for solving it. While the quadratic formula always works, it is
sometimes not the most efficient method. Given any quadratic equation in standard
form, ax 2 + bx + c = 0, general guidelines for determining the method for
solving it follow:
Solution: In this case, c = 0 and we can solve by factoring out the GCF.
Example 6: Solve: 3x 2 − 5 = 0.
√15
Answer: The solutions are ± 3
.
Example 7: Solve: 9x 2 − 6x − 7 = 0.
Since the discriminant is positive and not a perfect square, use the quadratic
formula and expect two real solutions.
1±2√2
Answer: The solutions are 3
.
Here
Since the discriminant is negative, the solutions are not real numbers.
Since the discriminant is 0, solve by factoring and expect one real solution, a double
root.
2√5
Answer: ± 5
Video Solution
In this section, the algebraic setups usually consist of a quadratic equation where
the solutions may not be integers.
Example 10: The height of a triangle is 2 inches less than twice the length of its
base. If the total area of the triangle is 11 square inches, then find the lengths of the
base and height. Round answers to the nearest hundredth.
Solution:
Use the coefficients, a = 1, b = −1, and c = −11, to determine the type of solutions.
In this problem, disregard the negative solution and consider only the positive
solution.
1+3√5
Answer: The base measures ≈ 3.85inches and the height is
⎯⎯ 2
Example 11: The sum of the squares of two consecutive positive integers is 481.
Find the integers.
Solution:
When the coefficients are large, sometimes it is less work to use the quadratic
formula instead of trying to factor it. In this case, a = 1, b = 1, and c = −240.
Substitute into the quadratic formula and then simplify.
Since the problem calls for positive integers, disregard the negative solution and
choose n = 15.
KEY TAKEAWAYS
TOPIC EXERCISES
Calculate the discriminant and use it to determine the number and type of solutions.
Do not solve.
1. x 2 + 2x + 3 = 0
2. x 2 − 2x − 3 = 0
3. 3x 2 − 1x − 2 = 0
4. 3x 2 − 1x + 2 = 0
5. 9y 2 +2=0
6. 9y 2 −2=0
7. 5x 2 +x=0
8. 5x 2 −x=0
9. 1
2
x 2 − 2x + 5
2
=0
10. 1
2
x2 − x − 1
2
=0
11. −x 2 − 2x + 4 = 0
12. −x 2 − 4x + 2 = 0
13. 4t 2 − 20t + 25 = 0
14. 9t 2 − 6t + 1 = 0
Part B: Solving
15. x 2 − 2x − 3 = 0
16. x 2 + 2x + 3 = 0
17. 3x 2 −x−2=0
18. 3x 2 −x+2=0
19. 9y 2 +2=0
20. 9y 2 −2=0
21. 5x 2 +x=0
22. 5x 2 −x=0
23. 1
2
x 2 − 2x + 5
2
=0
24. 1
2
x2 − x − 1
2
=0
25. −x 2 − 2x + 4 = 0
26. −x 2 − 4x + 2 = 0
27. 4t 2 − 20t + 25 = 0
28. 9t 2 − 6t + 1 = 0
29. y 2 − 4y − 1 = 0
30. y 2 − 6y − 3 = 0
33. 5t 2 −4=0
34. 2t 2 −9=0
35. 1
2
x2 − 9
4
x+1=0
36. 3x 2 + 1
2
x− 1
6
=0
37. 36y 2 = 2y
39. x (x − 6) = −29
40. x (x − 4) = −16
41. 4y (y + 1) = 5
42. 2y (y + 2) = 3
43. −3x 2 = 2x + 1
44. 3x 2 + 4x = −2
46. 2(x + 2) 2 = 7x + 11
47. 9t 2 = 4 (3t − 1)
48. 5t (5t − 6) = −9
49. (x + 1) (x + 7) = 3
50. (x − 5) (x + 7) = 14
Part C: Applications
Number Problems
51. A positive real number is 2 less than another. When 4 times the larger is
added to the square of the smaller, the result is 49. Find the numbers.
52. A positive real number is 1 more than another. When twice the smaller is
subtracted from the square of the larger, the result is 4. Find the numbers.
53. A positive real number is 6 less than another. If the sum of the squares of
the two numbers is 38, then find the numbers.
54. A positive real number is 1 more than twice another. If 4 times the
smaller number is subtracted from the square of the larger, then the result
is 21. Find the numbers.
Geometry Problems
55. The area of a rectangle is 60 square inches. If the length is 3 times the
width, then find the dimensions of the rectangle.
56. The area of a rectangle is 6 square feet. If the length is 2 feet more than
the width, then find the dimensions of the rectangle.
57. The area of a rectangle is 27 square meters. If the length is 6 meters less
than 3 times the width, then find the dimensions of the rectangle.
58. The area of a triangle is 48 square inches. If the base is 2 times the height,
then find the length of the base.
59. The area of a triangle is 14 square feet. If the base is 4 feet more than 2
times the height, then find the length of the base and the height.
60. The area of a triangle is 8 square meters. If the base is 4 meters less than
the height, then find the length of the base and the height.
61. The perimeter of a rectangle is 54 centimeters and the area is 180 square
centimeters. Find the dimensions of the rectangle.
62. The perimeter of a rectangle is 50 inches and the area is 126 square
inches. Find the dimensions of the rectangle.
Pythagorean Theorem
⎯⎯
65. If the sides of a square measure 10√ 6 units, then find the length of the
diagonal.
⎯⎯⎯⎯
66. If the diagonal of a square measures 3√ 10 units, then find the length
of each side.
⎯⎯
67. The diagonal of a rectangle measures 6√ 3 inches. If the width is 4
inches less than the length, then find the dimensions of the rectangle.
⎯⎯
68. The diagonal of a rectangle measures 2√ 3 inches. If the width is 2
inches less than the length, then find the dimensions of the rectangle.
69. The top of a 20-foot ladder, leaning against a building, reaches a height
of 18 feet. How far is the base of the ladder from the wall? Round off to the
nearest hundredth.
70. To safely use a ladder, the base should be placed about 1/4 of the ladder’s
length away from the wall. If a 20-foot ladder is to be safely used, then how
high against a building will the top of the ladder reach? Round off to the
nearest hundredth.
Business Problems
73. The profit in dollars of running an assembly line that produces custom
uniforms each day is given by the function
P(t) = −40t 2 + 960t − 4,000 , where t represents the number of
hours the line is in operation.
a. Calculate the profit on running the assembly line for 10 hours a day.
b. Calculate the number of hours the assembly line should run in order to
break even. Round off to the nearest tenth of an hour.
74. The profit in dollars generated by producing and selling x custom lamps
is given by the function P(x) = −10x 2 + 800x − 12,000 .
Projectile Problems
b. At what time after launch will the projectile reach a height of 128 feet?
79. The height of an object dropped from the top of a 144-foot building is
given by h (t) = −16t 2 + 144 . How long will it take to reach a point
halfway to the ground?
80. The height of a projectile shot straight up into the air at 80 feet/second
from the ground is given by h (t) = −16t 2 + 80t. At what time will the
projectile reach 95 feet?
81. Discuss the strategy of always using the quadratic formula to solve
quadratic equations.
82. List all of the methods that we have learned so far to solve quadratic
equations. Discuss the pros and cons of each.
ANSWERS
15: −1, 3
17: −2/3, 1
21: −1/5, 0
⎯⎯
25: −1 ± √5
27: 5/2
⎯⎯
29: 2 ± √5
2√5
33: ±
5
35: 1/2, 4
37: 0, 1/18
−1±√6
41:
2
47: 2/3
⎯⎯
49: −4 ± 2√ 3
⎯⎯ ⎯⎯
51: 3√ 5 and 3√ 5 −2
⎯⎯⎯⎯ ⎯⎯⎯⎯
53: 3 + √ 10 and −3 + √ 10
⎯⎯
65: 20√ 3 units
⎯⎯ ⎯⎯
67: Length: 2 + 5√ 2 inches; width: −2 + 5√ 2 inches
⎯⎯⎯⎯
69: 2√ 19 ≈ 8.72 feet
75: 5.5%
LEARNING OBJECTIVES
1. Graph a parabola.
2. Find the intercepts and vertex of a parabola.
3. Find the vertex of a parabola by completing the square.
We know that any linear equation with two variables can be written in the form
y = mx + b and that its graph is a line. In this section, we will see that any
quadratic equation of the form y = ax 2 + bx + c has a curved graph called a
parabola8.
Two points determine any line. However, since a parabola is curved, we should find
more than two points. In this text, we will determine at least five points as a means
to produce an acceptable sketch. To begin, we graph our first parabola by plotting
points. Given a quadratic equation of the form y = ax 2 + bx + c, x is the
independent variable and y is the dependent variable. Choose some values for x and
then determine the corresponding y-values. Then plot the points and sketch the
graph.
1514
Chapter 9 Solving Quadratic Equations and Graphing Parabolas
Answer:
the axis of symmetry11) is the vertical line through the vertex, about which the
parabola is symmetric.
For any parabola, we will find the vertex and y-intercept. In addition, if the x-
intercepts exist, then we will want to determine those as well. Guessing at the x-
values of these special points is not practical; therefore, we will develop techniques
that will facilitate finding them. Many of these techniques will be used extensively
as we progress in our study of algebra.
Next, recall that the x-intercepts, if they exist, can be found by setting y = 0 . Doing
this, we have 0 = a2 + bx + c, which has general solutions given by the quadratic
−b±√b2 −4ac
formula, x = 2a
. Therefore, the x-intercepts have this general form:
Using the fact that a parabola is symmetric, we can determine the vertical line of
symmetry using the x-intercepts. To do this, we find the x-value midway between
the x-intercepts by taking an average as follows:
We can use the line of symmetry to find the x-value of the vertex. The steps for
graphing a parabola are outlined in the following example.
Example 2: Graph: y = −x 2 − 2x + 3 .
Solution:
Here when y = 0, we obtain two solutions. There are two x-intercepts, (−3, 0) and
(1, 0).
Step 3: Determine the vertex. One way to do this is to use the equation for the line
b
of symmetry, x = − 2a , to find the x-value of the vertex. In this example, a = −1 and
b = −2:
Step 4: Determine extra points so that we have at least five points to plot. In this
example, one other point will suffice. Choose x = −2 and find the corresponding y-
value.
Step 5: Plot the points and sketch the graph. To recap, the points that we have
found are
y-intercept: (0, 3)
Vertex: (−1, 4)
Answer:
The parabola opens downward. In general, use the leading coefficient to determine
whether the parabola opens upward or downward. If the leading coefficient is
negative, as in the previous example, then the parabola opens downward. If the
leading coefficient is positive, then the parabola opens upward.
Example 3: Graph: y = 2x 2 + 4x + 5 .
Solution: Because the leading coefficient 2 is positive, note that the parabola opens
upward. Here c = 5 and the y-intercept is (0, 5). To find the x-intercepts, set y = 0.
In this case, a = 2, b = 4, and c = 5. Use the discriminant to determine the number and
type of solutions.
Since the discriminant is negative, we conclude that there are no real solutions.
Because there are no real solutions, there are no x-intercepts. Next, we determine
the x-value of the vertex.
Given that the x-value of the vertex is −1, substitute into the original equation to
find the corresponding y-value.
The vertex is (−1, 3). So far, we have only two points. To determine three more,
choose some x-values on either side of the line of symmetry, x = −1. Here we choose
x-values −3, −2, and 1.
To summarize, we have
y-intercept: (0, 5)
x-intercepts: None
Vertex: (−1, 3)
Answer:
Solution: Note that a = −2: the parabola opens downward. Since c = −18, the y-
intercept is (0, −18). To find the x-intercepts, set y = 0.
Solve by factoring.
Here x = 3 is a double root, so there is only one x-intercept, (3, 0). From the original
equation, a = −2, b = 12, and c = −18. The x-value of the vertex can be calculated as
follows:
Given that the x-value of the vertex is 3, substitute into the original equation to find
the corresponding y-value.
Therefore, the vertex is (3, 0), which happens to be the same point as the x-
intercept. So far, we have only two points. To determine three more, choose some x-
values on either side of the line of symmetry, x = 3 in this case. Choose x-values 1, 5,
and 6.
To summarize, we have
x-intercept: (3, 0)
Vertex: (3, 0)
Answer:
Example 5: Graph: y = x 2 − 2x − 1 .
In this case, solve using the quadratic formula with a = 1, b = −2, and c = −1.
Here we obtain two real solutions for x, and thus there are two x-intercepts:
Use the approximate answers to place the ordered pair on the graph. However, we
will present the exact x-intercepts on the graph. Next, find the vertex.
Given that the x-value of the vertex is 1, substitute into the original equation to find
the corresponding y-value.
To summarize, we have
Answer:
Answer:
Video Solution
It is often useful to find the maximum and/or minimum values of functions that
model real-life applications. To find these important values given a quadratic
function, we use the vertex. If the leading coefficient a is positive, then the parabola
opens upward and there will be a minimum y-value. If the leading coefficient a is
negative, then the parabola opens downward and there will be a maximum y-value.
Solution: Since a = −4, we know that the parabola opens downward and there will be
a maximum y-value. To find it, we first find the x-value of the vertex.
The x-value of the vertex is 3. Substitute this value into the original equation to find
the corresponding y-value.
The vertex is (3, 1). Therefore, the maximum y-value is 1, which occurs when x = 3,
as illustrated below:
Note
Solution: Since a = +4, the parabola opens upward and there is a minimum y-value.
Begin by finding the x-value of the vertex.
The vertex is (4, −2). Therefore, the minimum y-value of −2 occurs when x = 4, as
illustrated below:
Video Solution
Solution: First, note that since a = 1 is positive, the parabola opens upward. Hence
there will be a minimum y-value. To find that value, find the x-value of the vertex:
The vertex is (2, −1). The range consists of the set of y-values greater than or equal
to the minimum y-value −1.
Solution: Here a = −16, and the parabola opens downward. Therefore, the y-value
of the vertex determines the maximum height. Begin by finding the x-value of the
vertex:
The maximum height will occur in 9/4 = 2¼ seconds. Substitute this time into the
function to determine the height attained.
In this section, we demonstrate an alternate approach for finding the vertex. Any
quadratic equation y = ax 2 + bx + c can be rewritten in the form
Solution: When the equation is in this form, we can read the vertex directly from
the equation.
Here h = 3 and k = 1.
Often the equation is not given in this form. To obtain this form, complete the
square.
y = x 2 + 4x + 9 .
Solution: Begin by making room for the constant term that completes the square.
The idea is to add and subtract the value that completes the square, ( b2 ) , and then
2
Adding and subtracting the same value within an expression does not change it.
Doing so is equivalent to adding 0. Once the equation is in this form, we can easily
determine the vertex.
Here h = −2 and k = 5.
If there is a leading coefficient other than 1, then we must first factor out the
leading coefficient from the first two terms of the trinomial.
y = 2x 2 − 4x + 8 .
Solution: Since a = 2, factor this out of the first two terms in order to complete the
square. Leave room inside the parentheses to add a constant term.
Here h = 1 and k = 6.
y = −2x 2 − 12x + 3.
Video Solution
KEY TAKEAWAYS
vertex is (h, k) and can be read directly from the equation. To obtain this
form, take y = ax 2 + bx + c and complete the square.
TOPIC EXERCISES
1. y = x 2 − 9x + 20
2. y = x 2 − 12x + 32
3. y = −2x 2 + 5x + 12
4. y = −6x 2 + 13x − 6
5. y = 64 − x 2
6. y = −3x + 9x 2
7. y = x 2 + 4x − 12
8. y = x 2 − 13x + 12
9. y = 2x 2 + 5x − 3
10. y = 3x 2 − 4x − 4
11. y = −5x 2 − 3x + 2
13. y = 4x 2 − 25
14. y = 9x 2 − 49
15. y = x2 − x + 1
16. y = 5x 2 + 15x
17. y = −x 2 + 10x − 34
18. y = −x 2 − 6x + 1
20. y = −9x 2 + 6x + 2
21. y = 4x 2 − 1
22. y = x 2 − 16
Graph. Find the vertex and the y-intercept. In addition, find the x-intercepts if they
exist.
23. y = x 2 − 2x − 8
24. y = x 2 − 4x − 5
25. y = −x 2 + 4x + 12
26. y = −x 2 − 2x + 15
27. y = x 2 − 10x
28. y = x 2 + 8x
29. y = x2 − 9
30. y = x 2 − 25
31. y = 1 − x2
32. y = 4 − x2
33. y = x 2 − 2x + 1
34. y = x 2 + 4x + 4
36. y = −4x 2 − 4x + 3
37. y = x2 − 2
38. y = x2 − 3
39. y = −4x 2 + 4x − 3
40. y = 4x 2 + 4x + 3
41. y = x 2 − 2x − 2
42. y = x 2 − 6x + 6
43. y = −2x 2 + 6x − 3
44. y = −4x 2 + 4x + 1
45. y = x 2 + 3x + 4
46. y = −x 2 + 3x − 4
47. y = −2x 2 + 3
48. y = −2x 2 − 1
49. y = 2x 2 + 4x − 3
50. y = 3x 2 + 2x − 2
51. y = −x 2 − 6x + 1
52. y = −x 2 − 4x + 8
55. y = −x 2
56. y = 1 − 9x 2
58. y = 12x + 4x 2
59. y = 3x 2 − 4x − 2
60. y = 6x 2 − 8x + 5
Given the following quadratic functions, determine the domain and range.
67. The profit in dollars generated by producing and selling x custom lamps
is given by the function P(x) = −10x 2 + 800x − 12,000 . What is
the maximum profit?
69. The average number of hits to a radio station website is modeled by the
formula f (x) = 450t 2 − 3,600t + 8,000 , where t represents the
number of hours since 8:00 a.m. At what hour of the day is the number of
hits to the website at a minimum?
= −(x − 5) + 3
2
73. y
74. y = −2(x − 1) 2 + 7
75. y = 5(x + 1) 2 + 6
76. y = 3(x + 4) 2 + 10
77. y = −5(x + 8) 2 − 1
78. y = (x + 2) 2 − 5
79. y = x 2 − 14x + 24
80. y = x 2 − 12x + 40
81. y = x 2 + 4x − 12
82. y = x 2 + 6x − 1
83. y = 2x 2 − 12x − 3
84. y = 3x 2 − 6x + 5
85. y = −x 2 + 16x + 17
86. y = −x 2 + 10x
Graph.
87. y = x2 − 1
88. y = x2 + 1
89. y = (x − 1) 2
90. y = (x + 1) 2
91. y = (x − 4) 2 − 9
92. y = (x − 1) 2 − 4
93. y = −2(x + 1) 2 + 8
94. y = −3(x + 2) 2 + 12
95. y = −5(x − 1) 2
96. y = −(x + 2) 2
97. y = −4(x − 1) 2 − 2
98. y = 9(x + 1) 2 + 2
= (x + 5) − 15
2
99. y
= 2(x − 5) − 3
2
100. y
101. y = −2(x − 4) 2 + 22
102. y = 2(x + 3) 2 − 13
103. Write down your plan for graphing a parabola on an exam. What will
you be looking for and how will you present your answer? Share your plan
on the discussion board.
ANSWERS
1: Upward
3: Downward
5: Downward
23:
25:
27:
29:
31:
33:
35:
37:
39:
41:
43:
45:
47:
49:
51: Maximum: y = 10
53: Minimum: y = 4
55: Maximum: y = 0
57: Maximum: y = 10
67: $4,000
73: (5, 3)
75: (−1, 6)
87:
89:
91:
93:
95:
97:
99:
101:
LEARNING OBJECTIVES
Up to this point, the square root of a negative number has been left undefined. For
⎯⎯⎯⎯⎯
example, we know that √−9 is not a real a number.
There is no real number that when squared results in a negative number. We begin
the resolution of this issue by defining the imaginary unit12, i, as the square root of
−1.
⎯⎯⎯⎯⎯
12. Defined as i = √−1 and
i2 = −1.
1553
Chapter 9 Solving Quadratic Equations and Graphing Parabolas
⎯⎯⎯⎯⎯
If √−9 = 3i, then we would expect that 3i squared equals −9:
Therefore, the square root of any negative real number can be written in terms of
the imaginary unit. Such numbers are often called imaginary numbers13.
⎯⎯⎯⎯⎯
a. √−4
⎯⎯⎯⎯⎯
b. √−5
⎯⎯⎯⎯⎯
c. √−8
Solution:
Notation Note
⎯⎯ ⎯⎯
2i√2 = 2√2 i
where a and b are real numbers. Here a is called the real part15 and b is called the
imaginary part16. For example, 3 − 4i is a complex number with a real part, 3, and
an imaginary part, −4. It is important to note that any real number is also a complex
number. For example, the real number 5 is also a complex number because it can be
written as 5 + 0i with a real part of 5 and an imaginary part of 0. Hence the set of
real numbers, denoted R, is a subset of the set of complex numbers, denoted C.
Adding and subtracting complex numbers is similar to adding and subtracting like
terms. Add or subtract the real parts and then the imaginary parts.
Answer: 5 + i
To subtract complex numbers, subtract the real parts and subtract the imaginary
parts. This is consistent with the use of the distributive property.
Solution: Distribute the negative one and then combine like terms.
Answer: 1 − 9i
The distributive property also applies when multiplying complex numbers. Make
use of the fact that i2 = −1 to resolve the result into standard form: a + bi.
Answer: 20 + 15i
Solution:
Answer: 32 − i
Given a complex number a + bi, its complex conjugate17 is a − bi. We next explore
the product of complex conjugates.
Answer: 25
Note that the result does not involve the imaginary unit; hence the result is real.
This leads us to the very useful property:
1
Example 7: Divide: 1−2i .
18. The real number that results
from multiplying complex
(a + bi) (a − bi) = a + b in
conjugates: Solution: In this example, the conjugate of the denominator is 1 + 2i. Multiply by 1
2 2
. the form (1+2i)
(1+2i)
.
To express this complex number in standard form, write each term over the
common denominator 5.
Answer: 1
5
+ 2
5
i
Solution:
1
Answer: 13 − 18
13
i
5+5i
Try this! Divide: 1−3i .
Answer: −1 + 2i
Video Solution
Now that complex numbers are defined, we can complete our study of solutions to
quadratic equations. Often solutions to quadratic equations are not real.
Substitute these values into the quadratic formula and then simplify.
The equation may not be given in standard form. The general steps for solving
using the quadratic formula are outlined in the following example.
Solution:
Step 3: Substitute the appropriate values into the quadratic formula and then
simplify.
Here a = 1, b = 2, and c = 19. Substitute these values into the quadratic formula.
⎯⎯ ⎯⎯
Answer: The solutions are −1 − 3i√2 and −1 + 3i√2.
Notation Note
⎯⎯ ⎯⎯
−1 + 3i√2 = −1 + 3√2 i
⎯⎯
Both numbers are equivalent and −1 + 3√2i is in standard form, where the
⎯⎯
real part is −1 and the imaginary part is 3√2. However, this number is often
⎯⎯
expressed as −1 + 3i√2, even though this expression is not in standard form.
Again, this is done to avoid the possibility of misinterpreting the imaginary
unit as part of the radicand.
−4±i√6 √6
Answer: 2
= −2 ± 2
i
Video Solution
KEY TAKEAWAYS
TOPIC EXERCISES
Rewrite in terms of i.
⎯⎯⎯⎯⎯⎯⎯
1. √ −64
⎯⎯⎯⎯⎯⎯⎯
2. √ −81
⎯⎯⎯⎯⎯⎯⎯
3. √ −20
⎯⎯⎯⎯⎯⎯⎯
4. √ −18
⎯⎯⎯⎯⎯⎯⎯
5. √ −50
⎯⎯⎯⎯⎯⎯⎯
6. √ −48
⎯⎯⎯⎯⎯⎯⎯
7. −√ −45
⎯⎯⎯⎯⎯
8. −√ −8
9. √ −
⎯⎯⎯⎯⎯1⎯
4
10. √ −
⎯⎯⎯⎯⎯2⎯
9
15. ( 12 + 3
4
i) + ( 16 − 1
8
i)
16. ( 25 − 1
6
i) + ( 101 − 3
2
i)
21. ( 141 + 3
2
i) − ( 47 − 3
4
i)
22. ( 38 − 1
3
i) − ( 12 − 1
2
i)
23. 2i (7 − 4i)
24. 6i (1 − 2i)
26. −5i (2 − i)
27. (2 + i) (2 − 3i)
29. (1 − i) (8 − 9i)
31. (4 + 3i)2
32. (2 − 5i)
2
35. ( 12 + 2
3
i) ( 13 − 1
2
i)
36. ( 23 − 1
3
i) ( 12 − 3
2
i)
1
37.
5+4i
1
38.
3−4i
20i
39.
1−3i
10i
40.
1−2i
10−5i
41.
3−i
4−2i
42.
2−2i
5+10i
43.
3+4i
2−4i
44.
5+3i
1+2i
45.
2−3i
3−i
46.
4−5i
Solve by extracting the roots and then solve by using the quadratic formula. Check
answers.
47. x 2 +9=0
48. x 2 +1=0
49. 4t 2 + 25 = 0
50. 9t 2 +4=0
51. 4y 2 +3=0
52. 9y 2 +5=0
53. 3x 2 +2=0
54. 5x 2 +3=0
55. (x + 1) 2 + 4 = 0
56. (x + 3) 2 + 9 = 0
57. x 2 − 2x + 10 = 0
58. x 2 − 4x + 13 = 0
59. x 2 + 4x + 6 = 0
60. x 2 + 2x + 9 = 0
61. y 2 − 6y + 17 = 0
62. y 2 − 2y + 19 = 0
63. t 2 − 5t + 10 = 0
64. t 2 + 3t + 4 = 0
65. −x 2 + 10x − 29 = 0
66. −x 2 + 6x − 10 = 0
67. −y 2 −y−2=0
68. −y 2 + 3y − 5 = 0
71. 3y 2 − 2y + 4 = 0
72. 5y 2 − 4y + 3 = 0
73. 2x 2 + 3x + 2 = 0
74. 4x 2 + 2x + 1 = 0
75. 2x 2 − 1
2
x+ 1
4
=0
76. 3x 2 − 2
3
x+ 1
3
=0
77. 2x (x − 1) = −1
78. x (2x + 5) = 3x − 5
79. 3t (t − 2) + 4 = 0
80. 5t (t − 1) = t − 4
82. (2y + 5) − 12 (y + 1) = 0
2
83. −3 (y + 3) (y − 5) = 5y + 46
84. −2 (y − 4) (y + 1) = 3y + 10
85. 9x (x − 1) + 3 (x + 2) = 1
86. 5x (x + 2) − 6 (2x − 1) = 5
87. 3 (t − 1) − 2t (t − 2) = 6t
88. 3 (t − 3) − t (t − 5) = 7t
90. 5 (x + 1) (x − 1) − 3x 2 = −8
91. Explore the powers of i. Share your discoveries on the discussion board.
ANSWERS
1: 8i
⎯⎯
3: 2i√ 5
⎯⎯
5: 5i√ 2
⎯⎯
7: −3i√ 5
i
9:
2
11: 10 +i
13: −3 −i
15: 2
3
+ 5
8
i
17: −3 + 5i
21: − 1
2
+ 9
4
i
23: 8 + 14i
25: −8 − 6i
27: 7 − 4i
29: −1 − 17i
31: 7 + 24i
33: 20
35: 1
2
− 1
36
i
37: 5
41
− 4
41
i
39: −6 + 2i
41: 7
2
− 1
2
i
43: 11
5
− 2
5
i
45: − 4
13
+ 7
13
i
47: ±3i
5i
49: ±
2
i√3
51: ±
2
i√6
53: ±
3
55: −1 ± 2i
57: 1 ± 3i
⎯⎯
59: −2 ± i√ 2
⎯⎯
61: 3 ± 2i√ 2
63: 5
2
± √15
2
i
65: 5 ± 2i
67: − ± i
1 √7
2 2
69: 5
2
± 3
2
i
71: 1
3
± √11
3
i
73: − ± i
3 √7
4 4
75: 1
8
± √7
8
i
77: 1
2
± 1
2
i
79: 1 ± i
√3
3
81: 1
2
±i
83: 1
6
± √11
6
i
85: 1
3
± 2
3
i
87: 1
4
± √23
4
i
⎯⎯
89: ±i√ 5
1575
Chapter 9 Solving Quadratic Equations and Graphing Parabolas
REVIEW EXERCISES
1. x 2 − 16 = 0
9
2. y 2 = 4
3. x 2 − 27 = 0
4. x 2 + 27 = 0
5. 3y 2 − 25 = 0
6. 9x 2 −2=0
7. (x − 5) − 9 = 0
2
8. (2x − 1) 2 − 1 = 0
9. 16(x − 6) − 3 = 0
2
10. 2(x + 3) 2 − 5 = 0
11. (x + 3) (x − 2) = x + 12
12. (x + 2) (5x − 1) = 9x − 1
⎯⎯
13. ±√ 2
⎯⎯
14. ±2√ 5
17. x 2 − 12x + 1 = 0
18. x 2 + 8x + 3 = 0
19. y 2 − 4y − 14 = 0
20. y 2 − 2y − 74 = 0
21. x 2 + 5x − 1 = 0
22. x 2 − 7x − 2 = 0
23. 2x 2 +x−3=0
24. 5x 2 + 9x − 2 = 0
25. 2x 2 − 16x + 5 = 0
26. 3x 2 − 6x + 1 = 0
27. 2y 2 + 10y + 1 = 0
28. 5y 2 +y−3=0
29. x (x + 9) = 5x + 8
30. (2x + 5) (x + 2) = 8x + 7
Quadratic Formula
Identify the coefficients a, b, and c used in the quadratic formula. Do not solve.
31. x 2 −x+4=0
32. −x 2 + 5x − 14 = 0
33. x 2 −5=0
34. 6x 2 +x=0
35. x 2 − 6x + 6 = 0
36. x 2 + 10x + 23 = 0
37. 3y 2 −y−1=0
38. 2y 2 − 3y + 5 = 0
39. 5x 2 − 36 = 0
40. 7x 2 + 2x = 0
41. −x 2 + 5x + 1 = 0
42. −4x 2 − 2x + 1 = 0
45. (x − 3) 2 − 2x = 47
46. 9x (x + 1) − 5 = 3x
47. −x 2 + 5x + 1 = 0
48. −x 2 +x−1=0
49. 4x 2 − 4x + 1 = 0
50. 9x 2 −4=0
51. x 2 + 4x − 60 = 0
52. 9x 2 + 7x = 0
54. t 2 + 16 = 0
55. x 2 −x−3=0
56. 9x 2 + 12x + 1 = 0
57. 4(x − 1) 2 − 27 = 0
58. (3x + 5) − 4 = 0
2
59. (x − 2) (x + 3) = 6
60. x (x − 5) = 12
61. (x + 1) (x − 8) + 28 = 3x
63. The length of a rectangle is 2 inches less than twice the width. If the area
measures 25 square inches, then find the dimensions of the rectangle. Round
off to the nearest hundredth.
Graphing Parabolas
67. y = 2x 2 + 5x − 3
68. y = x 2 − 12
69. y = 5x 2 − x + 2
70. y = −x 2 + 10x − 25
71. y = x 2 − 6x + 1
72. y = −x 2 + 8x − 1
73. y = x 2 + 3x − 1
74. y = 9x 2 − 1
Graph. Find the vertex and the y-intercept. In addition, find the x-intercepts if they
exist.
75. y = x 2 + 8x + 12
76. y = −x 2 − 6x + 7
77. y = −2x 2 − 4
78. y = x 2 + 4x
79. y = 4x 2 − 4x + 1
80. y = −2x 2
81. y = −2x 2 + 8x − 7
82. y = 3x 2 − 1
83. y = x 2 − 10x + 1
84. y = −x 2 + 12x − 1
85. y = −5x 2 + 6x
86. y = 2x 2 − x − 1
Rewrite in terms of i.
⎯⎯⎯⎯⎯⎯⎯
89. √ −36
⎯⎯⎯⎯⎯⎯⎯
90. √ −40
91. √ −
⎯⎯⎯⎯⎯⎯⎯8 ⎯
25
92. −√ −
⎯⎯⎯⎯⎯1⎯
9
95. (2 − 3i) (5 + i)
4−i
96.
2−3i
Solve.
97. 9x 2 + 25 = 0
98. 3x 2 +1=0
99. y 2 −y+5=0
100. y 2 + 2y + 4
101. 4x (x + 2) + 5 = 8x
102. 2 (x + 2) (x + 3) = 3 (x 2 + 13)
SAMPLE EXAM
1. 4x 2 −9=0
2. (4x + 1) 2 − 5 = 0
3. x 2 + 10x + 19 = 0
4. x 2 −x−1=0
5. −2x 2 +x+3=0
6. x 2 + 6x − 31 = 0
7. (5x + 1) (x + 1) = 1
8. (x + 5) (x − 5) = 65
9. x (x + 3) = −2
10. 2(x − 2) 2 − 6 = 3x 2
11. The length of a rectangle is twice its width. If the diagonal measures
⎯⎯
6√ 5 centimeters, then find the dimensions of the rectangle.
12. The height in feet reached by a model rocket launched from a platform is
given by the function h(t) = −16t 2 + 256t + 3 , where t represents
time in seconds after launch. At what time will the rocket reach 451 feet?
Graph. Find the vertex and the y-intercept. In addition, find the x-intercepts if they
exist.
13. y = 2x 2 − 4x − 6
14. y = −x 2 + 4x − 4
15. y = 4x 2 − 9
16. y = x 2 + 2x − 1
20. The height in feet reached by a model rocket launched from a platform is
given by the function h(t) = −16t 2 + 256t + 3 , where t represents
time in seconds after launch. What is the maximum height attained by the
rocket.
⎯⎯⎯⎯⎯⎯⎯
22. Rewrite in terms of i: √ −60 .
4−2i
23. Divide: .
4+2i
Solve.
25. −2x 2 + 5x − 1 = 0
1: ±16
⎯⎯
3: ±3√ 3
5√3
5: ±
3
7: 2, 8
24±√3
9:
4
⎯⎯
11: ±3√ 2
13: x 2 −2=0
15: x 2 − 6x + 9 = (x − 3) 2
⎯⎯⎯⎯
17: 6 ± √ 35
⎯⎯
19: 2 ± 3√ 2
−5±√29
21:
2
23: −3/2, 1
8±3√6
25:
2
−5±√23
27:
2
⎯⎯
29: −2 ± 2√ 3
31: a = 1, b = −1 , and c = 4
33: a = 1, b = 0, and c = −5
⎯⎯
35: 3 ± √3
1±√13
37:
6
6√5
39: ±
5
5±√29
41:
2
43: −12, 24
⎯⎯
45: 4 ± 3√ 6
51: −10, 6
53: ±1/5
1±√13
55:
2
2±3√3
57:
2
59: −4, 3
⎯⎯
61: 5 ± √5
75:
77:
79:
81:
87: The car will have a minimum value 12 years after it is purchased.
89: 6i
2i√2
91:
5
93: 5 −i
95: 13 − 13i
5i
97: ±
3
1 √19
99:
2
± 2
i
i√5
101: ±
2
1: ± 3
2
⎯⎯
3: −5 ± √6
5: −1, 3/2
7: −6/5, 0
9: −2, −1
13:
15:
17: Maximum: y = −3
21: To maximize revenue, the company needs to produce and sell 100
bicycles a week.
3 4
23:
5
− 5
i
5±√17
25:
4
LEARNING OBJECTIVE
1.
1592
Chapter 10 Appendix: Geometric Figures
10.1 Plane
Area (A) is measured in square units, perimeter (P) is measured in units, and
circumference (C) is measured in units.
Square
Rectangle
Parallelogram
1593
Chapter 10 Appendix: Geometric Figures
Trapezoid
Triangle
Circle
10.2 Solid
Volume (V) is measured in cubic units and surface area (SA) is measured in square
units.
Cube
Rectangular Solid
1595
Chapter 10 Appendix: Geometric Figures
Sphere